M AY AY O C L I N I C INTERNAL MEDICINE BOARD REVIEW QUESIONS AND A NSWERS
M AY AY O C L I N I C INTERNAL MEDICINE BOARD REVIEW QUESIONS AND A NSWERS
Mayo Clinic Scienti�c Press Mayo Clinic Clinic Atlas of Regional Anesthesia Anesthesia and Ultrasound-Guided Ultrasound-Guided Nerve Blockade Blockade
Edited by James R. Hebl, MD, and Robert L. Lennon, DO Mayo Clinic Clinic Preventive Medicine Medicine and Public Public Health Board Board Review
Edited by Prathibha Varkey Varkey,, MBBS, MBBS , MPH, MHPE Mayo Clinic Clinic Challenging Images Images for Pulmonary Pulmonary Board Review
Edited by Edward C. Rosenow III, MD Mayo Clinic Clinic Gastroenterolog Gastroenterolog y and Hepatology Hepatology Board Review , Review , Fourth Edition
Edited by Stephen C. Hause Hauser, r, MD Mayo Clinic Clinic Infectious Diseases Board Board Review
Edited by Zelalem emesgen, emesgen, MD Mayo Clinic Clinic Antimicrobial Handbook: Handbook: Quick Guide , Second Edition
Edited by John W. Wilson, MD, and Lynn L. Estes, PharmD Just Enough Physiology Physiology
By James R. Munis, MD, PhD Mayo Clinic Clinic Cardiology: Concise Textbook Textbook , Fourth Edition
Edited by Joseph G. Murphy, Murphy, MD, and Margaret A. A . Lloyd, MD MD E dition Mayo Clinic Clinic Internal Medicine Medicine Board Review, Review, enth Edition Edited by Robert D. Ficalora, MD
MAYO CLINIC INTERNAL MEDICINE B OAR D REVI EW QUESIONS AND A NSWERS
EDIOR�IN�CHIEF
Robert D. Ficalora , MD C O N S U L A N , D I V I S I O N O F G E N E R A L I N E R N A L M E D I C I N E MAYO CLINIC, ROCHESER, MINNESOA A S S O C I A E P R O F E S S O R O F M E D I C I N E COLLEGE OF MEDICINE, M AYO CLINIC
EDIOR
Paul S. Mueller , MD A S S O C I A E E D I O R S
Thomas J. Beckman, MD
Nicole P. Sandhu , MD, PhD
Margaret Beliveau , MD
Amy T. Wang , MD
Mark C. Lee , MD
Christopher M. Wittich , MD
MAYO CLINIC SCIENIFIC PRESS
OXFORD UNIVERSIY PRESS
Te triple-shield Mayo logo and the words MAYO, MAYO CLINIC, and MAYO CLINIC SCIENIFIC PRESS are marks o Mayo Foundation or Medical Education and Research.
3 Oxord University Press is a department o the University o Oxord. It urthers the University’s objective o excellence in research, scholarship, and education by publishing worldwide. Oxord New York Auckland Cape own Dar es Salaam Hong Kong Karachi Kuala Lumpur Madrid Melbourne Mexico City Nairobi New Delhi Shanghai aipei oronto With offices in Argentina Austria Brazil Chile Czech Republic France Greece Guatemala Hungary Italy Japan Poland Portugal Singapore South Korea Switzerland Tailand urkey Ukraine Vietnam Oxord is a registered trademark o Oxord University Press in the UK and c ertain other countries. Published in the United States o America by Oxord University Press 198 Madison Avenue, New York, NY 10016 © Mayo Foundation or Medical Education and Research 2013 All rights reserved. No part o this publication may be reproduced, stored in a retrieval system, or transmitted, in any orm or by any means, electronic, mechanical, photocopying, recording, or otherwise, without the prior permission o Mayo Foundation or Medical Education and Research. Inquiries should be addressed to Scienti�c Publications, Plummer 10, Mayo Clinic, 200 First St SW, Rochester, MN 55905. Library o Congress Cataloging-in-Publication Data Mayo Clinic internal medicine board review : questions and answers / editor-in-chie Robert D. Ficalora ; editor, Paul S. Mueller ; associate editors, Tomas J. Beckman ... [et al.].—10th ed. p. ; cm.—(Mayo Clinic scienti�c press) Internal medicine board review : questions and answers Companion volume to: Mayo Clinic internal medicine board review. 10th ed. c2013. Includes bibliographical reerences and index. ISBN 978–0–19–998587–6 (alk. paper)—ISBN 978–0–19–932203–9 (alk. paper)—ISBN 978–0–19–932204–6 (alk. paper) I. Ficalora, Robert D. II. Mayo Clinic. III. Mayo Foundation or Medical Education and Research. IV. Mayo Clinic internal medicine board review. V. itle: Internal medicine board review : questions and answers. VI. Series: Mayo Clinic scienti�c press (Series) [DNLM: 1. Internal Medicine—Examination �uestions. WB 18.2] 616.0076—dc23
2013001640
Mayo Foundation does not endorse any particular products or services, and the reerence to any products or services in this boo k is or inormational purposes only and should not be taken as an endorsement by the authors or Mayo Foundation. Care has been taken to con�rm the accuracy o the inormation presented and to describe generally accepted practices. However, the authors, editors, and publisher are not responsible or errors or omissions or or any consequences rom application o the inormation in this book and make no warranty, express or implied, with respect to the contents o the publication. Tis book should not be relied on apart rom the advice o a quali�ed health care provider. Te authors, editors, and publisher have exerted efforts to ensure that drug selection and dosage set orth in this text are in accordance with current recommendations and practice at the time o publication. However, in view o ongoing research, changes in government regulations, and the constant �ow o inormation relating to drug therapy and drug reactions, readers are urged to check the package insert or each drug or any change in indications and dosage and or added wordings and precautions. Tis is particularly important when the recommended agent is a new or inrequently employed drug. Some drugs and medical devices presented in this publication have US Food and Drug Administration (FDA) clearance or limited use in restricted research settings. It is the responsibility o the heal th care providers to ascertain the FDA status o each drug or de vice planned or use in their clinical practice.
9 8 7 6 5 4 3 2 1 Printed in China on acid-ree paper
Dedicated to all the patients who help us, as internists, learn, practice, and master internal medicine. Robert D. Ficalora, MD
This page intentionally left blank
FOREWORD
One o the Department o Medicine’s strategic goals is to volume make both volumes more portable and user-riendly. provide premier education in the science and art o medicine. New questions and answers simulate the types o questions Established goals include leading the nation in the develop- included on the American Board o Internal Medicine examiment o lielong learning programs and educating physician nation. Te editors and associate editors added their depth o and nonphysician learners at all levels and along all points o experience to ensure that this edition is the �nest in the long the education continuum. Tese goals are attained by provid- history o this book. Te text is not only inormational but ing state-o-the-art graduate medical education. Te rapid also o great assistance in preparing or board certi�cation and pace at which medical knowledge is being discovered neces- recerti�cation, and it allows or the practical application o sitates requent updates. Mayo Clinic Internal Medicine Board knowledge to serve our patients. Review: Questions and Answers re�ects changes in the science o medicine and contains eatures that acilitate retention o Morie A. Gertz, MD the knowledge imparted. Te chapters in the companion textChair, Department o Internal Medicine book have been completely revised to correspond to American Mayo Clinic, Rochester, Minnesota Board o Internal Medicine objectives and include evidenceProessor o Medicine based recommendations. Bulleted points allow easy access to College o Medicine key points. Te questions and answers placed in this companion Mayo Clinic
vii
This page intentionally left blank
PREFACE
Mayo Clinic Internal Medicine Board Review: Questions and without the dedication o the associate editors who labored Answers is the result o the combined efforts o Mayo Clinic in isolation over the chapter drafs. We are all indebted to
physicians who practice in all the various subspecialties o staffs o the Department o Medicine; Section o Scienti�c Internal Medicine. Many have achieved certi�cates in medical Publications, Joseph G. Murphy, MD, Chair; LeAnn M. education and thus understand how to communicate inorma- Stee and Randall J. Fritz, DVM (editors), Kenna Atherton tion to our readers—physicians who are in training and prac- (manager), Jane M. Craig (editorial assistant), and Alissa K. ticing clinicians who are preparing or the American Board o Baumgartner (prooreader); and Section o Illustration and Internal Medicine (ABIM) certi�cation and maintenance-o- Design, Deb Veerkamp and Ryan Ledebuhr, at Mayo Clinic certi�cation examinations in internal medicine. or their contributions to this edition. I grateully acknowlOur annual Mayo Clinic Board Review Course, now in its edge the support and cooperation o the publisher, Oxord ��th year, gives the authors and editors the unique opportu- University Press. In particular, I am indebted to my adminnity to interact with our readers and tailor our approach to istrative partner, Michael O’Brien, whose unailing support the way the current generation o learners prepares or a high helped me through some very difficult times. stakes examination. In response to learner eedback, we have In the spirit o the previous editions, I trust that Mayo placed the questions and answers in this volume separate rom Clinic Internal Medicine Board Review: Questions and Answers the specialty-area content. More than ��� ABIM-ormat will serve our readers well in preparation or the primary certimultiple-choice questions with a single answer and explana- �cation or maintenance-o-certi�cation examination. tion are keyed to each chapter in the content textbook. I am grateul to the current and past authors or their Robert D. Ficalora, MD careul attention and hard work. Tis book would not exist Editor-in-Chie
ix
This page intentionally left blank
CONENS
Contributors
xiii
�. Preparing or the ABIM Examination �. Cardiology �uestions and Answers �. Gastroenterology and Hepatology �uestions and Answers �. Pulmonary Diseases �uestions and Answers �. Inectious Diseases �uestions and Answers �. Rheumatology �uestions and Answers �. Endocrinology �uestions and Answers �. Oncology �uestions and Answers �. Hematology �uestions and Answers ��. Nephrology �uestions and Answers ��. Allerg y �uestions and Answers ��. Psychiatry �uestions and Answers ��. Neurology �uestions and Answers ��. Dermatolog y �uestions and Answers ��. Cross-Content Area �uestions and Answers Index
� � �� �� �� �� �� �� �� �� ��� ��� ��� ��� ��� ���
xi
This page intentionally left blank
CONRIBUORS
Charles F. Abboud, MB, ChB
Robert D. Ficalora, MD
Consultant, Division o Endocrinology, Diabetes, Metabolism, & Nutrition Mayo Clinic, Rochester, Minnesota; and Associate Proessor o Medicine College o Medicine, Mayo Clinic
Consultant, Division o General Internal Medicine Mayo Clinic, Rochester, Minnesota; and Associate Proessor o Medicine College o Medicine, Mayo Clinic William W. Ginsburg, MD
Consultant, Division o Rheumatology Mayo Clinic, Jacksonville, Florida; and Associate Proessor o Medicine College o Medicine, Mayo Clinic
Tomas J. Beckman, MD
Consultant, Division o General Internal Medicine Mayo Clinic, Rochester, Minnesota; and Proessor o Medicine and o Medical Education College o Medicine, Mayo Clinic
Lyell K. Jones Jr, MD
Consultant, Department o Neurology Mayo Clinic, Rochester, Minnesota; and Assistant Proessor o Neurology College o Medicine, Mayo Clinic
Margaret Beliveau, MD
Consultant, Division o General Internal Medicine Mayo Clinic, Rochester, Minnesota; and Assistant Proessor o Medicine College o Medicine, Mayo Clinic
Mary J. Kasten, MD
Consultant, Division o Inectious Diseases Mayo Clinic, Rochester, Minnesota; and Associate Proessor o Medicine College o Medicine, Mayo Clinic
Consultant, Divisions o General Internal Medicine and Inectious Diseases Mayo Clinic, Rochester, Minnesota; and Assistant Proessor o Medicine College o Medicine, Mayo Clinic
Peter A. Brady, MB, ChB, MD
Kyle W. Klarich, MD
Consultant, Division o Cardiovascular Diseases Mayo Clinic, Rochester, Minnesota; and Associate Proessor o Medicine College o Medicine, Mayo Clinic
Consultant, Division o Cardiovascular Diseases Mayo Clinic, Rochester, Minnesota; and Associate Proessor o Medicine College o Medicine, Mayo Clinic
Maria L. Collazo-Clavell, MD
Mark C. Lee, MD
Consultant, Division o Endocrinology, Diabetes, Metabolism, & Nutrition Mayo Clinic, Rochester, Minnesota; and Associate Proessor o Medicine College o Medicine, Mayo Clinic
Consultant, Division o General Internal Medicine Mayo Clinic, Rochester, Minnesota; and Assistant Proessor o Medicine College o Medicine, Mayo Clinic
Lisa A. Drage, MD
Consultant, Division o General Internal Medicine Mayo Clinic, Rochester, Minnesota; and Proessor o Medicine College o Medicine, Mayo Clinic
Elie F. Berbari, MD
Scott C. Litin, MD
Consultant, Department o Dermatology Mayo Clinic, Rochester, Minnesota; and Assistant Proessor o Dermatology College o Medicine, Mayo Clinic
Conor G. Lofus, MD
Consultant, Division o Gastroenterology and Hepatology Mayo Clinic, Rochester, Minnesota; and Assistant Proessor o Medicine College o Medicine, Mayo Clinic
J. Christopher Farmer, MD
Consultant, Division o Pulmonary and Critical Care Medicine Mayo Clinic, Rochester, Minnesota; and Proessor o Medicine College o Medicine, Mayo Clinic xiii
Fabien Maldonado, MD
John J. Poterucha, MD
Consultant, Division o Pulmonary and Critical Care Medicine Mayo Clinic, Rochester, Minnesota; and Assistant Proessor o Medicine College o Medicine, Mayo Clinic
Consultant, Division o Gastroenterology and Hepatology Mayo Clinic, Rochester, Minnesota; and Proessor o Medicine College o Medicine, Mayo Clinic Abhiram Prasad, MD
Consultant, Division o Cardiovascular Diseases Mayo Clinic, Rochester, Minnesota; and Proessor o Medicine College o Medicine, Mayo Clinic
Robert D. McBane, MD
Consultant, Division o Cardiovascular Diseases Mayo Clinic, Rochester, Minnesota; and Proessor o Medicine College o Medicine, Mayo Clinic
Rajiv K. Pruthi, MBBS
Consultant, Division o Hematology Mayo Clinic, Rochester, Minnesota; and Associate Proessor o Medicine College o Medicine, Mayo Clinic
Kevin G. Moder, MD
Consultant, Division o Rheumatology Mayo Clinic, Rochester, Minnesota; and Associate Proessor o Medicine College o Medicine, Mayo Clinic
Qi Qian, MD
Consultant, Division o Nephrology & Hypertension Mayo Clinic, Rochester, Minnesota; and Associate Proessor o Medicine and o Physiology College o Medicine, Mayo Clinic
imothy J. Moynihan, MD
Consultant, Division o Medical Oncology Mayo Clinic, Rochester, Minnesota; and Associate Proessor o Oncology College o Medicine, Mayo Clinic
Nicole P. Sandhu, MD, PhD
Consultant, Division o General Internal Medicine Mayo Clinic, Rochester, Minnesota; and Assistant Proessor o Medicine College o Medicine, Mayo Clinic
Paul S. Mueller, MD
Chair, Division o General Internal Medicine Mayo Clinic, Rochester, Minnesota; and Proessor o Biomedical Ethics and o Medicine College o Medicine, Mayo Clinic
M. Rizwan Sohail, MD
Suzanne M. Norby, MD
Consultant, Division o Nephrology & Hypertension Mayo Clinic, Rochester, Minnesota; and Assistant Proessor o Medicine College o Medicine, Mayo Clinic
Consultant, Division o Inectious Diseases Mayo Clinic, Rochester, Minnesota; and Assistant Proessor o Medicine College o Medicine, Mayo Clinic Marius N. Stan, MD
Consultant, Division o Gastroenterology and Hepatology Mayo Clinic, Rochester, Minnesota; and Associate Proessor o Medicine College o Medicine, Mayo Clinic
Consultant, Division o Endocrinology, Diabetes, Metabolism, & Nutrition Mayo Clinic, Rochester, Minnesota; and Assistant Proessor o Medicine College o Medicine, Mayo Clinic
John G. Park, MD
Bruce Sutor, MD
Consultant, Division o Pulmonary and Critical Care Medicine Mayo Clinic, Rochester, Minnesota; and Assistant Proessor o Medicine College o Medicine, Mayo Clinic
Consultant, Division o Psychiatry and Psychology Mayo Clinic, Rochester, Minnesota; and Assistant Proessor o Psychiatry College o Medicine, Mayo Clinic
Naveen L. Pereira, MD
Consultant, Division o Cardiovascular Diseases Mayo Clinic, Rochester, Minnesota; and Assistant Proessor o Medicine College o Medicine, Mayo Clinic
Consultant, Division o Pulmonary and Critical Care Medicine Mayo Clinic, Rochester, Minnesota; and Associate Proessor o Medicine College o Medicine, Mayo Clinic
Axel P�ueger, MD, PhD
Seth R. Sweetser, MD
Consultant, Division o Nephrology & Hypertension Mayo Clinic, Rochester, Minnesota; and Proessor o Medicine College o Medicine, Mayo Clinic
Consultant, Division o Gastroenterology and Hepatology Mayo Clinic, Rochester, Minnesota; and Assistant Proessor o Medicine College o Medicine, Mayo Clinic
Amy S. Oxentenko, MD
Karen L. Swanson, DO
xiv • C O N R I B U O R S
Keith M. Swetz, MD
Amy . Wang, MD
Consultant, Division o General Internal Medicine Mayo Clinic, Rochester, Minnesota; and Assistant Proessor o Medicine College o Medicine, Mayo Clinic
Senior Associate Consultant, Division o General Internal Medicine Mayo Clinic, Rochester, Minnesota; and Assistant Proessor o Medicine College o Medicine, Mayo Clinic
Carrie A. Tompson, MD
Consultant, Division o Hematology Mayo Clinic, Rochester, Minnesota; and Assistant Proessor o Medicine College o Medicine, Mayo Clinic Pritish K. osh, MD
Mayo Clinic Scholar in Inectious Diseases Mayo Clinic, Rochester, Minnesota; and Assistant Proessor o Medicine College o Medicine, Mayo Clinic Gerald W. Volcheck, MD
Christopher M. Wittich, MD
Consultant, Division o General Internal Medicine Mayo Clinic, Rochester, Minnesota; and Assistant Proessor o Medicine College o Medicine, Mayo Clinic Alexandra P. Wolanskyj, MD
Consultant, Division o Hematology Mayo Clinic, Rochester, Minnesota; and Associate Proessor o Medicine College o Medicine, Mayo Clinic
Consultant, Division o Allergic Diseases Mayo Clinic, Rochester, Minnesota; and Associate Proessor o Medicine College o Medicine, Mayo Clinic
CONRIBUORS
•
xv
This page intentionally left blank
�. PREPARING FOR T HE AB IM E XAMINATION a Robert D. Ficalora , MD
EXA MINAION FOR MA
OVERVIEW
Since ����, more than �,��� individuals per year have taken Almost all o the questions are clinical and based on corthe American Board o Internal Medicine (ABIM) initial rect diagnosis and management. Because there is no penalty certi�cation examination, and between �,��� and �,��� indi- or guessing, candidates should answer every question. Most viduals per year have taken the Maintenance o Certi�cation questions are based on clinical cases. Among these, ��% are (MOC) examination. Pass rates have ranged rom ��% to related to the outpatient and emergency department settings, ��%. Pass rates or �rst-time takers on both examinations and the remainder are related to the inpatient setting, includexceed those o repeat takers. Tere is no doubt that careul ing the critical care unit and nursing home. Increasing emphaand serious preparation or the examination is valuable and sis is placed on patient saety and evidence-based quality o necessary. Although some individuals can take and pass the care. Selecting the correct answer to these questions requires examination with minimal preparation, most takers need integration o inormation provided rom several sources (eg, rigorous preparation. In recent years, board certi�cation has history, physical examination, laboratory test results, and assumed greater importance in the minds o patients. In a consultations), prioritization o alternatives, or use o clinical ���� Gallup poll o �,��� US adults aged �� years or older, judgment. Up to one-third o questions are experimental and ��% wanted their physicians to be board-certi�ed, ��% included to test question quality. Tey are not scored and canthought that the recerti�cation process was very important, not be identi�ed during the examination. Patient management and ��% would choose a new internist i their physicians’ with a cost-effective, evidence-based approach is stressed. Very ew questions require simple recall o medical acts. Tere are board certi�cation had expired. no intentional trick questions. EXAMINAION: BASIC INFORMAION
•
Te ABIM website (www.abim.org) has a wealth o inormation or test takers. No one should approach the examination without reading the ABIM Inormation and Statistics (http://www.abim.org/exam/prepare.aspx) and the ABIM Certi�cation and Recerti�cation Exam Guide (http://www. abim.org/exam/deault.aspx). Te “Exam Day: What to Expect” section, http://www.abim.org/exam/exam-day.aspx, has up-to-date inormation about changes to and navigation o the ABIM approach to computer-based testing, such as the ollowing:
E X A M I N A I O N C O N E N
Te questions in the examination cover a broad area o internal medicine. Tey are divided into primary and cross-content groups (http://www.abim.org/pd/blueprint/im_cert.pd ). Each session (� or initial certi�cation and � or maintenance o certi�cation) contains �� multiple-choice questions. Te question may include a case history, a brie statement, a radiograph, a graph, or a photograph (such as a blood smear or Gram stain). Each question has � possible answers, and the candidates should identiy the single-best answer. More than � answer may appear correct or partially correct or a question. Sample questions are included in the ABIM tutorial, http:// www.abim.org/exam/prepare.aspx.
How do I answer questions? How do I change answers? How do I make notes? How do I mark questions or review? •
Te ABIM examination has a uniorm question approach that stresses clinical reasoning over simple recall.
Review the ABIM website materials to understand the testing approach.
COMPUER�BASED ESING
Candidates currently take a computer-based certi�cation examination that has been designed to provide a �exible, quiet, and proessional environment or examination. Te
a
Reprinted rom Ficalora RD, editor. Mayo Clinic internal medicine board re view. ��th ed. Rochester (MN): Mayo Clinic Scienti�c Press and New York (NY): Oxord University Press; c����.
�
computer-based test is administered by about ��� centers in the United States. Candidates schedule their examination date according to the updated instructions on the ABIM website, http://www.abim.org/exam/. Candidates are well advised to access the online tutorial at http://www.abim. org/exam/prepare.aspx. Tis tutorial allows the candidate to become amiliar with answering questions, changing answers, making notes electronically, accessing the table o normal laboratory values, and marking questions or review. •
Candidates are advised to amiliarize themselves with the computer-based testing ormat by accessing the online tutorial.
Te sel-evaluation modules evaluate perormance in clinical skills, preventive services, practice perormance, und o medical knowledge, and eedback rom patients and colleagues. Successully completed sel-evaluation modules are valid or �� years. Candidates may apply to begin the MOC process any time afer initial certi�cation. Te ABIM recommends that completion o the sel-evaluation modules be spread out over the certi�cation period. A candidate should complete � sel-assessment module every � to � years. Te ABIM encourages candidates to enroll within � years o certi�cation in order to have adequate time to complete the program. •
Candidates who passed the ABIM certi�cation examination in internal medicine in ���� and thereafer have a certi�cate that is valid or �� years.
•
Te MOC process is called continuous proessional development and consists o a �-step process.
M A I N E N A N C E O F C E R I F I C A I O N
Te diplomate certi�cates issued to candidates who have passed the ABIM examination in internal medicine since ���� are valid or �� years. Te total number o candidates who took the ABIM MOC examination or the �rst time in ���� was �,���. O these, ��% passed. ENHANCEMEN S O MOC PROGRAM
In January ����, the ABIM enhanced the MOC program to increase �exibility and assess perormance in clinical practice. Te � retained general components (credentialing, sel-evaluation, and secure examination) and the added sel-evaluation module each have a point value. Every candidate must complete a total o ��� points in sel-evaluation modules. Unlike the previous system, renewal o more than � certi�cate does not necessitate taking additional sel-evaluation modules (ie, the same number o points, ���, satis�es the requirement to sit or these examinations). Candidates must complete at least �� points in medical knowledge and at least �� points in practice perormance. Te remaining �� points may be obtained rom completion o modules developed by ABIM or other organizations that meet the ABIM standards. Tus, one could combine an ABIM knowledge module (�� points) and an ABIM practice improvement module (�� points) with the American College o Physicians Medical Knowledge Sel-assessment Program (MKSAP) (� modules, �� points), or one could combine an ABIM practice improvement module (�� points) with � annual-update ABIM knowledge modules (�� points) and the ABIM peer and patient eedback module (�� points). All points are valid or �� years. Further re�nements to this process are likely. Tus, candidates should check or updates on the ABIM website.
� � I P S F O R E F F E C I V E E X A M I N A I O N PREPARAION �. HAVE A STUDY PLA N
We all have busy lives. Successul candidates stress that the most valuable preparation strategies must include scheduling a time to study. Preparing in small, discrete pieces improves recall, acilitates review, and makes the overall task less onerous. Spending � or � hours a week, using various approaches such as directed reading, practice questions, and group review, is enough to stay ocused. Simply reading by itsel is usually a bad strategy. You may not be able to retain much o the ABIM material by reading without ocus. Start with a question, a problem to solve, or a patient scenario in mind. Tis approach to a study session will help you and your group understand what you are studying, the clinical context, pathophysiology, and management and the reasons or it. Keep asking yoursel “why?” and “why not?” How can I study such a large mass o material? Plan a pace o no more than � major topics per hour. Survey the material. Consider the major subsections as potential questions. Review the material in each subsection careully to answer the question. Recite in your own words. Revise in your notes. ake notes! Even i you never look at them again, the act o synthesizing the inormation in writing will help you retain it.
•
MOC is a multistep process in addition to the examination.
•
o study, use active learning approaches to maximize efficiency.
•
Always check the ABIM website or inormation and updates.
•
Simply reading, no matter how much, is generally an ineffective preparation strategy.
�
•
M A YO C L I N I C I N E R N A L M E D I C I N E B O A R D R E V I EW : Q U E S I ON S A N D A N S W E R S
�. FORM A STUDY GROUP
I possible, orm a study group. You will be more likely to make and stay on a schedule i individuals eel a responsibility to the group’s progress. A group will boost everyone’s morale and give a common sense o purpose. A group size o only � to � candidates permits study o different textbooks, board review materials, and review articles in journals. Make sure that you have a committed, available group o study partners. Individuals who push ahead on their own and those who don’t keep their commitments can sabotage an otherwise productive group. Schedule regular meetings and assign individuals speci�c topics. Tis approach saves time, covers more topics per session, and allows everyone to retain more rom the discussion. ake turns acting as group moderator, to keep to the topic and schedule. Te moderator should be responsible, congratulating productive members and offering a riendly word to someone who might be slacking off. I everyone has a turn, no one person has to be the “bad guy.” Selected review articles on common and important topics, such as represented by the ABIM objectives, should be included in every session. Avoid indiscriminate reading o articles rom many journals. Remember that questions are tested or several examination cycles beore they are included in the examination. It is unlikely that new inormation or current controversies will be represented on your examination. Notes and other materials the candidates have gathered during residency training can be good sources o inormation. Finding the justi�cation or these “pearls” can cement one’s command o a particular topic. Tese clinical “pearls” gathered rom mentors will be o help in remembering certain important points. Always save some time each session to review questions and discuss the answers and their rationales. Don’t orget to discuss each o the options in detail. Tis will develop your thought process and sharpen your test-taking skills. •
Keep your study group small and stay ocused.
•
Make a schedule and read ahead o the discussion.
•
Discuss study material.
•
Do multiple-choice questions in groups.
•
Indiscriminate reading o articles rom many journals should be avoided.
•
Inormation in recent journals is unlikely to be included in the examination.
�. DETER MINE WHAT YOU NEED TO STUDY
For recent graduates attempting primary certi�cation, let your in-training examination subsection score results guide your study choices. In general, i your score in a given area was below the �fh decile, or �fieth percentile, you should consider that an area or intensive review and preparation. Use the section called “educational objectives,” which gives your perormance by content area, to guide your choice o preparation topics.
MOC candidates should use practice questions to guide your study choices. Do as many questions as you can, and monitor your perormance by the ABIM blueprint section, http://www.abim.org/pd/blueprint/im_cert.pd. Serious preparation or the examination actually starts at the beginning o residency training. In addition to daily reading and achieving subspecialty-based pro�ciency, most candidates require a minimum o � to � months o intense preparation or the examination. Cramming beore the examination, whether by yoursel or at a review course, is unlikely to be successul. Use a standard textbook o internal medicine. Ideally, you should use one good textbook and not jump rom one to another. Although online, just-in-time resources may be useul or act checking, they rarely give an inclusive, case-based review. Te most effective way to use the textbook is with patient-centered reading. Read the descriptions o the symptoms and signs careully because ofen they are part o the questions in the examination. able �.� provides several examples o the common descriptions o symptoms and signs that could be part o the examination. Rather than reading chapters at random, read the literature in a structured manner to assist in uture recall o acts. Tis book and similar books are excellent tools or brushing up on important board-relevant inormation several weeks to months beore the examination. Tey, however, cannot take the place o comprehensive textbooks o internal medicine. Tis book is designed as a study guide rather than a comprehensive textbook o medicine. Tereore, it should not be used as the sole source o medical inormation or the examination. •
Study �rst with a standard textbook o internal medicine.
•
Tis book is designed as a study guide and should not be used as the sole source o inormation or preparation or the examination.
•
Pay attention to the descriptions o signs and symptoms.
�. CREATE AND DEVOTE TIME
Board preparation should be part o your daily routine, like exercising, showering, or brushing your teeth. I you don’t regularly do some preparation, it will all off your routine, and your preparation just won’t happen. You can spend as little or as much time as you want on a particular activity. Ofen you can review amiliar topics in small discrete time periods (eg, beore or afer lunch). Less amiliar topics may require an hour in a quiet room, and this time may be best reserved or early in the morning or on a weekend. Keep in mind that the more time and energy you spend actively learning a topic, the better your command, and the less dependent you will be on rote memorization. Some people can study effectively while on a treadmill, on a train, or in a car. I you can do this, you can incorporate this into your studying routine. How much time it takes to perorm these tasks varies rom person to person and will improve as you solidiy your study habits. Regardless, every
�. PREPARING FOR HE ABIM EXAMINAION
•
�
learning task takes time, and you must budget or that time. range rom � to �� with some estimated to have � to �. Pace Only you can decide how much time you want to spend in your preparation by subject: solitary study, in groups, or in summary objective review. You can make great plans, but lie and work aren’t predictable, Subjects with a large estimated number o questions are so you should build in some catch-up time or unexpected very likely to be there. Master them. distractions. Subjects with a medium to low estimated number o You also have to consider how much time it takes to questions will be there in some orm. Review them. organize your studying. Review o cardiology may go quickly, whereas glomerulopathies may take a pad and penSubjects with a very low estimated number o questions cil to �gure out. You may have to travel to study sessions may be tangential or avorite board zebras. Depending on or spend time looking or inormation to ensure your com your available study time, it may be worthwhile or you to mand o a given objective. Many candidates try to set aside consider them only or last-minute review. large blocks o time. With our busy lives, that may be laudable but impractical. Many shorter sessions not only allow Plot out your objectives review on a calendar, mixing more or study and catch-up but also can be worked in around and less complex objectives. Leave time or discussion, litstanding commitments more effectively than large blocks, erature and online searches, and ollow-up or problem items and thus a missed session won’t be a major setback or that rom previous study sessions. Always plan to cover new mate week. Te time you spend will come back to you when you rial and to periodically cycle back to previously reviewed diffi pass, and a ailure only means you have to devote the time cult or detailed inormation. Imagine cases that might go with all over again. the material at hand. Certiying or recertiying board examinations can be stressTe most effective way to manage your study time is to ul. Te sheer mass o inormation can be overwhelming to periodically assess your progress through practice tests and some. Te press o occupational and personal responsibilities test questions. It is impossible to overemphasize the impormakes �nding the time to study very diffi cult, so many opt to tance o this point. “Boardsmanship” is a real skill, and there take a review course. A review course should be the �nal inte- is no substitute or amiliarity with the orm and content o grating activity once you have completed your own primary the ABIM-type questions. Tereore, you are strongly encour preparation. o be successul you should go to the review aged to take at least a ew mock examinations and simulate course prepared and ready to “�ll in” objectives you may have the actual testing environment (ie, no breaks, snacks, music,
learning task takes time, and you must budget or that time. Only you can decide how much time you want to spend in solitary study, in groups, or in summary objective review. You can make great plans, but lie and work aren’t predictable, so you should build in some catch-up time or unexpected distractions. You also have to consider how much time it takes to organize your studying. Review o cardiology may go quickly, whereas glomerulopathies may take a pad and pencil to �gure out. You may have to travel to study sessions or spend time looking or inormation to ensure your command o a given objective. Many candidates try to set aside large blocks o time. With our busy lives, that may be laudable but impractical. Many shorter sessions not only allow or study and catch-up but also can be worked in around standing commitments more effectively than large blocks, and thus a missed session won’t be a major setback or that week. Te time you spend will come back to you when you pass, and a ailure only means you have to devote the time all over again. Certiying or recertiying board examinations can be stressul. Te sheer mass o inormation can be overwhelming to some. Te press o occupational and personal responsibilities makes �nding the time to study very diffi cult, so many opt to take a review course. A review course should be the �nal integrating activity once you have completed your own primary preparation. o be successul you should go to the review course prepared and ready to “�ll in” objectives you may have missed or to learn rom experts’ objectives that you couldn’t understand on your own. Don’t expect a course to substitute or primary preparation. Attending a highly ocused, no-�uff course that delivers the inormation in a concentrated, high-yield manner right beore the examination may seem like an easy way out, but it is unlikely to be the difference between success and ailure. •
Residents: Prepare or the boards during residency. You will not �nd that kind o quality time afer your residency. Once residency is over and you start ellowship or a job, you will not �nd time to study.
range rom � to �� with some estimated to have � to �. Pace your preparation by subject: Subjects with a large estimated number o questions are very likely to be there. Master them. Subjects with a medium to low estimated number o questions will be there in some orm. Review them. Subjects with a very low estimated number o questions may be tangential or avorite board zebras. Depending on your available study time, it may be worthwhile or you to consider them only or last-minute review. Plot out your objectives review on a calendar, mixing more and less complex objectives. Leave time or discussion, literature and online searches, and ollow-up or problem items rom previous study sessions. Always plan to cover new material and to periodically cycle back to previously reviewed difficult or detailed inormation. Imagine cases that might go with the material at hand. Te most effective way to manage your study time is to periodically assess your progress through practice tests and test questions. It is impossible to overemphasize the importance o this point. “Boardsmanship” is a real skill, and there is no substitute or amiliarity with the orm and content o the ABIM-type questions. Tereore, you are strongly encouraged to take at least a ew mock examinations and simulate the actual testing environment (ie, no breaks, snacks, music, phones, pagers). aking mock questions is an effective use o a board review or board questions book. Practice material at intervals during your long-term test preparation schedule. Your schedule should be the most intense in the � months beore the test. Afer that, ocus on review, consolidating key points, and resolving previously difficult problems. Once you or your group has a schedule, stick to it. Add sessions, but never delete any. Stick to your start and stop times. •
Schedule your progress and build in assessment sessions.
•
Make changes, but no deletions.
•
Plan to review material that you have chosen several times (minimum twice).
•
MOC: Schedule the time.
•
•
Schedule multiple short preparation sessions rather than ewer long ones.
Stay ocused throughout the months beore the examination.
•
Board review preparation must be at its peak by � months beore the examination. I you have not yet ormed a study group, now is your last chance.
•
Do not rely solely on a review course; they are not a substitute or primary preparation.
�. PREPARE A PLAN AND SCHEDULE IT
�. ANSWER QUESTIONS SIMILAR TO EXAMINATION CONTE NT
All ABIM objectives are not equal. Review the relative percentage o the contents o the examination and the number o questions per objective (http://www.abim.org/pd/blue print/im_cert.pd ). Note that some areas may have an estimated number o questions o �� to ��, whereas many will
Te purpose o standardized testing is to measure a candidate’s command o the material so that scores rom different test dates can be reliably compared with one another. Te results must correlate statistically with the results o all the test-takers who have answered the same questions. Persons
�. PREPARING FOR HE ABIM EXAMINAION
•
�
who construct board review materials and questions go to • Familiarize yoursel with the teaching principle and the great pains to build them or the same content and content testing objective, which may give you insight into the questions and the possible responses. level as on the actual board examination. Te good ones are validated and have been tested to make sure that they perorm in a reliable, predictable manner and that they adequately test the content they purport to test. Any questions prepared or �. D O N ’ T F O R G E T A B O U T I M A G E S other courses, local residency, ellowship rotations, or other venues that cover similar material likely won’t test the mate- Every image-based question will also include text or a clinical rial in a way that predicts your perormance on the board case or both. Don’t simply ocus on the image without reading examination. Read a board review book, go to a board review the text. Familiarize yoursel with the image and its details afer course, and always practice answering questions. When you you have read the case, and then read it again. Photographs practice answering questions, do it as you would during the o skin disorders, radiographs, electrocardiograms, and other examination; just reading the book and reading the answers images given in board questions are generally easy ways to likely will not prepare you or the actual test. For every ques- score points. Reading the ABIM question stem last helps tion, identiy in your own mind the concept being tested. put the pieces o the puzzle in place. Methodically review a Make sure you read all the wrong answers and make sure you radiograph as you would in a patient encounter. Immediately understand why they weren’t the best answer or the question ocusing on an obvious abnormality can distract you rom a more subtle �nding that may alert you to the correct answer. asked. Sometimes candidates try to prepare by studying mate- You may miss the pneumothorax as the cause o the dyspnea i rials that are harder than the real test, such as subspecialty you ocus only on the heart size and the small pleural effusion. board–level review courses and practice tests. Te idea is that Likewise or skin �ndings; use your clinical skills to interpret becoming amiliar with something harder will make the real the �nding. Is it �at or raised, erythematous or pigmented? thing easier by comparison. Preparing with something more challenging can be a good idea in some types o athletic or • Approach an image question as you would a patient. endurance preparation, but it is a bad idea or the ABIM examination. Because the objectives are speci�c and pub- • Methodically examine the image. lic, preparing by using objectives or another examination • Use the text and the stem to ocus your inquiry. may cause you to misinterpret or overinterpret what is being asked. Reliably �nding the easiest approach to a test question requires being aware that the test can’t require you to use a cer�. SPEND SOME TIME BECOMING EXPERT IN tain higher-level data set or decision tree. Because the harder “BOARDSMANSHIP” material is testing a different skill set, reviewing this may lead to incorrect answers. Some candidates ail the examination despite intense preparaAs silly as this piece o advice may seem, read the ques- tion and the clinical competence necessary to pass the examitions careully! Doing so can make a big difference in your nation. Failing usually happens because they don’t understand score. I you read questions hastily, there is a high likelihood or interpret the questions properly. Te ability to understand that you will misinterpret them. Some questions offer incor- the nuances o the question ormat is sometimes reerred to as rect choices that are designed to answer a common misinter- “boardsmanship.” Intelligent interpretation o the questions pretation o the actual question. Be particularly careul with is very important or candidates who are not well versed in answers that have more than one part. Only one part may be the ormat o multiple-choice questions. Answer the quescorrect. Other distraction techniques include � responses that tions whose answers you know �rst, making sure you underare similar except or a word or phrase. Watch or responses stand what is being asked to ensure that they are answered that contradict others; usually, both o these can be ruled out. correctly. It is easy to become overcon�dent with such ques What i you read a question and the traditional correct answer tions, and thus you may ail to read the questions or the isn’t an option? What i more than one answer could be cor- answer options careully. Make sure you never make mistakes rect? Ten select the best option available. Be very careul on easy questions. Read the �nal sentence (that appears just o responses that are the longest or the unique answer. Tey beore the multiple answers) several times to understand how are no more likely or unlikely to be correct despite prevailing an answer should be selected. Recheck the question ormat wisdom. beore selecting the correct answer. Read each answer option completely. • Don’t try to read the board review material rom cover to Occasionally, a response may be only partially correct. At cover. times, the traditionally correct answer is not listed. In these situations, select the best alternative listed. Watch or quali�ers • Te best way to prepare is to review and always practice such as next, immediately, or initially. Avoid answers that conanswering questions. tain absolute or very restrictive words such as always, never, or • o improve your understanding, read the explanation, must. Answer options that contain absolutes are likely incorand look up additional inormation related to each o the rect. ry to think o the correct answer to the question beore choices—both correct and incorrect. looking at the list o potential answers. Assume you have �
•
M A YO C L I N I C I N E R N A L M E D I C I N E B O A R D R E V I EW : Q U E S I ON S A N D A N S W E R S
been given all the necessary inormation to answer the question. I the answer you had ormulated is not among the list o answers provided, you may have interpreted the question incorrectly. When a patient’s case is presented, think o the diagnosis beore looking at the list o answers. I you do not know the answer to a question, very ofen you are able to rule out one or several answer options. Determine whether your diagnosis is supported by any o the answers. I you can eliminate any answers as clearly wrong, you will improve your odds at guessing. Occasionally, you can use inormation presented in one question to help you answer other, diffi cult questions. Many questions are on the test or trial or validation purposes and are not scored. I a question seems to you to be a bad or conusing question, it may be in this categor y. It is best not to spend an inordinate amount o time trying to second guess this type o question. Come back to it afer you have �nished, i you still have time. •
When reading long multiple-choice cases: First read the actual “lead line” o the question
•
Once you understand what the question is asking: Stay ocused and look or clues in the long stem o the question.
•
As you read through the questions: Note the key acts and abnormal �ndings Skip questions about which you have no idea, and come back afer a complete �rst pass
�. USE YOUR REFLEXES
Associations, causes, complications, and other relationships between a phenomenon or disease and clinical eatures are important to remember and recognize. Each subspecialty has many common connections, and candidates or the ABIM and other examinations may want to prepare lists like this or different areas. For example, a case that presents a patient with health care–associated pneumonia should immediately bring to mind antipseudomonal antibiotics, not antibiotics traditionally used or community-acquired pneumonia. Combined knee and hip pain should have you considering a gait abnormality rather than abnormality in � joints simultaneously. Use the basic und o knowledge accumulated rom clinical experience and reading to solve the questions. Approaching the questions as real-lie encounters with patients is ar better than trying to second-guess the examiners or trying to analyze whether the question is tricky. As indicated above, the questions are never tricky, and there is no reason or the ABIM to trick the candidates into choosing wrong answers. Use examination techniques to your advantage. Look or target populations in questions. Start with a basic premise in mind, then modiy it as the inormation warrants. Examples are as ollows:
For young patients, aim or aggressive management. For elderly patients, aim or less aggressive alternatives, especially in those with multisystem disease. Beware o adverse medication effects and polypharmacy. For asymptomatic healthy patients, do nothing and observe. •
Use your existing und o knowledge o internal medicine and your previous clinical experience.
•
Approach each question as a real-lie patient encounter.
•
Tere are no trick questions.
� �. P L A N F O R T H E D A Y O F T H E E X A M I N AT I O N
You should have adequate time to read and answer all the questions; thereore, there is no need to rush or become anxious. Watch the time to ensure that you are at least halway through the examination when hal o the time has elapsed. Start by answering the �rst question and continue sequentially. Almost all o the questions ollow a case-presentation ormat. At times, subsequent questions will give you inormation that may help you answer a previous question. Do not be alarmed by lengthy questions; look or the question’s salient points. When aced with a conusing question, do not become distracted by that question. Mark it so you can �nd it later, then go to the next question and come back to the unanswered ones at the end. Extremely lengthy stem statements or case presentations are intended to test the candidate’s ability to separate the essential rom the unnecessary or unimportant inormation. You may want to highlight important inormation presented in the question in order to review this inormation afer reading the entire question and the answer options. Tere is no penalty or guessing, so you should never leave an answer blank. Every time you can eliminate just one choice you increase your chance o choosing a correct answer by ��%, so it’s best to guess among the remaining choices. I you truly have no idea about any o the choices, the “B” answer has been statistically more likely to be correct. It is better to choose “B” i you truly don’t know the answer. •
Look or the salient points in each question.
•
I a question is conusing, mark it to �nd it and come back to the unanswered questions at the end.
•
I you must guess, choose “B”; statistically, it is more likely to be correct.
It’s really not productive to discuss the questions or answers afer the examination with other candidates. Such discussions usually cause more consternation, although some candidates may derive a alse sense o having perormed well on the examination. In any case, the candidates are bound by their oath to the ABIM not to discuss or disseminate the questions. Do not study between examination sessions. o minimize stress, stick
�. PREPARING FOR HE ABIM EXAMINAION
•
�
to your daily routine; don’t start or stop exercising or using caffeine, and don’t skip meals or load up on carbohydrates. Be as rested and rereshed as you can be. Forget about your electronic devices such as pagers and cell phones. •
Don’t study the day beore the examination or between the examination sessions.
•
Discussing the examination questions with others raises anxiety and can adversely affect your perormance in the next session.
•
Maintain your normal routine.
�
•
SUMMARY
Preparation or the ABIM examination requires a serious and organized approach. Devote adequate time. Familiarize yoursel with the examination ormat and objectives. Use commonsense test-taking strategies, including practice tests and question analysis. reat the examination day as you would or any competitive event by preparing physically.
M A YO C L I N I C I N E R N A L M E D I C I N E B O A R D R E V I EW : Q U E S I ON S A N D A N S W E R S
�. CARDIOLOGY QUESTIONS AND ANSWERS
QUESIONS
�. A ��-year-old woman comes to your office with irregular palpitations, shortness o breath, and atigue. She has type � diabetes mellitus and hypertension, both o which she has kept under excellent control since she had a minor stroke at age ��. She has no other known medical problems o note. Examination �ndings are normal except that she has a sof, barely audible right carotid bruit. Te electrocardiogram (ECG) rom a week ago is shown (Figure �.Q�). A transthoracic echocardiogram shows normal lef ventricular unction and moderate lef atrial enlargement. You order another ECG today since her regular heart rate con�rms normal sinus rhythm. What anticoagulation therapy should you recommend or this patient at this time?
Multiple Choice (choose the best answer) ARRHYTHMIAS AND CLINICAL SYNDROMES
�. You are asked to see a ��-year-old man who under went a routine electrocardiographic (ECG) examination as part o a preemployment physical examination or a large manuacturing company. Other than having a airly sedentary liestyle, he is healthy and reports no symptoms. Cardiac examination �ndings are normal. You view the ��-lead ECG shown in Figure �.Q�. Which o the ollowing would be the best management strategy or this patient at this time? a. b. c. d. e.
a. Wararin (goal international normalized ratio, �.�–�.�) b. Aspirin ��� mg c. Catheter ablation o this arrhythmia to eliminate stroke risk d. Aspirin ��� mg and clopidogrel �� mg e. No anticoagulation since she is now in normal sinus rhythm and at low risk
Perorm an electrophysiology study and catheter ablation. Observe. Start therapy with metoprolol �� mg twice daily. Start therapy with procainamide ��� mg twice daily. Start therapy with digoxin �.��� mg daily.
I
aVR
V 1
V 4
II
aVL
V 2
V 5
III
aVF
V 3
V 6
II Figure �.Q�
�
I
aVR
V 1
V 4
II
aVL
V 2
V 5
III
aVF
V 3
V 6
II
V 1 V 5 Figure �.Q�
�. An ��-year-old man has had � separate syncopal episodes that occurred suddenly and without warning. On the �rst occasion, he was talking to his daughter on the phone—then he was lying on the �oor. Recovery was immediate, but he injured his lef hand. On the second occasion, he was in the kitchen making a sandwich. His daughter was present and was able to break his all. She noted that both arms made jerking movements, which resolved when he was placed into a recovery position on his lef side. On both occasions, a ull recovery occurred within a ew minutes without sequelae.
On examination, the patient was ully orientated and appeared younger than his age. His heart rate and blood pressure were satisactory, but his systolic pressure decreased by �� mm Hg on standing. Bilateral carotid bruits and a harsh �/� ejection murmur were present. Te murmur was loudest at the lef sternal border, with some radiation into the neck. Reversed splitting o the second heart sound (separation during expiration rather than inspiration) was also present, but no diastolic murmur was heard. Tere was no evidence o jugular venous distention, and his lungs were clear,
I
aVR
V 1
V 4
II
aVL
V 2
V 5
III
aVF
V 3
V 6
V 1
II V 5 Figure �.Q�
��
•
M A YO C L I N I C I N E R N A L M E D I C I N E B O A R D R E V I EW : Q U E S I O N S A N D A N S W E R S
a. b. c. d.
but a small amount o peripheral edema was present. Gait was normal and there were no neurologic signs o note. Te electrocardiogram is shown in Figure �.Q�. Tis patient’s presentation is most consistent with which o the ollowing? a. b. c. d. e.
Cardiogenic syncope A possible seizure disorder Stroke or transient ischemic attack Neurocardiogenic syncope Orthostatic response
�. A ��-year-old woman with nonischemic dilated cardiomyopathy and exertional dyspnea (New York Heart Association [NYHA] unctional class III) and �-pillow orthopnea underwent placement o an implantable cardioverter-de�brillator � years ago or sudden cardiac arrest prophylaxis. Now she returns and asks whether cardiac resynchronization therapy may be appropriate. Her current medications are lisinopril � mg daily, carvedilol �.�� mg twice daily, spironolactone �� mg daily, and urosemide �� mg daily. Physical examination and diagnostic study �ndings were as ollows: heart rate �� beats per minute, blood pressure ���/�� mm Hg, jugular venous pressure �� cm water, lungs clear, lateral displacement o the cardiac impulse, audible third heart sound, and no murmurs; electrocardiography showed lef bundle branch block (LBBB) with QRS duration o ��� ms, and transthoracic echocardiography showed lef ventricular ejection raction (LVEF) o ��%. What is the most appropriate management or this patient at this point? a. Tere is no indication or cardiac resynchronization therapy (CR)—the symptoms are not severe enough. b. Tere is no indication or CR—the QRS complex is not wide enough. c. Upgrade to CR now. d. Up-titrate the dosages o lisinopril and carvedilol.
�. A ��-year-old woman who received a diagnosis o peri partum cardiomyopathy (New York Heart Association class II) afer the birth o her second child presents with symptomatic paroxysmal atrial �brillation (AF). ypically, she experiences approximately � episodes o AF per year, each lasting � to � hours. During episodes, she notes palpitations, shortness o breath, and atigue, with heart rates around �� beats per minute, but she is able to continue working in her home. Although she has generally done well rom a heart ailure (HF) standpoint, she was hospitalized � weeks ago with acute decompensated HF. Current medications include carvedilol �� mg twice daily, lisinopril �� mg daily, urosemide �� mg daily, and digoxin �.��� mg daily. Physical examination and diagnostic study �ndings were as ollows: heart rate �� beats per minute (regular rhythm), blood pressure ���/�� mm Hg, jugular pressure not elevated, and lungs clear; electrocardiography showed normal sinus rhythm and lef bundle branch block. Which o the ollowing would you recommend at this time?
Add sotalol. Add dronedarone. Reer or catheter ablation. Continue the current regimen. C A R D I A C P H Y S I C A L E X A M I N AT I O N
�. A ��-year-old woman is reerred or a pre-insurance examination. She has no complaints. She is active and runs with her large dog about � hour daily (�–� miles). She has no exercise limitations and was hiking at an elevation o �,��� m in the Rockies last summer. On examination, her pulse rate is �� beats per minute, blood pressure is ���/�� mm Hg, and she is thin. On cardiac examination, there is a normal �rst heart sound and normal splitting; an early low-pitched diastolic sound is consistent with a third heart sound (S �). Tere is a grade � to � midsystolic murmur at the lef upper sternal border. Te murmur disappears with the Valsalva maneuver. Carotid upstrokes are normal, and the jugular venous pressure is normal. No systolic clicks can be appreciated at rest or with the squat-to-stand maneuver. Te electrocardiogram shows sinus bradycardia (heart rate �� beats per minute) but is otherwise normal. What urther cardiac testing needs to be done? a. Cardiac catheterization to measure a gradient across the mitral valve b. Stress echocardiography c. readmill ECG d. No additional testing e. Holter monitoring
�.
Which o the ollowing statements about shunts in the atrial septum is true? a. Te auscultatory �ndings associated with atrial septal deect (ASD) are paradoxical splitting o the second heart sound (S�) and a diastolic �ow rumble heard midsternum. b. Patent oramen ovale (PFO) is uncommon in healthy adults and is ound in approximately �.�% o adults. c. A �xed splitting o the S� and a right ventricular (RV) lif are ound in ASD with signi�cant shunting. d. Secundum ASD and PFO require subacute bacterial endocarditis prophylaxis.
�. A ��-year-old woman comes to your office or evaluation. She appears healthy and is not limited by physical activity. She says that she has occasional chest pains that occur at rest, ofen with a change in position. Her blood pressure is ���/�� mm Hg, and her heart rate is �� beats per minute and regular. Auscultatory �ndings are a midsystolic click ollowed by a �/� apical systolic murmur that radiates to her axilla. Which o the ollowing dynamic physical examination �ndings would con�rm your suspicion o mitral valve prolapse? a. b. c. d.
Tere is no change in the click with squatting or standing. A passive leg raise makes the click-murmur occur earlier. A squat maneuver makes the click-murmur occur later. Te mitral regurgitant murmur does not change with positional maneuvers.
�. CARDIOLOGY QUESIONS AND ANSWERS
•
��
�. A ��-year-old man, who is slender and athletic, comes to your office because o an episode o syncope immediately afer playing basketball. He exercises diligently but says that he has experienced mild dyspnea on exertion over the past � months. On physical examination, his blood pressure is ���/�� mm Hg, his carotid pulse is bounding, and there is an early systolic click immediately ollowed by a late peaking �/� systolic murmur heard best at the right second intercostal space and radiating to the carotids. Tere is no dynamic increase in murmur intensity with the Valsalva maneuver and no change in the timing o the click with squatting. Te apical impulse is sustained. A radial emoral delay is suspected on examination. What should be the next step in the evaluation or management o this patient? a. Coronary angiogram b. Use o a ��-hour Holter monitor c. Computed tomographic (C) scan o the chest d. Reassurance with recommendations or warm-up and cooldown routines with exertion e. Use o a ��-hour blood pressure monitor
��. A ��-year-old patient with a history o coronary artery bypass graf surgery � year ago presents with atigue, dyspnea, and progressive lower extremity edema. Examination reveals a blood pressure o ���/�� mm Hg and a pulse o �� beats per minute. Te lungs are clear. Te heart is quiet, with normal �rst and second heart sounds and no murmurs. Te jugular venous pressure (JVP) is elevated at midneck (approximately �� cm water). Tere is an increase in the JVP with inspiration with a rapid descent. What does this combination o �ndings strongly suggest? a. b. c. d.
Heart ailure due to biventricular dysunction Superior vena cava syndrome Constrictive pericarditis Failure o the bypass grafs and recurrent coronary artery disease
��. A patient comes to you or evaluation o peripheral edema and palpitations. On examination, a loud grade �/� holosystolic murmur is heard at the lef lower sternal border, and an intermittent third heart sound (S �) is heard that seems to be present only with inspiration. A large v wave is noted in the jugular venous pressure (JVP). What is your diagnosis? a. b. c. d. e.
Superior vena cava syndrome Atrial �brillation Constrictive pericarditis Signi�cant pulmonary stenosis Signi�cant tricuspid regurgitation
��. Which o the ollowing is true about the auscultatory �nding o a ourth heart sound (S �) on physical examination? a. b. c. d.
Ofen associated with hypertension Normal in young children Normal in athletes A hallmark o the onset o atrial �brillation ��
•
C O N G E S T I V E H E A R T F A I LU R E
��. A ��-year-old man presents with progressively increasing dyspnea and edema o the lower extremities. He denies having chest discomort or palpitations. He experiences dyspnea on minimal exertion, orthopnea, and occasional episodes o paroxysmal nocturnal dyspnea. He had a myocardial inarction �� years ago and underwent primary percutaneous coronary intervention then. His residual lef ventricular ejection raction (LVEF) afer that event was ��% by echocardiography. He has adhered to his medical regimen. He has been hospitalized or decompensated heart ailure repeatedly over the past � years. He received an implantable cardioverter-de�brillator with cardiac resynchronization therapy � years beore the current presentation. He has a history o hypertension and hyperlipidemia. His medical regimen consists o enalapril �� mg twice daily, carvedilol �.�� mg twice daily, simvastatin �� mg daily, aspirin �� mg daily, spironolactone �� mg daily, and urosemide �� mg daily. Examination �ndings were as ollows: aebrile, heart rate �� beats per minute, blood pressure ��/�� mm Hg, signi�cantly elevated jugular venous pressures with prominent v waves, lungs clear, a �/� pansystolic murmur at the apex, a third heart sound (S �), and pitting edema (�+). Laboratory test results were as ollows: hemoglobin �� g/dL, sodium ��� mEq/L, potassium �.� mEq/L, serum urea nitrogen �� mg/dL, and creatinine �.� mg/ dL. Te electrocardiogram showed a paced rhythm. A recent exercise stress test done with sestamibi imaging showed a large �xed deect in the anteroseptal wall with no evidence o reversible ischemia and an LVEF o ��%. Te patient was admitted to the hospital or intravenous diuretic therapy. Te edema improved signi�cantly, but he continued to experience dyspnea on minimal exertion. His creatinine increased to �.� mg/dL. What should be the next step in this patient’s management? a. b. c. d. e.
Increase the carvedilol dosage to ��.� mg twice daily. Optimize cardiac resynchronization therapy. Add metolazone � mg daily. Perorm a positron emission tomographic viability scan. Reer or implantation o a lef ventricular assist device (LVAD).
��. Which statement is false regarding serum brain natriuretic peptide (BNP) levels? a. Serum BNP levels may be in the reerence range in patients with advanced heart ailure. b. A persistently elevated serum BNP level despite optimal medical therapy is a marker o poor prognosis in heart ailure. c. Optimization o medical therapy based on serum BNP levels decreases hospitalizations in heart ailure. d. Serum N -terminal pro-BNP has a longer hal-lie than serum BNP. e. Te serum BNP level is requently elevated in idiopathic constrictive pericarditis.
MAYO CLINIC INERNAL MEDICIN E BOARD REVIEW: QUESIONS AND ANSWER S
��. A ��-year-old woman presents with dyspnea on exertion and tingling numbness in both lower extremities. On physical examination, she is aebrile, her heart rate is �� beats per minute, and her blood pressure is ���/�� mm Hg in a sitting position and ��/�� mm Hg while standing. She has jugular venous distention accentuated with inspiration and prominent x and y descents. Bibasilar crackles are present posteriorly. A ourth heart sound is present. She has mild bilateral pitting edema o the lower extremities. Sensation is diminished in both eet, but re�exes and motor power are preserved. Laboratory test results were as ollows: hemoglobin �� g/dL, sodium ��� mEq/L, potassium �.� mEq/L, serum urea nitrogen �� mg/dL, and creatinine �.� mg/dL. Te urinanalysis is remarkable or mild proteinuria. Te electrocardiogram shows sinus rhythm, low voltage, and an old anteroseptal inarct pattern. An echocardiogram shows a lef ventricular ejection raction o ��%, severely thickened lef ventricular walls with a speckled appearance, and mild mitral regurgitation. What is the best treatment option or this patient? a. b. c. d. e.
Stem cell transplant Angiotensin-converting enzyme inhibitors Combined heart and liver transplant Diuretic therapy Hospice care
��. A ��-year-old man with a history o diabetes mellitus and hypertension presents to the hospital with new orthopnea and edema. His blood pressure is ��/�� mm Hg, and his heart rate is �� beats per minute. He has clear lungs, a displaced cardiac apex with a third heart sound but no cardiac murmurs, and no lower extremity edema. Laboratory test results were as ollows: hemoglobin �� g/dL, sodium ��� mEq/L, potassium �.� mEq/L, and serum creatinine �.� mg/dL. Chest radiography shows pulmonary congestion with cardiomegaly. Te electrocardiogram shows sinus rhythm, nonspeci�c S- changes, and requent premature ventricular complexes (PVCs). Echocardiography shows a lef ventricular ejection raction o ��% and a dilated lef ventricle. What should be the next step in evaluating this patient? a. b. c. d. e.
Endomyocardial biopsy Stress testing Coronary angiography Holter monitoring Sleep study
��. A ��-year-old Arican American woman with pulmonary edema is transerred to your hospital. Her heart rate is ��� beats per minute and regular; her blood pressure is ���/�� mm Hg. Her jugular venous pressure is normal. Her lungs are clear. She has a lef ventricular lif and a �/� pansystolic murmur at the apex. A summation gallop is present. She has no hepatosplenomegaly, ascites, or edema. Chest radiography is consistent with pulmonary congestion and cardiomegaly. Te electrocardiogram shows sinus rhythm with a lef bundle branch block. Te echocardiogram shows global
hypokinesis, ejection raction ��%, and severe mitral regurgitation. Coronary angiography is normal. Which o the ollowing treatment options would not prolong this patient’s survival? a. b. d. e. e.
β-Blockers Biventricular pacing Isosorbide dinitrate in combination with hydralazine Spironolactone Digoxin
��. A ��-year-old man presents with syncope. He has a amily history o sudden cardiac death. On physical examination, his blood pressure is ���/�� mm Hg and his heart rate is �� beats per minute. Jugular venous pressures show a prominent a wave. Carotid upstroke is rapid; lef ventricular impulse is sustained and dis placed laterally. He has a prominent ourth heart sound and a �/� systolic ejection murmur at the lef sternal border radiating to the right second intercostal space and apex. He has a �/� pansystolic murmur at the apex. Te murmur at the lef sternal border is accentuated with the squat-to-stand maneuver. His lungs are clear, he has no hepatomegaly, and his extremities are not edematous. Which o the ollowing statements regarding this patient’s disease state is false? a. A majority o patients are asymptomatic. b. Te pathophysiology o the disease is characterized by abnormalities in myocardial energetics. c. Unexplained syncope is an indication or an implantable cardioverter-de�brillator (ICD). d. Patients with heart ailure symptoms should be offered surgery. e. All �rst-degree relatives should undergo screening or this disorder.
��. A ��-year-old woman presents with progressive dyspnea on exertion. She denies having chest pain. She has a long-standing history o hypertension and chronic atrial �brillation. Her medications include wararin � mg daily, enalapril � mg daily, and digoxin �.��� mg daily. On physical examination, her blood pressure is ���/�� mm Hg and her heart rate is �� beats per minute and irregular. Jugular venous distention is present. A sof �/� pansystolic murmur was heard at the lef sternal border. Her lungs were clear. Her extremities had pitting edema (�+). Te electrocardiogram showed atrial �brillation with lef ventricular hypertrophy. Chest radiography showed cardiomegaly with mild pulmonary venous hypertension. Echocardiography showed severe biatrial enlargement and lef ventricular hypertrophy with normal lef ventricular cavity dimensions and an ejection raction o ��%. Which o the ollowing would not be included in the differential diagnosis or this patient’s symptoms? a. b. c. d. e.
Constrictive pericarditis Arteriovenous �stula Amyloidosis Coronary artery disease Hypertensive heart disease
�. CARDIOLOGY QUESIONS AND ANSWERS
•
��
CORONARY ARTERY DISEASE A ND MYOCARDIAL INFARCTION
��. A ��-year-old woman presents to a community hospital with a �-day history o intermittent chest pressure at rest. Te pain is persistent on the day o admission. She has a history o chronic stable angina treated with aspirin ��� mg daily, metoprolol �� mg twice daily, and isosorbide mononitrate �� mg daily. Her past medical history is signi�cant or hypertension and diabetes mellitus. Additional medications include hydrochlorothiazide �� mg daily. On physical examination, her pulse is regular at �� beats per minute, and her blood pressure is ���/�� mm Hg. Her jugular venous pressure and all peripheral pulses are normal. On auscultation, the heart sounds are normal and the lungs are clear. Tere is trace peripheral edema. Te electrocardiogram (ECG) shows a �- to �-mm S-segment depression in the inerior leads and in leads III, aVF, and V �. Te tro ponin level is elevated (�.�� ng/mL). Afer treatment with intravenous heparin and nitroglycerin, the patient becomes asymptomatic with complete resolution o the ECG abnormalities. Later that day, recurrent, transient chest pressure and S-segment depression develop. In addition to arranging transer to a hospital with acilities or coronary angiography, which o the ollowing is the most appropriate next step? a. b. c. d. e.
Increase the dosage o metoprolol to �� mg twice daily. Perorm exercise sestamibi testing. Administer morphine intravenously. Administer reteplase �� units intravenously over � minutes. Initiate an inusion o epti�batide, a glycoprotein IIb/IIIa inhibitor.
��. A ��-year-old woman presents to the emergency department with dyspnea and retrosternal chest pain that developed � hours ago, soon afer she had an intense argument with her daughter. Her past medical history is remarkable or hypertension and diabetes mellitus. Te chest discomort is not relieved by sublingual nitroglycerin. Her heart rate is ��� beats per minute, and her blood pressure is ���/�� mm Hg. Her jugular venous pressure, carotid pulse, and peripheral pulses are normal. On auscultation, heart sounds are normal and the lungs are clear. Te cardiac troponin level is �.�� ng/ mL (reerence range ≤�.�� ng/mL). Te ��-lead electrocardiogram shows a �.�- to �-mm S-segment elevation in precordial leads V � through V�. An emergency coronary angiogram shows mild coronary atherosclerosis. A lef ventriculogram shows severe hypokinesis o the apical and midsegments o the heart with normal unction at the base. Which o the ollowing is the most likely diagnosis? a. b. c. d. e.
Myocarditis Apical ballooning syndrome (takotsubo cardiomyopathy) Acute coronary syndrome Dilated cardiomyopathy Pericarditis ��
•
��. A ��-year-old woman presents with chest pain on exertion. Past medical history is remarkable or hyperlipidemia, hypertension, abdominal aortic aneurysm repair, and a stroke rom which she has made a good neurologic recovery. Current treatment includes aspirin �� mg daily, lisinopril �� mg daily, and atorvastatin �� mg daily. On physical examination, she weighs �� kg, her pulse is regular at �� beats per minute, and her blood pressure is ���/�� mm Hg. Jugular venous pressure is normal, and all peripheral pulses are normal. On auscultation, heart sounds are normal and the lungs are clear. Tere is no peripheral edema. A stress test is perormed: she exercises or � minutes in a Bruce protocol, and typical symptoms develop with �-mm S-segment depression in multiple leads. A coronary angiogram shows a ��% stenosis in the midsection o the lef anterior descending coronary artery. A drug-eluting stent is deployed without complications, and the patient is discharged with clopidogrel �� mg daily in addition to her usual medications. Five days later, she calls and says that an intense pruritic rash has developed on her trunk and limbs. She believes that it is due to the clopidogrel. Which o the ollowing is an absolute contraindication or the use o prasugrel as an alternative to clopidogrel in this patient? a. b. c. d. e.
Female sex Age older than �� years Weight less than �� kg Percutaneous coronary intervention History o stroke
��. A ��-year-old woman wants your advice about a recent cardiology evaluation. Having had angina or � years with increasingly limiting symptoms, she was advised to have coronary angiography, which showed severe �-vessel disease. An echocardiogram showed that the lef ventricular ejection raction was ��%. Current medications include aspirin �� mg daily, simvastatin �� mg daily, atenolol �� mg daily, and long-acting isosorbide dinitrate �� mg daily. Physical examination �ndings include a pulse o �� beats per minute, blood pressure o ���/�� mm Hg, and normal jugular venous pressure and heart sounds. Tere are no murmurs, and the lungs are clear on auscultation. Which o the ollowing is the most appropriate recommendation or this patient to treat her symptoms and improve the prognosis? a. Increase the dosage o atenolol to �� mg daily. b. Add ranolazine to her treatment and reassess in � month. c. Coronary artery bypass graf (CABG) surgery is the optimal strategy. d. Percutaneous coronary intervention (PCI) is the optimal strategy. e. PCI and CABG are equivalent strategies.
��. A ��-year-old man returns or a clinic visit afer an anterior myocardial inarction � months ago. He was treated successully with primary percutanteous coronary intervention. He has made an uncomplicated recovery, completed cardiac rehabilitation, and returned to
MAYO CLINIC INERNAL MEDICIN E BOARD REVIEW: QUESIONS AND ANSWER S
work as a salesman. His risk actors or vascular disease include smoking and hypertension. His current medications are aspirin �� mg daily, metoprolol �� mg twice daily, clopidogrel �� mg daily, simvastatin �� mg daily, and lisinopril �� mg daily. On examination, his body mass index is ��. His pulse rate is �� beats per minute, his blood pressure is ���/�� mm Hg, no murmurs or gallops are noted on heart examination, and the lungs are clear. Complete blood cell count and serum chemistry results are all within the reerence ranges. Results o a asting lipid panel are as ollows: low-density lipo protein cholesterol (LDL-C) �� mg/dL, high-density lipoprotein cholesterol (HDL-C) �� mg/dL, and triglycerides ��� mg/dL. Fasting blood glucose is ��� mg/ dL. For secondary prevention, in addition to smoking cessation, what is the most important next step? a. Intensiy weight management and physical activity. b. Increase the dosage o simvastatin to �� mg daily. c. Measure the plasma high-sensitivity C-reactive protein (hs-CRP) level. d. Continue the cardiovascular rehabilitation program long-term. e. Measure the plasma lipoprotein (a) level.
��. A ��-year-old woman presents with progressive angina. Her past medical history is signi�cant or hyperlipidemia, hypertension, and diabetes mellitus. On physical examination, her heart rate is �� beats per minute and her blood pressure is ���/�� mm Hg. Te heart sounds are normal, and a ourth heart sound is present. Te lungs are clear on auscultation, and the peripheral pulses are normal. Current daily medications include aspirin ��� mg, atenolol �� mg, lisinopril �� mg, atorvastatin �� mg, insulin, and metormin. Laboratory test results are a normal complete blood cell count and creatinine �.� mg/dL. A stress test is markedly positive or ischemia. Te cardiologist has recommended proceeding with coronary angiography. In addition to discontinuing use o metormin, which o the ollowing steps would be most important beore angiography? a. Increase the dosage o atenolol to �� mg daily. b. Ensure that the patient is not dehydrated beore the procedure. c. Add slow-release isosorbide mononitrate �� mg daily. d. Decrease the dosage o aspirin to �� mg daily. e. Start clopidogrel therapy with �� mg daily.
��. A ��-year-old man presents with chest pain on exertion. He can walk approximately one-hal mile beore symptoms develop. He has a history o hypertension and hyperlipidemia. Current treatment includes lisinopril �� mg daily and atorvastatin �� mg daily. On physical examination, his pulse is regular at �� beats per minute and his blood pressure is ���/�� mm Hg; his jugular venous pressure is normal, and all peripheral pulses are normal. On auscultation, heart sounds are normal and the lungs are clear. Tere is no peripheral edema. During a stress test, he exercises or � minutes in a Bruce
protocol; toward the end o the test, typical symptoms and �-mm S-segment depression develop. A coronary angiogram shows a solitary ��% stenosis in the midsection o the right coronary artery. Te patient elects medical therapy and returns to your clinic to discuss whether he should have percutaneous coronary inter vention (PCI). What is the most appropriate advice on the merits o medical therapy compared with PCI or this patient? a. PCI will reduce the need or optimal risk actor management. b. PCI will reduce the risk o myocardial inarction. c. PCI will result in much less angina compared with medical therapy. d. Both medical therapy and PCI will result in similar survival. e. Te patient will eventually require PCI i medical therapy is the initial treatment strategy. VA S C U L A R M E D I C I N E
��. A ��-year-old man is evaluated or a �-year history o progressive right leg pain. He has a history o hypertension but not diabetes mellitus. His medications include hydrochlorothiazide and aspirin. On examination, his blood pressure is ���/�� mm Hg and his pulse is �� beats per minute and regular. Cardiac examination �ndings are normal. Vascular examination identi�es palpable but diminished pulses in the popliteal and posterior tibial arteries bilaterally with an absent dorsalis pedis pulse on the lef. Which o the ollowing eatures would be most useul to distinguish intermittent claudication rom pseudoclaudication as the primary cause o his symptoms? a. b. c. d.
History o nicotine addiction Symptoms brought on by exertion Sof bruit over the right common emoral artery Ankle-brachial index o �.�� on the right and �.�� on the lef e. Symptom relie with sitting only
��. A ��-year-old man presents to the emergency department with a �-hour history o back pain radiating to the anterior chest and neck. He describes the pain as “tearing” and severe. His pain came on suddenly while watching a ootball game on the television. His past medical history includes hypertension and ongoing tobacco use. On examination, he is uncomortable. His blood pressure is ���/��� mm Hg in the right arm and ���/�� mm Hg in the lef arm with a regular pulse o ��� beats per minute. His chest is clear on auscultation. On cardiac examination, there is a �/� holosystolic murmur at the apex and a sof ourth heart sound. Vascular examination identi�es a slight delay in the lef radial and ulnar pulses with a bruit over the lef subclavian artery. Femoral, popliteal, and posterior tibial pulses are symmetrical but diminished. On computed tomographic (C) imaging, a type B aortic dissection
�. CARDIOLOGY QUESIONS AND ANSWERS
•
��
extends to the aortic biurcation. Which o the ollowing is an indication or surgical intervention? a. b. c. d.
ype B aortic dissection Nonperused lef kidney identi�ed on the C scan Severe hypertension Discordant brachial blood pressures, indicating compromise o the lef subclavian artery by the dissection e. Presence o the murmur, indicating disruption o the aortic valve annulus by the dissection
��. A ��-year-old woman is evaluated or an abdominal aortic aneurysm (AAA) ound at a vascular screening air at her local supermarket. Her risk actors or atherosclerosis include hypertension and hyperlipidemia. She has not smoked in the past and has no history o diabetes mellitus. On examination, her blood pressure is ���/�� mm Hg in both arms with a regular pulse o �� beats per minute. Her lungs are clear. Te carotid upstrokes are normal. On abdominal examination, an enlarged pulsatile mass is superior to the umbilicus. Femoral, popliteal, and posterior tibial pulses are normal. Which o the ollowing �ndings should prompt surgical reerral or intervention? a. b. c. d. e.
A �.�-cm inrarenal AAA Annual growth rate o �.� cm Accompanying lef iliac artery aneurysm measuring �.� cm Father who died o a ruptured AAA ender aneurysm on examination
��. A ��-year-old man presents or evaluation in the emergency department with acute onset o dyspnea and severe cough. He has had no recent travel, trauma, or surgery. He notes that his lef leg has been painul or � days, and this morning he noted some swelling. He is dyspneic. His blood pressure is ��/�� mm Hg with a regular pulse o ��� beats per minute. His chest is clear. Cardiac examination identi�es mild jugular venous distention with a subtle right ventricular lif. His lef leg is slightly edematous. Computed tomographic angiogra phy o the chest identi�es bilateral pulmonary emboli. Duplex ultrasonography identi�es an extensive lef emoral-popliteal deep vein thrombosis (DV). Which o the ollowing �ndings should prompt inerior vena cava (IVC) �lter placement?
c. Right popliteal deep vein thrombosis, heterozygous actor V Leiden, and right �bular racture � days prior d. Symptomatic PE �� days afer a �-hour �ight e. Lef ovarian vein thrombosis afer vaginal hysterectomy
��. A ��-year-old right-handed woman is being evaluated or a transient ischemic attack (IA) that she experienced earlier this morning. While clearing the breakast dishes, she noted a �-minute episode o slurred speech and right hand and acial numbness. She has a history o hypertension but has been healthy otherwise with no additional diagnoses. On examination, her blood pressure is ���/�� mm Hg with a regular pulse o �� beats per minute. Her chest is clear on auscultation. Cardiac examination identi�es a normal jugular venous pulse and apical impulse. On auscultation, there is no murmur or gallop. A bruit is heard over both carotid arteries. Carotid upstrokes are normal. Te electrocardiogram shows normal sinus rhythm with normal inter vals and axes. Which o the ollowing statements is true or symptomatic carotid disease? a. Afer a IA in a patient with an ipsilateral internal carotid artery stenosis greater than ��%, the combined outcomes o stroke or death are improved with carotid endarterectomy surgery compared with medical management. b. Most IAs result rom a symptomatic ipsilateral carotid lesion. c. For an ipsilateral symptomatic carotid stenosis greater than ��%, surgery and medical therapy result in similar �-year stroke-ree survival rates. d. For the patient presented, carotid artery stenting with a distal protection device will provide superior short- and long-term stroke-ree survival compared with carotid endarterectomy. e. At � years, the rate o recurrent stenosis (>��%) is equal or carotid stenting and endarterectomy.
��. A ��-year-old man with diabetes mellitus, ongoing tobacco use, hypertension, and hyperlipidemia complains o �-block claudication in both legs. His symptoms improve with standing and are consistent rom day to day. He has no rest pain or ulceration. His symptoms have been stable or the past � years. His blood pressure is ���/�� mm Hg with a regular pulse o �� beats per minute. His chest is clear on auscultation. His carotid upstrokes are normal without bruit. a. Hemodynamic instability with coexistent DV and a large Cardiac examination identi�es a normal jugular venous pulmonary embolism (PE) pressure and apical impulse. Findings on auscultation b. Mobile thrombus in the lef emoral vein identi�ed on ultraare normal. He has normal pulses in the emoral and sonographic imaging popliteal arteries bilaterally without bruit. Te pedal c. Family history o PE pulses are not palpable. His extremities do not have d. History o PE � years ago ulcers, ischemic �ssures, dependent rubor, or elevation e. Retroperitoneal hemorrhage afer initiation o heparin pallor. Te ankle-brachial index is �.�� on the right and therapy �.�� on the lef. An angiogram perormed at his local ��. Which o the ollowing conditions should prompt promedical acility � year ago showed severe inrapopliteal longed secondary prophylaxis with wararin? arterial occlusive disease with diseased but patent proxa. Portal vein thrombosis � weeks afer colonic resection or imal arteries bilaterally. What is the most appropriate ulcerative colitis treatment regimen or this patient with intermittent b. Bilateral pulmonary embolism (PE) in a ��-year-old woman claudication? who uses oral contraception
��
•
M A YO C L I N I C I N E R N A L M E D I C I N E B O A R D R E V I EW : Q U E S I ON S A N D A N S W E R S
a. Computed tomographic angiography (CA) o the legs b. Magnetic resonance angiography (MRA) o the legs c. Risk actor modi�cation and the Canadian walking program d. Angioplasty and stenting e. Bypass surgery HYPERTENSION
��. A ��-year-old man with no complaints comes to you or a general medical examination. His body mass index is ��, his blood pressure is ���/�� mm Hg, and his pulse is �� beats per minute. Te rest o the examination �ndings are normal. He takes acetaminophen or occasional arthritis pain. He has no other signi�cant past medical or surgical history. What would be your next step in management? a. b. c. d. e.
Recheck blood pressure in � years. Recheck blood pressure in � year. Recheck blood pressure in � months. Recheck blood pressure in � months. Evaluate and treat blood pressure now.
��. A ��-year-old sexually active woman has a blood pressure o ���/�� mm Hg that has been con�rmed on several office visits and with sel-monitoring despite a �-month trial o liestyle modi�cations. Which o the ollowing drugs would be most appropriate or this patient? a. b. c. d. e.
Lisinopril �� mg once daily Doxazosin � mg once daily Losartan �� mg once daily Hydrochlorothiazide ��.� mg once daily Aliskiren ��� mg once daily
��. A ��-year-old woman is reerred or urther evaluation o an elevated office blood pressure (BP) o ���/�� mm Hg. Her riend, who is a nurse, has measured her BP several times at home, where it ranged rom ���/�� to ���/�� mm Hg. Te patient eats a high-salt diet and does not exercise. Her mother has hypertension. On examination, the patient’s BP is ���/�� mm Hg; examination �ndings are otherwise normal. Routine laboratory test results are within the reerence ranges. What is the most appropriate next step in evaluating or treating this patient? a. Discuss liestyle modi�cations and begin therapy with atenolol. b. Begin therapy with a low dose o hydrochlorothiazide. c. Obtain a duplex ultrasonogram o the renal arteries. d. Begin therapy with a low dose o hydrochlorothiazide in combination with lisinopril. e. Obtain a ��-hour ambulatory BP recording.
��. A ��-year-old man has an elevated blood pressure (BP) (���/�� mm Hg) that has been con�rmed on several office visits. He is a smoker with stable, mild claudication. He has hyperlipidemia, which is controlled with diet and statin therapy. Liestyle modi�cation and drug therapy are initiated or BP control. What is the recommended BP goal or this patient? a. Less than ���/�� mm Hg measured in the office b. Less than ���/�� mm Hg measured at home c. Less than ���/�� mm Hg measured in the office and at home d. Less than ���/�� mm Hg measured in the offi ce e. Less than ���/�� mm Hg measured at home
��. A ��-year-old woman has long-standing hypertension. wo days afer total hip arthroplasty, her blood pressure is ���/��� mm Hg, which is con�rmed on a subsequent measurement. She reports having substernal chest pressure and mild dyspnea. An electrocardiogram shows S-segment depression in the inerior leads. What is the most appropriate parenteral antihypertensive drug to consider or this patient? a. Sodium nitroprusside b. Hydralazine c. Labetalol d. Nitroglycerin e. Nicardipine
��. A ��-year-old woman has had episodes o headache associated with diaphoresis and nausea. Tese episodes begin suddenly and vary in duration rom �� to �� minutes. Recently, a blood pressure o ���/��� mm Hg was measured during an episode. Her amily history is signi�cant or pheochromocytoma in her mother. On examination, her blood pressure is ���/�� mm Hg. Results o routine laboratory tests were normal. What is the most appropriate next step in her evaluation? a. b. c. d. e.
Measure plasma ree metanephrines. Begin drug treatment with metoprolol. Obtain a computed tomogram o the abdomen. Obtain a duplex ultrasonogram o the renal arteries. Measure plasma and urine catecholamines.
��. A ��-year-old man with a history o metabolically active calcium oxalate nephrolithiasis has hypertension that was recently diagnosed and is not controlled with liestyle modi�cations. Which o the ollowing drugs would be the most appropriate initial choice or treating his hypertension? a. Lisinopril b. Furosemide c. Losartan d. Atenolol e. Chlorthalidone
�. CARDIOLOGY QUESIONS AND ANSWERS
•
��
ANSWERS
�. Answer b.
be permanent pacemaker implantation. Te other causes o syncope are most unlikely in this patient.
Te ECG is typical or Wolff-Parkinson-White syndrome. Since the patient is asymptomatic, there is no indication or drug therapy or an electrophysiology study unless he is in a high-risk occupation (eg, pilot, military member). With the onset o symptoms (palpitations, class I indication), the preerred approach would be an electrophysiology study and catheter ablation (>��% success rate with a low risk o complications).
�. Answer d.
CR is indicated or patients who have LVEF less than ��%, LBBB with QRS duration o more than ��� ms, and congestive heart ailure in NYHA class III or IV despite receiving the maximally tolerated medical therapy. Te most appro priate management is to up-titrate the dosages o lisinopril and carvedilol now and reevaluate the patient over the next � to � months to determine whether CR is indicated.
�. Answer a.
Te clinical decision on how best to treat the patient’s atrial �brillation (rate control or rhythm control) should be based on symptom burden and adequacy o rate control, independently o stroke risk, as determined by calculating her CHADS� (congestive heart ailure, hypertension, age >�� years, diabetes mellitus, and previous stroke) risk score. Her CHADS� score is greater than �. According to current guidelines, wararin is indicated.
�. Answer d.
Te patient is well compensated rom an HF standpoint. Her episodes o AF are inrequent and not particularly long in duration. Tereore, use o a daily antiarrhythmic drug (eg, sotalol) is not warranted (toxicity could occur over the long term with limited bene�t). Dronedarone is contraindicated in patients with HF or recent decompensation o HF. (See Køber et al in the “Suggested Reading” list.)
�. Answer a.
�. Answer d.
Te symptoms and presentation are consistent with cardiogenic syncope and conduction system disease (there is electrocardiographic evidence o high-grade atrioventricular block and lef bundle branch block). reatment would
Tis patient is asymptomatic and presents with bradycardia that may be readily explained by her athletic conditioning. Her murmur is systolic, sof (less than grade �), and heard
Presence o cardiac murmur
Systolic murmur
Grade 1 or 2 & midsystolic
Asymptomatic & no associated �ndings
Diastolic or continuous murmur
Grade 3 or higher holosystolic or late systolic
Other signs or symptoms o cardiac disease
Echocardiography
Catheterization & angiography i required
No urther workup
(Adapted rom Bonow RO, Carabello B, De Leon AC Jr, Edmunds LH Jr, Fedderly BJ, Freed MD, et al. ACC/AHA guidelines or the management o patients with valvular heart disease : a report o the American College o Cardiology/American Heart Association. ask Force on Practice Guidelines [Committee on Management o Patients with Valvular Heart Disease]. J Am Coll Cardiol. ���� Nov;��[�]:����–���. Used with permission.) Figure �.A�.
��
in the pulmonary position, and it disappears with the Valsalva maneuver. I you were worried about hypertrophic cardiomyopathy, the response to this maneuver airly well argues against it. Tere are no associated systolic clicks o mitral valve prolapse or bicuspid aortic valve, no diastolic snaps to suggest mitral stenosis, and the electrocardiogram is normal with the exception o the slow heart rate. Tus, the murmur is most likely a benign �ow murmur due to a thin chest wall, and you are likely hearing pulmonary out�ow, anatomically located just below the lef upper sternal border. An S� is a normal �nding in a young athlete. Tere is no need or urther testing (Figure �.A�). �. Answer c.
is not indicated in ASD or PFO. Chest radiography may be helpul. Classically, with large shunts radiography shows enlargement o the right atrium (bulging o the right heart border), RV (decreased retrosternal clear space), and pulmonary artery and increased pulmonary vascular markings. Common anatomical types o ASD are ostium secundum ASD, ostium primum ASD, and sinus venosus ASD. Sinus venosus ASD is commonly accompanied by anomalous pulmonary venous return. Ostium primum ASD is commonly associated with clef mitral valve and other complex congenital heart diseases. (See Levin et al, O’oole et al, and Wilson et al in the “Suggested Reading” list.) �. Answer c.
Several clinical �ndings in ASD provide clues to the condition: �) RV lif or sternal lif; �) palpable pulmonary artery pulse consistent with pulmonary artery volume and pressure overload o the RV (associated with signi�cant shunting); �) �xed splitting o the S � (the hallmark o ASD); �) accentuation o the tricuspid valve closure leading to splitting o the �rst heart sound; �) midsystolic pulmonary ejection murmur due to increased volume o �ow through the RV out�ow tract; and �) i a large shunt is present, a mid- to late-diastolic rumble may also be appreciated. PFO is usually not associated with any signi�cant physical examination �ndings, and it is a relatively common �nding in autopsy studies; on average, PFO is seen in ��% o otherwise healthy people. Subacute bacterial endocarditis prophylaxis
Tis ��-year-old patient presents with atypical chest pain, which may be associated with mitral valve prolapse. Mitral valve prolapse is essentially a mismatch o the lef ventricular cavity size and the mitral valve lea�ets, which are redundant. Te classic bedside maneuver is the squat-tostand maneuver; however, any maneuver that increases the lef ventricular cavity size will delay midsystolic clicks and the mitral regurgitant murmur (Figure �.A�). Te Valsalva maneuver decreases venous return, which would result in a smaller lef ventricle and thus an earlier mitral valve prolapse. A postextrasystolic beat (due to the compensatory pause), the passive leg raise, and squat (all would improve lef ventricular �lling) should actually delay midsystolic clicks and murmur.
Supine
S1
S2
C
Standing
S1
S2
C
Squatting
S1 C
S2
C indicates click; S �, �rst heart sound; S�, second heart sound. (Adapted rom Shaver JA, Leonard JJ, Leon DF. Examination o the heart. Part �: Auscultation o the heart. Dallas [X]: American Heart Association; c����. p. ��. Used with permission.) Figure �.A�.
�. CARDIOLOGY QUESIONS AND ANSWERS
•
��
�. Answer c.
��. Answer e.
Tis patient, with a systolic click murmur in the absence o dynamic change, likely has bicuspid aortic valve (AV) stenosis. Tis diagnosis is more commonly associated with a coarctation o the aorta whose �ow murmur is best heard over the posterior aspect between the shoulder blades, slightly to the lef o the midline. In the presence o hypertension and radial-emoral delay, there is enough evidence on examination to support imaging o the aorta to rule out coarctation. A C scan (or magnetic resonance imaging) o the chest would show the coarctation. Another appro priate test would be transthoracic echocardiography to con�rm the diagnosis o a bicuspid AV and to assess the degree o aortic stenosis. Echocardiography is ofen used to view the descending thoracic aorta to determine the degree o coarctation; however, i the aorta is not well visualized by echocardiography, other imaging is indicated to rule out the coarctation. A coronary angiogram is unlikely to be helpul unless it is coupled with an aortogram, which might provide the diagnosis, but there may be an increased danger due to the potential or coarctation, which could be difficult to cross with the catheter. Tis patient most likely does not have coronary artery disease. Te murmur and clinical presentation are not benign, and a thorough evaluation is warranted. Tis patient’s syncopal event is related to aortic stenosis or coarctation with signi�cant stenosis and the inability to adequately increase cardiac output, especially with postexercise vasodilatation. ��. Answer c.
Constrictive pericarditis is a relatively inrequent complication o open heart surgery, yet cardiac surgery is now the number one cause o constrictive pericarditis causing heart ailure. Te heart ailure ofen begins slowly, with symptoms o dyspnea, peripheral edema, and signs o elevated neck veins and the Kussmaul sign—elevation o JVP with inspiration. Another �nding ofen reported on the JVP is rapid descent. ��. Answer e.
Te presence o a prominent v wave in the JVP pro�le is consistent with signi�cant tricuspid regurgitation. Te right-sided S� may be intermittent and more prominent with the inspiratory phase o respiration. Mitral regurgitation will produce a v wave into the pulmonary circulation, which can be seen with Swan-Ganz catheter tracings, but the v wave is generally not transmitted to the JVP. Constrictive pericarditis has very rapid y descents. Pulmonary stenosis is associated with a giant a wave due to increased atrial contraction caused by right ventricular pressure overload. In superior vena cava (SVC) syndrome, the pulsatile waves are ofen lost because o extra-cardiac obstruction o the SVC. ��. Answer a.
An S� indicates increased lef ventricular stiffness and is commonly heard in adults with hypertension. It is caused by the increased �lling in late diastole during atrial contraction. An S� is never normal. ��
•
Te patient presents with stage D congestive heart ailure. He has several poor prognostic indicators, including recurrent hospitalizations or heart ailure exacerbations despite optimal medical therapy, low blood pressure, anemia, renal insuffi ciency, and a decreasing LVEF. His blood pressure would not allow an increase in β-blocker dose. Optimization o cardiac resynchronization therapy is unlikely to help patients with stage D heart ailure and a low output state. Adding a thiazide diuretic when edema has improved and renal insuffi ciency has worsened would not be an appropriate next step. Te patient had a remote history o myocardial inarction and had no active angina pectoris; hence, determining myocardial viability with a view toward revascularization is unlikely to provide much bene�t toward improving the patient’s symptoms o heart ailure or survival. Patients with dilated cardiomyopathy, an LVEF o ��% or less, and New York Heart Association (NYHA) class IV symptoms should be considered or LVAD implantation afer optimization o medical and electrical therapy. LVADs have been shown to prolong sur vival, improve NYHA unctional class, and improve quality o lie. ��. Answer e.
Although serum BNP levels correlate with clinical severity, some patients with advanced heart ailure have normal BNP levels. Several large clinical trials have shown that a persistently elevated BNP level despite ongoing medical therapy is a marker o poor prognosis. A BNP-guided approach to titrate medical therapy has been shown to decrease hospitalizations compared with standard therapy, but the bene�t may result rom more intense medical sur veillance and subsequent up-titration o medical therapy; thereore, its routine use or this purpose cannot be recommended. Although the serum BNP level is requently elevated in constrictive pericarditis due to cardiac surgery or radiotherapy, it is requently within the normal range in patients with idiopathic constrictive pericarditis. ��. Answer c.
Te patient has severe heart ailure with preserved ejection raction. Te cause o heart ailure in this patient is cardiac amyloidosis, which is consistent with the low voltage on the electrocardiogram and the thickened lef ventricular walls with a classic speckled pattern on the echocardiogram. Te concomitant presence o peripheral neuropathy is consistent with a diagnosis o amilial amyloidosis. In amilial amyloidosis, the source o the amyloidogenic variant o the transthyretin protein is the liver. Hence, the best treatment option or this patient is combined heart and liver transplant. ��. Answer c.
Te patient has dilated cardiomyopathy. Te most common cause o dilated cardiomyopathy, especially in a person with atherosclerotic risk actors, is coronary artery disease.
MAYO CLINIC INERNAL MEDICIN E BOARD REVIEW: QUESIONS AND ANSWER S
Te most appropriate next step or this patient would be to perorm coronary angiography, the best diagnostic test in this circumstance. Stress testing may have alse-positive or alse-negative results in patients with dilated cardiomyopathy. Endomyocardial biopsy should not be routinely perormed in the evaluation o patients with heart ailure. Holter monitoring is useul in detecting the PVC burden over a ��-hour period, and i the PVC burden were especially high, it could be implicated as a rare cause o heart ailure. However, Holter minitoring would not be the next step in evaluation and should not be perormed without initiating medical therapy because indicated medical treatment (eg, β-blockers) can attenuate PVCs. Sleep apnea could be a requent accompaniment o decompensated heart ailure, but a sleep study should be perormed only afer patients are optimally treated. ��. Answer e.
β-Blockers, spironolactone, isosorbide dinitrate in combination with hydralazine, and biventricular pacing have improved survival among patients with severe heart ailure and idiopathic dilated cardiomyopathy. However, the Digitalis Investigation Group trial did not show a survival bene�t with the use o digoxin in heart ailure. Te lack o bene�t may have been related to a higher serum digitalis level resulting in a higher mortality presumably due to a proarrhythmic effect. Digitalis retains a role in treating patients who have persistently symptomatic heart ailure and patients who have atrial �brillation with rapid ventricular response and heart ailure. ��. Answer d.
hypertensive heart disease. However, she should be evaluated or possible constrictive pericarditis and amyloidosis. Rarely, arteriovenous �stulae can also cause high-output heart ailure. Coronary artery disease can be present in patients with heart ailure and normal ejection raction and requently needs to be excluded, but it would not account or the patient’s symptoms and signs o heart ailure. ��. Answer e.
Te patient presents with a non–S-elevation myocardial inarction. Her age, ECG changes, and elevated levels o biomarkers give her a high-risk pro�le. An initially conser vative medical management strategy is reasonable because she presented to a community hospital. However, recurrence o chest pain is a clear indication or transerring her to a acility with a cardiac catheterization laboratory. Several studies have shown that glycoprotein IIb/IIIa inhibitors are effective in the management o non–S-segment elevation acute coronary syndromes, particularly in high-risk patients who require percutaneous coronary intervention. Initiating therapy with a glycoprotein IIb/IIIa inhibitor such as epti�batide is effective in reducing recurrent ischemia and recurrent myocardial inarction beore the coronary intervention and improves outcomes afer percutaneous coronary intervention. Increasing the dose o the β-blocker would not be advisable in view o the bradycardia. Stress testing is absolutely contraindicated or patients with acute ischemic symptoms. Morphine may be administered to relieve symptoms but is not a de�nitive therapy or severe ischemia. Trombolytics such as reteplase are not indicated in non–S-segment elevation acute coronary syndromes.
Te diagnosis is hypertrophic cardiomyopathy (HCM) and ��. Answer b. is made primarily by the physical �ndings. Although hos pitalized patients present with syncope, chest pain, heart Te patient presents with symptoms consistent with an ailure, or sudden cardiac death, most patients with HCM acute coronary syndrome with S-segment elevation on the are asymptomatic. Te pathophysiology o this autosomal electrocardiogram. Tereore, it was reasonable to perorm dominant disease has been linked to genetic mutations, emergency angiography, which demonstrated normal cor predominantly in the sarcomere and mitochondria, that onary arteries. Te differential diagnosis includes coronary result in either increased energy use or decreased energy spasm, coronary embolism, pericarditis, and myocarditis. production, which ultimately promotes myocyte growth. However, this patient had a very characteristic regional Hence, all �rst-degree relatives should be evaluated with wall motion abnormality on the lef ventriculogram, which electrocardiography and echocardiography. Patients with a involved the mid and apical segments o the lef ventricle personal or amily history o sudden cardiac death, unex with sparing o the basal segments. Tis entity has recently plained syncope, nonsustained ventricular tachycardia on been recognized as the apical ballooning syndrome (takoHolter monitoring, severe lef ventricular hypertrophy, or tsubo cardiomyopathy). Apical ballooning syndrome an abnormal blood pressure response to exercise are at the occurs predominantly in postmenopausal women and is highest risk or sudden cardiac death and should be considrequently preceded by mental or physical stress. Acute ered or ICD implantation. reatment o the patient with coronary syndrome was ruled out by coronary angiography. HCM and heart ailure is initially medical, with β -blockers Myocarditis is a differential diagnosis, although it typically or calcium channel blockers (or both), but i symptoms are produces global lef ventricular dysunction. Pericarditis is progressive despite medical therapy, myomectomy or alconot associated with systolic dysunction. hol septal ablation should be considered. ��. Answer e.
��. Answer d.
Tis patient presents with evidence o heart ailure with a normal lef ventricular ejection raction. She has the classic presentation o diastolic heart ailure likely due to
Prasugrel is a new antiplatelet agent that acts by irreversibly blocking adenosine diphosphate receptors on platelets, preventing their activation and aggregation. It has been approved or use in patients with unstable angina,
�. CARDIOLOGY QUESIONS AND ANSWERS
•
��
non–S-segment elevation myocardial inarction, or S-segment elevation myocardial inarction managed with percutaneous coronary intervention. Its use is absolutely contraindicated in patients with active pathologic bleeding or a history o transient ischemia attack or stroke. It is generally not recommended (but may be used) in patients who are �� years or older or who weigh less than �� kg. It is not contraindicated in women, but women have an increased risk o bleeding. ��. Answer c.
Te patient has severe �-vessel disease with reduced ejection raction and signi�cant angina despite medical therapy. CABG surgery is the treatment o choice to relieve symptoms and to improve long-term prognosis. PCI is increasingly being perormed or multivessel disease but is not the standard o care or revascularization in patients with severe �-vessel disease. Medical therapy alone in patients with �-vessel disease and decreased ejection raction is associated with worse outcomes compared with surgery ollowed by optimal medical therapy. ��. Answer a.
Tis patient has premature coronary artery disease. Aggressive risk actor management is required. Smoking cessation and adequate treatment o hypertension are essential. Te patient’s blood pressure appears to be below the recommended goal o ���/�� mm Hg. Te Adult reatment Panel III (AP III) guidelines or managing hyperlipidemia recommend an optimal LDL-C goal o less than �� mg/dL. Tis patient has achieved that goal. Te AP III guidelines also recommend screening and treating or the metabolic syndrome as a secondary therapeutic goal or managing hyperlipidemia. Tis patient meets criteria or metabolic syndrome: the level o triglycerides is high (≥��� mg/dL), HDL-C is low (≤�� mg/dL), and asting blood glucose is high (≥��� mg/dL). Te guidelines state that the initial step in managing metabolic syndrome is to treat underlying causes, such as overweight or obesity and physical inactivity. I these measures are inadequate, pharmacologic therapy or low HDL-C or elevated triglycerides (or both) is indicated. Increasing the dose o simvastatin rom �� to �� mg is not indicated since the patient has achieved the LDL-C goal and an increase in the dose is unlikely to substantially increase the HDL-C level. Elevated levels o hs-CRP are associated with an increased risk o uture events, but the precise clinical utility o measuring hs-CRP has not been established. Long-term cardiovascular rehabilitation therapy has not been shown to improve outcome. Elevated plasma lipoprotein (a) levels are associated with an increased risk o cardiovascular events; however, no therapeutic intervention has been shown to improve outcomes among patients who have elevated lipoprotein (a) levels. ��. Answer b.
A patient with diabetes mellitus and chronic renal ailure has an increased risk o contrast nephropathy. Reasonable ��
•
prophylactic measures include administering intravenous �uid and N -acetylcysteine beore the procedure. Initiating the use o clopidogrel beore angiography is not routine practice. Te most appropriate time or adjusting the medical therapy would be afer coronary angiography as part o the overall management strategy. In the presence o abnormal renal unction, it is important to discontinue the use o metormin beore coronary angiography. Tere is an increased risk o lactic acidosis among diabetic patients who have renal impairment and contrast nephropathy and are receiving metormin. Tereore, it is best practice to discontinue the use o metormin beore elective procedures and to resume it �� hours afer the administration o radiologic contrast material i the patient has an uncomplicated recovery. ��. Answer d.
Tis patient has moderate symptoms with single-vessel coronary artery disease. PCI is an established, effective therapy or coronary heart disease; however, it has not been shown to reduce the risk o myocardial inarction or improve sur vival among patients with stable angina. Previous studies have suggested that PCI reduces the amount o medications required and provides better symptomatic relie. Te Clinical Outcomes Utilizing Revascularization and Aggressive Drug Evaluation (COURAGE) trial con�rmed that with optimal medical therapy, PCI does not offer signiicant advantage. (See Boden et al in the “Suggested Reading” list.) However, PCI was associated with slightly lower use o antianginal medications and slightly better symptom control. Tus, or this particular patient both treatment strategies are equal or survival and risk o myocardial inarction. PCI is likely to lead to less antianginal medication use and better control o angina in the short term. ��. Answer e.
Distinguishing true claudication rom pseudoclaudication can be accomplished with a good history and physical examination. Patients with true claudication typically describe cramping pain with muscle atigue occurring in the cal and progressing to the thigh and buttock with continued exercise. Te classic description includes exertional pain involving the cal that impedes walking, resolves within �� minutes o rest, and neither begins at rest nor resolves on walking. In contrast, pseudoclaudication occurs both with walking and with prolonged standing or any activity associated with spinal extension. Patients with pseudoclaudication must sit (with spinal �exion) to relieve the pinching o the involved nerve roots. Sorting out the cause o patients’ symptoms can be difficult since many have both true claudication and pseudoclaudication. Te patient described in this question has peripheral artery disease (PAD), but rom the data provided, his symptoms cannot be de�nitely attributed to PAD. Only ��% o patients with PAD have classic symptoms o claudication. Indeed, more than hal o patients with documented PAD have no symptoms o claudication. (See McDermott et al in “Suggested Reading” list.)
M A YO C L I N I C I N E R N A L M E D I C I N E B O A R D R E V I EW : Q U E S I ON S A N D A N S W E R S
��. Answer b.
Tis patient has a descending aortic dissection. By de�nition, the proximal tear occurs distal to the lef subclavian artery ori�ce and adjacent to the ligamentum arteriosum. In contrast to type A (ascending) thoracic aortic dissections, which are managed surgically (mortality is �% per hour without surgery), type B dissections are typically managed medically. Indications or surgical intervention or type B dissections include dissection progression despite medical management, persistent or recurrent pain despite medical management, and end-organ ischemia due to branch luminal compromise rom the dissection �ap. In this patient, the dissection does not involve the lef subclavian artery by de�nition. Te discordant brachial blood pressures are a result o prolonged tobacco exposure and associated subclavian arterial atherosclerosis. Te murmur described is rom mitral insufficiency, not rom aortic valve disruption. (See Hiratzka et al in the “Suggested Reading” list.) ��. Answer e.
iming o AAA repair (either surgically or by endograf) is important and requires balancing the risk o rupture with the risk associated with the intervention. Currently, AAA repair is recommended i the diameter is �.� cm. Tis measurement should be anteroposterior and perpendicular to the axis, which can be difficult with tortuous aorta. Other indications or repair include annual growth rates exceeding �.� cm or any symptoms suggesting instability, such as tenderness o the aorta on examination. Te iliac artery aneurysm is small and would not require surgery. Te typical cutoff or iliac artery aneurysms is �.� cm. Te amily history o ruptured AAA is not a criterion or surgery. Te guidelines support a one-time screening or AAA in men aged �� to �� who have never smoked but who have a �rst-degree relative who required repair o an AAA or died o a ruptured AAA (grade �C). A one-time screening or AAA with abdominal ultrasonography is also recommended or men aged �� to �� who have smoked (grade �A). Screening or asymptomatic women is not recommended; however, Medicare allows or a one-time screening or AAA in women with a amily history o AAA. (See Hirsch et al in the “Suggested Reading” list.) ��. Answer e.
More IVC �lters are placed in medical institutions in the United States than in any other country. Current indications or IVC �lter placement include inability to provide anticoagulation because o active bleeding or circumstances in which the risk o initiating major bleeding is high (eg, recent surgery). Te proper treatment o hemodynamic instability with DV and a PE would not be placement o an IVC �lter but rather consideration o thrombolytic therapy. Te �nding o mobile thrombi (rather than immobile thrombi) on cross-sectional imaging does not increase the risk o embolism. A prior personal history o PE would not justiy IVC �lter placement; however, it would suggest the need or prolonged secondary prophylaxis with
wararin therapy. Te amily history o PE is not relevant to this decision. For cancer patients with recurrent thrombosis despite adequate wararin therapy, the proper treatment is conversion to low-molecular-weight heparin rather than IVC �lter placement. (See Baglin et al in “Suggested Reading” list.) ��. Answer d.
When de�ning appropriate venous thromboembolism treatment (ie, the �rst � months o therapy) and duration o secondary prophylaxis (ie, the time beyond the initial � months o therapy), it is important to distinguish provoked rom unprovoked or spontaneous thrombotic events. In general, events provoked by a transient risk actor (as in answer choices a-c and e) are treated with � months o wararin therapy. While heterozygous actor V Leiden is a congenital risk actor, it carries a rather weak propensity or venous thrombosis, with a hazard ratio o approximately �. Tereore, neither heterozygous actor V Leiden nor heterozygous prothrombin G�����A mutations require prolonged secondary prophylaxis in and o themselves. One might argue that a �-hour �ight �� days beore the event was a provocation, but the hazard associated with such an exposure is low. A stronger argument is that the PE was unrelated to the air travel and was an unprovoked event. Idiopathic or unprovoked events carry a higher rate o recurrence. Furthermore, i this patient has a recurrent event, it is twice as likely to be another PE. PE, and especially recurrent PE, carries a high mortality rate. Severe symptomatic PE in the period immediately afer travel is extremely rare afer �ights o less than � hours. In �ights longer than �� hours, the rate is � per million. (See Kearon et al and see Watson and Baglin in the “Suggested Reading” list.) ��. Answer a.
Nearly ���,��� strokes occur each year in the United States, and stroke is the third leading cause o death and the leading cause o long-term disability. Most strokes are ischemic and thromboembolic in nature. Te prevalence o moderate to severe carotid disease in the United States is �% to �% in persons older than ��, accounting or �.� to �.� million Americans. Te annual prevalence o IA is nearly � million. O these, one-third occur in patients with moderate to severe carotid stenosis. Both the Asymptomatic Carotid Atherosclerosis Study (ACAS) and the North American Symptomatic Carotid Endarterectomy rial (NASCE) showed that surgery improved stroke-ree survival at � years compared with medical management in symptomatic patients. Te Endarterectomy Versus Angioplasty in Patients With Symptomatic Severe Carotid Stenosis (EVA-�S) trial and the International Carotid Stenting Study showed that carotid endarterectomy was the procedure o choice or symptomatic carotid stenosis compared with carotid stenting (with embolic protection devices). Te Stent-Protected Angioplasty Versus Carotid Endarterectomy (SPACE) study ound that carotid stenting resulted in a signi�cantly greater rate o restenosis
�. CARDIOLOGY QUESIONS AND ANSWERS
•
��
(>��%) at � years (��.�% vs �.�%) compared with endarterectomy. (See Ederle et al, European Carotid Surgery rialists’ Collaborative Group, Mas et al, North American Symptomatic Carotid Endarterectomy rial Collaborators, and Ringleb et al in the “Suggested Reading” list.)
hypertension may evolve into sustained hypertension over time. Tus, when hypertension is diagnosed, patients should be advised on liestyle modi�cations associated with lowering BP and be ollowed. Sel-measurement o BP should be encouraged. ��. Answer c.
��. Answer c.
Tis patient has a classic presentation o inrapopliteal arterial occlusive disease with long-standing diabetes mellitus. Both the latest physical examination and the previous con ventional angiogram support this impression. It would not be helpul to repeat either MRA or CA now. Angioplasty and stenting o these distal arteries would have poor durability. Tere is no indication or surgical bypass since the patient has neither rest pain nor ulceration. Te best option or this patient is risk actor modi�cation (smoking cessation, statin therapy, hypertension control, and diabetes therapy). (See Hirsch et al in the “Suggested Reading” list.) ��. Answer d.
Te Seventh Report o the Joint National Committee on Prevention, Detection, Evaluation and reatment o High Blood Pressure (JNC �) provides ollow-up recommendations based on the classi�cation o blood pressure. Patients with normal blood pressure should be rechecked in � years. Prehypertensive patients should be rechecked in � year. Tose with stage � hypertension should be rechecked in � months. Tose with stage � hypertension should have more urgent ollow-up. Te patient in this question has stage � hypertension and should thereore be rechecked within � months. ��. Answer d.
Angiotensin-converting enzyme inhibitors, angiotensin receptor blockers, and direct renin inhibitors are contraindicated in pregnancy because their use has been associated with serious etal abnormalities (l imb deects, lung hypoplasia, cranioacial deormities, and renal dys plasia). raditionally, etal abnormalities were thought to develop afer the �rst trimester; however, the latest inormation suggests that they can occur at any time during pregnancy. Tus, it is recommended that these drugs be avoided in hypertensive women who are sexually active and can become pregnant. Doxazosin is not recommended as an initial drug or the treatment o hypertension. ��. Answer e.
Approximately ��% to ��% o patients with elevated offi ce BP have normal readings outside the clinic environment. Tis is reerred to as office or white coat hypertension. Tese persons are at low risk and do not require drug therapy. Sel-measured BP consistently less than ���/�� mm Hg con�rms the diagnosis. I the sel-measured systolic BP is ��� to ��� mm Hg and the sel-measured diastolic BP is �� to �� mm Hg, the diagnosis is best con�rmed with noninvasive ��-hour ambulatory BP monitoring. Offi ce ��
•
In general, the office BP goal or persons with hypertension is less than ���/�� mm Hg and the home BP goal is less than ���/�� mm Hg. Current guidelines recommend a more aggressive BP goal o less than ���/�� mm Hg or persons who have diabetes mellitus, chronic kidney disease, known vascular disease, or a high risk o vascular disease. Tis same goal applies or both office and sel-measured readings. A goal o less than ���/�� mm Hg is appropriate or patients who have renal disease associated with proteinuria. ��. Answer d.
Tis case should be considered a hypertensive emergency because the patient has evidence o organ injury (coronary ischemia and inarction). Tereore, an immediate reduction in blood pressure with a parenteral agent is indicated. O the available drugs, nitroglycerin is preerred with myocardial ischemia. It is a balanced arterial and venous dilator and lessens myocardial oxygen demand by reducing both preload and aferload. Hydralazine is a direct arterial vasodilator and may worsen myocardial ischemia. Te other medications listed would be acceptable second-line agents or this patient. O these, sodium nitroprusside is the best studied. ��. Answer a.
Te presentation is highly suggestive o amilial pheochromocytoma. Te �rst step is to establish the diagnosis by measurement o ree metanephrines in the plasma, which is the screening test o choice, especially i a amilial disorder is suspected. Plasma and urine catecholamines lack diagnostic accuracy. Computed tomography or magnetic resonance imaging o the abdomen is appropriate as the initial test to locate the tumor since ��% are located in � or both adrenal glands and ��% are located in the abdomen. A search or the tumor should ollow biochemical con�rmation o the diagnosis. Ten an α-blocker should be used as initial therapy. β -Blocker monotherapy can be associated with a paradoxical increase in blood pressure, but a β-blocker may be used to treat tachycardia that may occur with adequate α-blocker therapy. ��. Answer e.
Tiazide diuretics are associated with decreased urinary excretion o calcium and are ofen used to treat calcium nephrolithiasis to reduce the risk o recurrent stone ormation. Tus, or this patient, chlorthalidone would be an appropriate �rst agent to treat hypertension. Furosemide is associated with an increase in urinary calcium excretion and should not be considered i a patient has concomitant calcium stone disease.
MAYO CLINIC INERNAL MEDICIN E BOARD REVIEW: QUESIONS AND ANSWER S
SUGGESED RE ADING Baglin P, Brush J, Streiff M; British Committee or Standards in Haematology Writing Group. Guidelines on use o vena cava �lters. Br J Haematol. 2006 Sep;134(6):590–5. Epub 2006 Jul 26. Boden WE, O’Rourke RA, eo KK, Hartigan PM, Maron DJ, Kostuk WJ, et al; COURAGE rial Research Group. Optimal medical therapy with or without PCI or stable coronary disease. N Engl J Med. 2007 Apr 12;356(15):1503–16. Epub 2007 Mar 26. Ederle J, Dobson J, Featherstone RL, Bonati LH, van der Worp HB, de Borst GJ, et al; International Carotid Stenting Study investigators. Carotid artery stenting compared with endarterectomy in patients with symptomatic carotid stenosis (International Carotid Stenting Study): an interim analysis o a randomised controlled trial. Lancet. 2010 Mar 20;375(9719):985–97. Epub 2010 Feb 25. Erratum in: Lancet. 2010 Jul 10;376(9735):90. Nasser, H-C [corrected to Nahser, H-C]. European Carotid Surgery rialists’ Collaborative Group. MRC European Carotid Surgery rial: interim results or symptomatic patients with severe (70–99%) or with mild (0–29%) carotid stenosis. Lancet. 1991 May 25;337(8752):1235–43. Hiratzka LF, Bakris GL, Beckman JA, Bersin RM, Carr VF, Casey DE Jr, et al; American College o Cardiology Foundation/ American Heart Association ask Force on Practice Guidelines; American Association or Toracic Surgery; American College o Radiology; American Stroke Association; Society o Cardiovascular Anesthesiologists; Society or Cardiovascular Angiography and Interventions; et al. 2010 ACCF/AHA/AAS/ACR/ASA/SCA/ SCAI/SIR/SS/SVM g uidelines or the diagnosis and management o patients with Toracic Aortic Disease: a report o the American College o Cardiology Foundation/American Heart Association ask Force on Practice Guidelines, American Association or Toracic Surgery, American College o Radiology, American Stroke Association, Society o Cardiovascular Anesthesiologists, Society or Cardiovascular Angiography and Interventions, Society o Interventional Radiology, Society o Toracic Surgeons, and Society or Vascular Medicine. Circulation. 2010 Apr 6;121(13):e266– 369. Epub 2010 Mar 16. Erratum in: Circulation. 2010 Jul 27;122(4):e410. Hirsch A, Haskal ZJ, Hertzer NR, Bakal CW, Creager MA, Halperin JL, et al; American Association or Vascular Surgery; Society or Vascular Surgery; Society or Cardiovascular Angiography and Interventions; Society or Vascular Medicine and Biology; Society o Interventional Radiology; ACC/AHA ask Force on Practice Guidelines Writing Committee to Develop Guidelines or the Management o Patients With Peripheral Arterial Disease; et al. ACC/AHA 2005 Practice Guidelines or the management o patients with peripheral arterial disease (lower extremity, renal, mesenteric, and abdominal aortic): a collaborative report rom the American Association or Vascular Surgery/Society or Vascular Surgery, Society or Cardiovascular Angiography and Interventions, Society or Vascular Medicine and Biology, Society o Interventional Radiology, and the ACC/AHA ask Force on Practice Guidelines (Writing Committee to Develop Guidelines or the Management o Patients With Peripheral Arterial Disease): endorsed by the American Association o Cardiovascular and Pulmonary Rehabilitation; National Heart, Lung, and Blood
Institute; Society or Vascular Nursing; ransAtlantic Inter-Society Consensus; and Vascular Disease Foundation. Circulation. 2006 Mar 21;113(11):e463–654. Kearon C, Kahn SR, Agnelli G, Goldhaber S, Raskob GE, Comerota AJ; American College o Chest Physicians. Antithrombotic therapy or venous thromboembolic disease: American College o Chest Physicians Evidence-Based Clinical Practice Guidelines (8th Edition). Chest. 2008 Jun;133(6 Suppl):454S-545S. Erratum in: Chest. 2008 Oct;134(4):892. Køber L, orp-Pedersen C, McMurray JJ, Gøtzsche O, Levy S, Crijns H, et al; Dronedarone Study Group. Increased mortality afer dronedarone therapy or severe heart ailure. N Engl J Med. 2008 Jun 19;358(25):2678–87. Erratum in: N Engl J Med. 2010 Sep 30;363(14):1384. Levin AR, Spach MS, Boineau JP, Canent RV Jr, Capp MP, Jewett PH. Atrial pressure-�ow dynamics in atrial septal deects (secundum type). Circulation. 1968 Apr;37(4):476–88. Mas JL, Chatellier G, Beyssen B, Branchereau A, Moulin , Becquemin JP, et al; EVA-3S Investigators. Endarterectomy versus stenting in patients with symptomatic severe carotid stenosis. N Engl J Med. 2006 Oct 19;355(16):1660–71. McDermott MM, Greenland P, Liu K, Guralnik JM, Criqui MH, Dolan NC, et al. Leg symptoms in peripheral arterial disease: associated clinical characteristics and unctional impairment. JAMA. 2001 Oct 3;286(13):1599–606. North American Symptomatic Carotid Endarterectomy rial Collaborators. Bene�cial effect o carotid endarterectomy in symptomatic patients with high-grade carotid stenosis. N Engl J Med. 1991 Aug 15;325(7):445–53. O’oole JD, Reddy PS, Curtiss EI, Shaver JA. Te mechanism o splitting o the second heart sound in atrial septal deect. Circulation. 1977 Dec;56(6):1047–53. Ringleb PA, Allenberg J, Bruckmann H, Eckstein HH, Fraedrich G, Hartmann M, et al; SPACE Collaborative Group. 30 day results rom the SPACE trial o stent-protected angioplasty versus carotid endarterectomy in symptomatic patients: a randomised non-ineriority trial. Lancet. 2006 Oct 7;368(9543):1239–47. Erratum in: Lancet. 2006 Oct 7;368(9543):1238. Watson HG, Baglin P. Guidelines on travel-related venous thrombosis. Br J Haematol. 2011 Jan;152(1):31–4. Epub 2010 Nov 18. Wilson W, aubert KA, Gewitz M, Lockhart PB, Baddour LM, Levison M, et al; American Heart Association Rheumatic Fever, Endocarditis, and Kawasaki Disease Committee; American Heart Association Council on Cardiovascular Disease in the Young; American Heart Association Council on Clinical Cardiology; American Heart Association Council on Cardiovascular Surgery and Anesthesia; �uality o Care and Outcomes Research Interdisciplinary Working Group. Prevention o inective endocarditis: guidelines rom the American Heart Association: a guideline rom the American Heart Association Rheumatic Fever, Endocarditis, and Kawasaki Disease Committee, Council on Cardiovascular Disease in the Young, and the Council on Clinical Cardiology, Council on Cardiovascular Surgery and Anesthesia, and the �uality o Care and Outcomes Research Interdisciplinary Working Group. Circulation. 2007 Oct 9;116(15):1736–54. Epub 2007 Apr 19. Erratum in: Circulation. 2007 Oct 9;116(15):e376–7.
�. CARDIOLOGY QUESIONS AND ANSWERS
•
��
This page intentionally left blank
�. GASTROENTEROLOGY AND HEPATOLOGY QUESTI ONS AND ANSWERS
QUESIONS
a. Oral prednisone b. Bismuth subsalicylate c. Budesonide d. Cipro�oxacin e. Metronidazole
Multiple Choice (choose the best answer) COLON A ND PANCREAS
�. A ��-year-old woman presented to the emergency department with a �-hour history o epigastric pain, nausea, and vomiting. She has a history o hypertension and hyperlipidemia. Her medications are aspirin �� mg daily, lisinopril �� mg daily, and simvastatin �� mg daily. Her heart rate is ��� beats per minute, her blood pressure is ���/�� mm Hg, and her tem perature is ��.�°C. On clinical examination, the patient has moderate epigastric tenderness and reduced bowel sounds. Laboratory test results include the ollowing: hemoglobin �� g/dL, white blood cell count ��×���/L, amylase �,��� U/L, lipase �,��� U/L, aspartate aminotranserase (AS) ��� U/L, alanine aminotranserase (AL) ��� U/L, and bilirubin �.� mg/dL. What should you recommend next?
�. A ��-year-old man presents or colon cancer screening. He is asymptomatic, and he states that he is not aware o any amily member with a prior history o either colon cancer or colon polyps. Te patient undergoes colonoscopy and is ound to have the ollowing polyps and histologic diagnoses: �-mm cecal polyp and �-mm transverse polyp (tubular adenomas, low-grade dysplasia), �-mm sigmoid polyp (tubulovillous adenoma, low-grade dys plasia), and �-mm rectal polyp (hyperplasia). I those � lesions were completely removed, what should you recommend or ongoing colon surveillance? a. b. c. d. e.
Colonoscopy in � year Colonoscopy in � years Colonoscopy in � years Yearly ecal occult blood test and colonoscopy in � years Computed tomographic colonography in � years
a. Computed tomographic (C) scan o the abdomen with intravenous (IV) contrast medium b. Ultrasonography o the abdomen c. Emergent endoscopic retrograde cholangiopancreatography (ERCP) d. Plain abdominal radiography e. IV �uid, bowel rest, and observation
�. A ��-year-old woman with ulcerative colitis o � years’ duration presents with a red eye. She has no pain or headache, and her vision is normal. Her stools have been somewhat looser over recent weeks. She has been taking mesalamine at her usual dosage o �.� g daily, but she has missed doses occasionally. She recently quit smoking. What is the most likely cause o her red eye? a. b. c. d. e.
�. A ��-year-old man presents to the emergency department with lef lower abdominal pain. He has not had ever or a change in bowel habit. He is eating without diffi culty. He has never had similar symptoms in the past. He has not undergone colon cancer screening. He has no comorbid conditions. On examination, he has mild tenderness in the lef lower abdomen without peritoneal signs. Te white blood cell count is ��.����/L. Computed tomography (C) shows changes consistent with diverticulitis without abscess. What should you recommend as the next step or this patient?
Uveitis Giant cell arteritis Episcleritis Nicotine withdrawal and insomnia Viral conjunctivitis
�. A ��-year-old woman presents with a �-week history o diarrhea. She has had � to � large-volume watery stools daily. Tere has been no blood in the stool. She has not had ever or other systemic symptoms. Recently, she has not taken antibiotics or changed medication. Clinical examination �ndings were normal. Laboratory test results were unremarkable. Stool examination or ecal leukocytes was negative. On colonoscopy, the colon appeared normal. On biopsy, a thickened subepithelial collagen band was apparent. What initial therapy should you recommend?
a. b. c. d. e.
��
Hospital admission, bowel rest, and intravenous antibiotics Outpatient antibiotics Colonoscopy Surgical consultation C colonography
ESOPHAGUS AND STOMACH
�. A ��-year-old man undergoes esophagogastroduodenoscopy (EGD) and colonoscopy as part o an evaluation or mild iron de�ciency anemia. He denies having melena or hematochezia, and he has not lost weight. Te only medication he takes is ibuproen or intermittent joint aches, and he is otherwise healthy. His amily history is unremarkable. Te EGD showed multiple linear antral erosions and a �.�-cm polyp in the body o the stomach. Biopsies rom throughout the stomach showed a chemical gastritis with no Helicobacter pylori , and biopsies o the polyp identi�ed a tubular adenoma with low-grade dysplasia. Small bowel biopsy �ndings were normal. Te colonoscopy showed only scattered sigmoid diverticula. Which o the ollowing is the best next step? a. b. c. d. e.
�.
No urther testing Helicobacter pylori stool antigen test
Endoscopic ultrasonography EGD now with ull removal o the polyp EGD in � year
A ��-year-old man presents with hoarseness and intermittent heartburn symptoms. He notes that in the past he had heartburn caused by triggering oods several times per month; symptoms now occur several times per week without obvious precipitants. He also notes that his voice is hoarse, which has affected his singing in the church choir. He reports no dysphagia, odynophagia, nausea, vomiting, early satiety, or weight loss. Proton pump inhibitor (PPI) therapy twice daily beore meals has provided only partial relie o symptoms. Esophagogastroduodenoscopy (EGD) showed several cystic undic gland polyps but was otherwise normal. Which o the ollowing is the best next step? a. b. c. d. e.
An H� receptor blocker at bedtime Gastric scintigraphy with a solid meal A ��-hour ambulatory pH probe Video �uoroscopic swallowing test Esophageal manometry
�. A ��-year-old man presents or evaluation o new swallowing problems. He states that or the past � months, ood gets stuck when he swallows, although the ood eventually passes spontaneously. He also notes di�culty swallowing liquids and senses ullness in his chest or a prolonged period afer drinking any beverage. He reports regurgitation o �uid into the back o his throat. He has a ��–pack-year smoking history, and a long-standing history o re�ux, well-controlled with PPI therapy as needed. He has lost �.� kg over the past ew months. He had esophagogastroduodenoscopy (EGD) � year ago to screen or Barrett esophagus; results were negative. With his new symptoms, he now undergoes a barium esophagram, which shows a bird’s beak narrowing at the distal esophagus; esophageal manometry shows an elevated pressure in the lower esophageal sphincter, which does not relax afer a ��
•
swallow, and aperistalsis. Which o the ollowing is the best next step? a. b. c. d. e.
Perorm another EGD now. Inject botulinum toxin into the lower esophageal sphincter. Reer the patient to a surgeon or myotomy. Perorm computed tomography o the chest. est or anticentromere antibodies. SMALL BOWEL AND IN TESTINE
�. A ��-year-old man presents with a �-month history o diarrhea. He has approximately �� watery bowel movements daily, and he has lost �.� kg while he has had diarrhea. Physical examination, complete blood cell count, and chemistry panel results were normal. A ��-hour stool collection showed �,��� g o stool with �� g o at per �� hours. Stool electrolyte concentrations were as ollows: sodium �� mEq/L and potassium �� mEq/L. From these �ndings, what is the most likely cause o this patient’s diarrhea? a. b. c. d. e.
Whipple disease Vasoactive intestinal peptide tumor Celiac sprue Chronic pancreatitis Lactase de�ciency
��. A ��-year-old woman who has iron de�ciency anemia began receiving oral iron therapy without response. She reports no gastrointestinal tract complaints or heavy menses. She has a normal appetite and reports no weight loss. Tere is no amily history o colon cancer or in�ammatory bowel disease. Fecal occult blood testing o the stool is negative. Which test should be perormed next? a. Measurement o serum IgA and IgG tissue transglutaminase antibodies b. Upper endoscopy with small bowel biopsies c. Capsule endoscopy d. Small bowel ollow-through e. Stool evaluation or ova and parasites
��. A ��-year-old white man presents with a �-month history o weight loss and arthralgias. He reports no anorexia but has had diarrhea with up to � loose stools daily. He has migratory pain involving the shoulders, elbows, and knees. On physical examination, he has skin hyperpigmentation and oculomasticatory myorhythmia. A ��-hour stool collection shows �� g o at per �� hours. Which test is most likely to be positive or diagnostic or this patient? a. Measurement o serum IgA and IgG tissue transglutaminase antibodies b. Measurement o serum IgA and IgG deamidated gliadin antibodies c. Small bowel biopsy showing enlarged villi with periodic acid-Schiff (PAS)-positive macrophages d. Small bowel biopsy showing villous atrophy and crypt hyperplasia e. Duodenal aspirates showing more than ���,��� colony-orming units (CFU)/mL
MAYO CLINIC INERNAL MEDICIN E BOARD REVIEW: QUESIONS AND ANSWER S
��. A ��-year-old woman presents with abdominal discomort and diarrhea. Almost every day, she has variable abdominal discomort with up to � or � watery stools. She has associated abdominal bloating and �atulence. Eating and stress aggravate symptoms, and abdominal discomort is relieved by deecation. She reports no anorexia, weight loss, or blood in the stool. You suspect irritable bowel syndrome (IBS). Laboratory study results were normal or complete blood cell count, erythrocyte sedimentation rate, and C-reactive protein. Fecal leukocytes are present, but culture or enteric pathogens and testing or Clostridium diffi cile are negative. Which intervention is appropriate at this time? a. Loperamide b. Riaximin c. Colonoscopy with biopsies d. Reassuring and counseling the patient e. Stool-bulking agents
��. A ��-year-old woman with a history o mitral valve disease and atrial �brillation awakens rom sleep with a sudden onset o sharp central abdominal pain and the sudden urge to have a bowel movement. An explosive stool is passed without blood; however, the pain persists. On examination, her abdomen is sof, with minimal tenderness and no rebound or guarding. Te physician notes that her pain is out o proportion to the �ndings on physical examination. Laboratory test data are normal, including results or a complete blood cell count, blood chemistry panel, amylase, lipase, and serum lactate. An electrocardiogram shows normal sinus rhythm. A computed tomographic (C) scan o the abdomen shows a nonspeci�c gas pattern. What should be the next step in management? a. Observation b. A vascular imaging study c. Laparotomy d. Laparoscopy e. Stool studies
��. A ��-year-old woman who had extensive abdominal radiotherapy � years earlier or endometrial cancer has chronic symptoms o postprandial bloating and periumbilical discomort. Over the past several weeks she has had persistent diarrhea. Results o routine stool studies or inectious agents are negative. Which o the ollowing treatments would most likely result in sustained improvement in her diarrhea? a. b. c. d. e.
Oral mesalamine Bile acid–binding resin Cipro�oxacin A proton pump inhibitor Low-at diet
��. A ��-year-old woman with Crohn disease and a prior ileal resection receives a bile acid–binding agent or treatment o chronic diarrhea. However, the diarrhea worsens. What is the most likely explanation?
a. Bacterial overgrowth requently develops in patients with ileal resections. b. Bile acid–binding resins cause diarrhea. c. Te bile acid binder has urther depleted the bile acid pool. d. Bile acid binders may cause vitamin de�ciencies. e. Te patient most likely increased her consumption o poorly digestible carbohydrates.
LIVER
��. A ��-year-old man who comes or a preoperative assessment beore hernia repair is mildly jaundiced. Other than a symptomatic inguinal hernia, he eels well. en years ago, the patient had a brie period o illicit drug use. He currently drinks � alcoholic beverages daily. He takes niacin or hyperlipidemia. Physical examination is notable only or mild jaundice. Laboratory test results include the ollowing: hemoglobin �� g/dL, platelet count ������/L, aspartate aminotranserase (AS) �� U/L, alanine aminotranserase (AL) �� U/L, alkaline phosphatase �� U/L, total bilirubin �.� mg/dL, and direct bilirubin �.� mg/dL. Which o the ollowing is the most likely diagnosis? a. b. c. d. e.
Hepatitis C Hepatitis B Niacin-induced liver disease Alcoholic hepatitis Gilbert syndrome
��. A ��-year-old woman presents with increasing values on liver tests. Tree months ago, at a general assessment, the level o her alanine aminotranserase (AL) was �� U/L and she had hyperlipidemia. Evaluation o the elevated liver test results was negative except or steatosis noted on ultrasonography. She began taking simvastatin. One month afer starting simvastatin, her AL is �� U/L. She continues to eel well and her examination is unremarkable except or obesity. Her only other medication is metormin. She drinks � glass o wine every � months. Which o the ollowing would you advise at this time? a. b. c. d. e.
Stop all alcohol intake. Undergo liver biopsy. Stop use o simvastatin. Stop use o metormin. Follow with serial monitoring o AL.
��. A ��-year-old woman is reerred because o an elevated alkaline phosphatase level discovered during evaluation or atigue. She has had no previous blood tests. Physical examination is notable or xanthelasmas. Laboratory study results include the ollowing: alkaline phosphatase ��� U/L, alanine aminotranserase �� U/L, and total cholesterol ��� mg/dL; bilirubin, international normalized ratio, and albumin are all normal. Which o the ollowing tests is most likely to establish a diagnosis? a. Antimitochondrial antibody testing b. Magnetic resonance cholangiopancreatography
�. GASROENEROLOGY AND HEPAOLOGY QUESIONS AND ANSWERS
•
��
c. Antinuclear antibody testing d. Anti–smooth muscle antibody testing e. Angiotensin-converting enzyme level measurement
��. A ��-year-old alcoholic man is reerred with a �-month history o abdominal distention, leg edema, and dyspnea. Physical examination is notable or mild jugular venous distention, distant heart sounds, marked ascites, and leg edema. Chest radiography shows mild cardiomegaly. Laboratory test results are as ollows: total bilirubin �.� mg/dL, albumin �.� g/dL, and international normalized ratio �.�. Liver Doppler ultrasonography shows ascites, a coarse echotexture o the liver, and patent hepatic and portal veins. In the abdominal �uid, the protein level is �.� g/dL and the albumin level is �.� g/dL. Which o the ollowing would you advise next? a. Hepatic venography b. ransjugular intrahepatic portosystemic shunt c. Liver biopsy d. Echocardiography e. Laparoscopy
��. A ��-year-old woman is reerred with a �-week history o atigue and vague right upper quadrant pain. She drinks � glasses o wine daily and occasionally more on weekends. Her medical history is notable or hypothyroidism. She reports no prior blood transusions, illegal drug use, or a amily history o liver disease. Her only medications are ibuproen ��� mg up to � times daily, acetaminophen ��� mg up to � times daily, and thyroxine. Physical examination �ndings are normal. Laboratory study results are as ollows: aspartate aminotranserase (AS) ��� U/L, alanine aminotranserase (AL) ��� U/L, alkaline phosphatase ��� U/L, and γ-globulin �.� g/dL. Bilirubin and albumin levels and the international normalized ratio (INR) are normal. Serologic studies are negative or hepatitis A, B, and C. Te patient is observed, and � weeks later her laboratory test results are as ollows: AS ��� U/L, AL ��� U/L, total bilirubin �.� mg/dL, and INR �.�. Ultrasonography shows a coarse echotexture to the liver, borderline splenomegaly, and gallbladder stones without bile duct dilatation. wo weeks later the AL is unchanged, but the bilirubin is �.� mg/dL. Which o the ollowing is the most likely diagnosis? a. b. c. d. e.
��. A ��-year-old man is admitted rom the emergency department with upper gastrointestinal tract bleeding. He has a history o chronic hepatitis C and cirrhosis but elt well until early this morning when he had hematemesis. Esophagogastroduodenoscopy in the emergency department showed large esophageal varices, which were ligated. He currently eels weak but has had no other complaints. Physical examination �ndings include the ollowing: blood pressure ���/�� mm Hg, heart rate �� beats per minute, alert and oriented, splenomegaly, and no ascites or edema. Laboratory study results are as ollows: hemoglobin �.� g/L, platelet count �����/L, aspartate aminotranserase �� U/L, alanine aminotranserase �� U/L, total bilirubin �.� mg/dL, albumin �.� g/dL, and international normalized ratio (INR) �.�. Abdominal ultrasonography shows no liver mass, a patent portal vein, splenomegaly, and no ascites. Which o the ollowing would you advise now? a. b. c. d. e.
ransjugular intrahepatic portosystemic shunt Nor�oxacin ransusion o � units o packed red blood cells ransusion o � units o resh rozen plasma Pegylated intereron and ribavirin
��. A ��-year-old Asian woman receives a diagnosis o non-Hodgkin lymphoma, and chemotherapy is advised. She has a history o hepatitis B without complications. Her mother also had hepatitis B. On examination, the patient has cervical adenopathy consistent with lymphoma and no stigmata o chronic liver disease. Laboratory test results are as ollows: platelet count ������/L, alanine aminotranserase �� U/L, total bilirubin �.� mg/dL, hepatitis B surace antigen positive, hepatitis B e antigen (HBeAg) negative, antibody to HBeAg positive, IgG antibody to hepatitis B core antigen positive, and hepatitis B virus (HBV) DNA undetectable. Which o the ollowing should you advise at this time? a. b. c. d. e.
Hepatitis B vaccination Surveillance or hepatocellular carcinoma Lamivudine Pegylated intereron Nothing urther at this time except chemotherapy
Autoimmune hepatitis Ibuproen hepatotoxicity Acetaminophen hepatotoxicity Alcoholic hepatitis Cholelithiasis
��
•
M A YO C L I N I C I N E R N A L M E D I C I N E B O A R D R E V I EW : Q U E S I ON S A N D A N S W E R S
ANSWERS
�. Answer b.
without difficulty, outpatient management may be pursued. I there were evidence o a complication (eg , abscess), or i the patient could not tolerate oral intake, hospitalization would be necessary. Since this is the patient’s �rst episode o diverticulitis, and it is uncomplicated, surgery is not indicated. Colonoscopy is contraindicated with acute diverticulitis, but since this patient has not undergone colon cancer screening, it would be reasonable to perorm colonoscopy � to � weeks afer the acute symptoms have resolved.
Hyperplastic polyps do not iner an increased risk o colon cancer. Tereore, the rectal lesion is o no clinical signi�cance. Te patient had � clinically signi�cant polyps. Although each lesion was smaller than � cm, the interval to the next colonoscopy would be � years because there were � or more polyps and because � o the lesions had a villous component. �. Answer c.
A patient with in�ammatory bowel disease who has a red eye most likely has either episcleritis or uveitis. Uveitis is associated with pain in the eye. Episcleritis is typically painless. Giant cell arteritis typically does not maniest with red eye and is usually associated with headache or visual loss.
�. Answer d.
Tis patient has iron de�ciency anemia that is likely due to the antral erosions that appear to have been induced by the nonsteroidal anti-in�ammatory drug. However, he also has a gastric polyp that is a gastric adenoma; similar to colonic adenomas, gastric adenomas are deemed premalignant and require ull endoscopic removal. Te patient should have another EGD now with polypectomy since the polyp was simply biopsied and not ully removed during his previous EGD. o do no urther testing would be inadequate because this polyp could continue to grow and progress to gastric cancer. While gastric erosions can be caused by H pylori, this patient’s histologic examination was negative or H pylori, and he was not taking any medications that could lead to alse-negative testing or H pylori (proton pump inhibitor, antibiotics, etc); thereore, urther testing or H pylori would not be needed. I the gastric polyp had been malignant, endoscopic ultrasonography would be needed to assess the depth o invasion and to complete locoregional staging, but that is not needed or an adenoma o this size. o wait to repeat the EGD or � year is not recommended since the polyp could continue to grow, progress to cancer, or cause bleeding, all o which could be prevented by removal now.
�. Answer b.
Te clinical presentation and the colonoscopic and histologic �ndings are typical o microscopic colitis (in this case, collagenous colitis). Initial therapy or mild to moderate disease (�–� bowel movements daily) is usually with either an antidiarrheal, such as loperamide hydrochloride, or bismuth subsalicylate. In more severe cases (>� bowel movements daily), budesonide may be considered or initial therapy. Prednisone would be used only in cases o microscopic colitis that did not respond to the aorementioned therapies. �. Answer e.
Te patient has acute pancreatitis as evidenced by the clinical presentation and the elevated levels o amylase and lipase. Te cause is most likely gallstone disease (AS and AL were moderately elevated). Te most important initial step in the management o patients with acute pancreatitis is to ensure excellent hydration to optimize pancreatic perusion and thereby decrease the risk o pancreatic necrosis. Te patient does not have evidence o cholangitis (absence o ever, pain localizing to the right upper quadrant, and jaundice), so there is no indication or emergent ERCP. C scan o the abdomen with IV contrast medium may be indicated later in the clinical course to evaluate or pancreatic necrosis, but it is not indicated at initial presentation. Ultrasonography o the abdomen is a reasonable step but only afer IV �uid resuscitation has been initiated.
�. Answer c.
Tis patient has eatures suggestive o gastroesophageal disease. He has not only progressive heartburn but also the extra-esophageal eature o hoarseness. Since he has not ully improved with PPI therapy and the EGD was negative or esophagitis, a ��-hour ambulatory pH probe would be helpul to document whether he truly has acid re�ux and whether there is symptom correlation. When this test incor porates impedance testing, symptoms can be correlated with acid-mediated re�ux or with non–acid-mediated re�ux (eg, seen with someone receiving PPI therapy). At this point,
�. Answer b.
Tis patient is presenting with a �rst episode o uncom plicated diverticulitis. Since he is tolerating oral intake ��
establishing a diagnosis would be helpul to direct uture therapy rather than adding additional acid suppression with an H� receptor blocker at bedtime. While patients with delayed gastric emptying can present with worsening acid re�ux, this patient does not report having any o the other symptoms that commonly occur with gastroparesis (nausea, vomiting, early satiety), making it less likely at this point. Although hoarseness may result rom neuromuscular weakness, which can cause oropharyngeal dysphagia, this patient does not report swallowing diffi culties that would suggest that a video swallowing test would be o value. Esophageal manometry can be useul or evaluating motility disorders o the esophagus, such as achalasia, but would not have a role here in the evaluation o persistent gastroesophageal re�ux disease–related symptoms in the absence o dysphagia. �. Answer a.
Tis patient has clinical, radiographic, and manometric eatures consistent with achalasia; however, given his age, the rapid onset o his symptoms, and the weight loss, pseudoachalasia due to malignancy needs to be considered and ruled out. An EGD should be perormed now to rule out esophageal or gastric cardia malignancy because his most recent EGD was � year ago (beore the onset o his current symptoms), and an early lesion could have been missed. o reer this patient or any therapy targeted at achalasia, such as botulinum toxin injection into the lower esophageal sphincter or myotomy, would be premature until EGD has been perormed to rule out cancer. I a patient with clinical eatures o pseudoachalasia has negative �ndings on EGD, imaging o the chest may then be considered, especially with a smoking history. A pulmonary or mediastinal malignancy can in�ltrate the lower esophageal sphincter complex and cause pseudoachalasia symptoms; however, this testing should not take place beore another EGD is perormed, allowing direct mucosal inspection. Anticentromere antibodies can be seen in CRES syndrome associated with scleroderma; similar to patients with achalasia, these patients may also have dysphagia to solids and liquids and are at increased risk o esophageal cancer. However, patients with esophageal involvement with scleroderma typically have a decreased lower esophageal sphincter tone, which is the opposite o what is seen in this case. �. Answer b.
Tis patient has a stool osmotic gap (��� − � [�� + ��]) o less than �� mOsm/kg, suggesting a secretory cause o diarrhea. Causes o secretory diarrhea include toxins rom cholera and enterotoxigenic Escherichia coli and peptides produced rom endocrine tumors (vasoactive intestinal peptide). Te distinction between secretory and osmotic diarrhea helps in the differential diagnosis and evaluation o patients with chronic diarrhea. Te � main methods to help distinguish between secretory and osmotic diarrhea are by calculating the stool osmotic gap and assessing the response to asting. Secretory diarrhea will not decrease substantially during a ast, whereas osmotic diarrhea will. Te other answer choices (Whipple disease, celiac sprue, ��
•
chronic pancreatitis, and lactase de�ciency) are causes o osmotic diarrhea and are thereore incorrect. ��. Answer b.
Tis woman has iron de�ciency without evidence o gastrointestinal tract or menstrual blood loss, which suggests malabsorption o iron. Te most common maniestation o celiac disease is iron de�ciency anemia. Iron is mainly absorbed in the duodenum. Celiac disease preerentially affects the proximal small bowel, interering with iron uptake. Tereore, upper endoscopy with small bowel biopsies should be perormed to evaluate or celiac disease. A small bowel series or capsule endoscopy may suggest the diagnosis o celiac disease but does not provide tissue or diagnosis. Positive serologic testing (tissue transglutaminase antibodies) supports the diagnosis o celiac disease but, i negative, does not exclude the diagnosis in this patient with a high pretest probability o celiac disease. In a patient with iron de�ciency and no gastrointestinal tract symptoms, stool evaluation or ova and parasites would be low yield. Furthermore, a parasitic inection (eg, strongyloidiasis) would likely be detected on small bowel biopsy. ��. Answer c.
Te diagnosis o Whipple disease should be suspected with the combination o steatorrhea, weight loss, and migratory arthralgias in a middle-aged white man. Whipple disease may cause central nervous system involvement maniesting with the �nding o oculomasticatory myorhythmia in ��% o patients. Oculomasticatory myorhythmia is pathognomonic or Whipple disease and consists o continuous rhythmic jaw contractions that are synchronous with dissociated pendular vergence oscillations. Whipple disease occurs predominantly in middle-aged white men and is caused by chronic inection with Tropheryma whipplei . In most patients with Whipple disease, the intestinal tract is involved regardless o the presence or absence o gastrointestinal tract symptoms. Tus, the primary diagnostic approach to a patient with clinically suspected Whipple disease is upper endoscopy with mucosal biopsy. Intestinal biopsies show the characteristic �ndings o macrophages with PAS-staining particles, which indicate the presence o T whipplei bacilli. Polymerase chain reaction assays may assist in the detection o T whipplei DNA in the intestinal mucosa. issue transglutaminase antibodies, deamidated gliadin antibodies, and small bowel biopsy with villous atrophy and crypt hyperplasia are characteristic o celiac disease and do not �t this clinical scenario. Duodenal aspirates showing more than ���,��� CFU/mL are diagnostic o small intestinal bacterial overgrowth, which can cause steatorrhea and arthralgias but would not maniest with oculomasticatory myorhythmia. ��. Answer c.
Tis patient has signs and symptoms consistent with IBS. Te only test required or patients who have typical diarrhea-predominant IBS symptoms and no alarm eatures is serologic testing or celiac disease. However, this
MAYO CLINIC INERNAL MEDICIN E BOARD REVIEW: QUESIONS AND ANSWER S
patient has ecal leukocytes. Tis �nding suggests colonic in�ammation and warrants urther investigation with colonoscopy and biopsy to evaluate or in�ammatory bowel disease or microscopic colitis. Reassurance, antidiarrheals, and stool-bulking agents are therapies to consider or the patient with IBS. Riaximin is a nonabsorbable antibiotic used to treat traveler’s diarrhea, recurrent hepatic encephalopathy, and small intestinal bacterial overgrowth. Although riaximin was recently ound to alleviate IBS symptoms, it is not approved by the US Food and Drug Administration or this indication, and this patient with ecal leukocytes requires urther evaluation with colonoscopy. ��. Answer b.
Tis patient has the classic clinical history or superior mesenteric artery embolus. Since the C scan o the abdomen was negative, selective mesenteric angiography should be the next step in management. Mesenteric ischemia should be diagnosed promptly. Te mortality rate exceeds ��%, even i the embolus is removed and all inarcted bowel is resected, because decreased splanchnic blood �ow may initiate persistent and irreversible mesenteric vasoconstriction. ��. Answer c.
Long-term radiotherapy injury to the small bowel may maniest as impaired motility or stricture, as suggested by the patient’s chronic stable symptoms. Patients with impaired motility or stricture are predisposed to small bowel bacterial overgrowth due to stasis. reatment with a course o antibiotics, such as cipro�oxacin, would be the therapy most likely to result in sustained improvement o the diarrhea. ��. Answer c.
Diarrhea afer an ileal resection is usually caused either by bile acid malabsorption or bile salt–induced colonic secretion (resection ≤��� cm) or by bile salt malabsorption with bile salt pool depletion and atty acid–induced colonic secretion (resection >��� cm). Te clinical �nding o worsening diarrhea with treatment with binding resins suggests urther bile salt depletion.
��. Answer e.
Te patient presents with a mildly increasing AL � month afer she began taking simvastatin. She almost certainly has nonalcoholic atty liver disease. It is now well documented that patients with nonalcoholic atty liver disease do not have an increased risk o liver toxicity with statins. Mild increases in aminotranserases afer starting a statin are common but are nearly always transient; consequently, serial monitoring o AL is the correct answer. Patients with nonalcoholic atty liver disease have an increased risk o death, although much o that increase in mortality is due to cardiovascular disease rather than liver disease. Tereore, control o cardiovascular risk actors is an important management issue or these patients. Te amount o alcohol consumed by the patient is irrelevant. Unless there would be other changes in the patient’s clinical condition, liver biopsy is not required but could be considered i there are urther increases in AL. Metormin is a rare cause o elevated liver test results; thereore, metormin may be continued. Simvastatin should be discontinued only i the AL level increases to � to � times the upper limit o the reerence range. (See orres and Harrison, and Chalasani in the “Suggested Reading” list.) ��. Answer a.
Te presence o xanthelasmas and a cholestatic liver pro�le in a woman o this age is highly suggestive o primary biliary cirrhosis (PBC), and antimitochondrial antibody testing should be done. Magnetic resonance cholangiopancreatography would be useul i biliary obstruction were likely. Te absence o pain, history o ulcerative colitis (associated with primary sclerosing cholangitis), or an elevation in the bilirubin level makes biliary obstruction less likely than PBC. Antinuclear and anti–smooth muscle antibodies are used to diagnose autoimmune hepatitis, which elevates predominantly aminotranserases rather than alkaline phosphatase. An angiotensin-converting enzyme level is not necessary in the absence o other clinical eatures o sarcoidosis. (See Kaplan and Gershwin in the “Suggested Reading” list.) ��. Answer d.
��. Answer e.
Tis patient has an indirect hyperbilirubinemia with normal levels o liver enzymes and without evidence o hemolysis. Te most likely diagnosis is Gilbert syndrome, which occurs in about �% o the general population. Further diagnostic workup is unnecessary. Te prior history o illicit drug use puts the patient at risk or hepatitis C and hepatitis B. He should be tested, although it would be unusual or him to have viral hepatitis with normal liver enzyme levels, and Gilbert syndrome is much more common. Patients with alcoholic hepatitis can have hyperbilirubinemia, but they generally have abnormal liver tests, with the AS being higher than the AL. Niacin-induced liver disease is accompanied by elevated liver enzymes. (See Kamath in the “Suggested Reading” list.)
Te serum-ascites albumin gradient is �.�, which suggests portal hypertension. Te �uid protein level greater than �.� g/dL is suggestive o hepatic venous out�ow obstruction. Te hepatic veins are patent, and the most likely cause or the ascites and the other symptoms is heart a ilure, perhaps due to alcoholic cardiomyopathy. Echocardiography should be the next test. ransjugular intrahepatic portosystemic shunt is used to treat reractory ascites associated with cirrhosis. Liver biopsy might be useul later, but it is invasive and should not be used unless noninvasive tests do not lead to a diagnosis. Laparoscopy should be reserved or a suspicion o peritoneal carcinomatosis or inection, both o which would have a serum-ascites albumin gradient less than �.�. (See Runyon et al in the “Suggested Reading” list.)
�. GASROENEROLOGY AND HEPAOLOGY QUESIONS AND ANSWERS
•
��
��. Answer a.
��. Answer c.
Te combination o AL that is persistently abnormal to this degree and hypergammaglobulinemia in a woman with a history o thyroid disease and no other identi�able cause or liver disease is most suggestive o autoimmune hepatitis. Determination o antinuclear and anti–smooth muscle antibodies and a liver biopsy would be the next steps in management. oxicity rom ibuproen is rare, and acetaminophen causes an acute, usually marked, increase in aminotranserases. Alcoholic hepatitis almost never increases aminotranserases to more than ��� U/mL. Gallbladder stones can cause abnormalities in liver ducts either through severe in�ammation or through passage o a common bile duct stone. Te patient does not have clinical eatures o in�ammation, and a common bile duct stone causes transient aminotranserase elevations and severe abdominal pain, neither o which are present in this patient. (See Czaja et al in the “Suggested Reading” list.)
Patients with hepatitis B who need immunosuppressive therapy are at risk or reactivation o disease and should receive hepatitis B treatment. An oral nucleoside or nucleotide analogue, such as lamivudine, is preerred because o the reliable antiviral effect and lack o toxicity. Ideally, hepatitis B treatment is started � weeks beore initiation o chemotherapy and continued or several months afer completion o the lymphoma treatment. Hepatitis B vaccination is not useul i the patient already has hepatitis B. Surveillance or hepatocellular carcinoma (HCC) is advised in the ollowing hepatitis B patients: patients who have cirrhosis, Asian women older than �� years, Asian men older than �� years, Aricans older than �� years, patients with a amily history o HCC, and patients with persistently elevated liver test results and high HBV DNA levels. Tis patient does not meet any o those criteria. (See Lok et al in the “Suggested Reading” list.)
��. Answer b.
Patients with cirrhosis who are admitted with gastrointestinal tract bleeding should receive prophylactic antibiotics even i there is no ascites. Oral nor�oxacin is probably sufficient, although a recent trial suggested that a third-generation cephalosporin may be more effective i there is a high prevalence o quinolone resistance. Much o the recent reduction in mortality related to variceal bleeding is probably due to the increasing use o prophylactic antibiotics in patients with cirrhosis and gastrointestinal tract bleeding. ransjugular intrahepatic portal systemic shunt is not necessary unless bleeding cannot be controlled with endoscopy or medical therapy. Although the hemoglobin level should be careully monitored, the patient is hemodynamically stable and does not need transusion now. Overtransusion, when hemoglobin is greater than � to � g/dL, can precipitate recurrence o bleeding and �uid overload. Fresh rozen plasma is also not necessary since the INR is relatively normal. A decision about pegylated intereron and ribavirin treatment o hepatitis C should be deerred until the patient has recovered rom this bleeding episode. (See Sanyal et al in the “Suggested Reading” list.)
��
•
SUGGESED RE ADING Chalasani N. Statins and hepatotoxicity: ocus on patients with atty liver. Hepatology. 2005 Apr;41(4):690–5. Czaja AJ, Freese DK; American Association or the Study o Liver Disease. Diagnosis and treatment o autoimmune hepatitis. Hepatology. 2002 Aug;36(2):479–97. Kamath PS. Clinical approach to the patient with abnormal liver test results. Mayo Clin Proc. 1996 Nov;71(11):1089–94; quiz 1094–5. Kaplan MM, Gershwin ME. Primary biliary cirrhosis. N Engl J Med. 2005 Sep 22;353(12):1261–73. Erratum in: N Engl J Med. 2006 Jan 19;354(3):313. Lok AS, McMahon BJ; Practice Guidelines Committee, American Association or the Study o Liver Diseases (AASLD). Chronic hepatitis B: update o recommendations. Hepatology. 2004 Mar;39(3):857–61. Runyon BA; Practice Guidelines Committee, American Association or the Study o Liver Diseases (AASLD). Management o adult patients with ascites due to cirrhosis. Hepatology. 2004 Mar;39(3):841–56. Sanyal AJ, Bosch J, Blei A, Arroyo V. Portal hypertension and its complications. Gastroenterology. 2008 May;134(6):1715–28. orres DM, Harrison SA. Diagnosis and therapy o nonalcoholic steatohepatitis. Gastroenterology. 2008 May;134(6):1682–98.
MAYO CLINIC INERNAL MEDICINE BOARD REVIEW: QUESIONS AND ANSWERS
�. PULMONARY DISEASES QUESTIONS AND ANSWERS
QUESIONS
a. Haloperidol b. Lorazepam c. Hydromorphone d. Midazolam e. Fentanyl
Multiple Choice (choose the best answer) CRITICAL CARE MEDICINE
�. A ��-year-old man comes to the emergency department with clumsiness o his right (dominant) hand that began abruptly � hours earlier. He has hypertension, or which he takes a β-blocker. His temperature is ��°C, his heart rate is �� beats per minute, his respirations are �� per minute, and his blood pressure is ���/�� mm Hg. Oxygen saturation as measured by pulse oximetry is ��% with room air. Physical examination reveals an intact sensorium, no obvious cranial nerve de�cits, �/� motor strength in the right upper arm with inability to perorm repetitive hand movements (ataxia), and �/� motor strength in the right lower leg. No sensory de�cits are noted. Emergent computed tomography o the head does not show a stroke. All screening laboratory study results are normal except or a serum cholesterol level o ��� mg/dL. It is now � hours afer onset o symptoms. Which o the ollowing is the de�nitive treatment?
�. A ��-year-old man underwent a radical prostatectomy or prostate cancer �� hours ago. Since hospital admission, he has received a total o �.� L o intravenous isotonic crystalloid and has had a total urine output o �.� L. Te hospital physician on duty examined him earlier this evening or evaluation o substernal chest pain. Te patient’s electrocardiogram (ECG) was normal. He received intravenous urosemide and oral nitroglycerin and had �.� L o urine output. He became symptomatically worse, so he was transerred to the intensive care unit or urther evaluation. When you initially examine the patient, his blood pressure is ���/��� mm Hg, his heart rate is ��� beats per minute, his respiration rate is �� breaths per minute, and his temperature is �� °C. He reports chest pain and shortness o breath. Tere is no jugular vein distention. You hear bibasilar crackles posteriorly and a harsh systolic murmur across the precordium. His extremities are well perused with good pulses. An ECG shows sinus tachycardia and delayed precordial transition, but no other changes are noted. While you wait or results o additional diagnostic studies, which o the ollowing should you administer now?
a. Alteplase b. Aspirin c. Clopidogrel d. Continuous-inusion epti�batide e. Continuous-inusion heparin
a. Furosemide b. Metoprolol c. Nitroglycerin d. Aspirin e. Heparin
�. A ��-year-old man is admitted to the hospital because o acute dyspnea and a syncopal episode during which he ell. He is hypotensive and requires intubation and norepinephrine to maintain adequate blood pressure and oxygenation. A computed tomographic (C) scan o the chest shows a saddle pulmonary embolism, and the patient is administered tissue plasminogen activator (tPA) in the emergency department. An echocardiogram shows right ventricular dilatation and ailure. en hours later, the norepinephrine dosage has been incrementally increased rom �� to �� mcg per minute. Oxygen saturation as measured by pulse oximetry is ��% with a raction o inspired oxygen o �.��. His heart rate is ��� beats per minute, his blood pressure is ���/�� mm Hg, and his central venous pressure is � to � mm Hg. Te hemoglobin level has decreased rom �� g/dL on admission to � g/dL. Activated partial thromboplastin time is �� seconds. A ormal reading o the admission C scan shows lef rib ractures in
�. A ��-year-old man underwent surgical repair and �xation o multiple traumatic orthopedic injuries � days ago. He also has bilateral lung contusions and has required mechanical ventilation. He did not have any head injuries. His condition is improving, and physiologically he is ready to begin being weaned rom the ventilator. His oxygenation is good. However, when the propool inusion is decreased, he maniests a severe, agitated delirium with ventilator dyssynchrony and an inability to ollow commands. Neurologic examination �ndings are otherwise normal. Laboratory test results are normal, computed tomography o the head is normal, and the electrocardiogram is normal. Which o the ollowing should you order now? ��
addition to pulmonary embolism. A subsequent chest radiograph is clear. A subsequent echocardiogram shows a poorly contracting right ventricle. Gastric aspirate is clear, and the stool is negative or occult blood. In addition to transusion, which o the ollowing should you recommend? a. Perorm a C scan o the abdomen. b. Repeat the dose o tPA. c. Stop the heparin and place an inerior vena cava (IVC) �lter. d. Continue heparin and place an IVC �lter. e. Perorm a surgical embolectomy.
�. A ��-year-old man is admitted to the intensive care unit (ICU) with decelerating injury afer a motor vehicle collision. Te patient required on-scene mechanical extrication rom the vehicle. He has bilateral lower extremity ractures that required surgical intervention beore he arrived in the ICU. Tus ar, he has received �� L o crystalloid and � units o packed red blood cells. Over the �rst � hours, his blood pressure and urine output decrease and partially respond to an additional � L o crystalloid. Currently, his blood pressure is ��/�� mm Hg, his heart rate is ��� beats per minute, his respiratory rate is �� breaths per minute, and he is normothermic. Other than his lower extremity injuries, no abnormal �ndings are noted on his examination. His hemoglobin is � g/dL and his coagulation values are normal. His total creatine kinase (CK) is ��� U/L. A computed tomographic scan o the abdomen rom the emergency department is normal. What should you do next? a. b. c. d.
Administer a colloid �uid bolus. Obtain an echocardiogram. Administer methylprednisolone. Perorm a ocused assessment with sonography or trauma (FAS). e. Begin a bicarbonate inusion.
�. Te incidence o transusion-related acute lung injury (RALI) is greatest with transusion o which o the ollowing? a. b. c. d. e.
�.
Fresh rozen plasma Packed red blood cells Pooled platelets Single-donor platelets Salt-poor albumin
Which o the ollowing best describes the effects o critical illness on physiologic sleep? a. Opiate-benzodiazepine combinations promote physiologic sleep in patients receiving mechanical ventilatory support. b. In critical illness, the proportion o rapid eye movement (REM) sleep is decreased. c. In critical illness, sleep has predominant waveorms consistent with deep sleep. d. Te total duration o sleep during a ��-hour period is increased. e. Te total duration o sleep during a ��-hour period is markedly decreased.
��
•
D I F F U S E L U N G D I S E A S E A N D O C C U P AT I O N A L LUNG DISEA SE
�. A ��-year-old emale nonsmoker presents with a �-week history o mild cough and dyspnea afer a �ulike illness with ever, arthralgias, and tender erythematous lesions on the anterior aspects o the legs. She has no history o asthma or signi�cant medical illnesses. No environmental or occupational high-risk exposures are noted. Examination reveals clear lung �elds and no other abnormalities. A chest radiograph shows prominent bilateral hilar lymphadenopathy without parenchymal in�ltrates. What should you do next ? a. b. c. d. e.
Set up blood cultures. Perorm human immunode�ciency virus serology testing. Determine the erythrocyte sedimentation rate. Perorm Lyme serology testing. Observe and repeat the chest radiograph in �� weeks.
�. A ��-year-old man, a ormer smoker, presents with a �-year history o progressive dry cough and dyspnea. He has no extrapulmonary symptoms. No occupational or environmental exposures are noted. Findings on examination include bibasilar coarse rales and digital clubbing. A chest radiograph shows prominent interstitial in�ltrates in the middle and lower lung �elds. Te antinuclear antibody titer is borderline elevated at �:��. Serum protein electrophoresis shows a polyclonal gammopathy. Te rheumatoid actor titer is also borderline elevated at �:��. A high-resolution com puted tomographic (C) scan o the chest shows sub pleural honeycombing with thickened alveolar septa in both lower lobes with bilateral mediastinal �.�-cm lymph nodes. No ground-glass opacities are present. Which treatment is most likely to result in clinical improvement? a. b. c. d. e.
Azathioprine Systemic corticosteroids Cyclophosphamide Systemic corticosteroids with azathioprine No treatment
��. A ��-year-old man, a current smoker (�� pack-years), is examined or acute dyspnea and right-sided chest pain. He denies having ever, chills, sweats, cough, sputum production, or hemoptysis. On auscultation o the lungs, diminished breath sounds are heard throughout, with more on the right than the lef. Te chest radiograph and computed tomographic scan o the chest show scattered interstitial changes with cystic and nodular abnormalities, which are more prominent in the mid and upper lung zones, and a right-sided pneumothorax. What is the most likely diagnosis? a. Lymphangioleiomyomatosis b. Pulmonary Langerhans cell histiocytosis (histiocytosis X) c. Cystic �brosis d. Aspiration pneumonia e. Idiopathic pulmonary �brosis
MAYO CLINIC INERNAL MEDICINE BOARD REVIEW: QUESIONS AND ANSWERS
��. A ��-year-old man, a ormer smoker, presents with a �- to ��-month history o dry cough and dyspnea. He denies ever, chest pain, or hemoptysis. No re�ux or dysphagia is reported. His past history is signi�cant or diabetes mellitus, hyperlipidemia, and benign prostatic hypertrophy. No known signi�cant hobby, travel, or environmental exposures are noted. Past occupational history includes ship reurbishing while in the armed services. Medications include pravastatin, glyburide, and prazosin. His vital signs are stable, and he is aebrile. No adenopathy is noted. Bibasilar crackles are present. Te heart rhythm is regular. His abdomen is normal on palpation and auscultation, and no edema is noted. Clubbing is present. Blood test results are normal. Pulmonary unction testing indicates mild restriction and a low diffusing capacity. A chest radiograph and high-resolution computed tomographic scan o the chest show bilateral lower lobe in�ltrates along with some honeycombing, pleural thickening, diaphragmatic calci�cation, and an area o consolidation in the right lower lobe consistent with rounded atelectasis. What should you do next? a. b. c. d. e.
Begin systemic corticosteroids. Begin azathioprine. Begin methotrexate. Begin cyclophosphamide. Observe only.
��. A ��-year-old man who was previously healthy presents to the emergency department with an acute onset o shortness o breath. He denies having chest pain but reports a signi�cant nonproductive cough and generalized malaise with myalgias that have developed over the past ew days. He has been ebrile, with temperatures up to ��.�°C, but without chills. He takes no medication (including over-the-counter or herbal medications) and has never had lung or heart problems beore. A day beore the onset o symptoms, he attended a graduation party and smoked several cigars with his riends, none o whom got ill aferward. He has severe hypoxemia with increased work o breathing and is admitted to the intensive care unit. Shortly afer admission, he requires endotracheal intubation, and mechanical ventilation is initiated. A chest radiograph showed diuse alveolar in�ltrates. Tere was no pleural effusion, and the heart size was normal. Results o the ollowing blood tests were normal: a complete blood cell count and differential count, electrolyte levels, and a coagulation pro�le. Results o human immunode�ciency virus testing and toxicology screening were negative. He has no relevant travel history. Arterial blood gas results indicated proound hypoxemia with normal pH and Pa���. Immediately afer intubation (and beore initiation o broad-spectrum antibiotics), bronchoalveolar lavage was perormed. Results with Gram, ungal, and microbiological stains were negative. Te differential leukocyte count showed ��% eosinophils. What is the most appropriate next step?
a. Continue cefriaxone and levo�oxacin or severe community-acquired pneumonia. b. Add vancomycin or linezolid or community-acquired methicillin-resistant Staphylococcus aureus inection. c. Check stool samples or ova and parasites. d. Repeat bronchoscopy with transbronchial lung biopsies. e. Start treatment with corticosteroids.
��. A ��-year-old man who has never smoked and who has a past medical history o gastroesophageal re�ux disease and hypertension presented or recurrent pneumonias. Approximately � months beore presentation, he received a diagnosis o community-acquired pneumonia and was treated with macrolide antibiotics with good resolution o his symptoms. He has since had recurrent similar episodes o �ulike illness characterized by the acute onset o ever, cough, and occasional myalgias and arthralgias. Tese episodes last � to � weeks and generally seem to respond to short courses o antibiotics. Results o human immunode�ciency virus testing and toxicology screening were negative. Chest radiographs during those episodes showed multilobar alveolar opacities, mostly peripheral, without evidence o pleural effusion. Te precise location o these in�ltrates seems to vary over time without predilection or particular lobes. Findings rom all microbiological studies have been repeatedly negative, including multiple blood cultures, urinary antigens or Streptococcus pneumoniae and Legionella pneumophila, and sputum ungal and mycobacterial cultures. A computed tomographic scan o the chest shows triangular-shaped, pleural-based in�ltrates without evidence o an endobronchial lesion, an abscess, or pleural effusion. Some o these in�ltrates are characterized by ground-glass attenuation surrounded by more consolidated opacities. Bronchoscopy with biopsies and bronchoalveolar lavage show lymphocytic predominance without evidence o inection, and biopsies show the presence o plugs o granulation tissue within the alveolar spaces. Which o the ollowing is true? a. Te location o the in�ltrates suggests septic emboli and the need or an echocardiogram. b. Te presentation is typical o tuberculosis, and the patient should be treated with antituberculous medications. c. A trial o corticosteroids without histologic con�rmation is appropriate. d. Aspiration is the likely cause, and a proton pump inhibitor should be prescribed. e. A bronchogenic carcinoma should be excluded with surgical lung biopsy.
��. An ��-year-old woman who has never smoked is admitted or the subacute onset o shortness o breath that limits her daily activities. Her past medical history is signi�cant or gastroesophageal re�ux disease, osteoporosis, hypertension, and recurrent urinary tract inections. Her usual treatment includes lisinopril, aspirin, omeprazole, alendronate, and daily nitrourantoin.
�. PULMONARY DISEASES QUESIONS AND ANSWERS
•
��
She denies having cough or sputum production but has noticed tingling and numbness in the lower extremities. She denies having a rash or photosensitivity, arthritis, and Raynaud phenomenon. Her chest radiograph shows reticular opacities preerentially located in the bases o the lungs bilaterally, and a computed tomographic scan o her chest con�rms the presence o dense �brotic-appearing �brotic-appeari ng changes in both bases without pleural effusions. Results o the ollowing blood tests were unremarkable: unre markable: complete blood cell count and differential count, electrolytes, and coagulation pro�le. Tere is nothing to suggest aspiration. At this point, what is the best next step? a. Proceed with bronchoscopy bronchoscopy with bronchoalveolar bronchoalveolar lavage and transbronchial biopsies. b. Initiate treatment with broad-spe broad-spectrum ctrum antibiotics antibiotics.. c. Discontin Discontinue ue use o nitrourantoin. nitrourantoin. d. Reer the patient or lung transpla transplant nt since no treatment has proved effective or or this disease. e. Start treatment with corticosteroi corticosteroids. ds. VA S C U L A R D I S E A S E , E M B O L I S M , A N D HYPERTENSION
��. A ��-year-old ��-year- old woman who had a stroke � months ago presentss or evaluation. present ev aluation. Her Her initial symptoms included right upper extremity weakness that has completely resolved. Magnetic resonance imaging o the brain shows no underlying pathology other than the inarct. As part o her evaluation, she underwent carotid ultrasonography, which was normal, and transthoracic echocardiography with a bubble study, which was suggestive o a right-to-lef shunt. Physical examination �ndings were as ollows: temperature ��.� °C, pulse rate �� beats per p er minute, respirations respirations �� per minute, blood pressure ���/�� mm Hg, and oxygen saturati saturation on with room air ��%. Lip and �nger telangiectases were noted. What test should be perormed next? next? a. Cerebral angiography b. Positro Positronn emission tomography or computed computed tomographic (C) scan c. C o the chest d. ransesophageal echocardiography e. Electroencephalography e.
��. A ��-year-old ��-year-o ld woman presents with dyspnea on exertion and right pleural effusion over the past � years. Cardiac catheterization �ndings were normal. Te patientt uses diuretic patien diuretics; s; however however,, her lower extremit extremities ies have continued to swell. She has had � thoracenteses over the years or symptomatic relie. Physical examination �ndings are as ollows: temperature ��.� °C, pulse rate �� beats per minute, respirations �� per minute, and blood pressure ���/�� mm Hg. Dullness to percussion and diminished breath sounds are apparent in the right lung base. Heart examination �ndings and jugular venous pressure are normal. She has pretibial edema (�+). Tere is no clubbing; however, her nails are short ��
•
and discolored with onycholysis. Laboratory �ndings include the ollowing: hematocri hematocritt ��%, leukocyte count � ��× �� ×�� /L, serum creatinine �.� mg/dL, and serum calcium �.� mg/dL. Chest radiography shows right pleural effusion and a normal heart size. On diagnostic right thoracentesis, thoracen tesis, there is serosanguineous �uid with a lactate dehydrogenase (LDH) level o ��� U/L. What is the most likely diagnosis? a. b. c. d. e.
Congestive heart ailure Malignant pleural effusion Connectivee tissue disease Connectiv Yellow nail syndrome Asbestos-related pleural effusion
��. A ��-year-old man with alcoho alcoholic lic cirrhosis and portal hypertension presents with progressive dyspnea over the past year. He denies having chest pain, lower extremity edema, ascites, or ever. On physical examination, his pulse rate is ��� beats per minute, his blood pressure is ��/�� mm Hg, and his temperatu temperature re is ��.�°C. Auscultation o the heart and lungs is normal, and there is no edema. Oxygen saturation in the seated position is ��% with room air air.. Laboratory �ndings include the ollowing: hemoglobin �� g/dL, leukocyte count �.�× �.�×���/L, serum sodium ��� mEq/L, serum potassium �.� mEq/L, and serum creati creatinine nine �.� mg/ dL. Computed tomography o the chest is negative or pulmonary embolism, and pulmonary unction testing testing is normal with the exception o the diffusion capacity, which is ��% o the the predicted value. Which test test should be perormed next to help determine the underlying diagnosis? a. b. c. d. e.
Pulmonary angiography ransthoraci ransthoracicc contrast echoca echocardiogra rdiography phy Cardiopulmonary Cardiopulmon ary exercise exercise testing Cardiac catheterization Overnight oximetry
��.. A ��-yea �� ��-year-old r-old man with nonalcoho nonalcoholic lic steatohepatitis and portal hypertensi hypertension on presents with progressive dys pnea on exertion, chest tightness, and lower extremity edema. On physical examination, his pulse rate is ��� beats per minute, his blood pressure is ��/�� mm Hg, and his temperature is ��.�° ��.� °C. Auscultation o the lungs is normal. Jugular venous pressure is elevated at � cm water,, and cardiac auscultat water auscultation ion identi identi�es �es an accentu accentuated ated pulmonic valve component (P�) and a right ventricular heave. Te liver eels pulsatile with positive hepatojugular re�ux. Laboratory �ndings include the ollowing: hemoglobin concentration ��.� g/dL, leukocyte count �.�× �.� ×���/L, serum sodium so dium �� ��� mEq/L, serum potassium po tassium � mEq/L, serum creatinine �.� mg/dL, and N -terminal -terminal B-type natriuretic peptide precursor (N-pro-BNP) �,��� pg/mL. ransthoracic echocardiography shows a dilated inerior vena cava with no inspiratory collapse and an estimated right ventricular systolic pressure o �� mm Hg. His right ventricle is moderately enlarged with reduced reduced systolic unction and and a D-shaped lef ventricle. On right heart catheterization, pulmonary artery
MAYO CLINIC INERNAL MEDICIN E BOARD REVIEW: REVIEW: QUESIONS AND ANSWER S
pressure is ��/�� mm Hg, pulmonary artery arter y occlusion pressure is is �� �� mm Hg, and cardiac output output is �.� �.� L/min. What is the most likely likely diagnosis? a. b. c. d. e.
Idiopathic pulmonary artery artery hypertension hypertension Diastolic dysunction Valvular heart disease Portopulmonary Portopulmon ary hypertension hypertension Pulmonary embolism
PULMONARY SIGNS AND SYMPTOMS AND CHEST RADIOGRAPHY
��. An ��-year-old ��-year- old man with moderately severe chronic obstructive pulmonary disease (COPD) present presentss to the emergency department again or progressively worsenworsening dyspnea and an increasingly productive cough. He lives alone. Most recently, he was dismissed rom the hospital � weeks ago afer treatment o COPD exacerbations. His initial vital signs include oxygen saturation o ��% with room air, respiratory rate �� breaths per minute, blood blo od pressure ���/�� ���/�� mm Hg, and pulse ��� beats per minute. Afer nebulizer treatment with albuterol and ipratropium, his respiratory rate improves to �� breaths per minute and his oxygen saturation is ��% on room air. Which o the ollowing is true about urther management o COPD or this patient? a. He should be given oral corticosteroids and dismissed to home with ollowollow-up up with with his prim primary ary care physic physician ian in � days. b. He should begin receiving tiotropium and dismissed to b. home with ollow-up with his primary care physician in � days. c. He should be hospitalized hospitalized or urther management management o his COPD exacerbation. d. He should be intubated intubated and admitted to the the intensive intensive care unit. e. Afer another treatment with bronchodilators, he should be dismissed and receive a home visit by a nurse tomorrow tomorrow.
��. A ��-year-old man who had been healthy without signi�cant past medical history is admitted to your intensive care unit with severe shock. Endotracheal intubation was perormed beore he arrived. Te initial chest radiograph is shown in Figure �.Q��A. Even afer receiving several liters o �uid and vasopressors, he remains hypotensive. You decide to place a pulmonary artery (P ( PA) catheter. catheter. Te procedure goes smoothly and the vessel is cannulated at �rst pass. You request another chest radiograph to con�rm placement placement (Figure �.Q��B). Which o the ollowing is the cause or the �nding on the second radiograph? a. Vessel injury resulting in hemothorax b. Lobar collapse due to mucous plug c. Pneumothorax c. d. Hemopneumothorax d. e. Pulmonary inarct due to overwedging o the the PA PA catheter catheter
Figure �.Q��A
��. A ��-year-old woman presents presents with progressive progressive dyspnea at rest. Chest radiography shows signi�cant lef-sided effusion. Results o the thoracentesis and blood tests are shown in able �.Q��. Table �.Q�� C O MP O N E N
S E RU M
H O R AC E N E S I S F LU ID
Protein, g/dL
�.�
�.�
Lactate dehydrogenase, U/L
���
���
pH
…
�.�
Which o the ollowing is not a a possible cause or the pleural �uid? a. Pulmonary embolism b. Empyema b. c. Rheumatoid effusion d. uberculosis d. e. Malignancy e.
Figure �.Q��B
�. PULMONARY DISEASES QUESIONS AND ANSWERS
•
��
��. A �� ��-year-year-old old man presents with dyspnea. Pulmonary Pulmonar y unction test (PF) results are shown in able able �.Q��. Table �.Q�� VA LU E
PERCENAGE OF PREDICED VALUE
otal lung capacity, L
�.��
��
Residual volume, L
�.��
���
Forced vital capacity (FVC), L
�.��
��
Forced expiratory volume in the �rst second o expiration (FEV�), L
�.��
��
C O MP O N E N
FEV�/F VC
��.�
…
Diffusing capacity o lung or carbon monoxide, mL·min−�·mm Hg−�
��.�
��
Te results may be suggestive o which o the ollowing? a. a. b. b. c. d. e.
Asthma Obesity Chronic obstructive pulmonary disease Severe pulmonary hypertension hypertension Idiopathic pulmonary �brosis
��. A ��-year-o ��-year-old ld man with severe chronic obstructive pulmonary disease (COPD) presents or ollow-up afer hospitalization. He was admitted with respiratory ailure and COPD exacerbati ex acerbation on or the second time in the past � months. He eels that his dyspnea is at baseline. Which o the ollowing has not been been shown to reduce exacerbation or rehospitalization rates?
��
•
a. iotropium a. b. Salmeterol b. c. Salmeterol with �uticasone d. Long-term oxygen therapy e. N -acetylcysteine -acetylcysteine
��. A ��-year-old woman comes come s to your yo ur office with her inant because she is concerned about her recurrent sinusitis. Although she has never been hospitalized, her younger brother was hospitalized or recurrent pancreatitis. pancreat itis. She describes a chronic productive cough with dyspnea. Examinatio Ex amination n reveals wheezing and digital clubbing. What is the recommended initial test to diagnose this disorder? a. b. c. d.
No urther testing Sweat chlorid chloridee testing esting or a CFTR genetic mutation Computed tomographic tomographic (C) scan o the chest with an intravenous contrast agent e. C scan o the sinuses ollowed ollowed by magnetic resonance imaging o the brain i abnormalities are detected
��. A �� ��-year-year-old old man presents or excessive sleepiness and difficulty unctioning at his work. work . His wie wi e has comcom plained o very disruptive snoring and has witnessed requent apneic episodes. His body mass index is ��. His blood pressure is ���/�� mm Hg. His neck size is �� cm. Which o the ollowing tests should be perormed next to con�rm your suspicion? a. b. c. d. e.
Overnight pulse oximetry Use o a ��-hour ambulatory ambulatory blood pressure monitor monitor Overnight polysomnography Carotid Carot id duplex duplex ultrasonography ultrasonography Adrenal imaging with with computed computed tomography tomography
MAYO CLINIC INERNAL MEDICIN E BOARD REVIEW: REVIEW: QUESIONS AND ANSWER S
ANSWERS
�. Answer b.
mechanism o accident injury and his elevated CK. A bedside echocardiogram can quickly con�rm this diagnosis. Te other answer choices are less plausible.
Tis patient has hypertrophic cardiomyopathy with out�ow obstruction that is exacerbated by intravascular volume contraction. His diagnosis is suggested by �) the murmur, �) tachycardia (shortened lef ventricular diastolic �lling time), �) lung crackles, �) absence o jugular vein distention, and �) worsening with urosemide-induced diuresis. His �uid output and input are matched since admission, but his surgical procedure is associated with increased third-space �uid losses. Tereore, his intravascular volume is likely depleted. O the answer choices given and the various differential diagnostic possibilities (congestive heart ailure, myocardial inarction, pulmonary embolism, and methemoglobinemia), only metoprolol will improve his out�ow obstruction.
�. Answer c.
Te risk o RALI is related to the potential number o exposures to plasma rom different sources. Te greatest risk occurs when multiple pooled sources are used, as in platelet transusions. Te transusion o pooled products rom women, in particular resh rozen plasma, has also been associated with an increased risk o RALI. (See Gajic et al and oy et al in the “Suggested Reading” list.) �. Answer b.
In critical illness, disrupted and ragmented sleep leads to impaired cognitive unction. Opiates and benzodiazepines alter normal sleep architecture, but the total duration o physiologic sleep during a ��-hour period has been ound to be normal. Sleep patterns in critical illness have a predominance o super�cial stages N� and N� and a low pro portion o slow-wave deep sleep (stage N�). REM sleep is severely decreased or absent. (See Cooper et al and Friese et al in the “Suggested Reading” list.)
�. Answer a.
Tis patient has an acute, agitated delirium. Benzodiazepines are not indicated; they may worsen delirium and merely suppress its maniestations. Similarly, this is not a pain syndrome, and opioids will suppress but not treat the underlying pathophysiology. �. Answer a.
Emerging data suggest that the interval o time rom symptom onset to administration o thrombolytic therapy can be prolonged in selected patients with acute ischemic stroke, including patients not at increased risk o bleeding. Tese patients had improved survival i they received thrombolytic therapy up to �.� hours afer onset o symptoms. (See Hacke et al in the “Suggested Reading” list.)
�. Answer e.
Sarcoidosis is a granulomatous disease most ofen affecting the lungs and lymph nodes. It can occur afer a �ulike illness and may be diagnosed by a speci�c constellation o symptoms and signs when presenting with L ögren syndrome (erythematous nodosum, bilateral hilar lymphadenopathy, ever, and polyarthritis). In most other instances, a diagnosis o sarcoidosis requires a compatible history, �ndings o noncaseating granulomas by biopsy, and exclusion o other possible causes o granulomatous in�ammation. I there is systemic involvement, blood work may show abnormalities, including hypercalcemia, anemia, and elevated liver enzymes. Serum angiotensin-converting enzyme levels are neither speci�c nor sensitive to use as a diagnostic tool but, when elevated, may be helpul or ollowing disease activity. Bronchoscopy can con�rm granulomatous disease in over ��% o patients with hilar adenopathy and parenchymal lung involvement. Rales are uncommon in sarcoidosis even when parenchymal interstitial changes are present. Incidence, clinical course, and prognosis o sarcoidosis are in�uenced by ethnic and genetic actors. Computed tomographic scan may show nodular opacities with bronchovascular and subpleural distribution, thickened intralobular
�. Answer a.
Tis patient presents with a massive pulmonary embolism and shock. Trombolysis or embolectomy is indicated. Te lack o improvement re�ects either ailure o primary thrombolysis or development o a new complication. Te central venous pressure is much lower than would be expected in a patient with pulmonary embolism and acute right heart ailure. Te rib ractures and decreased hemoglobin suggest an occult site o bleeding, potentially in the abdomen or retro peritoneum, which needs to be identi�ed beore a decision is made on possible rescue therapy or the pulmonary embolism. (See Meneveau et al in the “Suggested Reading” list.) �. Answer b.
Tis patient has myocardial contusion afer trauma, with possible lef ventricular ailure. Tis is suggested by the ��
septa, architectural distortion, or conglomerate masses (late stage). obacco use has not been associated with development o sarcoidosis. Extrapulmonary involvement rom sarcoidosis may involve the heart, liver, spleen, eyes, bone, skin, bone marrow, parotid glands, pituitary gland, and reproductive organs. Tis patient’s presentation is most consistent with Lögren syndrome, which carries a very good prognosis; symptoms resolve without treatment. Tus, observation with ollow-up chest radiography is appropriate. I symptoms are more bothersome, symptomatic treatment such as nonsteroidal anti-in�ammatory agents may be considered. For progressive pulmonary and extrapulmonary disease, corticosteroids or immunosup pressive therapy should be considered. �. Answer e.
Te combination o interstitial lung in�ltrates predominantly involving the lower lung zones, lack o occupational exposure, duration o symptoms, and peripheral honeycombing make the diagnosis o idiopathic pulmonary �brosis (IPF) most likely. Favorable prognostic actors in IPF–usual interstitial pneumonia include age younger than ��, emale sex, shorter duration o symptoms beore presentation, presence o ground-glass opacities on C scan o the chest, and lymphocytosis on examination o bronchoalveolar lavage �uid. Pulmonary unction tests in IPF usually indicate restrictive impairment. Patients with IPF generally do not respond to corticosteroids or other immunosuppressive therapies. No clearly effective treatment options are currently available. Oxygen extends survival among patients with chronic obstructive pulmonary disease, but this bene�t has not been shown or IPF patients. Familial clusters o IPF patients suggest a potential genetic predisposition in some cases o IPF. ��. Answer b.
Tis patient’s presentation is most consistent with adult pulmonary Langerhans cell histiocytosis, which, in most cases, is a orm o smoking-related interstitial lung disease. Smoking cessation is the primary orm o treatment. Stabilization or improvement occurs in up to two-thirds o patients with smoking cessation alone. Other therapies including systemic corticosteroids and immunosuppressives have been used with limited success. No role o plasmapheresis has been described. ��. Answer e.
Asbestosis typically has a basilar predominance, and rounded atelectasis is suggestive o asbestos exposure. Known or remembered histories o occupational asbestos exposure may not always be present, but they can be helpul. Common �ndings include pleural plaques or diaphragmatic calci�cations (or both). Malignant mesothelioma is strongly associated with asbestos exposure but not smoking. Pleural suraces are generally abnormal and involved with asbestos-related parenchymal lung disease. However, pleural and parenchymal abnormalities may, in some instances, occur independently o each other. Asbestos ��
•
�bers typically are dormant or decades beore pulmonary �brosis develops. A clinical response to corticosteroids or other medication is not expected in the treatment o asbestos-related pulmonary �brosis. Smoking in the presence o asbestos-related pulmonary �brosis increases the rate o progression o �brosis and the risk o bronchogenic carcinoma. uberculosis is not a common complication o asbestosis. No therapy has been effective in preventing progressive pulmonary �brosis due to asbestos exposure. ��. Answer e.
Acute eosinophilic pneumonia is a rare but increasingly recognized cause o acute respiratory distress syndrome. Te pathophysiology is unclear, but a recent onset o smoking and an increase in smoking patterns have been recognized as common precipitating actors. Clinically, acute eosino philic pneumonia is characterized by the acute onset o diuse alveolar in�ltrates with pulmonary eosinophilia; unlike in chronic eosinophilic pneumonia, peripheral eosinophilia is rare. Radiologically, the in�ltrates are diffuse but nonspeci�c, and treatment with corticosteroids generally results in dramatic improvement. Bronchoscopic lung biopsies are not generally needed or the diagnosis. Other causes o pulmonary eosinophilia include allergic bronchopulmonary aspergillosis, Churg-Strauss syndrome, parasitic inections, some ungal inections, drug-induced lung diseases, chronic eosinophilic pneumonia, and, rarely, connective tissue disease–related interstitial lung disease and malignancies. ��. Answer c.
Te likely diagnosis is cryptogenic organizing pneumonia (ormerly known as idiopathic bronchiolitis obliterans with organizing pneumonia, or BOOP). Organizing pneumonia is characterized pathologically by the presence o plugs o �broblasts and myo�broblasts occupying the distal airspaces without disruption o the underlying architecture o the lung. Cryptogenic organizing pneumonia typically maniests with recurrent or persistent episodes o pneumonia-like illness that occasionally responds to antibiotics, particularly macrolides. Te treatment consists o a prolonged course o corticosteroids (≥� months), and the absence o response to treatment should suggest an underlying cause such as hematologic malignancy, drug-induced lung disease, or connective tissue disease (ie, secondary organizing pneumonia). While usually very effective, rebound afer discontinuation o treatment is requent. ��. Answer c.
Te possibility o drug-induced lung disease should always be considered when evaluating a case o diffuse parenchymal lung disease. Since nitrourantoin is a common offender (with bleomycin, methotrexate, and amiodarone), its use should be careully investigated because patients may not always volunteer that inormation. Nitrourantoin lung toxicity can maniest either as an acute orm o lung toxicity (when it is usually associated with peripheral eosinophilia and elevated in�ammatory markers) or, less requently (as in this case), as a chronic orm o �brotic lung disease.
MAYO CLINIC INERNAL MEDICINE BOARD REVIEW: QUESIONS AND ANSWERS
Discontinuation o the drug is warranted in both cases and should take precedence over any other intervention. Te role o corticosteroids, sometimes recommended or the acute orm o the disease, is less clear in chronic cases, which may progress and even be atal in up to ��% o patients. ��. Answer c.
Tis young patient has lip and �nger telangiectases, an inappropriately low oxygen saturation, and evidence o a right-to-lef shunt on bubble echocardiography. Her stroke was likely caused by paradoxical embolism, and her low oxygen saturation suggests a pulmonary arteriovenous malormation, so that C o the chest is the best choice. With the telangiectases, the shunt, and the suggestion o a pulmonary arteriovenous malormation, the most likely diagnosis is hereditary hemorrhagic telangiectasia. Te clinical criteria or this diagnosis include telangiectases (lips, mouth, nose, and �ngertips), visceral arteriovenous malormation, epistaxis, and amily history o any o the preceding criteria. Stroke in a young person should prompt one to think o pulmonary arteriovenous malormation. ��. Answer d.
Tis patient presents with dyspnea, recurrent right pleural effusion, lower extremity edema, and discolored, dystrophic nails. Her thoracentesis �uid is exudative, with an LDH o ��� U/L. Te most likely diagnosis is yellow nail syndrome, which consists o the triad o yellow nails, lym phedema, and respiratory tract illness (pleural effusions, bronchiectasis, and recurrent pneumonias). Te nails usually do not grow, and patients will wonder why they do not have to cut their nails. ��. Answer b.
Tis man has alcoholic cirrhosis, portal hypertension, and hypoxemia. Te most likely diagnosis is hepatopulmonary syndrome. Te clinical triad in the diagnosis is �) presence o liver disease with portal hypertension, �) intrapulmonary shunting, and �) hypoxemia. Te transthoracic contrast echocardiogram will show the presence o intra pulmonary shunting by the passage o bubbles into the lef heart afer � to � cardiac cycles. Te polycythemia is due to the chronic hypoxemia, which is present at rest and worsens with exertion. Liver transplant is the treatment o choice i the patient is a surgical candidate otherwise. ��. Answer d.
Tis patient has severe pulmonary artery hypertension with liver disease and portal hypertension. Te most likely diagnosis is portopulmonary hypertension because o the ollowing criteria: �) liver disease or portal hypertension, �) mean pulmonary artery pressure greater than �� mm Hg, �) pulmonary vascular resistance greater than ��� dynes ·s·cm−�, and �) pulmonary artery occlusion pressure less than �� mm Hg. Liver transplant or this patient is contraindicated, and he should be considered or vasodilator therapy to try to control the pulmonary artery hypertension.
��. Answer c.
Tis patient has multiple risk actors requiring in-patient management o his COPD exacerbation. As noted by the Global Initiative or Chronic Obstructive Lung Disease (GOLD), indications or hospital admission or COPD exacerbation include the ollowing: marked increase in intensity o symptoms, severe underlying disease, onset o new physical signs, ailure o exacerbation to respond to initial medical management, signi�cant comorbidities, newly occurring arrhythmias, diagnostic uncertainty, requent exacerbations, older age, and insufficient home support. Although the patient’s symptoms improved some what initially, with the persistence o abnormal vital signs and other risk actors noted above, he should be admitted or urther management. (See Rabe et al in the “Suggested Reading” list.) ��. Answer a.
Pleural luid analysis and the ratio o pleural luid lactate dehydrogenase (LDH) to serum LDH (���:���) show that the luid is an exudate. For luid to be considered an exudate, the luid needs to meet only � o the Light criteria (ratio o pleural luid protein to serum protein >�.�, ratio o pleural luid LDH to serum LDH >�.�, pleural luid LDH greater than two-thirds o the upper limit o the reerence range or serum LDH). While the protein ratio is less than �.�, the LDH ratio is greater than �.�; thus, this luid is an exudate. Furthermore, the low pH suggests a certain diagnosis. When thoracentesis luid pH is less than �.�, diagnostic possibilities include empyema, esophageal rupture, rheumatoid arthritis, trauma, tuberculosis, and malignancy. Pleural luid in pulmonary embolism may be either a transudate or an exudate, but pH o the pleural luid should not be severely acidic. ��. Answer b.
While all the answer choices are potential complications o PA catheter placement, careul inspection o the radiograph shows that lobar collapse is the best answer. Note the elevation o the minor �ssure and the elevation o the right hemidiaphragm. Tese are characteristic �ndings o a lobar collapse. Hemothorax should not be limited to the upper lobes only; rather, �uid would accumulate in the lower portions o the chest, thereby blunting the costo phrenic angle. Pneumothorax should result in collapse o the lower lobes as well in this previously healthy patient. Furthermore, pneumothorax should create an air interace and thus appear black, not white, on the radiograph. A hemopneumothorax should create an air-liquid interace along the lateral margin o the lung, which is not apparent in this radiograph. Te region o involvement would be too large or pulmonary inarct due to overwedging. Tis patient underwent bronchoscopic clearing o the airways; the chest radiograph �� minutes later is shown in Figure �.A��.
�. PULMONARY DISEASES QUESIONS AND ANSWERS
•
��
is recommended. (See Farrell et al and McMullen et al in the “Suggested Reading” list.) ��. Answer c.
Tis patient has all the risk actors or at least moderately severe obstructive sleep apnea (OSA). Tereore, overnight polysomnography should be perormed next to con�rm the diagnosis. I OSA is con�rmed, treatment should begin with a continuous positive airway pressure device. Overnight oximetry is occasionally used as a screening tool, but the results are not sufficient to establish the diagnosis even though they may be suggestive o OSA. Other tests have no role in establishing the diagnosis o OSA. SUGGESED RE ADING Figure �.A��
��. Answer e.
PF results show a moderately severe restrictive deect with a markedly reduced diffusing capacity o lung or carbon monoxide (D���). O all the options, only idiopathic pulmonary �brosis �ts the PF �ndings. A restrictive deect would not be expected in asthma or chronic obstructive pulmonary disease. Obesity may result in a restrictive lung deect, but the D��� is ofen normal to elevated. While the D��� may be reduced in severe pulmonary hypertension, lung volumes should not be reduced. ��. Answer d.
Several studies have examined readmission rates and measures that mitigate them. iotropium reduced exacerbations, among other outcomes, during a �-year trial. Another trial compared the effect o salmeterol alone, �uticasone alone, or the � drugs in combination. Exacerbation rates were signi�cantly reduced in all groups compared with placebo, and combination therapy reduced hospitalization rates. Furthermore, use o N -acetylcysteine reduced the risk o readmissions in a study o �,��� patients. While long-term oxygen therapy reduces overall mortality, its use was associated with increased risk o readmissions in those with moderate-to-severe COPD. (See Calverley et al, Gerrits et al, Gonzalez et al, and ashkin et al in the “Suggested Reading” list.) ��. Answer b.
Tis patient has recurrent sinusitis, wheezing, digital clubbing, and a amily member with recurrent pancreatitis. Cystic �brosis must be considered as the underlying disorder. Women with cystic �brosis can be ertile i they have adequate nutritional and pulmonary reserve. In contrast, men ofen present with azoospermia. Te Cystic Fibrosis Foundation recommends sweat chloride testing as the initial diagnostic test. I the sweat chloride concentration is more than �� mEq/L, the diagnosis o cystic �brosis is con�rmed; i it is �� to �� mEq/L, testing or CFTR mutations ��
•
Calverley PM, Anderson JA, Celli B, Ferguson G, Jenkins C, Jones PW, et al; ORCH investigators. Salmeterol and �uticasone propionate and survival in chronic obstructive pulmonary disease. N Engl J Med. 2007 Feb 22;356(8):775–89. Cooper AB, Tornley KS, Young GB, Slutsky AS, Stewart E, Hanly PJ. Sleep in critically ill patients requiring mechanical ventilation. Chest. 2000 Mar;117(3):809–18. Erratum in: Chest 2001 Mar;119(3):993. Farrell PM, Rosenstein BJ, White B, Accurso FJ, Castellani C, Cutting GR, et al; Cystic Fibrosis Foundation. Guidelines or diagnosis o cystic �brosis in newborns through older adults: Cystic Fibrosis Foundation consensus report. J Pediatr. 2008 Aug;153(2):S4-S14. Friese RS, Diaz-Arrastia R, McBride D, Frankel H, Gentilello LM. �uantity and quality o sleep in the surgical intensive care unit: are our patients sleeping? J rauma. 2007 Dec;63(6):1210–4. Gajic O, Rana R, Winters JL, Yilmaz M, Mendez JL, Rickman OB, et al. ransusion-related acute lung injury in the critically ill: prospective nested case-control study. Am J Respir Crit Care Med. 2007 Nov 1;176(9):886–91. Epub 2007 Jul 12. Gerrits CM, Herings RM, Leuens HG, Lammers JW. N -acetylcysteine reduces the risk o re-hospitalisation among patients with chronic obstructive pulmonary disease. Eur Respir J. 2003 May;21(5):795–8. Gonzalez C, Servera E, Marin J. Importance o noninvasively measured respiratory muscle overload among the causes o hospital readmission o COPD patients. Chest. 2008 Apr;133(4):941–7. Epub 2008 Feb 8. Hacke W, Kaste M, Bluhmki E, Brozman M, Davalos A, Guidetti D, et al; ECASS Investigators. Trombolysis with alteplase 3 to 4.5 hours afer acute ischemic stroke. N Engl J Med. 2008 Sep 25;359(13):1317–29. McMullen AH, Pasta DJ, Frederick PD, Konstan MW, Morgan WJ, Schechter MS, et al. Impact o pregnancy on women with cystic �brosis. Chest. 2006 Mar;129(3):706–11. Meneveau N, Seronde MF, Blonde MC, Legalery P, Didier-Petit K, Briand F, et al. Management o unsuccessul thrombolysis in acute massive pulmonary embolism. Chest. 2006 Apr;129(4):1043–50. Rabe KF, Hurd S, Anzueto A, Barnes PJ, Buist SA, Calverley P, et al; Global Initiative or Chronic Obstructive Lung Disease. Global strategy or the diagnosis, management, and prevention o chronic obstructive pulmonary disease: GOLD executive summary. Am J Respir Crit Care Med. 2007 Sep 15;176(6):532–55. Epub 2007 May 16. ashkin DP, Celli B, Senn S, Burkhart D, Kesten S, Menjoge S, et al; UPLIF Study Investigators. A 4-year trial o tiotropium in chronic obstructive pulmonary disease. N Engl J Med. 2008 Oct 9;359(15):1543–54. Epub 2008 Oct 5. oy P, Popovsky MA, Abraham E, Ambruso DR, Holness LG, Kopko PM, et al; National Heart, Lung and Blood Institute Working Group on RALI. ransusion-related acute lung injury: de�nition and review. Crit Care Med. 2005 Apr;33(4):721–6.
MAYO CLINIC INERNAL MEDICIN E BOARD REVIEW: QUESIONS AND ANSWER S
�. INFECTIOUS DISEASES QUESTIONS AND ANSWERS
QUESIONS
no clubbing or cyanosis. Te total leukocyte count is ��.����/L. Results o serum chemistry tests are normal. Te chest radiograph shows a new dense consolidation in the lef upper lung �eld. Which o the ollowing antimicrobial regimens would you initiate?
Multiple Choice (choose the best answer)
a. b. c. d. e.
PNEUMONIA, ZO ONOSES, TRAVEL, AND BIOTERRORISM
�. A ��-year-old man with uncomplicated type � diabetes mellitus and hypertension presents to his primary care physician with a �-day history o ever, productive cough, and shortness o breath. He has been in good health otherwise, has never been hospitalized, and has good social support at home. Examination �ndings include the ollowing: temperature ��.�°C, blood pressure ���/�� mm Hg, heart rate �� beats per minute with regular rate and rhythm, and respiratory rate �� breaths per minute. Oxygen saturation is ��% with room air. He is awake and alert and oriented to person, place, and time; he responds to questions appropriately. Inspiratory crackles are audible in the right lower lung �eld, his abdomen has active bowel sounds and is not tender, and his extremities have no clubbing or cyanosis. Which o the ollowing should be done next?
Azithromycin orally Cefriaxone intravenously and azithromycin orally Cipro�oxacin intravenously Ampicillin intravenously and clindamycin orally Meropenem intravenously
�. A ��-year-old male business executive is admitted to the intensive care unit or evers, diarrhea, cough, and shortness o breath progressing over the past �� hours. His past medical history includes cigarette smoking and weekly binge drinking but no other chronic medical problems. He is rom St Louis, Missouri, but was attending a conerence in Chicago, Illinois, � days beore admission. Abdominal pain, diarrhea, cough, and shortness o breath developed � days beore admission. He had mentioned that other people rom the convention had similar illnesses. He has had no other recent travel and has not had any exposure to animals. Antimicrobial therapy has been started and blood and sputum cultures are pending. Examination �ndings include the ollowing: temperature ��.� °C, blood pressure ��/�� mm Hg, heart rate �� beats per minute with regular rate and rhythm, and respiratory rate �� breaths per minute. Oxygen saturation is ��% with �� L o oxygen per minute by ace mask. He is drowsy and does not reliably ollow commands, and he is oriented to person only. Diffuse bilateral crackles are audible on lung auscultation, his abdomen has active bowel sounds and is not tender, and his extremities have no clubbing or cyanosis. Remarkable diagnostic test results were leukocytosis (��.�×���/L) and hyponatremia (��� mEq/L). A chest radiograph showed bilateral patchy in�ltrates. Gram staining o a sputum showed scant polymorphonuclear cells and no bacteria. What is the most likely cause o the patient’s respiratory inection?
a. Outpatient observation only with ollow-up in � days b. Outpatient consultation with an inectious diseases specialist c. Hospital admission and treatment with levo�oxacin d. Intensive care unit admission and treatment with levo�oxacin e. Outpatient treatment with levo�oxacin
�. A ��-year-old woman is admitted to an internal medicine hospital ward with a �-day history o ever, cough, progressively purulent sputum, and shortness o breath. She has been well otherwise, has never been hospitalized, and has a past medical history o well-controlled type � diabetes mellitus and hypertension. She has not had any recent antimicrobial exposure, oreign travel, or animal exposure. She has no known drug allergies. Examination �ndings include the ollowing: temperature ��.�°C, blood pressure ���/�� mm Hg, heart rate �� beats per minute with regular rate and rhythm, and respiratory rate �� breaths per minute. Oxygen saturation is ��% with � L o oxygen per minute by nasal cannula. She is awake and alert and oriented to person, place, and time. Inspiratory crackles are audible in the lef upper lung �eld, her abdomen has active bowel sounds and is not tender, and her extremities have
a. Legionella pneumophila b. Respiratory syncytial virus (RSV) c. Enterovirus d. Chlamydophila psittaci e. Coccidioides immitis
�. Which o the ollowing is a recommended intervention to reduce the risk o ventilator-associated pneumonia among patients receiving mechanical ventilation?
��
a. Preerence or invasive ventilation over noninvasive ventilation b. Preerence or nasotracheal intubation over orotracheal intubation c. Keeping patients supine during enteral eeding d. Preerence or parenteral nutrition over enteral nutrition e. Maintaining endotracheal tube cuff pressure greater than �� cm water
�. For which pair o pathogens should hospital antimicrobial resistance rates be considered when an empirical antimicrobial regimen is chosen to treat hospital-acquired pneumonia? a. Acinetobacter baumannii and Candida albicans b. Streptococcus pneumoniae and Haemophilus in�uenzae c. Pseudomonas aeruginosa and Staphylococcus aureus d. Klebsiella pneumoniae and in�uenza virus e. Serratia marcescens and Enterococcus faecium
�. A previously healthy ��-year-old man presents to his primary care physician afer his cat bit his arm earlier in the morning. Te cat’s vaccinations are current, and the cat has not been attacked by other animals. Te patient says that the bite was deep enough to draw blood, although the bleeding has stopped. You cleanse the wound, apply a bandage, and update his tetanus-diphtheria vaccination. Examination �ndings include the ollowing: tem perature ��.�°C, blood pressure ���/�� mm Hg, heart rate �� beats per minute, and respiratory rate �� breaths per minute. A �.�-cm laceration is evident on the dorsal aspect o the lef orearm without any erythema or purulence. Which o the ollowing antimicrobial regimens should be prescribed? a. b. c. d. e.
�.
Amoxicillin-clavulanic acid Dicloxacillin Cephalexin Clindamycin No antimicrobials
A previously healthy ��-year-old man is admitted to an intensive care unit (ICU) with a �-hour history o rapidly progressive ever, shortness o breath, and cough. He had spent most o the previous � days at the county air. Upon arrival at the hospital, the patient underwent endotracheal intubation, was given �uid resuscitation, and received vasopressor medications and mechanical ventilation. Examination �ndings include the ollowing: temperature ��.�°C, blood pressure ��/�� mm Hg with vasopressors, heart rate ��� beats per minute (tachycardic), and respiratory rate �� breaths per minute. Oxygen saturation is ��% with assist/control ventilation and ��% raction o inspired oxygen. He is sedated and does not ollow commands. Diffuse bilateral crackles are audible on lung auscultation, his abdomen has active bowel sounds and is not tender, and his extremities have no clubbing or cyanosis. A chest radiograph shows diffuse pulmonary in�ltrates and a widened mediastinum. Blood samples drawn in the emergency department � hours ago or cultures ��
•
are growing a gram-positive rod in � out o � bottles. Terapy with cefriaxone and levo�oxacin was started upon admission to the ICU. What is the most likely etiologic agent? a. Streptococcus pneumoniae b. Avian in�uenza virus H�N� c. Francisella tularensis d. Bacillus anthracis e. Listeria monocytogenes
SKIN AND SOFT TISSUE IN FECTIONS, BONE AND JOINT INFECTIONS, AND MYCOBACTERIAL INFEC TIONS
�. A ��-year-old diabetic man underwent abdominal hernia repair � days ago afer receiving preoperative ceazolin. A nurse calls you to evaluate him or ever, hypotension, and tachycardia. He has a toxic appearance; his incision is tender and erythematous with some sero-sanguinous drainage. Which group o bacteria should your therapy target? a. Gram-positive cocci b. Gram-positive cocci and gram-positive bacilli c. Gram-positive cocci and gram-positive and gram-negative bacilli d. Gram-negative bacilli e. Gram-positive bacilli
�. A ��-year-old man underwent septoplasty or a deviated nasal septum and recurrent sinusitis. He presents to an emergency department �� hours later with headache, ever, chills, myalgia, nausea, vomiting, and abdominal cramping. His temperature is ��.�°C, his pulse is ��� beats per minute, his respiratory rate is �� breaths per minute, and his blood pressure is ��/�� mm Hg. He has generalized erythroderma. On head and neck examination, the nasal passages are hyperemic but not purulent. Laboratory test results included the ollowing: leukocyte count ��.�×���/L, hemoglobin ��.� g/dL, and platelet count ��×���/L. Te results o liver unction tests, serum creatinine, and amylase were normal. Which pathogen is most likely to cause this syndrome? a. Pseudomonas aeruginosa b. Haemophilus in�uenzae c. Moraxella catarrhalis d. Streptococcus pneumoniae e. Staphylococcus aureus
��. Which o the ollowing patients is best suited or out patient parenteral antibiotic therapy (OPA) or the inection? a. A ��-year-old injection drug user with Staphylococcus aureus tibial osteomyelitis b. A ��-year-old man with enterococcal prosthetic valve endocarditis with a new Wenckebach heart block c. A ��-year-old woman with a diabetic oot ulcer and Pseudomonas metatarsal osteomyelitis that requires imipenem-cilastatin every � hours
MAYO CLINIC INERNAL MEDICIN E BOARD REVIEW: QUESIONS AND ANSWER S
d. A ��-year-old woman with pneumococcal meningitis that is improving on day � o a ��-day cefriaxone regimen e. An ��-year-old man with early Alzheimer disease treated with cefriaxone or community-acquired pneumonia
��. A ��-year-old man with diabetes mellitus and severe peripheral vascular disease presents with a �-week history o erythema and induration surrounding a �-cm plantar ulcer. oday he is nauseated, ebrile, and tachycardic. You can insert a metallic probe through the open wound to the bone surace. Tere is surrounding redness and drainage o oul-smelling pus. Which o the ollowing would be the next appropriate step in management? a. Swabbing the patient’s nose or methicillin-resistant Staphylococcus aureus (MRSA) b. Magnetic resonance imaging (MRI) o the oot c. Parenteral vancomycin and piperacillin-tazobactam d. Bone scintigraphy e. Plain radiography o the oot
��. For which o the ollowing would use o the �uantiFERON-B Gold test or Mycobacterium tuberculosis be preerable to a puri�ed protein derivative (PPD) skin test? a. A ��-year-old resident physician rom India who received BCG vaccine as a child b. A ��-year-old health care worker with recent exposure to someone with tuberculosis in the past � months c. A ��-year-old man who recently returned rom � year o volunteer work in Rwanda d. A ��-year-old Somali woman with a new diagnosis o human immunode�ciency virus inection (CD� count ��� cells/μL) e. A ��-year-old nurse undergoing annual required tuberculosis screening
��. A ��-year-old Mexican immigrant with human immunodi�ciency virus inection (CD� count ��� cells/ μL) has been treated with isoniazid, riampin, pyrazinamide, ethambutol, and pyridoxine or pulmonary tuberculosis or the past � months. He is not currently taking antiretroviral medications. He now presents with severe pain, swelling, and redness o the lef great toe. Which o the ollowing medications is most likely associated with this condition? a. b. c. d. e.
Isoniazid Riampin Pyridoxine Ethambutol Pyrazinamide
��. A ��-year-old woman with hypertension and type � diabetes mellitus is seen or worsening hypertension and poor glycemic control. She is taking verapamil and glyburide. Recently, a methicillin-resistant Staphylococcus aureus inection developed in her lef prosthetic hip joint, or which she underwent débridement and com ponent retention and is at a nursing home receiving vancomycin and riampin. Her erythrocyte sedimentation
rate and C-reactive protein level have been normal. What is the most likely cause o the change in her blood glucose level and blood pressure? a. b. c. d. e.
Ongoing inection and need or resection arthroplasty Poor control o diet Medication interaction Inadequate pain control Poor adherence to drug therapy
��. Each o the ollowing patients underwent a tuberculin skin test (S) or appropriate indications. Which result would be considered a positive S reaction (ie, a positive puri�ed protein derivative skin test [PPD])? a. A ��-year-old man who is positive or human immunode�ciency virus (HIV )—an �-mm induration b. A ��-year-old Mexican native migrant worker—a �-mm induration last year and a ��-mm induration this year c. A ��-year-old man whose chest radiograph suggests old tuberculosis—a �-mm induration d. A ��-year-old diabetic woman—a ��-mm induration e. All o the above
��. Which o the ollowing tuberculosis-suspect patients should be considered inectious? a. A ��-year-old man had a smear positive or acid-ast bacilli (AFB) � weeks ago ; no urther specimens were obtained. He has received sel-administered antitubercular therapy or the past � days and continues to cough. b. A ��-year-old school teacher with pulmonary tuberculosis has been receiving directly observed therapy or � weeks and is asymptomatic. Tree sputum smears have been evaluated; the �rst was positive and the rest were negative. c. Cough and malaise developed in a ��-year-old Asian man visiting his grandchildren. A tuberculin skin test and �uantiFERON-B Gold test were both positive. His chest radiograph was normal, and � sputum samples were negative or AFB. d. A ��-year-old woman with a cough has lost weight. Te �uantiFERON-B Gold test was positive, and her chest radiograph was normal. e. A ��-year-old health care worker had a newly positive puri�ed protein derivative skin test and a normal chest radiograph.
��. A ��-year-old male landscaper with human immunode�ciency virus (HIV), sickle cell anemia, and chronic renal ailure has a hot, painul, swollen right knee. Results o an aspiration o the knee joint are as ollows: the leukocyte count is �� ���/L with ��% polymorphonuclear cells, and Gram staining is negative or microorganisms. Which o the ollowing is the least likely cause o his symptoms? a. b. c. d. e.
Acute sickle cell crisis Acute gout Acute gonococcal arthritis Acute Salmonella arthritis Acute Staphylococcus aureus arthritis
�. INFECIOUS DISEASES QUESIONS AND ANSWERS
•
��
HIV INFECTI ON
��. A ��-year-old man who is positive or human immunode�ciency virus (HIV) presents with progressive headache, irritability, and low-grade ever over the past month. A riend who brought him to the emergency department today is worried about his persistent complaints. Te patient has not been seen in the clinic or � years and has not taken antiretrovirals or more than � years. At the initial diagnosis � years ago, cytomegalovirus (CMV) serology was negative, serum Toxoplasma IgG antibody test results were positive, hepatitis serology tests were negative, and the tuberculin skin test was negative. His CD� count was ��� cells/μL but increased to ��� cells/ μL with treatment. He reports that his headache has become progressively disabling. He has not worked as a waiter or the past week. His temperature is ��°C, his blood pressure is ���/�� mm Hg, and his heart rate is �� beats per minute. He has questionable nuchal rigidity, but no other neurologic �ndings are noted. Te ophthalmoscopic examination is normal, without papilledema. His CD� cell count is �� cells/ μL, and HIV-� RNA is ���,��� copies/mL. A computed tomographic (C) scan is normal. Which o the ollowing is most appropriate? a. b. c. d. e.
Empirical �uconazole therapy Empirical treatment with pyrimethamine and suladiazine Lumbar puncture and cerebrospinal �uid (CSF) studies CMV serology Magnetic resonance imaging (MRI) o the head
��. A ��-year-old man has a severe cough and is ound to have lobar pneumonia. He is treated with levo�oxacin and improves over the next � weeks. He denies having risk actors or sexually transmitted and blood-borne diseases. A human immunode�ciency virus (HIV) test is done, and results o both the enzyme-linked immunosorbent assay (ELISA) and the Western blot are positive. Which o the ollowing is the most likely explanation? a. Levo�oxacin can interere with HIV testing, and the test should be repeated in � month. b. Acute inections can cause a alse-positive HIV antibody test result, and he can be reassured that it is likely a alse-positive result. Te test should be repeated in � months. c. Te result is a true-positive, and he has not been orthcoming about his risk actors. d. Te result is likely a alse-positive rom laboratory error. e. He should be asked whether he is taking any over-the-counter herbal supplements since products containing ginseng can interere with the test.
��. A ��-year-old married woman presents with a history o low-grade ever, malaise, sore throat, anorexia, abdominal discomort, and diarrhea o �� days’ duration. She denies having risk actors or human immunode�ciency virus (HIV) inection. On physical examination, her temperature is ��.�°C, her blood ��
•
pressure is ���/�� mm Hg, her heart rate is �� beats per minute, and her respiratory rate is �� breaths per minute. She has cervical lymphadenopathy and mild diffuse abdominal tenderness. Findings on the rest o the examination, including mouth, lungs, heart, and pelvis, are unremarkable. Laboratory evaluation shows the ollowing: hemoglobin ��.� g/dL, leukocyte count �.����/L, and platelet count ��� ���/L; atypical lymphocytes are noted on the peripheral smear, the serum creatinine is normal (�.� mg/dL), and liver unction test values (aspartate aminotranserase, alanine aminotranserase, bilirubin, and alkaline phosphatase) are all within the reerence range. Which statement is false? a. Results o HIV antibody testing will likely be negative. b. I the HIV test results are positive, genotyping should be perormed. c. esting or acute HIV, Epstein-Barr virus, and cytomegalovirus should be done i her symptoms persist or several more weeks, but it is not currently indicated. d. I she has acute HIV, her HIV viral load by polymerase chain reaction should be more than ���,��� copies/mL. e. Patients with undiagnosed HIV inection ofen deny being at risk.
��. A ��-year-old woman with long-standing human immunode�ciency virus (HIV) inection and hepatitis C began a salvage program o stavudine/tenoovir/lami vudine/darunavir with boosted ritonavir in combination with raltegravir about � months ago. She is also taking pravastatin and �sh oil or hyperlipidemia. In a routine appointment � weeks ago, she reported mild nausea and atigue. A pregnancy test was negative. Her viral load was suppressed or the �rst time in � years. Her CD� count had increased rom ��� to ��� cells/ μL. Her alanine aminotranserase (AL) increased slightly rom �� to �� U/L. She reports eeling increasingly unwell, and her appetite has been poor. She believes that she may have lost weight. In the past � days she has had new diuse abdominal discomort. Laboratory values are as ollows: AL ��� U/L, glucose ��� mg/dL, leukocyte count �.�×���/L, hemoglobin ��.� g/dL, CD� count ��� cells/μL, HIV viral load nondetectable, and lactate �� mmol/L. Which, i any, o her medications is likely responsible or her symptoms and elevated lactate? a. Te protease inhibitor darunavir b. Nucleoside reverse transcriptase inhibitors (NRIs), especially stavudine c. Noneo her HIV medications, but instead an HIV-associated inection d. Pravastatin combined with the ritonavir e. Raltegravir
��. A ��-year-old surgical nurse was stuck with a suture needle while assisting with emergent surgery o a patient who is human immunode�ciency virus (HIV) positive. She saw blood in her glove and called the employee health offi ce or recommendations. Which o the ollowing is false?
M A YO C L I N I C I N E R N A L M E D I C I N E B O A R D R E V I EW : Q U E S I ON S A N D A N S W E R S
a. Initiation o postexposure prophylaxis with zidovudine in combination with lamivudine will signi�cantly decrease her risk o acquiring HIV. b. Postexposure prophylaxis should be continued i tolerated or �� weeks. c. Use o � drugs should be considered i the surgical patient has known resistance to lamivudine. d. Te risk o acquiring HIV rom a needlestick injury is approximately � in �,��� events. e. Postexposure prophylaxis is unlikely to be helpul i not started within �� hours.
��. A ��-year-old man with a history o AIDS, including a past history o pneumocystis pneumonia (PCP) and cryptococcal meningitis, did well while incarcerated and treated with eavirenz, tenoovir, and emtricitabine. His CD� count gradually improved to ��� cells/ μL, and his viral load was suppressed or over � years. When released, he was told to continue the � drugs and to ollow up with a community human immunode�ciency virus clinic, but he never sought care. His sister convinced him to seek care � years afer he was incarcerated, and he seems interested in restarting therapy. He reports eeling well except or mild atigue, diarrhea (� sof stools daily accompanied by urgency) and a acial rash suggestive o seborrheic dermatitis. On examination, he has mild cervical, axillary, and inguinal adenopathy and thrush. Which o the ollowing is not currently indicated? a. Restarting trimethoprim-sulamethoxazole (MP-SMX) prophylaxis against PCP b. Checking the viral load with genotyping c. Puri�ed protein derivative skin test or �uantiFERON-B Gold blood test or tuberculosis d. Azithromycin prophylaxis against Mycobacterium aviumintracellulare (MAI) complex e. Ketoconazole cream and shampoo
��. A ��-year-old man who is regularly sexually active with multiple partners, both male and emale, asks what he can do (other than decreasing his sexual activity) to lower his risk o acquiring human immunode�ciency virus (HIV). Which o the ollowing will not decrease his risk o acquiring HIV? a. Use o latex condoms b. Addition o nonoxynol spermicide in addition to condoms c. aking tenoovir in combination with emtricitabine on a preventive basis d. Circumcision e. Engaging in only insertive sex
INFECTIOUS DISEA SE SYNDROMES: URINARY TRACT INFE CTION, SEXUALLY TRANSMITTED INFECTION, AND GASTROINTESTINAL TRACT INFECTION
��. An ��-year-old man presents with pelvic pain and dysuria. He has had � urinary tract inections in the past � months. A urologic evaluation last month showed an
enlarged prostate but normal urodynamics. Urinalysis results are the ollowing: �� to �� leukocytes per high-power �eld, a positive leukocyte esterase test, a negative nitrite test, and numerous gram-negative rods. Which o the ollowing is the best choice or treatment o his symptoms? a. b. c. d.
Nitrourantoin ��� mg daily or � days Amoxicillin ��� mg � times daily or � days Cipro�oxacin ��� mg twice daily or �� days rimethoprim-sulamethoxazole � tablet twice daily or �� days e. Levo�oxacin ��� mg once daily or �� days
��. A ��-year-old diabetic woman rom Hyderabad, India, has had unrelenting ever and night sweats or � weeks. Blood cultures have been negative and an abdominal computed tomographic scan shows a ��-cm hypodense lesion in the lef lobe o her liver. Which o the ollowing should be done next or this patient? a. b. c. d. e.
Ultrasound-guided needle aspiration Serology test on blood Stool test or ova and parasites Piperacillin-tazobactam �.��� g intravenously every � hours Puri�ed protein derivative (PPD) skin test
��. A ��-year-old woman reports that her urine has been cloudy or the past � days. She is asymptomatic but has missed her period or � months. A urine pregnancy test is positive and a urine culture shows more than ���,��� colonies o a gram-negative rod. Which o the ollowing should be recommended or management o her bacteriuria? a. Cipro�oxacin ��� mg twice daily or � days b. rimethoprim-sulamethoxazole � tablet twice daily or �� days c. Nitrourantoin ��� mg daily or � days d. Cefriaxone � g intravenously daily or �� days e. No treatment
��. A ��-year-old woman with acute myelogenous leukemia (AML) is treated with daunorubicin and cytarabine. She has been neutropenic or � weeks and ebrile or � weeks despite taking meropenem, vancomycin, and acyclovir. A computed tomographic scan o her chest shows a large, wedge-shaped peripheral in�ltrate with some central clearing in the lef lower lobe. Which o the ollowing antiungal agents should be most effective or treatment? a. b. c. d. e.
Caspoungin Itraconazole Voriconazole Amphotericin B Fluconazole
��. Which o the ollowing is true about Clostridium dif �cile inection? a. Vancomycin is less effective than metronidazole or treatment o severe disease. b. Most relapses are due to antibiotic resistance.
�. INFECIOUS DISEASES QUESIONS AND ANSWERS
•
��
c. Vancomycin-resistant enterococcal (VRE) colonization occurs more requently with oral vancomycin treatment than with metronidazole. d. A single recurrence o inection increases the likelihood o urther relapses. e. A stool sample or Clostridium difficile toxin should be obtained at the end o treatment to document cure.
��. A ��-year-old man comes to your offi ce with a �-day history o malaise, a diffuse rash involving his trunk and extremities, swollen glands, and headache. His symptoms began � days afer taking cipro�oxacin or gonorrhea. At that time, his syphilis IgM and IgG were positive (titer �:��). His rapid plasma reagin (RPR) titer was �:���, and his human immunode�ciency virus (HIV) quantitative viral load and HIV antibody test were both negative. Which o the ollowing should you also recommend? a. b. c. d. e.
Lopinavir-ritonavir plus zidovudine-lamivudine Doxycycline ��� mg twice daily or � days Benzathine penicillin �.� million units intramuscularly Lumbar puncture Azithromycin � g orally
��. A ��-year-old morbidly obese diabetic woman, recently treated or abdominal wall cellulitis, now presents with a �-day history o ever, right �ank pain, and dysuria. A urinalysis is esterase positive and a Gram stain shows gram-negative bacilli too numerous to count. Which o the ollowing treatments should you recommend or management o her urinary tract inection? a. b. c. d.
Cefriaxone � g intravenously daily or �� days Ampicillin � g intravenously every � hours or �� days Cipro�oxacin ��� mg orally twice daily or � days rimethoprim-sulamethoxazole(MP-SMX )���–��� mg orally twice daily or � days e. Nitrourantoin ��� mg orally twice daily or �� days INFECTIOUS DISEASE SYNDROMES: CARDIOVASCULAR, BLOODSTREAM, AND CENTRA L NERVOUS SYSTEM INFECTIONS
��. A ��-year-old woman is undergoing her second cycle o chemotherapy or breast cancer. On her third hospital day, she has a ever (��.�°C) and you notice a red track along her Hickman catheter tunnel. Cultures rom catheter-drawn blood and rom peripheral vein blood are negative at �� hours. Which o the ollowing should you recommend or management o her ebrile illness? a. Intravenous vancomycin and ceepime or �� days b. Intravenous vancomycin and vancomycin lock therapy or �� days c. Removal o the Hickman catheter and intravenous vancomycin or �� days d. ransesophageal echocardiography to determine the duration o antibiotic therapy e. opical neomycin, polymixin B, and bacitracin to the exit site and intravenous vancomycin or � days
��
•
��. A ��-year-old man with a recently implanted permanent pacemaker (� months ago) is admitted to the hos pital because he has had increasing pain, swelling, and erythema at the site o his pacemaker pocket. He has a history o type � diabetes mellitus, coronary artery disease, and third-degree heart block. His symptoms started � days ago and are rapidly progressing. He is ebrile on admission, and blood cultures are growing gram-positive cocci resembling staphylococci at �� hours. A transesophageal echocardiogram is negative or any evidence o endocarditis. What is the most appropriate management or this patient? a. Start intravenous vancomycin and delay device explantation until subsequent blood cultures are negative. b. Start intravenous vancomycin and immediately proceed to device explantation. c. Device removal is not necessary since a pacemaker pocket inection can be cured with combination therapy with vancomycin and riampin. d. Start intravenous ceazolin and proceed with device removal as soon as possible. e. Start intravenous daptomycin and oral riampin and reassess in �� hours.
��. Which o the ollowing has not been shown to reduce the incidence o central venous catheter–related bloodstream inections (CRBSIs)? a. b. c. d. e.
Chlorhexidine patch applied to the exit site Routine catheter exchange over a guidewire at �� hours Minocycline-riampin–coated central venous catheters Silver suladiazine–coated central venous catheters A standardized catheter insertion bundle
��. A ��-year-old woman with recurrent episodes o sinusitis presents to the emergency department with a �-week history o ever and constant headache. A computed tomographic scan o the head shows opaci�cation o the rontal and sphenoid sinuses and an abscess in the lef rontal lobe. Which o the ollowing would be the best initial antimicrobial regimen or this patient? a. b. c. d.
Vancomycin Piperacillin-tazobactam Ceazolin and metronidazole Vancomycin, cefriaxone, and a lipid ormulation o amphotericin e. Vancomycin, cefriaxone, and metronidazole
��. Which o the ollowing patients should receive antibiotic prophylaxis or inective endocarditis? a. A ��-year-old man who has a dual chamber permanent pacemaker and is undergoing dental extraction b. A ��-year-old woman who has a history o aortic valve endocarditis and is scheduled or placement o an orthodontic appliance c. A ��-year-old man who has severe mitral valve regurgitation and is undergoing endoscopy or evaluation o epigastric pain
MAYO CLINIC INERNAL MEDICIN E BOARD REVIEW: QUESIONS AND ANSWER S
d. A ��-year-old heart transplant recipient who has cardiac valvulopathy and is undergoing a dental extraction e. A ��-year-old woman who has a history o surgically repaired congenital heart disease in childhood and is undergoing a root canal
��. A ��-year-old woman who has a history o mitral valve replacement with a mechanical prosthesis is admitted with a �-week history o ever, malaise, and shortness o breath. Admission blood cultures are positive or methicillin-resistant Staphylococcus aureus (MRSA), and a transesophageal echocardiogram is consistent with a �-mm vegetation on the anterior lea�et o the mitral valve. Which o the ollowing is the most appro priate antibiotic regimen or this patient? a. Vancomycin or � weeks b. Vancomycin and gentamicin or � weeks c. Daptomycin and gentamicin or � weeks, with riampin or the �rst � weeks o therapy d. Vancomycin and riampin or � weeks, with gentamicin or the �rst � weeks o therapy e. Vancomycin and riampin or � weeks
��. A ��-year-old diabetic man is seen in the emergency department or ever, productive cough, headache, and altered mental status or the past � hours. On examination, he is ebrile (��°C) and lethargic, and he has prominent neck stiness. A chest radiograph shows a dense right lobar consolidative iniltrate. His leukocyte count is ��.� ×���/L, and his creatinine level is �.� mg/dL. A lumbar puncture shows an opening pressure o �� mm water, leukocyte count ��.�×���/L (��% neutrophils), protein ��� mg/dL, and glucose �� mg/dL. Gram staining shows gram-positive diplococci. Which o the ollowing is the most appropriate regimen or empirical treatment? a. b. c. d. e.
Ampicillin, cefriaxone, and vancomycin Cefriaxone and dexamethasone Cefriaxone and vancomycin Cefriaxone, vancomycin, and acyclovir Cefriaxone, vancomycin, and dexamethasone
�. INFECIOUS DISEASES QUESIONS AND ANSWERS
•
��
ANSWERS
�. Answer e.
Legionella inections can progress rapidly and are ofen asso-
ciated with diarrhea and other gastrointestinal tract symptoms. Alcohol abuse is a known epidemiologic risk actor, and Legionella ofen causes outbreaks among persons with common-source exposure. Relative bradycardia and mental status changes requently occur in patients with Legionella inection. Leukocytosis, hyponatremia, and patchy bilateral in�ltrates occur requently in Legionella inections, but those eatures may not be as helpul or discriminating betweenothermicrobiologic causesocommunity-acquired pneumonia. RSV is unlikely to cause respiratory illness o this severity in an adult without immunocompromising conditions. Enterovirus is a cause o encephalitis but is not a cause o community-acquired pneumonia. When C psittaci causes atypical community-acquired pneumonia, it is generally not rapidly progressive or associated with relative bradycardia or gastrointestinal tract symptoms. Te patient has not been to a region where C immitis is endemic.
Initial assessment o the severity o community-acquired pneumonia is important or internal medicine physicians to reduce unnecessary hospitalization and to identiy patients who are at higher risk o death or who need more immediate intervention. Illness severity scores such as the pneumonia severity index (PSI) and CURB-�� ( c onusion, urea nitrogen, r espiratory rate, blood pressure, and �� years or older) have been developed to help in the decision or site o care or patients with community-acquired pneumonia. In the outpatient setting, the modi�ed CRB-�� ( c onusion, r espiratory rate, blood pressure, and �� years or older) score is useul since it does not require laboratory or radiographic evidence to determine the severity score. In this question, the patient is not conused, does not have a respiratory rate o �� breaths per minute or more, does not have a systolic blood pressure less than �� mm Hg or a diastolic blood pressure o �� mm Hg or less, but is �� years or older. His CRB-�� score is �. Patients with scores o � or � can generally be treated as outpatients i they can reliably take oral antimicrobials and have outpatient support resources. (See Mandell et al and Capelastegui et al in the “Suggested Reading” list.)
�. Answer e.
Tis question relates to knowledge o modi�able risk actors to reduce the risk o ventilator-associated pneumonia among patients receiving mechanical ventilation. Te correct answer and incorrect answers (modi�ed to be negative) were taken directly rom the guideline recommendations on health care–associated pneumonia. (See American Toracic Society and Inectious Diseases Society o America in the “Suggested Reading” list.)
�. Answer b.
Tis question is related to appropriate initial antimicrobial treatment o community-acquired pneumonia in a patient with medical comorbidities who requires hospitalization but not intensive care unit admission. Recommended treatment is either a respiratory �uoroquinolone or a β-lactam antibiotic in combination with a macrolide antibiotic. Azithromycin alone is not a recommended regimen in patients with medical comorbidities, including diabetes mellitus. Cipro�oxacin does not have sufficient coverage or Streptococcus pneumoniae and is thus not considered a respiratory �uoroquinolone. Although intravenous ampicillin would be an acceptable β-lactam antibiotic antimicrobial choice, clindamycin is not a macrolide antibiotic and the patient does not have risk actors to warrant empirical coverage o methicillin-resistant Staphylococcus aureus. Meropenem is a very broad-spectrum antimicrobial and is not recommended or routine use or hospitalized patients with community-acquired pneumonia who do not have risk actors or inection with Pseudomonas aeruginosa. (See Mandell et al in the “Suggested Reading” list.)
�. Answer c.
Tis question is based on an understanding o the epidemiology o hospital-acquired pneumonia and how it relates to the choice o empirical antimicrobials or its treatment. Te hospital prevalence o methicillin-resistant S aureus and multidrug-resistant gram-negative organisms, such as P aeruginosa, A baumannii, and the enterobacteriaceae, need to be considered when deciding on an antimicrobial regimen. Te only correct pair o choices is S aureus and P aeruginosa. Enterococci and C albicans are respiratory pathogens in only select hosts and are very rare causes o hospital-acquired pneumonia. Although in�uenza virus, S pneumoniae, and H in�uenzae can cause hospital-acquired pneumonia, they generally are involved in a relatively small proportion o nosocomial cases and their resistance pro�les are generally those o the community-acquired strains. (See American Toracic Society and Inectious Diseases Society o America in the “Suggested Reading” list.)
�. Answer a.
Tis question involves recognition o Legionella as a cause o community-acquired pneumonia. In healthy hosts, ��
�. Answer a.
��. Answer c.
Tis question is based on an understanding o the microbiology and subsequent antimicrobial prophylaxis rami�cations o cat bites. Major pathogens isolated rom cat bites include numerous anaerobes, streptococci, and staphylococci but most commonly Pasteurella multocida . Prophylactic antimicrobials are recommended unless the bite wound is very super�cial, and P multocida is generally resistant to dicloxacillin, cephalexin, clindamycin, and erythromycin but is generally susceptible to amoxicillin-clavulanic acid. (See Oehler et al in the “Suggested Reading” list.) �. Answer d.
Tis diabetic man has peripheral vascular disease and a etid oot ulcer with surrounding cellulitis, which can be probed to the bone. He is maniesting systemic toxicity. Te �rst step in management would be initiation o antimicrobial therapy to cover MRSA and a mixed inection. Te inection will be polymicrobial, and a culture would have limited value. A nasal swab or MRSA does not establish the presence o MRSA in the wound. An MRI offers no additional immediate value at this stage, especially since the wound can be probed to the bone and is likely osteomyelitic. ��. Answer a.
Tis question relates to a syndrome o a potential bioterrorism agent. Te patient was previously healthy, and a lie-threatening illness developed quickly, suggesting a highly aggressive pathogen. Te widened mediastinum is suggestive o inhalational anthrax but could also be seen with diseases such as tularemia, histoplasmosis, and tuberculosis. Te quick positivity o the blood cultures with gram-positive bacilli in this clinical scenario strongly suggests anthrax, which should not be excluded as a contaminant. (See �uintiliani and �uintiliani in the “Suggested Reading” list.) �. Answer c.
Te patient has type � necrotizing asciitis, which is most ofen a mixed inection involving aerobic and anaerobic organisms. Risk actors or this type o inection are gastrointestinal tract procedures, diabetes mellitus, or vascular disease. �. Answer e.
Tis patient has staphylococcal toxic shock syndrome, which may be associated with the retained packing afer the nasal procedure. Ofen patients receive clindamycin until the packing is removed. Te eatures o staphylococcal toxic shock syndrome are due to toxin-mediated cytokine activation and can occur �) afer surgical and postpartum procedures; �) with mastitis, sinusitis, burns, and skin and sof tissue inections (especially o the extremities, perianal area, and axillae); and �) with respiratory inections afer in�uenza. ��. Answer d.
Tere are OPA guidelines or most inectious diseases. Ofen OPA is used or inections (eg, bone and joint inections) that require prolonged parenteral therapy. Key tenets o OPA include the ollowing: �) the patient or caregiver (or both) is willing to participate and can saely, effectively, and reliably deliver OPA; �) a physician or home care agency is available and accessible or communications about problems and or monitoring; �) the active inectious diseases are stable and the patient has no active, new problems, is not at increased risk o complications, and has received more than � days o therapy or bacterial meningitis; and �) the patient can pay or treatment, has a sae and adequate home or outpatient environment to sup port care, and is not actively using illicit drugs.
Te �uantiFERON-B Gold test can be done in all circumstances in which a PPD test is indicated. Its principal role may be in sorting out alse-positive tuberculin skin test results in patients who received BCG vaccine since it does not cross-react with nontuberculous mycobacteria. ��. Answer e.
Pyrazinamide can cause hyperuricemia and gout. Uric acid levels may need to be monitored in patients at risk who are receiving pyrazinamide. Key side effects o riampin are rash, drug interactions due to induction o hepatic microsomal enzymes, hepatotoxicity, and orange secretions. Key side effects o isoniazid are hepatotoxicity, peripheral neuropathy Lupus-like syndrome, and monoamine (histamine-tyramine) poisoning. Generally, pyridoxine has no side effects except or remote peripheral neuropathy. Side effects o ethambutol include retrobulbar neuritis and decreased red-green color discrimination. ��. Answer c.
Tere are many drug interactions with riampin that have potentially serious consequences. Riampin is an important inducer o hepatic cytochrome P��� enzymes, which decrease the effects o drugs such as glyburide, verapamil, wararin, phenytoin, and many others. Tese interactions should always be considered when the use o riampin is begun or stopped since doses may need to be adjusted or medications may need to be changed. For example, verapamil may need to be changed to an alternative antihypertensive medication. ��. Answer e.
Tese cases ocus on the de�nitions o a positive PPD skin test according to the guidelines o the Centers or Disease Control and Prevention. A S result o more than � mm is positive in a person who meets any o the ollowing criteria: is positive or HIV, had recent contact with a person who had pulmonary tuberculosis (ie, a new conversion), has a chest radiograph consistent with old untreated tuberculosis, or has received an organ transplant or tumor necrosis actor inhibitor treatment. ��. Answer a.
Tis patient’s treatment was sel-administered or only � week, and he is still symptomatic; thus, he should be
�. INFECIOUS DISEASES QUESIONS AND ANSWERS
•
��
considered inectious. Te other patients are no longer considered inectious. ��. Answer a.
Te patient has large joint monoarticular arthritis and in�ammatory synovial �uid. Te leukocyte count o �����/L with ��% polymorphonuclear cells suggests in�ammation more than purulent inection. All the choices except acute sickle cell crisis could give this clinical picture. Te differential diagnosis includes crystalline and inectious arthritis. Sickle cell crisis usually causes a hemorrhagic �uid and is not associated with this type o cell count unless coinected with Salmonella . Gonococcal arthritis may appear in�ammatory and in a patient with another sexually transmitted disease (HIV), this should be considered. Te leukocyte count is usually greater than �����/L (typically >��% polymorphonuclear cells). In gonococcal arthritis, intracellular gram-negative diplococci are ound in less than ��% o synovial �uid aspirates. Te synovial �uid should be cultured on prewarmed chocolate agar or highest yield (positive �ndings in only ��% o patients with gonococcal arthritis and ��% to ��% o patients with disseminated gonococcal inection). Genital and oral sites should be cultured or Neisseria gonorrhoeae. ��. Answer c.
Te patient is clinically stable, so empirical �uconazole or ertapenem is not indicated. Since he does not have a mass lesion, empirical treatment or toxoplasmosis is inappro priate. His symptoms are consistent with possible central nervous system cryptococcosis. His CD� count is less than ��� cells/μL, placing him at risk. A CSF examination with a cryptococcal antigen test is most appropriate, and a lumbar puncture should be sae in the absence o papilledema. I the CSF is normal, an MRI can be reconsidered to look or small lesions or other abnormalities that may be missed on the C scan. ��. Answer c.
Medications and herbs are not known causes o alse-positive HIV antibody tests. Laboratory error can occur but is unusual, and positive results should be con�rmed. Acute inection can lead to cross-reacting antibodies and, occasionally, to a alse-positive ELISA result; however, the Western blot is very speci�c, and it should be negative or indeterminate i a patient is not inected with HIV and has not participated in a vaccine trial. Patients are ofen reluctant to speak openly about their risk actors or HIV because o the stigma attached to many o these behaviors and a desire to be respected by their caregivers. Te Centers or Disease Control and Prevention recommends that all patients undergo screening at least once, regardless o risk, between ages �� and ��. Pneumonia (particularly pneumococcal pneumonia) is more common in patients with HIV than in uninected adults even when the CD� count is relatively preserved. ��. Answer c.
Symptomatic acute HIV inection occurs in up to ��% o persons with HIV. A mononucleosis-like syndrome is ��
•
recognized as the most common presentation, but other symptoms, including prominent gastrointestinal tract symptoms, requently occur. Unortunately, acute HIV is ofen overlooked as a possibility, even when patients seek medical care. Symptomatic acute HIV does spontaneously resolve, and, i acute HIV is being considered, observation is not appropriate. HIV antibody testing with the acute retroviral illness should be negative or indeterminate. HIV antibody testing will usually seroconvert by � months but may take up to � months afer exposure to become positive. Patients should have genotyping done at diagnosis since resistance is becoming more common. HIV ofen reverts back to wild-type virus, although the resistant virus is still present in the patient. Genotyping is thereore recommended as soon as possible afer diagnosis. Many patients who engage in behaviors that place them at risk o acquiring HIV deny being at risk because o the stigma attached to these behaviors and because others do not know that they are at risk since they do not realize that their partner is engaging in risky behavior or is HIV positive. ��. Answer b.
NRIs have been associated with asymptomatic, mild ele vations in lactate levels (which can generally be ignored) and with more severe symptomatic lactic acidosis, which can lead to proound illness and death. Te most common symptoms o lactic acidosis in HIV-inected patients are the subacute development o nausea, vomiting, abdominal pain, atigue, weakness, and weight loss. achypnea, dys pnea with exertion, arrhythmias, and neurologic �ndings have also been reported in the absence o gastrointestinal tract symptoms. Liver test results are generally abnormal. Te combination o stavudine and didanosine, although popular in the past, is no longer recommended because it carries an increased risk o lactic acidosis and neuropathy. Patients with certain risk actors (preexisting liver disease, lower CD� counts, decreased glomerular �ltration rate, and being emale or pregnant) seem to be at increased risk o symptomatic lactic acidosis. Stavudine is thought to be the NRI most strongly associated with lactic acidosis, but all NRIs can probably cause lactic acidosis and the more NRIs used at a time, the more likely lactic acidosis is to occur. ��. Answer b.
A case control study has shown a ��% decreased risk o acquiring HIV rom use o zidovudine or � weeks promptly afer a percutaneous exposure. Zidovudine in combination with lamivudine should be equally or more effective. enoovir in combination with emtricitabine is an alternative that needs to be taken only once daily. Prophylaxis is recommended or � weeks, and there is no inormation that a longer period will result in ewer inections afer exposure. Te addition o a third drug (generally a protease inhibitor) should be considered when the patient has known drug resistance, or the risk o transmission is elt to be high because o the type o exposure (ie, deep puncture wound, hollow needle with visible blood, or
MAYO CLINIC INERNAL MEDICIN E BOARD REVIEW: QUESIONS AND ANSWER S
a very high viral load). Te estimated risk o acquiring HIV rom a needlestick is � in �,���, but, as noted above, some exposures are more risky than others. In animal studies, prompt initiation o treatment (within hours) was associated with decreased transmission; initiation afer �� hours is unlikely to be o any bene�t. ��. Answer d.
Trush is an indication or PCP prophylaxis. MP-SMX prophylaxis needs to be restarted, and the patient should restart antiretroviral medication as soon as possible. A viral load with genotype should �rst be determined to guide therapy. It is likely that his seborrheic dermatitis, atigue, and diarrhea will improve with immune reconstitution. His past history o cryptococcal meningitis puts him at high risk o a recurrence when his CD� count is less than ��� cells/ μL, and �uconazole therapy should be restarted despite his lack o symptoms i his CD� count is less than ��� cells/ μL. Azithromycin is not indicated unless his CD� count is less than �� cells/μL and symptomatic MAI inection has been excluded. reatment o his seborrheic dermatitis with ketoconazole cream and shampoo would be reasonable therapy. ��. Answer b.
I used correctly, latex condoms decrease the risk o acquiring or transmitting HIV by ��% among heterosexuals who consistently use a condom. Condom ailure may be more common with anal intercourse. Nonoxynol-� spermicide does not decrease the risk o acquiring HIV and can increase the risk in women by causing mucosal irritation. Using a condom with nonoxynol-� is saer than not using a condom. A randomized study o men who have sex with men showed that continuous treatment with tenoovir in combination with emtricitabine decreased the relative risk o acquiring HIV by ��% overall and by ��% among those who reported over ��% adherence. Circumcision signi�cantly decreases the risk o HIV acquisition by men in heterosexual relationships by ��%. Insertive anal or vaginal sex is less risky or acquiring HIV than receptive anal or vaginal sex. ��. Answer c.
Tis elderly man has had recurrent episodes o urinary tract inection. In the absence o a neurogenic bladder or other secondary cause, this almost always re�ects an inected prostate. Te diagnosis o chronic prostatitis is a clinical one, based on symptoms and signs o recurrent urinary tract inection as seen in this patient. Antibiotics that penetrate well into the prostate include �uoroquinolones, trimethoprim-sulamethoxazole, and doxycycline. Amoxicillin and nitrourantoin penetrate poorly into the prostate. Te duration o therapy should be prolonged to reduce the rates o recurrence. Tus, a ��-day course o cipro�oxacin would be the best choice or management o this patient’s chronic prostatitis due to Escherichia coli. ��. Answer b.
Tis diabetic woman rom India has a large abscess in the lef lobe o her liver. Since blood cultures have been
negative in the absence o antimicrobial therapy, a bacteremic seeding o the liver is less likely. Common bacterial causes o liver abscess include enteric organisms rom the biliary tree, viridans group streptococci rom the digestive tract, and Staphylococcus aureus secondary to bacteremia. In India, Entamoeba histolytica inection is common and may maniest as a ebrile illness with a ocal liver mass. Te best test is blood serology or E histolytica. Ultrasound-guided needle aspiration is useul or many liver lesions but should be delayed pending the serology results. A stool test or ova and parasites is not speci�c, and ameba may not be seen at the stage o liver abscess. Empirical antibiotic therapy should be avoided until the diagnosis is established. A PPD skin test is likely to be positive and not helpul in establishing the cause o the liver lesion. ��. Answer c.
Tis patient is pregnant and has asymptomatic bacteriuria with a gram-negative rod. She appears to be in the second trimester. Asymptomatic bacteriuria in pregnancy is de�ned as more than ���,��� colony-orming units per milliliter and is associated with worse pregnancy outcomes. Tus, it is one o the ew situations in which asymptomatic bacteriuria should be treated. Cipro�oxacin and other �uoroquinolones are contraindicated in pregnancy and should be avoided. wo weeks o trimethoprim-sulamethoxazole is excessive or asymptomatic bacteriuria. Cefriaxone is useul or treatment o urinary tract inections in pregnancy but is not needed or asymptomatic bacteriuria with a susceptible organism. Nitrourantoin can be saely used in pregnancy and a �- to �-day course is adequate or asymptomatic bacteriuria. A urine culture should be repeated � weeks afer completion o the treatment. ��. Answer c.
Tis woman has AML with prolonged ebrile neutropenia and a wedge-shaped peripheral pulmonary in�ltrate that has not resolved with broad-spectrum antibiotics. Te leading inectious cause is invasive ungal inection, particularly aspergillosis or, less requently, mucormycosis. Te appropriate therapy in the absence o voriconazole prophylaxis is the addition o empirical voriconazole to treat presumed invasive aspergillosis while attempting to establish the diagnosis. Caspoungin is ofen used as second-line or add-on therapy. Amphotericin B is more toxic and may be less effective. Fluconazole has no activity against Aspergillus, and itraconazole is less effective than voriconazole. ��. Answer d. Clostridium difficile is the most common cause o inec-
tious diarrhea in hospitalized patients. Tose who have � episode are predisposed to recurrences and those with � or more episodes are at highest risk or multiple relapses. Oral vancomycin is more effective than metronidazole or treatment o severe disease. Most relapses result rom reinection or germination o spores, not rom antibiotic resistance. VRE colonization ofen occurs in similar populations o patients who have received broad-spectrum
�. INFECIOUS DISEASES QUESIONS AND ANSWERS
•
��
antibiotics; however, there is no difference in VRE colonization whether patients receive vancomycin or metronidazole or treatment o C difficile inection. Because stool toxin assays may remain positive during and afer successul treatment, ollow-up stool toxin assays or test o cure should be avoided. ��. Answer d.
�-coated catheters and chlorhexidine patches, have also been effective. Coated catheters are used when other measures do not reduce the rate o central venous CRBSIs. Routine guidewire exchange o catheters does not reduce central venous CRBSI rates. (See O’Grady et al in the “Suggested Reading” list.) ��. Answer e.
Tis ��-year-old man was recently treated or gonorrhea and has secondary syphilis with multiple systemic eatures, including headache. He has a high positive RPR titer (>�:��), which should indicate the need to consider a lumbar puncture to exclude neurosyphilis beore initiating therapy. ��. Answer c.
Tis woman has acute uncomplicated pyelonephritis. She has been recently exposed to antimicrobials, so she is at increased risk or MP-SMX and ampicillin resistance. Tus, an oral �uoroquinolone is the drug o choice. Intravenous therapy is not necessary. Nitrourantoin, although effective or cystitis, is not appropriate or pyelonephritis. ��. Answer c.
Tis patient has tunnel inection o her indwelling catheter. She is not bacteremic. Essentially, this is a sof tissue inection with a device in place (similar to a pocket inection or a pacemaker). Te most appropriate management is removal o the catheter, along with antimicrobial therapy directed at the likely cause o the sof tissue inection: coagulase-negative staphylococci, streptococci, or Staphylococcus aureus. wo weeks o therapy without removal o the catheter is not satisactory, nor is lock therapy, which is useul or intraluminal inections but not tunnel inections. opical therapy is effective or only localized exit site inections. (See Mermel et al in the “Suggested Reading” list.) ��. Answer b.
Complete device removal is necessary in all cases o pacemaker inections regardless o clinical presentation (ie, pocket inection or endocarditis). Delaying device explantation until a blood culture is negative is not recommended. Because staphylococci ( Staphylococcus aureus and coagulase-negative staphylococci) are the most common pathogens, vancomycin is the preerred drug or empirical therapy. Ceazolin is not appropriate or empirical treatment because o the high rate o methicillin resistance in staphylococci (up to ��% in coagulase-negative staphylococci). Combination therapy with riampin is not recommended in pacemaker inections. (See Baddour et al, ����, and Sohail et al in the “Suggested Reading” list.) ��. Answer b.
Prevention o central venous CRBSIs has become a major national issue. Studies have shown that use o a bundle o evidence-based practices is effective at reducing central venous CRBSIs. Several different technologies, including ��
•
Tis patient has a brain abscess rom a contiguous ocus originating rom the sinuses. Te microorganisms to cover empirically are those ofen associated with sinus inections, which are usually polymicrobial inections involving streptococci, Haemophilus in�uenzae, staphylococci, and anaerobes. Because o increasing methicillin-resistant Staphylococcus aureus (MRSA) in the community, initial therapy should cover MRSA; thus, ceazolin would not be the best choice. Empirical antiungal therapy is not needed, nor is anti- Pseudomonas coverage. (See Carpenter et al in the “Suggested Reading” list.) ��. Answer d.
In ����, the American Heart Association issued revised guidelines or antimicrobial prophylaxis or endocarditis: Te only patients who should receive antibiotic prophylaxis are those who are scheduled to undergo invasive dental procedures and who have a prosthetic heart valve, a prior history o endocarditis, surgically uncorrected congenital heart disease, or cardiac valvulopathy afer undergoing cardiac transplant. In�asive dental procedures are de�ned as those that involve manipulation o gingival tissue or the periapical region o teeth or peroration o oral mucosa. Administration o antibiotic prophylaxis solely to prevent endocarditis is not recommended or patients undergoing gastrointestinal tract or genitourinary tract procedures. Te presence o a permanent pacemaker with transvenous cardiac leads is not an indication or antibiotic prophylaxis beore invasive dental procedures. (See Wilson et al in the “Suggested Reading” list.) ��. Answer d.
A patient with MRSA prosthetic valve endocarditis should receive antibiotic therapy or at least � weeks. With bio�lm ormation by staphylococci on prosthetic valves, the addition o riampin signi�cantly improves cure rates o S aureus prosthetic valve endocarditis. Gentamicin is recommended or the �rst � weeks o therapy, not or the entire �-week course. Daptomycin is not superior to vancomycin and is reserved or situations in which vancomycin cannot be used. (See Baddour et al, ����, in the “Suggested Reading” list.) ��. Answer e.
Tis diabetic man has pneumococcal pneumonia and meningitis. Te most appropriate empirical therapy includes the initial use o dexamethasone in combination with the � antimicrobials to ensure coverage or β-lactam–resistant pneumococci. Dexamethasone (�.�� mg/kg every � hours or �–� days) should be started beore or with the �rst dose
MAYO CLINIC INERNAL MEDICIN E BOARD REVIEW: QUESIONS AND ANSWER S
o antibiotics. Te greatest bene�t is achieved in the most severely ill patients. Tere is no bene�t when corticosteroids are given afer antibiotic therapy has already begun. Tere is no need to use acyclovir because the clinical presentation and the spinal �uid �ndings are not consistent with herpes simplex encephalitis. (See unkel et al in the “Suggested Reading” list.) SUGGESED RE ADING American Toracic Society; Inectious Diseases Society o America. Guidelines or the management o adults with hospital-acquired, ventilator-associated, and healthcare-associated pneumonia. Am J Respir Crit Care Med. 2005 Feb 15;171(4):388–416. Baddour LM, Epstein AE, Erickson CC, Knight BP, Levison ME, Lockhart PB, et al; American Heart Association Rheumatic Fever, Endocarditis, and Kawasaki Disease Committee; Council on Cardiovascular Disease in Young; Council on Cardiovascular Surgery and Anesthesia; Council on Cardiovascular Nursing; Council on Clinical Cardiology; Interdisciplinary Council on �uality o Care; et al. Update on cardio vascular implantable electronic device inections and their management: a scienti�c statement rom the American Heart Association. Circulation. 2010 Jan 26;121(3):458–77. Epub 2010 Jan 4. Baddour LM, Wilson WR, Bayer AS, Fowler VG Jr, Bolger AF, Levison ME, et al; Committee on Rheumatic Fever, Endocarditis, and Kawasaki Disease; Council on Cardiovascular Disease in the Young; Councils on Clinical Cardiology, Stroke, and Cardiovascular Surgery and Anesthesia; American Heart Association; Inectious Diseases Society o America. Inective endocarditis: diagnosis, antimicrobial therapy, and management o complications: a statement or healthcare proessionals rom the Committee on Rheumatic Fever, Endocarditis, and Kawasaki Disease, Council on Cardiovascular Disease in the Young, and the Councils on Clinical Cardiology, Stroke, and Cardiovascular Surgery and Anesthesia, American Heart Association: endorsed by the Inectious Diseases Society o America. Circulation. 2005 Jun 14;111(23): e394–434. Erratum in: Circulation. 2007 Apr 17;115(15):e408. Circulation. 2008 Sep 16;118(12):e497. Circulation. 2007 Nov 20;116(21):e547. Circulation. 2005 Oct 11;112(15):2373. Capelastegui A, Espana PP, �uintana JM, Areitio I, Gorordo I, Egurrola M, et al. Validation o a predictive rule or the management o community-acquired pneumonia. Eur Respir J. 2006 Jan;27(1):151–7. Carpenter J, Stapleton S, Holliman R. Retrospective analysis o 49 cases o brain abscess and review o the literature. Eur J Clin Microbiol Inect Dis. 2007 Jan;26(1):1–11.
Mandell LA, Wunderink RG, Anzueto A, Bartlett JG, Campbell GD, Dean NC, et al; Inectious Diseases Society o America; American Toracic Society. Inectious Diseases Society o America/American Toracic Society consensus guidelines on the management o community-acquired pneumonia in adults. Clin Inect Dis. 2007 Mar 1;44 Suppl 2:S27–72. Mermel LA, Allon M, Bouza E, Craven DE, Flynn P, O’Grady NP, et al. Clinical practice guidelines or the diagnosis and management o intravascular catheter-related inection: 2009 Update by the Inectious Diseases Society o America. Clin Inect Dis. 2009 Jul 1;49(1):1–45. Erratum in: Clin Inect Dis. 2010 Feb 1;50(3):457. Clin Inect Dis. 2010 Apr 1;50(7):1079. Dosage error in article text. Oehler RL, Velez AP, Mizrachi M, Lamarche J, Gomp S. Biterelated and septic syndromes caused by cats and dogs. Lancet Inect Dis. 2009 Jul;9(7):439–47. Erratum in: Lancet Inect Dis. 2009 Sep;9(9):536. O’Grady NP, Alexander M, Burns LA, Dellinger EP, Garland J, Heard SO, et al; Healthcare Inection Control Practices Advisory Committee (HICPAC). Guidelines or the prevention o intravascular catheter-related inections. Clin Inect Dis. 2011 May;52(9):e162–93. Epub 2011 Apr 1. �uintiliani R Jr, �uintiliani R. Inhalational anthrax and bioterrorism. Curr Opin Pulm Med. 2003 May;9(3):221–6. Sohail MR, Uslan DZ, Khan AH, Friedman PA, Hayes DL, Wilson WR, et al. Management and outcome o permanent pacemaker and implantable cardioverter-de�brillator inections. J Am Coll Cardiol. 2007 May 8;49(18):1851–9. Epub 2007 Apr 23. unkel AR, Hartman BJ, Kaplan SL, Kauman BA, Roos KL, Scheld WM, et al. Practice guidelines or the management o bacterial meningitis. Clin Inect Dis. 2004 Nov 1;39(9):1267–84. Epub 2004 Oct 6. Wilson W, aubert KA, Gewitz M, Lockhart PB, Baddour LM, Levison M, et al; American Heart Association Rheumatic Fever, Endocarditis, and Kawasaki Disease Committee; American Heart Association Council on Cardiovascular Disease in the Young; American Heart Association Council on Clinical Cardiology; American Heart Association Council on Cardiovascular Surgery and Anesthesia; �uality o Care and Outcomes Research Interdisciplinary Working Group. Prevention o inective endocarditis: guidelines rom the American Heart Association: a guideline rom the American Heart Association Rheumatic Fever, Endocarditis, and Kawasaki Disease Committee, Council on Cardiovascular Disease in the Young, and the Council on Clinical Cardiology, Council on Cardiovascular Surgery and Anesthesia, and the �uality o Care and Outcomes Research Interdisciplinary Working Group. Circulation. 2007 Oct 9;116(15):1736–54. Epub 2007 Apr 19. Erratum in: Circulation. 2007 Oct 9;116(15):e376–7.
�. INFECIOUS DISEASES QUESIONS AND ANSWERS
•
��
This page intentionally left blank
�. RHEUMATOLOGY QUE STIONS AND A NSWERS
QUESIONS
becomes atigued i she tries to comb her hair. She is a nonsmoker and denies having any respiratory com plaints. On examination, her temperature is ��.�°C. Her right radial pulse is decreased compared with the lef, and the blood pressure in her right arm is decreased compared with the lef. Laboratory studies show mild normochromic anemia with a hemoglobin o ��.� g/dL (reerence range >��.� g/dL), mildly elevated erythrocyte sedimentation rate (ESR) at �� mm/h (reerence range <�� mm/h), negative antineutrophil cytoplasmic autoantibody (ANCA) test, and normal blood chemistry panel results. Urinalysis and chest radiograph �ndings are normal. Which o the ollowing is the most likely diagnosis?
Multiple Choice (choose the best answer) N O N A R T I C U L A R R H E U M AT I S M A N D VA S C U L IT I S
�. A ��-year-old woman presents with complaints o new-onset headache. She is atigued and has lost about �.� kg over the past month. Te patient describes daily bitemporal headaches. On urther questioning, she describes morning stiffness lasting about � hour and aching in the shoulders and hips. She reports transient loss o vision in her right eye or about �� minutes yesterday beore her vision returned. Physical examination �ndings are unremarkable or her age. Initial laboratory studies show a mild normochromic anemia and elevated erythrocyte sedimentation rate (ESR) (��� mm/h). Which would be the most appropriate next step? a. b. c. d. e.
a. b. c. d. e.
Polymyalgia rheumatica (PMR) Giant cell arteritis (GCA) Buerger disease Wegener granulomatosis akayasu arteritis
�. A ��-year-old man has a �-week history o low back discomort. He says that he has been moving urniture over the past several weeks, but he does not recall a speci�c injury. Te pain is worse i he is active; it improves i he is at rest. He denies having pain radiating to the legs. Te patient has been taking ibuproen ��� mg twice daily with ood, and this seems to help. He has no prior history o lower back pain. Neurologic examination �ndings are normal, with a downgoing Babinski sign, equal and symmetrical knee jerks, and normal strength in the lower extremities. He has somewhat diffuse tenderness over the lumbar spine. Laboratory study results are normal or the complete blood cell count, erythrocyte sedimentation rate, and blood chemistry panel. What would be the most appropriate recommendation at this point?
Request a temporal artery biopsy. Prescribe prednisone � mg/kg orally daily. Prescribe prednisone � mg orally � times daily. Request computed tomography (C) o the head. Reer the patient to a neurologist.
�. A ��-year-old man presents with a �-week history o lef knee pain. He does not recall an injury. He says that the ront o his leg is tender. On examination, he is aebrile and overweight. Tere is no lef knee effusion, but the patient is markedly tender over the anterior medial tibia just distal to the knee joint. Tis area seems somewhat puffy but is not particularly erythematous. Te patient has good range o motion in both hips, but the area over the lef trochanteric bursa is tender. What would be the most appropriate step or pain relie ? a. Inject the lef knee joint with corticosteroid and local anesthetic. b. Inject the lef trochanteric bursa with corticosteroid and local anesthetic. c. Inject the lef pes anserine bursa with corticosteroid and local anesthetic. d. Prescribe propoxyphene �� mg orally every � hours. e. Prescribe prednisone �� mg orally daily or � days.
a. b. c. d. e.
Bed rest or � weeks Radiograph o the lumbar spine Electromyographic (EMG) study Neurologic consultation HLA-B�� testing
�. A ��-year-old woman presents to your office because she aches all over. She tells you that this condition has been present or several years but has gotten worse over the past � months. She has problems getting to sleep and does not eel rested when she wakes up. She denies having depression. She is stiff or � minutes in the morning and has not noticed any joint swelling. On examination,
�. A ��-year-old woman presents or evaluation o low-grade evers that have been present about � months. She also has lost �.� kg and has noted some arthralgias and myalgias. She says that her right arm
��
her body mass index (BMI) is ��. Her muscle strength is normal, and there is no synovitis. Tere is no rash. She has multiple tender points. Results o the ollowing laboratory studies are normal: complete blood cell count, erythrocyte sedimentation rate, blood chemistry panel, and sensitive thyrotropin. What would be the most appropriate next step? a. b. c. d. e.
Obtain an electromyogram (EMG). Prescribe prednisone �� mg orally daily. Prescribe duloxetine �� mg orally daily. Obtain overnight oximetry results. Prescribe oxycodone � mg orally every � hours, as needed or pain control.
�. A ��-year-old man has sinus drainage, cough, and hemoptysis. He also says that he has had joint pain or several weeks and swelling in the eet. On examination, he has synovitis o several proximal interphalangeal joints o the hands and bilateral lower extremity edema. Results o laboratory studies show a normochromic anemia (hemoglobin �.� mg/dL), an elevated erythrocyte sedimentation rate (�� mm/h), and an elevated creatinine level (�.� mg/dL). Te urinalysis shows proteinuria (�+) and red blood cell casts. Chest radiography shows multiple nodular lesions in both lungs. What would be the most appropriate next test to help establish a diagnosis? a. Renal biopsy b. Open lung biopsy c. Antineutrophil cytoplasmic autoantibody (ANCA) panel or vasculitis d. Rheumatoid actor test e. Cyclic citrullinated peptide (CCP) antibody test
�.
A ��-year-old woman presents with a several-week history o severe headaches and episodes that resemble a transient ischemic attack (IA). She has no history o smoking or o having asthma. On examination, she is aebrile but dysarthric. Her laboratory test results are normal or complete blood cell count with differential count, erythrocyte sedimentation rate, creatinine, and urinalysis. Te antineutrophil cytoplasmic autoantibody (ANCA) test is negative. Chest radiography is unrevealing, but magnetic resonance imaging o the head shows multiple areas suggestive o inarctions in several arterial distributions. What is the most likely diagnosis? a. b. c. d. e.
Wegener granulomatosis (WG) Giant cell arteritis (GCA) Buerger disease Isolated central nervous system (CNS) vasculitis Churg-Strauss vasculitis
O S T E O A R T H R I T I S , R H E U M AT O I D A R T H R I T I S , AND ANTIRHEUMATIC DRUGS
�. A ��-year-old woman has bothersome, but not disabling, osteoarthritis o her right knee. She has pain i she walks more than � blocks but denies locking, ��
•
catching, or giving way o the knees. She takes glucosamine chondroitin sulate (�,��� mg daily) and acetaminophen (up to �,��� mg � times daily as needed or pain). She has hypertension and mild renal insu�ciency (serum creatinine �.� mg/dL, reerence range ≤�.� mg/dL). She also has a history o coronary artery disease. On examination, she has a small amount o effusion in her right knee, mild genu varus deormity, and mild tenderness along the medial joint line o the right knee. Te knees appear stable on examination. Radiographs show medial joint space narrowing o the right knee. Which would be the best next step in her care? a. Obtain a magnetic resonance imaging (MRI) scan o the right knee. b. Administer an intra-articular corticosteroid injection. c. Administer a series o injections with hylan G-F ��. d. Prescribe naproxen ��� mg twice daily. e. Obtain an orthopedic consultation or possible arthroscopic surgery.
�. A ��-year-old man with rheumatoid arthritis (RA) has noticed an increase in small rheumatoid nodules on his hands over the past � months. Otherwise, he eels well. His RA has been well controlled with methotrexate ��.� mg weekly, etanercept �� mg subcutaneously weekly, olic acid � mg daily, and nabumetone ��� mg twice daily. On examination, he is aebrile. Tere are numerous small nodules on the extensor surace o the �ngers, but the patient’s RA appears quiescent, and he does not have active synovitis in the hands or wrists. Which o the ollowing would be the most appropriate recommendation? a. Stop taking nabumetone. b. Add trimethoprim-sulamethoxazole � double-strength tablet twice daily. c. Add prednisone �� mg daily. d. Decrease the methotrexate dosage to �.� mg weekly. e. Switch etanercept to in�iximab.
��. A ��-year-old woman with rheumatoid arthritis presents to your offi ce or optimization o her therapy. She is taking methotrexate �� mg weekly, prednisone � mg daily, and nabumetone ��� mg daily. She saw a television advertisement or in�iximab and wants to try it. She has a past medical history signi�cant or multiple sclerosis (MS) at age ��, which has not been active in recent years. On examination, she does have synovitis in several joints. Which o the ollowing would be most appropriate to tell her about her request to add in�iximab? a. She will need a puri�ed protein derivative (PPD) skin test beore starting in�iximab therapy. b. Etanercept would be a better choice or her. c. In�iximab can be given either orally or subcuteneously. d. In�iximab cannot be used with prednisone. e. In�iximab is relatively contraindicated in patients with a history o MS.
MAYO CLINIC INERNAL MEDICIN E BOARD REVIEW: QUESIONS AND ANSWER S
��. A ��-year-old woman comes to your office or mild knee discomort. She says that i she walks more than � miles, both knees hurt. With daily activity, she is not symptomatic. She currently is not taking any medications or her occasional knee pain. She is most interested in what she can do to prevent progression o her condition. On examination, you note mild varus deormity o both knees and tenderness over the medial joint line. What would be the best advice or her? a. b. c. d.
ake naproxen ��� mg twice daily. ake acetaminophen �,��� mg � times daily. ake glucosamine chondroitin sulate �,��� mg daily. Undergo magnetic resonance imaging (MRI) scanning o both knees. e. Undergo ultrasound-guided aspiration o � knee.
��. A ��-year-old woman has had seropositive rheumatoid arthritis or � years. She takes methotrexate �� mg on � day weekly, hydroxychloroquine ��� mg daily, olic acid � mg daily, and dicloenac-misoprostol �� mg up to twice daily as needed or joint pain. She says that these medications have helped her. She weighs �� kg and is normotensive. On examination, you �nd only � tender joints and no swollen joints. Radiographs o the hands and eet do not show any erosions. She was told by an ophthalmologist that she has early cataracts in both eyes. O the ollowing, what would be the best recommendation or her? a. Stop taking hydroxychloroquine. b. Increase the methotrexate dosage to �� mg weekly, and stop taking hydroxychloroquine. c. Increase the hydroxychloroquine dosage to ��� mg daily. d. Have an ophthalmologic examination at least annually while taking hydroxychloroquine. e. Stop taking the medications since therapy or rheumatoid arthritis is no longer needed.
��. A ��-year-old woman presents to your office or treatment o newly diagnosed rheumatoid arthritis (RA). She has had symptoms or about � months. She is not unctionally limited. So ar, she has been taking ibuproen ��� mg � times daily and has received some relie. On examination, she has � swollen joints and � tender joints. She does not have any rheumatoid nodules or other extra-articular eatures o RA. Te rheumatoid actor test is mildly positive, but the cyclic citrullinated peptide antibody test is negative. Her radiographs do not show any erosions. She would like to start therapy with a tumor necrosis actor (NF) inhibitor. What would be the best advice or this patient? a. b. c. d. e.
Start in�iximab � mg/kg intravenously. Start etanercept �� mg/wk subcutaneously. Start methotrexate �� mg orally on � day weekly. Additional therapy is not required now. Start glucosamine chondroitin sulate �,��� mg daily.
��. A ��-year-old man presents to you with a ��-week history o joint pain. His mother has severe rheumatoid arthritis (RA). On examination, he has �� swollen joints
and �� tender joints. Laboratory test results include the ollowing: mild normochromic anemia (hemoglobin ��.� g/dL), elevated erythrocyte sedimentation rate (�� mm/h), negative rheumatoid actor test (�� international units/mL, reerence range <�� international units/mL), weakly positive antinuclear antibody (ANA) test (�.� units, reerence range <�.� units), and an increased level o antibodies to cyclic citrullinated peptide (anti-CCP) (>��� units, reerence range <� units). What can you tell him rom these results? a. You are reerring him to a hematologist because o anemia. b. Patients with RA do not have positive ANA tests. c. Te anti-CCP results suggest that he is at higher risk o radiographic progression than RA patients who have negative anti-CCP results. d. Since his rheumatoid actor test is negative, it is unlikely that he has RA. e. Anti-CCP antibodies occur only in RA. SPONDYLOARTHROPATHIES
��. What is the goal with allopurinol therapy in the treatment o hyperuricemia in gout patients? a. Decrease the uric acid level to �.� mg/dL or less (reerence range ≤�.� mg/dL). b. Prevent urther gout attacks without regard to uric acid level. c. Decrease the uric acid level to less than �.� mg/dL (reerence range ≤�.� mg/dL). d. Decrease urinary uric acid excretion to normal. e. Prevent uric acid–induced renal disease.
��. In a patient with ankylosing spondylitis, which o the ollowing would be most helpul or treating back pain and preventing episodes o uveitis? a. Nonsteroidal anti-in�ammatory drugs b. Methotrexate c. Sulasalazine d. Etanercept e. Adalimumab
��. A ��-year-old man presents with a �-week history o pain and swelling o the right Achilles tendon and lef ankle. He has a previous history o uveitis on � occasions. He describes stiffness in his lower back, which is worse in the morning. Which disease best accounts or his symptoms? a. Ankylosing spondylitis b. Rheumatoid arthritis c. Lupus d. Gout e. Pseudogout
��. A ��-year-old woman experiences her �rst attack o gout in her great toe. She takes niedipine or hypertension, simvastatin or hyperlipidemia, and naproxen or osteoarthritis o her hips. Laboratory study results (and reerence ranges) include the ollowing: uric acid �.� mg/dL (<� mg/dL), creatinine �.� mg/dL (<�.� mg/dL),
�. RHEUMAOLOGY QUESIONS AND ANSWERS
•
��
and hemogloblin ��.� g/dL (>�� g/dL). Besides treating her with tapering doses o methylprednisolone, which o the ollowing should be done next? a. b. c. d. e.
Stop naproxen. Stop simvastatin. Begin colchicine. Begin allopurinol. Begin probenecid.
��. Acute monoarthritis o the lef knee developed in a ��-year-old man receiving long-term hemodialysis. Joint aspiration analysis is negative with Gram staining, and polarization microscopy shows bireringent bipyramidal crystals that stain with alizarin red, indicative o calcium. What is the most likely diagnosis? a. Gout b. Inection c. Basic calcium phosphate disease (hydroxyapatite deposition disease) d. Pseudogout e. Calcium oxalate arthropathy
��. A ��-year-old man has had a �-year history o progressive psoriatic arthritis unresponsive to nonsteroidal anti-in�ammatory drugs. On examination, he has in�ammatory arthritis, and radiographs show erosive changes in his hands and eet. He currently is being treated or hepatitis C inection, which he acquired rom a blood transusion �� years ago. Which o the ollowing would be the best treatment option at this time? a. Methotrexate b. Prednisone �� mg daily c. Prednisone �� mg daily d. Etanercept e. Rituximab
��. An ��-year-old woman had sudden onset o pain and swelling in her lef knee. Tere was no trauma. She had no previous episodes or history o ever. She was taking hydrochlorothiazide or hypertension. Joint aspiration analysis is negative with Gram staining, the synovial �uid leukocyte count is ��.����/L, and polarization microscopy shows weakly positive rhomboid bireringent crystals. Laboratory study results include the ollowing: hemoglobin ��.� g/dL, leukocyte count �.����/L, erythrocyte sedimentation rate �� mm/h, creatinine �.� mg/dL, and uric acid �.� mg/dL (reerence range <� mg/dL). Which o the ollowing is the cause o her acute monoarthritis? a. Gout b. Pseudogout c. Basic calcium phosphate disease d. Calcium oxalate arthropathy e. Septic joint
��
•
M I S C E L L A N E O U S R H E U M AT O L O G I C CONDITIONS
��. A ��-year-old man rom Rhode Island has bilateral intermittent painul knee effusions. Synovial �uid anal ysis is negative with Gram staining and culture, with a leukocyte count o �.����/L. No crystals are seen. Tere is no history o skin rash or low back pain. Tere is no other joint involvement. Afer a �ulike illness � years previously, the patient did have Bell palsy, which resolved. His ather has gout. Which o the ollowing diseases is most likely to account or his symptoms? a. Rheumatoid arthritis b. Lyme disease c. Systemic lupus erythematosus (SLE) d. Gout e. Spondyloarthropathy
��. A ��-year-old woman with a ��-year history o CRES syndrome presents with a �-month history o dyspnea with walking. A technetium c ��m sestamibi study with exercise does not show any evidence o ischemia. A chest radiograph is normal. Pulmonary unction test results are normal except or an isolated decrease in the diusing capacity o lung or carbon monoxide (D���) to ��% o the predicted value. Which o the ollowing would be the most likely diagnosis? a. b. c. d. e.
Deconditioning Atypical angina Recurrent pulmonary embolism Interstitial lung disease Pulmonary hypertension
��. A ��-year-old male intravenous drug user has arthralgias and biopsy-proven cutaneous leukocytoclastic vasculitis. Laboratory study results are shown in able �.Q��. Table �.Q�� COMPONEN
RESUL
Hemoglobin, g/dL
��.�
Leukocyte count, ���/L
�.�
Erythrocyte sedimentation rate, mm/h
��
Rheumatoid actor
�:���
C�
Low
Aspartate aminotranserase, U/L (AS)
� times upper limit o reerence range
Cryoglobulins
Positive
M A YO C L I N I C I N E R N A L M E D I C I N E B O A R D R E V I EW : Q U E S I ON S A N D A N S W E R S
Which o the ollowing tests would be most likely to establish the diagnosis? a. b. c. d. e.
Anti–cyclic citrullinated peptide Antinuclear antibody Anti-dsDNA antibody Human immunode�ciency virus (HIV) Hepatitis C serology
Table �.Q�� COMPONEN
Hemoglobin, g/dL
��.�
Leukocytes, ���/L
�.�
Platelets, ���/L
���
��. A ��-year-old woman with polymyositis began taking Erythrocyte sedimentation rate, mm/h prednisone �� mg daily � months ago. Her initial creatine kinase (CK) level was �,��� U/L (reerence range Creatinine, mg/dL <��� U/L). Now her CK is in the reerence range and her Anti-dsDNA antibody prednisone dose has been decreased to �� mg. She still has signi�cant proximal muscle weakness. What would C� be the best course o action at this time to improve her Anti-Sm antibody muscle strength? a. Increase the dose o prednisone. b. Continue to decrease the prednisone dose and have her begin physical therapy. c. Begin methotrexate. d. Begin azathioprine. e. Begin intravenous immune globulin.
��. A ��-year-old woman has a ��-year history o systemic lupus erythematosus. Multiple �ares o her disease during the past several years have necessitated high-dose prednisone therapy. She now presents with a �-week history o pain in the lef groin with ambulation. She denies having evers, chills, or other symptoms that she had equated with a �are in the past. Findings on routine radiography o her pelvis, including hips, are normal. Which o the ollowing would be most helpul in determining the cause o her symptoms? a. Electromyography b. Erythrocyte sedimentation rate and anti-nDNA antibody determination c. Magnetic resonance imaging (MRI) o the hips d. Empirical trial o a corticosteroid injection into the hip under �uoroscopy e. MRI o the lumbar spine
��. A ��-year-old woman with systemic lupus erythematosus is considering having a child. She is asymptomatic. She has no history o a previous pregnancy or o renal disease. She takes hydroxychloroquine ��� mg daily. On the basis o her laboratory study results (able �.Q��), which o the ollowing would she be at increased risk or during a pregnancy?
RESUL
� �.� Negative Normal Negative
Anti-RNP antibody
Negative
Anti–SS-A antibody
Positive
Anti–SS-B antibody
Negative
Antiphospholipid antibody
Negative
Antibodies
IgG, IgM
a. b. c. d. e.
Deep vein thrombosis Miscarriage secondary to placental inarction Lupus renal disease Pregnancy complications with etal heart block Hemolytic anemia
��. A ��-year-old man noted slowly progressive muscle weakness in his arms and legs over the past � years. He considers himsel healthy otherwise. He has had no pain. He takes simvastatin and hydrochlorothiazide. He is weak both proximally and distally. An electromyogram (EMG) shows both myopathic and neuropathic �ndings. Laboratory test results include the ollowing: hemoglobin ��.� g/dL, erythrocyte sedimentation rate � mm/h, creatinine �.� mg/dL, antinuclear antibodies �:�� (negative <�:��), and creatine kinase (CK) ��� U/L (reerence range <��� U/L). What is the most likely diagnosis? a. b. c. d. e.
Polymyositis Amyotrophic lateral sclerosis (ALS) Simvastatin toxicity McArdle disease Inclusion body myositis (IBM)
�. RHEUMAOLOGY QUESIONS AND ANSWERS
•
��
ANSWERS
�. Answer b.
�. Answer c.
In a patient o this age with a new onset o headache, anemia, and highly elevated ESR, giant cell arteritis (GCA) would be the number one consideration. emporal artery biopsy would be appropriate, but with the high degree o suspicion or GCA (because o the loss o vision), prednisone therapy should begin without waiting or the biopsy results. Te appropriate dose o prednisone or GCA is � mg/kg daily. Lower doses are appropriate or polymyalgia rheumatica. A C scan o the head would not be helpul or a patient with suspected GCA. Likewise, reerral to a neurologist would not be helpul.
Tis patient presents with �bromyalgia-like symptoms. Te patient does not have muscle weakness, so an EMG would not be indicated. Her symptoms, physical examination �ndings, and laboratory test results are not suggestive o in�ammation, so prednisone is not indicated. Duloxetine has been shown to be efficacious treatment or �bromyalgia in women. Although the patient’s sleep is nonrestorative, she is young and has a low BMI, so sleep apnea is not likely. She has a chronic pain syndrome, and it is usually best to avoid using narcotics to treat �bromyalgia pain.
�. Answer c.
�. Answer c.
Tis patient has pes anserine bursitis. Injection o the bursa is ofen helpul. Tere is not an indication to inject the lef knee joint. Although the area over the lef trochanteric bursa is tender, the patient is not symptomatic in this area, so there is no indication or an injection there. Propoxyphene has been taken off the market because o saety concerns, including accidental overdose, suicide, and arrhythmias. High-dose prednisone is typically not used or bursitis.
Tis patient has eatures o Wegener granulomatosis (WG) with upper and lower respiratory tract involvement and renal involvement. An ANCA panel would likely be positive in a c-ANCA pattern with a positive proteinase � enzyme-linked immunosorbent assay (PR� ELISA). A renal biopsy would be expected to show glomerulonephritis, which would not be speci�c or WG. An open lung biopsy likely would show histologic eatures o WG (granulomatous vasculitis), but the procedure is invasive and may not be necessary i the ANCA tests are positive. Patients with WG can have synovitis. Te rheumatoid actor and CCP tests are or rheumatoid arthritis. Patients with rheumatoid arthritis would not typically have hemoptysis, although they could have pulmonary involvement, and they would not usually have glomerulonephritis.
�. Answer e.
O the choices listed, this patient would most likely have akayasu arteritis. Tis would be most common in women younger than ��. Weight loss, arthralgias, myalgias, and low-grade evers are all common eatures. In addition, patients ofen present with upper extremity claudication. Te decreased radial pulse in the right arm compared with the lef and the decreased blood pressure in the right arm suggest upper aortic arch involvement. ypically, mild anemia is present. Some patients have active disease but a normal ESR. Many patients with akayasu arteritis have a negative ANCA test. Both PMR and GCA occur in an older age group. Buerger disease occurs in smokers. Patients with Wegener granulomatosis usually have respiratory symptoms and a positive ANCA test; they ofen have upper and lower respiratory involvement and kidney involvement.
�. Answer d.
Patients with WG usually have upper and lower respiratory involvement and ofen renal disease, which this patient does not have. Also, most patients with WG have a positive ANCA test. GCA would not occur in this age group and usually does not maniest with IAs. Buerger disease affects the peripheral circulation and occurs in smokers. Churg-Strauss vasculitis occurs in patients with a history o asthma and is usually associated with eosinophilia. Isolated CNS vasculitis can maniest with IAs and headaches. Ofen patients do not have other eatures o systemic vasculitis, so the term isolated CNS vasculitis is used. Most patients with isolated CNS vasculitis are ANCA negative.
�. Answer b.
New onset o lower back pain afer age �� would be an indication or radiograph. Bed rest or more than � days is not helpul. Tere is no suggestion o radiculopathy, so EMG and neurologic consultation are not indicated. Te patient is not describing in�ammatory back pain and is older than ��, so HLA-B�� testing or spondyloarthropathy would not be helpul.
�. Answer b.
Tis patient has osteoarthritis. Her examination, history, and radiographs do not suggest another process, so MRI is likely to yield little more inormation. Because she has ��
renal insufficiency, hypertension, and a history o coronary artery disease, nonsteroidal anti-in�ammatory drugs are relatively contraindicated. A corticosteroid injection would be a consideration, especially since she has pain in only � joint. I the patient had no response to the corticosteroid injection, viscosupplementation would be a consideration. Te patient does not have mechanical symptoms, so there is no clear indication or an arthroscopic procedure. �. Answer d.
Methotrexate can cause increased nodulosis in some RA patients. Depending on how problematic it is, one may need to consider decreasing the dose or even stopping the medication. rimethoprim-sulamethoxazole can interact with methotrexate and potentially cause toxicity, so it should be avoided, especially i there is not a speci�c reason to add it. Te RA appears relatively quiescent, so there would not be a need to add high-dose prednisone. Likewise, there would not be a reason to switch to a different tumor necrosis actor inhibitor. ��. Answer e.
All the tumor necrosis actor (NF) inhibitors have been reported to potentially cause demyelinating lesions in patients. rials o in�iximab in patients with MS lead to worsening o the disease. Tereore, NF inhibitors are relatively contraindicated or patients with a history o MS. In�iximab can be used in patients who are taking nonsteroidal anti-in�ammatory drugs, methotrexate, and prednisone. A PPD skin test should be checked beore starting any o the NF inhibitors because tuberculosis has reactivated in some patients. In�iximab is given by intravenous inusion only. ��. Answer c.
Nonsteroidal anti-in�ammatory drugs and acetaminophen are analgesics, but they do not prevent progression o osteoarthritis. Tere is some support rom evidence-based medicine or the use o glucosamine chondroitin sulate to retard progression o osteoarthritis o the knee. Tere is no indication or MRI or ultrasound-guided arthrocentesis o the knee. ��. Answer d.
Hydroxychloroquine is typically used at a dosage o up to �.� mg/kg daily. Higher doses may be associated with increased ocular toxicity and so are generally avoided. Te toxicity is retinal toxicity, and hydroxychloroquine use is not associated with cataract ormation. Patients taking hydroxychloroquine should have an ophthalmologic examination at least annually. Patients with rheumatoid arthritis usually require long-term therapy with a disease-modiying antirheumatic drug. Tis patient’s disease is well controlled with the current therapy, so there is no indication to change her regimen. ��. Answer c.
According to present treatment guidelines, initiation o a NF would not be indicated or this patient as the �rst
therapy. It would be appropriate to start a disease-modiying antirheumatic drug, so methotrexate would be a reasonable choice. Glucosamine chondroitin sulate has been shown to be potentially helpul in osteoarthritis o the knee, but there is no support rom evidence-based medicine that it helps RA. ��. Answer c.
Te patient likely has an anemia o chronic disease related to in�ammatory arthritis. High levels o anti-CCP antibodies are speci�c or RA; occasionally, lower values are present with other conditions, including systemic lupus erythematosus. A positive ANA test, especially a weakly positive result such as this patient’s, is common in RA. Te anti-CCP test is more speci�c than the rheumatoid actor test or RA but, depending on the assay used, may be less sensitive than the rheumatoid actor test. Anti-CCP antibodies sometimes occur in patients who are rheumatoid actor–negative; occasionally, anti-CCP antibodies occur beore rheumatoid actor occurs in patients with RA. Te presence o anti-CCP antibodies has been associated with more radiographic progression in patients with RA. ��. Answer c.
Te goal o allopurinol therapy in the treatment o gout is to decrease the uric acid level to less than �.� mg/dL. Te solubility quotient o uric acid is �.� mg/dL. Tereore, decreasing the uric acid level to less than �.� mg/dL promotes shrinkage o tophi and results in better clinical effi cacy. ��. Answer e.
Although both etanercept and adalimumab would be helpul in treating the back pain, only adalimumab has been shown to decrease the requency o uveitis. ��. Answer a.
Achilles tendinitis (enthesopathy), low back pain with morning stiffness, and uveitis are characteristic o spondyloarthropathies, such as ankylosing spondylitis. Te other diseases do not predispose characteristically to back pain, uveitis, or enthesopathy. ��. Answer a.
Naproxen should be stopped because o the elevated creatinine. Stopping naproxen may also result in decreased creatinine and uric acid levels. Only i the patient continued to have recurrent attacks o gout would allopurinol be used to lower the uric acid. With an elevated creatinine, probenecid would not be effective. ��. Answer e.
Calcium oxalate arthropathy usually occurs in patients receiving long-term hemodialysis, and the bipyramidal crystals stain positive or calcium. Dialysis patients are also at increased risk or basic calcium phosphate disease, gout, and a septic joint. Negative Gram staining rules against a
�. RHEUMAOLOGY QUESIONS AND ANSWERS
•
��
septic joint, and no urate crystals indicative o gout were seen. In basic calcium phosphate disease, crystals are seen only with electron microscopy, not with polarization microscopy. ��. Answer d.
Etanercept is approved or both psoriasis and psoriatic arthritis and has no deleterious effect on viral titers or treatment o hepatitis C. Methotrexate is contraindicated or a patient with hepatitis C. A high dose o prednisone can increase viral titers, and a low dose o prednisone should be used only to give temporary bene�t until a disease-modiying antirheumatic agent provides bene�t. Rituximab is not approved or psoriatic arthritis. ��. Answer b.
Weakly positive rhomboid bireringent crystals are characteristic o pseudogout. Tis patient had an elevated uric acid level, but monosodium urate crystals, which are needle shaped and strongly negative bireringent, were not seen. Gram staining was negative, which rules against a septic joint. Calcium oxalate crystals are bipyramidal. In basic calcium phosphate disease (calcium hydroxyapatite), no crystals are seen with polarization microscopy. ��. Answer b.
Not all patients with Lyme disease recall a tick bite or have erythema chronicum migrans. Tis patient had �ulike symptoms and Bell palsy several years beore the onset o his Lyme arthritis, which characteristically involves the knees several years afer the initial inection. Bell palsy is a characteristic neurologic maniestation o Lyme disease. Rheumatoid arthritis would be less likely without polyarticular involvement and would not be associated with Bell palsy. No crystals were seen with joint aspiration, which would rule against gout. Tere is no history o low back pain or uveitis, which would make a spondyloarthropathy unlikely. Tere is no clinical history to suggest SLE. ��. Answer e.
Pulmonary hypertension is more likely to develop in patients with CRES syndrome than in patients with diuse systemic sclerosis (scleroderma). Te isolated decrease
��
•
in the D��� suggests pulmonary hypertension, and the patient should undergo right heart catheterization. ��. Answer e.
Hepatitis C inection can cause cryoglobulinemia, which is responsible or the positive rheumatoid actor and cryoglobulinemic vasculitis. Te low C� is also associated with the cryoglobulinemic vasculitis. Te elevated AS is indicative o ongoing hepatitis liver disease. Rheumatoid arthritis, systemic lupus erythematosus, and HIV do not characteristically cause cryoglobulinemia. ��. Answer b.
Tis patient has steroid myopathy. With the CK value being normal, the polymyositis is under good control. Tere is no reason to add a steroid-sparing agent at this time. A slow, continued corticosteroid taper in conjunction with physical therapy is indicated. ��. Answer c.
Te most helpul test would be MRI o the hips. Tis patient has � risk actors or avascular necrosis o the hip: �) systemic lupus erythematosus and �) prednisone use. ��. Answer d.
Te positive results or anti–SS-A antibody give this patient a �% risk o a complete heart block developing in the etus in the second or third trimester. Tere are no antiphospholipid antibodies, so there is no increased risk o deep vein thrombosis or miscarriage. Te patient has had no previous renal disease, and with no anti-dsDNA there is not an increased risk o active renal disease during pregnancy. ��. Answer e.
IBM occurs in older patients and causes both proximal and distal muscle weakness. In contrast, polymyositis causes predominantly proximal weakness. In IBM, the elevation o CK values is usually relatively small, and characteristically the EMG has both myopathic and neuropathic �ndings. Polymyositis, simvastatin toxicity, and McArdle disease would not show neuropathic �ndings on EMG. ALS can be associated with an elevated CK, but the muscle weakness is usually asymmetrical and EMG would show only neuropathic �ndings.
MAYO CLINIC INERNAL MEDICIN E BOARD REVIEW: QUESIONS AND ANSWER S
�. ENDOCRINOLOGY QUESTIONS AND ANSWERS
QUESIONS
�. A ��-year-old man has new-onset atrial �brillation and abnormal results on thyroid tests (thyrotropin <�.�� mIU/L, ree thyroxine �.� ng/dL). He begins antiarrhythmic therapy and is reerred to you. He reports palpitations and weight loss or � months along with redness, swelling, and pain over his eyes. His past medical history is signi�cant or hypertension, congestive heart ailure, and ongoing nicotine dependence. On examination, his heart rate is irregular at �� beats per minute, his blood pressure is ���/�� mm Hg, and his lungs are clear. Te thyroid is nontender, about twice the normal size, and increased in consistency. He has bilateral exophthalmos with red and signi�cantly swollen eyelids and injected conjunctiva. Tere is pitting edema (trace) o the lower extremities and an area o brownish thickening o the skin over the pretibial areas. What is the best management or his thyroid dysunction at this point?
Multiple Choice (choose the best answer) THYROID DISEA SE
�. A ��-year-old armer is evaluated as a new patient. He has a ��-year history o hypothyroidism and hypogonadism. He takes levothyroxine (�� mcg daily) and testosterone gel �% (�.� g daily) and eels well. He weighs �� kg and has a small thyroid. Physical examination �ndings are unremarkable. Laboratory test results include the ollowing: thyrotropin �.� mIU/L (reerence range, �.�–�.� mIU/L) and ree thyroxine �.� ng/dL (reerence range, �.�–�.� ng/dL). You decide to decrease the levothyroxine dosage to �� mcg daily. Afer � months, thyrotropin was �.� mIU/L, ree thyroxine was �.� ng/dL, and he eels tired and cold. Which o the ollowing should you do now?
a. Observation, antiarrhythmic treatment, and reevaluation in � weeks or possible hypothyroidism b. Propylthiouracil (PU) c. Methimazole treatment d. otal or near-total thyroidectomy e. Radioactive iodine treatment
a. Decrease the levothyroxine dosage to �� mcg daily and recheck in � weeks. b. Stop levothyroxine altogether and evaluate thyroid uptake or hyperthyroidism. c. Discontinue the testosterone gel since it increases the thyroxine-binding globulins. d. Check his thyroglobulin to distinguish between primary and secondary hypothyroidism. e. Continue levothyroxine at ��� mcg daily, get his old records, and consider cortisol testing.
DIABETES MELL ITUS, HYPOGLYCEMIA, AND HYPERLIPIDEMIA
�. A ��-year-old woman comes to your office or her routine �-month ollow-up or the management o type � diabetes mellitus. She has no immediate concerns. Over the past year, she has consulted a dietitian and has been trying to adhere to her diabetic diet. She has also started a walking program. Her medications include metormin �,��� mg twice daily, simvastatin �� mg daily, lisinopril �� mg daily, a multivitamin daily, and a calcium supplement daily. She monitors her blood glucose levels at home in the morning � or � times per week, and the metered glucose values (in milligrams per deciliter) are in the mid ���s. She denies having hypoglycemia. She works outside the home but eats meals at regular times. On physical examination, her blood pressure is ���/�� mm Hg, her heart rate is �� beats per minute, her height is ��� cm, her weight is �� kg, and her body mass index is ��. Other examination �ndings are unremarkable. Laboratory test results include hemoglobin A�c (HbA�c) �.�% and normal serum creatinine and liver transaminase values. What is your best management option to improve this patient’s glycemic control?
�. A ��-year-old woman has a �-year history o primary hypothyroidism due to Hashimoto thyroiditis. She has been treated with ��� mcg o levothyroxine with a good clinical and biochemical response so ar. A ew months ago, she decided to start taking oral contraceptives because she had persistently irregular periods related to polycystic ovary syndrome. Her periods are regular now, but she eels tired and has cold intolerance. Her thyrotropin is ��.� mIU/L; ree thyroxine is �.� ng/dL. Which o the ollowing should you do next? a. Check the titer o thyroperoxidase antibodies to assess the severity o Hashimoto thyroiditis. b. Advise the patient to improve her adherence to the medical therapy and recheck in � to � months. c. Advise the patient to take the oral contraceptive �� hours afer taking levothyroxine, and recheck in � to � months. d. Add liothyronine to the levothyroxine or a better clinical response. e. Increase the levothyroxine dose to ��� mcg, and recheck thyrotropin in � to � months. ��
a. Make no changes in her current glucose-lowering regimen, and congratulate her. b. Increase the metormin dose to �,��� mg daily, with a goal HbA�c o less than �%. c. Add a sulonylurea to her regimen, with a goal HbA�c o less than �%, and discuss the risk o hypoglycemia. d. Add rosiglitazone to her regimen, with a goal HbA�c o less than �%, and discuss the risk o heart ailure and cardiovascular events. e. Add a single injection o insulin glargine to her regimen, with a goal HbA�c o less than �%, and discuss the risk o hypoglycemia.
�. A ��-year-old man comes to your office or a routine physical examination. He eels well but is concerned about his amily medical history. Both his mother and his ather have type � diabetes mellitus, and his ather (age �� and an ex-smoker) had coronary bypass surgery recently. Te patient’s medical history is positive or hypertension. He is a nonsmoker. Current medications include hydrochlorothiazide-triamterene �� mg/��.� mg � tablet daily and �sh oil capsules �,��� mg daily. On physical examination, his pulse is �� beats per minute, his blood pressure is ���/�� mm Hg, his height is ��� cm, his weight is ��� kg, and his body mass index is ��. He has central obesity. Te other cardiac and physical examination �ndings are unremarkable. Diagnostic test results are shown in able �.Q�. Table �.Q� COMPONEN
���
Hemoglobin A�c, %
�.�
Serum creatinine, mg/dL
�.�
otal cholesterol, mg/dL
���
riglycerides, mg/dL
���
High-density lipoprotein cholesterol, mg/dL
��
Calculated low-density lipoprotein cholesterol, mg/dL
���
Non–high-density lipoprotein cholesterol, mg/dL
���
Which o the ollowing statements about this patient is false? a. He has prediabetes, and his �-year risk or diabetes mellitus can be as high as ��%. b. Liestyle changes that lower his intake o saturated ats, increase his exercise, and cause modest weight loss can lower his risk or type � diabetes mellitus by ��%. c. His lipid pro�le puts him at increased risk or a premature cardiac event.
•
�. A ��-year-old woman with a history o type � diabetes mellitus or the past � years comes to your office with complaints o worsening paresthesias involving her hands. She has peripheral neuropathy involving her lower extremities; the neuropathy is unchanged and has been attributed to diabetes. She eels more atigued, which she attributes to poor sleep as a result o her neuropathic symptoms. Additional medical diagnoses are hypertension and hyperlipidemia. She is up-to-date or all her medical preventive services. Her current medications are metormin �,��� mg twice daily, lisinopril �� mg daily, and atorvastatin �� mg daily. Physical examination �ndings include the ollowing: height ��� cm, weight ��.� kg, body mass index ��, pulse �� beats per minute, blood pressure ���/�� mm Hg, dry skin on the lower extremities, no areas o skin breakdown, decreased sensation to mono�lament in the lower extremities to the ankles, and absent ankle re�exes bilaterally. Te upper extremities have normal hand grip and normal re�exes, but the patient describes paresthesias. Laboratory test results include the ollowing: hemoglobin ��.� g/dL, hemoglobin A �c �.�%, and normal values or serum creatinine, liver transaminases, and serum thyrotropin. What should be your next step in the evaluation and management o this patient? a. b. c. d.
VALUE
Fasting blood glucose, mg/dL
��
d. Insulin sensitizers such as metormin and rosiglitazone have been shown to slow the progression to type � diabetes mellitus with minimal risk.
�.
Obtain an electromyogram. Determine the vitamin B�� level. Start therapy with a tricyclic antidepressant. Intensiy her glucose-lowering therapy by adding a second glucose-lowering agent.
A ��-year-old man with type � diabetes mellitus comes to your office or a routine visit. He has a history o coronary artery disease and had a myocardial inarction � years ago. He is currently ree o any cardiac symptoms. He is an ex-smoker. He monitors his blood glucose levels daily beore breakast, and the majority o values are less than ��� mg/dL. He denies having hypoglycemia. Current medications are metormin �,��� mg twice daily, glimepiride � mg twice daily, simvastatin �� mg daily, metoprolol �� mg daily, quinapril �� mg daily, and aspirin �� mg daily. Physical examination �ndings include the ollowing: height ��� cm, weight ��� kg, pulse �� beats per minute, blood pressure ���/�� mm Hg, body mass index ��, overweight with central obesity, systolic ejection murmur �/� at the lef sternal border, and decreased sensation to pinprick in the eet up to the ankles with absent ankle re�exes bilaterally. Laboratory test results are shown in able �.Q�.
MAYO CLINIC INERNAL MEDICIN E BOARD REVIEW: QUESIONS AND ANSWER S
Table �.Q� COMPONEN
VALUE
Hemoglobin A�c, %
�.�
Serum creatinine, mg/dL
�.�
otal cholesterol, mg/dL
���
riglycerides, mg/dL
���
High-density lipoprotein cholesterol, mg/dL
��
Calculated low-density lipoprotein cholesterol, mg/dL
��
Non–high-density lipoprotein cholesterol, mg/dL
���
Which intervention is associated with a lower risk or a cardiovascular event? a. Increasing his aspirin dosage to ��� mg daily b. Adding eno�brate to his lipid-lowering regimen c. Adding insulin to improve his glycemic control until the hemoglobin A�c is less than �.�% d. Increasing his simvastatin dosage to �� mg daily e. Continuing metormin therapy
�. A ��-year-old woman is seen in your office accompanied by her husband or ollow-up afer dismissal rom her local emergency department (ED) or urther evaluation o reactive hypoglycemia. For years, she has had intermittent symptoms o diaphoresis and excessive hunger that resolve afer eating. She remembers having a glucose tolerance test, during which her blood glucose level decreased to about �� mg/dL. She was advised to see a dietitian, eat regularly, and always have hard candy available to abort her symptoms. She has been experiencing more requent symptoms and has gained weight over the past year. She is healthy otherwise. She has no history o diabetes. She is currently taking no medications. Te day o her evaluation in the ED, she was involved in her usual activities at home when she experienced her usual symptoms related to hypoglycemia, diaphoresis, and hunger. She recalled missing breakast but does not recall any additional events until she was in the ED. Her husband recognized that she was not well and was worried that she was having a stroke, so he called the paramedics. She was sweaty and would not respond to his questions. At the ED, several laboratory tests were perormed. She was told that her blood glucose was low. She was given �uids and dextrose intra venously. She was dismissed and advised to ollow up as an outpatient. Which test results support abnormal endogenous insulin secretion as a cause or hypoglycemia (reerence ranges: C peptide, �.�–�.� ng/mL; β-hydroxybutyrate, <�.� mmol/L)? a. Glucose �� mg/dL, C peptide �.� ng/mL, β-hydroxybutyrate �.� mmol/L b. Glucose �� mg/dL, C peptide �.� ng/mL, β-hydroxybutyrate �.� mmol/L
c. Glucose �� mg/dL, C peptide �.� ng/mL, β-hydroxybutyrate �.� mmol/L with negative results or the sulonylurea screening test d. Glucose �� mg/dL, C peptide �.� ng/mL, β-hydroxybutyrate �.� mmol/L e. Glucose �� mg/dL, C peptide �.� ng/mL, β-hydroxybutyrate �.� mmol/L with negative results or the sulonylurea screening test
�. A ��-year-old man comes to your office or ollow-up o his diabetes mellitus. Te diagnosis was made a year ago with a random blood glucose value above ��� mg/dL. Te patient has been struggling to adhere to the dietary recommendations and has been unable to establish an activity program. He monitors his blood glucose levels a ew times a week, generally in the morning beore breakast. All blood glucose values are over ��� mg/dL. He is a nonsmoker. He has no history o cardiovascular disease. His current medications are metormin �,��� mg twice daily, glimepiride � mg twice daily, atorvastatin �� mg daily, lisinopril �� mg twice daily, hydrochlorothiazide-triamterene �� mg/�� mg � tablet daily, and �sh oil capsules �,��� mg twice daily. Physical examination �ndings include the ollowing: height ��� cm, weight �� kg, body mass index ��, pulse �� beats per minute, and blood pressure ���/�� mm Hg. Laboratory test results are shown in able �.Q�. Table �.Q� COMPONEN
VALUE
Hemoglobin A�c, %
�.�
Serum creatinine, mg/dL
�.�
otal cholesterol, mg/dL
���
riglycerides, mg/dL
���
High-density lipoprotein cholesterol, mg/dL
��
Calculated low-density lipoprotein cholesterol, mg/dL
��
Non–high-density lipoprotein cholesterol, mg/dL
���
Which o the ollowing statements about the management o this patient is false? a. Use o pioglitazone (a thiazolidinedione [ZD]) should be avoided because o the higher risk o congestive heart ail ure and cardiovascular events. b. Use o glucagon-like peptide � analogues (exenatide, liraglutide) will improve glycemic control with the potential or weight loss. c. Use o dipeptidyl-peptidase-� inhibitors (sitagliptin, saxagliptin) will improve glycemic control without weight loss or gain. d. Insulin therapy will improve glycemic control but with a higher risk o weight gain and hypoglycemia.
�. ENDOCRINOLOGY QUESIONS AND ANSWERS
•
��
OBESITY AND NUTRITION
��. A ��-year-old woman comes to your office or her routine physical examination. She has no immediate concerns except that her weight has been increasing in the past ew years. She has a clerical job and a busy home lie. Her eating habits have not changed dramatically, although she admits that she eats at ast ood restaurants several times a week on her way to extracurricular activities. She currently has no exercise program, although she owns a treadmill. She is other wise healthy. She has regular menses. She has had tubal ligation. She has a amily history o type � diabetes mellitus; her mother received the diagnosis in her ��s. Te patient’s medications are vitamin D supplement ��� international units daily and calcium supplement ��� mg daily. Physical examination �ndings include the ollowing: height ��� cm, weight �� kg, body mass index ��, pulse �� beats per minute, and blood pressure ���/�� mm Hg. She is overweight with central obesity. Examination �ndings are otherwise unremarkable. Which is not an appropriate step in the management o this patient? a. Screen or hypothyroidism with a sensitive thyrotropin (sSH) test. b. Screen or type � diabetes mellitus by measuring the asting blood glucose level. c. Advise that she start a walking program o ��- to ��-minute sessions, working toward a goal o ��� minutes o walking per week. d. Recommend that she eat a low-carbohydrate diet, which is superior or promoting and maintaining a weight loss o �% to ��%.
a. She meets the National Institutes o Health consensus criteria or considering bariatric surgery. b. Laparoscopic adjustable gastric banding and Roux-en-Y gastric bypass are associated with similar outcomes or inducing remission o type � diabetes mellitus. c. Untreated obstructive sleep apnea is a risk actor or higher perioperative morbidity and mortality with bariatric surgery. d. Te use o nonsteroidal anti-in�ammatory drugs is contraindicated afer Roux-en-Y gastric bypass surgery.
��. You are called to the emergency department to see one o your patients. He is a ��-year-old man who under went a Roux-en-Y gastric bypass or the management o his obesity � months ago. He has severe nausea and vomiting and has not been able to keep any ood or �uids down or the past �� hours. He has abdominal pain, particularly in the epigastric region. He eels dizzy and complains o tingling and numbness o his hands. He has lost �� kg since his surgery. He has a diagnosis o type � diabetes mellitus and takes metormin, which was discontinued at his hospital dismissal afer surgery. Other diagnoses are hypertension, hyperlipidemia, and obstructive sleep apnea, or which he uses continuous positive airway pressure. Current medications (which he has not been able to take regularly) are simvastatin �� mg daily, losartan ��� mg daily, chewable multivitamin � tablet twice daily, calcium carbonate ��� mg � times daily, and vitamin B �� �,��� mcg subcutaneously monthly. Physical examination �ndings include the ollowing: height ���.� cm, weight ��� kg, pulse ��� beats per minute, blood pressure ���/�� mm Hg, and temperature ��.�°C. Te patient appears uncomortable and has requent dry heaves. Cardiac auscultation is unremarkable except or tachycardia. Lung sounds are normal. Te abdomen has hyperactive bowel sounds and is tender to palpation in the epigastric region. Tere are no acute peritoneal signs. Laboratory test results are shown in able �.Q��.
��. A ��-year-old woman comes to your office to discuss bariatric surgery. She has struggled to lose weight during most o her adult lie. She has participated in indi vidual and commercial weight loss programs with no long-term success at managing her weight, and she is concerned about her health. She has type � diabetes mellitus, hypertension, hyperlipidemia, and joint pain. Her joint pain and atigue interere with her ability to Table �.Q�� be physically active. She does not smoke or drink alcoCOMPONEN holic beverages. She has no prior psychiatric history. Afer she recently spoke to a riend who had bariatric Sodium, mg/dL surgery and is doing very well, she became motivated to consider this option. Her current medications are met- Potassium, mg/dL ormin �,��� mg twice daily, glipizide �� mg twice daily, Glucose, mg/dL atorvastatin �� mg daily, eno�brate ��� mg daily, osinopril �� mg daily, hydrochlorothiazide-triamterene Creatinine, mg/dL �� mg/��.� mg � tablet daily, aspirin �� mg daily, and Aspartate aminotranserase, U/L naproxen ��� mg twice daily as needed. Physical examination �ndings include the ollowing: height ��� cm, Alkaline phosphatase, U/L weight ��� kg, body mass index ��, pulse �� beats per Lipase, U/L minute, and blood pressure ���/�� mm Hg. She is overweight with central obesity. Examination �ndings Hemoglobin, g/dL are otherwise unremarkable. Which statement about Leukocyte count, ���/L bariatric surgery in the management o this patient is Platelet count, ���/L false? ��
•
VALUE
��� �.� ��� �.� �� ��� �� ��.� ��.� ���
M A YO C L I N I C I N E R N A L M E D I C I N E B O A R D R E V I EW : Q U E S I ON S A N D A N S W E R S
You started intravenous �uid hydration with �.�% sodium chloride solution and �� mEq o potassium per liter at ��� mL/h. You have consulted the general surgeon on call. Te patient is scheduled or computed tomography o the abdomen within the next hour. What is the most appropriate next step in the management o this patient? a. Give the patient ��� mg o thiamine intravenously. b. Call a gastroenterologist to perorm an emergent esophagogastroduodenoscopy or the possibility o an anastomotic stricture. c. Give the patient �,��� mcg o cyanocobalamin (vitamin B��) intramuscularly. d. Give the patient a multivitamin injection intravenously.
��. A ��-year-old man comes to your office or a routine physical examination. He tries to live a healthy liestyle. He ollows a low-at diet, rich in ruits and vegetables, and limits his intake o red meats. He exercises regularly, � to � times per week or �� to �� minutes each time. He does not smoke. His medical history is positive or hypertension, hyperlipidemia, benign prostatic hypertrophy, and erectile dysunction. He does not have diabetes mellitus. He is interested in disease prevention and wonders about his nutritional supplements. His amily history is signi�cant or coronary artery disease in his ather, who had a myocardial inarction at age �� years, and or type � diabetes mellitus in his mother. His current medications are atorvastatin �� mg daily, metoprolol �� mg daily, tamsulosin �.� mg daily, tadala�l �� mg as directed beore intercourse, olic acid �.� mg daily, �sh oil �,��� mg daily, vitamin E ��� international units daily, and a multivitamin daily. Physical examination �ndings include the ollowing: height ��� cm, weight �� kg, body mass index ��, pulse �� beats per minute, and blood pressure ���/�� mm Hg. He appears well and physically �t. Tere are no abnormal �ndings on the cardiac examination. His prostate has mild diuse enlargement without discrete nodularities. Te remainder o the examination �ndings are unremarkable. Which o the ollowing statements is false about vitamin and nutritional supplementation to prevent disease? a. Routine multivitamin supplementation has not been associated with health bene�ts in population-based studies. b. Antioxidant vitamin supplementation (beta carotene, vitamin E) has not been associated with a lower risk o cardiovascular events or mortality in randomized, placebo-controlled clinical trials. c. Supplementation with �sh oil (�,��� mg daily) is associated with a greater decrease in cardiovascular mortality compared with consumption o � or � meals o atty �sh (salmon) per week. d. Folic acid supplementation with normalization o homocysteine levels has not been associated with a lower risk o cardiovascular events in randomized, placebo-controlled clinical trials.
PITUITARY, GONADAL, AND ADRENAL DISORDERS
��. A ��-year-old married white man presents with breast enlargement and tenderness o � months’ duration. He reports normal sexual unctions and has � biologic children. He is not taking any medications, he does not smoke or drink, and he has not used any recreational drugs. Physical examination �ndings are unremarkable except or bilateral, tender, symmetrical gynecomastia. His secondary sex characteristics, external genitalia, and testicular size and consistency are all normal. Laboratory test results are normal or serum total and ree testosterone, prolactin, thyrotropin, and dehydroepiandrosterone-sulate (DHEA-S). Other results (and reerence ranges) include serum estradiol �� pg/mL (��–�� pg/mL) and β-human chorionic gonadotropin (hCG) ��,��� IU/L (<�.� IU/L). Which o the ollowing tests is the most appropriate next step? a. Ultrasonography o the testicles b. Computed tomography imaging o the adrenals c. Magnetic resonance imaging o the pituitary d. Mammography e. Liver biopsy
��. A ��-year-old woman presents at � weeks post partum with headaches, proound weakness, nausea, and vomiting. She had been breast-eeding. Physical examination �ndings are unremarkable. Magnetic resonance imaging (MRI) o the head shows a sellar mass with suprasellar extension but without chiasmal compression. Laboratory test results (and reerence ranges) include the ollowing: serum sodium ��� mEq/L (���–��� mEq/L), serum prolactin �� μg/L (�–�� μg/L), � �� serum cortisol � μg/dL (�–�� μg/dL), serum adrenocorticotropic hormone (ACH) �� pg/mL (��–�� pg/mL), and normal values or serum ree thyroxine and thyrotropin. Which o the ollowing is the most likely diagnosis? a. b. c. d. e.
Primary adrenal insufficiency (Addison disease) Prolactin-producing pituitary tumor Nonunctioning pituitary tumor Pituitary apoplexy (Sheehan syndrome) Lymphocytic hypophysitis
��. A ��-year-old woman presented with amenorrhea o � months’ duration. Physical examination �ndings were unremarkable except or bilateral expressible galactorrhea. Laboratory test results included a negative pregnancy test, evidence o a hypogonadotropic state, and a prolactin level o ��� μg/L. Findings on magnetic resonance imaging o the pituitary were consistent with a pituitary microadenoma. Te patient was unable to tolerate dopaminergic-agonist drugs, and a decision was made to treat her surgically. On the second day afer transsphenoidal microadenomectomy, postoperative diabetes insipidus developed but resolved by the third postoperative day. Her medications at dismissal included an analgesic and replacement doses o hydrocortisone. She was instructed in hydrocortisone dosage modi�cations during an acute
�. ENDOCRINOLOGY QUESIONS AND ANSWERS
•
��
illness. A week afer her hospital dismissal, she was evaluated in an emergency department or headache, atigue, lethargy, conusion, and nausea. She had been taking her medications regularly. Physical examination �ndings were unremarkable. Which o the ollowing is the most important laboratory test to perorm? a. b. c. d. e.
Pregnancy test Serum ree thyroxine Serum cortisol Serum sodium Serum prolactin B O N E A N D P A R AT H Y R O I D D I S E A S E
��. A ��-year-old woman is reerred to you or a preoperative evaluation rom an otolaryngology colleague who diagnosed a medullary thyroid carcinoma. Tat evaluation was prompted by a call rom � o the patient’s sisters who recently received the same diagnosis. Te patient has no complaints, she has no past medical problems, and she does not take any medications. Her vital signs are normal. You can eel a �.�-cm right thyroid lobe nodule without cervical adenopathy. Te rest o the examination �ndings are unremarkable. Which o the ollowing should you do preoperatively? a. b. c. d.
Measure levels o serum calcium, gastrin, and insulin. Request magnetic resonance imaging o the abdomen. Obtain genetic testing or RET proto-oncogene mutation. Recommend proceeding with surgery i the patient’s blood pressure is normal. e. Measure levels o serum calcium and plasma ractionated metanephrines.
��. A ��-year-old woman broke her hip � months ago by tripping on a stair. Subsequent evaluation (her �rst) with dual energy x-ray absorptiometry (DEXA) identi�ed low bone mineral density with scores less than −�.� at the contralateral hip and spine. She has come to you with her daughter to discuss osteoporosis treatment. You are told that she had normal periods until menopause at age ��, she never took estrogens but has begun taking �,��� mg o elemental calcium daily since she returned home rom the hospital, and she takes ��� international units o vitamin D daily. Her calcium level is normal. She has had diffi culty swallowing tablets afer she had a stroke � years ago, and she wants to discuss the use o intravenous (IV) bisphosphonates. You discuss their side effects, including the risk o hypocalcemia. You plan to identiy and minimize that risk. Which o the ollowing should you do next?
��
•
a. b. c. d.
Check the parathyroid hormone (PH) level. Check the ��-hydroxyvitamin D level. Perorm a parathyroid scan. Emphasize the use o acetaminophen i �ulike symptoms occur. e. Use bisphosphonates only i her creatinine clearance is greater than �� mL/min.
��. A ��-year-old man was hospitalized because o anterior chest pain. His vital signs were normal. An urgent coronary angiogram led to angioplasty, which was successully carried out, and the patient was discharged home � days later. During the next month, he had some nervousness, anxiety, and a ��-kg weight loss while eating his usual diet. His cardiologist recently determined that the patient had atrial �brillation and prescribed a calcium channel blocker. When you see him a ew days later in the emergency department, he has a ventricular rate o ��� beats per minute and shortness o breath. On examination, he is in no acute distress at rest, but he is in atrial �brillation with a rate o ��� to ��� beats per minute. He has no tremors and no signs o Graves ophthalmopathy or dermopathy. His thyroid is low lying, multinodular, nontender, and slightly enlarged. You learn o a long-standing history o nontoxic multinodular goiter. Because o the atrial �brillation, thyroid tests were perormed, and the results were as ollows: thyrotropin (SH) less than �.�� mIU/L, ree thyroxine �.� ng/dL, and negative or thyroid peroxidase (PO) antibodies. What is the most likely cause o this patient’s thyrotoxicosis? a. b. c. d. e.
Lymphocytic thyroiditis Autonomous thyroid nodule Iodine-induced hyperthyroidism Graves disease SH-producing pituitary adenoma
��. A ��-year-old woman has an elevated ionized calcium level o �.�� mg/dL during an inertility evaluation. Her only complaints are atigue and occasional headaches. She denies having a history o constipation, nausea, vomiting, kidney stones, hematuria, or ractures. Her appetite was normal without signi�cant weight changes. She denies having any known amily history o hypercalcemia, metabolic bone disease, or kidney stones. Her medications include exoenadine and multivitamins. Examination �ndings are unremarkable: heart rate �� beats per minute and regular, weight �� kg, and blood pressure ���/�� mm Hg. Laboratory test results are shown in able �.Q��.
M A YO C L I N I C I N E R N A L M E D I C I N E B O A R D R E V I EW : Q U E S I O N S A N D A N S W E R S
Table �.Q�� COMPONEN
FINDING
Ionized calcium, mg/dL
�.��
otal calcium, mg/dL
��.�
Albumin, g/dL
�.�
Phosphorus, mg/dL
�.�
Creatinine, mg/dL
�.�
Parathyroid hormone, pg/mL
��
Serum protein electrophoresis
Normal
��-hydroxyvitamin D, ng/mL
��
Blood cell counts
Normal
Urinary creatinine, mg/d
�,���
Urinary calcium, mg/d
���
Calcium clearance to creatinine clearance ratio
�.����
What is the best test or diagnosing amilial hypocalciuric hypercalcemia (FHH)? a. b. c. d. e.
��. A ��-year-old woman with recurrent kidney stones receives a diagnosis o hyperparathyroidism (total calcium ��.� mg/dL, parathyroid hormone [PH] ��� pg/mL). An experienced parathyroid surgeon perorms a minimally invasive parathyroidectomy and removes a large parathyroid adenoma (� g). Postoperatively, the patient eels well and begins eating a normal diet. At �� hours postoperatively, she begins experiencing perioral tingling and the Chvostek sign is positive. Te calcium level decreases to �.� mg/dL, and calcium carbonate (��� mg elemental calcium � tablets daily) is given orally. On postoperative day �, the tingling continues and the results o her laboratory tests are as ollows: total calcium �.� mg/dL, phosphorus �.� mg/dL, albumin �.� g/dL, and creatinine �.� mg/dL. You administer intravenous calcium, increase her oral calcium dosage, and begin calcitriol therapy, and her symptoms improve. What is the best explanation or these �ndings? a. b. c. d.
Hungry bone syndrome Severe vitamin D de�ciency ransient hypoparathyroidism rom surgical manipulation ransient hypoparathyroidism rom long-term suppression o normal parathyroids e. Lack o absorption o oral calcium
Another ��-hour urine collection Parathyroid scan Neck ultrasonography Screening or FHH among amily members Calcium-sensing receptor mutational analysis
�. ENDOCRINOLOGY QUESIONS AND ANSWERS
•
��
ANSWERS
�. Answer e.
eye condition, lead to a transient increase in thyroid hormone values, and not achieve euthyroidism or another � to � months. Surgery would be associated with additional risks because this patient has several cardiac comorbidities. Te saest and most effective treatment or him would be antithyroid drug treatment. Methimazole would be the �rst choice. PU is a second-line drug because o its association with severe liver ailure and death, but PU is still preerred in the �rst trimester o pregnancy and during thyroid storm. (See Bahn et al and Bartalena et al in the “Suggested Reading” list.)
Te replacement dose o levothyroxine is relatively low or this patient’s weight (the usual dose is �.� mcg/kg body weight), and it would be unlikely to cause iatrogenic thyrotoxicosis. Te ree thyroxine level decreased with a decrease in the levothyroxine dosage without a signi�cant change in the thyrotropin level. Te patient has symptoms o hypothyroidism, and he has hypogonadism. His old records might explain why he has central hypothyroidism. I those records are not available, testing his adrenal axis and imaging his pituitary would be the next step. For the same reason, decreasing the dosage to �� mcg daily or a ��-kg person or evaluating him or hyperthyroidism would not address his symptoms or biochemical response to the �rst change in dosage. Androgens actually decrease the level o thyroxine-binding globulins. (See Daly et al and Singer et al in the “Suggested Reading” list.)
�. Answer c.
Metormin and sulonylurea medications have been extensively studied as monotherapy and combination therapy or type � diabetes mellitus. For the overweight or obese patient, metormin is the recommended �rst-line therapy because o its insulin-sensitizing effects, low risk o hypoglycemia, and neutral impact on weight. Sulonylurea medications provide additional glucose lowering to the patient who has inadequate glycemic control with optimal doses o metormin and a recognized risk o hypoglycemia and weight gain. No additional glucose-lowering effect is observed with metormin doses higher than �,��� mg daily. Te use o rosiglitazone is restricted because o recognized risks o heart ailure and cardiovascular events, and it is not appropriate or this patient. (See Nathan et al in the “Suggested Reading” list.)
�. Answer e.
Increased estrogen levels, which were likely present in this patient during oral contraceptive therapy, lead to an increase in thyroxine-binding globulins, and that requires an increase in the levothyroxine replacement dose in hypothyroid patients to maintain the ree thyroxine levels unchanged. Tyroperoxidase antibodies are not pathogenic, and their titer does not correlate with the replacement dose. Te patient’s hypothyroidism had been well controlled with a stable dosage o levothyroxine or several years, so that nonadherence is probably not a problem. Te estrogen action is sustained, is not related to the timing o the administration o levothyroxine and oral contraceptive, and does not interere with the absorption o levothyroxine. Te combination o levothyroxine and liothyronine has not been proved to be more effective clinically than levothyroxine monotherapy, but it has been associated with signi�cantly more iatrogenic thyrotoxicosis in clinical trials; thereore, it is not recommended. (See Ain et al and Sawka et al in the “Suggested Reading” list.)
�. Answer d.
Tis patient meets the body mass index (BMI) criterion or obesity (BMI >��) and is at risk or type � diabetes mellitus over the next � years. Many patients are unaware o their risk or progression toward type � diabetes mellitus. Interventions to delay progression to type � diabetes mellitus should be discussed. Liestyle changes with dietary modi�cation, regular physical activity, and modest weight loss can substantially lower his risks or progressing to type � diabetes mellitus. Several medications have been studied in type � diabetes mellitus prevention, including metormin, rosiglitazone, acarbose, and orlistat. However, rosiglitazone was associated with signi�cant risk or weight gain and heart ailure despite the younger population studied (mean age, �� years) in the Diabetes Reduction Assessment With Ramipril and Rosiglitazone Medication (DREAM) trial. Since the DREAM trial, the use o rosiglitazone has been signi�cantly restricted because
�. Answer c.
Tis patient has thyrotoxicosis with Graves ophthalmopathy and dermopathy, which are pathognomonic or Graves disease. Tereore, the possibility o thyroiditis is effectively excluded. Te presence o moderate to severe ophthalmopathy in a smoker is a relative contraindication or the use o radioactive iodine, which might exacerbate his ��
it is associated with a higher risk o cardiovascular events. (See American Diabetes Association, Gerstein et al, and Knowler et al in the “Suggested Reading” list.) �. Answer b.
In patients with diabetes mellitus, neuropathic symptoms are ofen attributed to the underlying disease. However, in a large percentage o patients (��%), long-term metormin therapy can lead to vitamin B �� malabsorption and subsequent de�ciency. I not recognized, a treatable cause or irreversible neurologic symptoms would be overlooked. (See Bell in the “Suggested Reading” list.) �. Answer e.
Among the interventions listed, metormin therapy is the only one that has been clearly associated with a lowering o cardiovascular events and mortality in the United Kingdom Prospective Diabetes Study. Te Action to Control Cardiovascular Risk in Diabetes (ACCORD) trial showed a trend toward lower cardiovascular events only when eno�brate was used with a statin and triglyceride values were greater than ��� mg/dL. For a patient with a low-density lipoprotein cholesterol level less than �� mg/ dL, there would be no additional bene�t to increasing the statin dosage. (See Stratton et al and oth in the “Suggested Reading” list.) �. Answer e.
Te normal physiologic response to hypoglycemia is low insulin secretion with gradual development o ketosis. Endogenous insulin secretion is best determined not only by measuring insulin levels but by measuring C peptide, the peptide cleaved rom the insulin molecule afer secretion renders it active. o assure that true excessive endogenous insulin secretion is involved, the presence o insulin secretagogues must be ruled out as a potential explanation or hypoglycemia with elevated or inappropriately normal insulin and C peptide levels. (See Cryer et al in the “Suggested Reading” list.) �. Answer a.
Tis patient is receiving maximal dosages o the recommended �rst- and second-line therapies or the management o type � diabetes mellitus. Te choice o a third agent should take several actors into consideration. Te use o rosiglitazone, a ZD, is signi�cantly restricted because o the recognized risk o cardiovascular events. Te same risks may not apply to pioglitazone. (See Loke et al in the “Suggested Reading” list.) ��. Answer d.
For a patient with recent weight gain, it is reasonable to screen or thyroid unction abnormalities with an sSH test. Other endocrinopathies such as Cushing syndrome are rare, and screening should likely not be pursued in the absence o other suspicious physical �ndings such as acial plethora, purple striae, hirsutism, and hypertension. Te
patient currently meets criteria or obesity, and, with her amily history, screening by measuring the asting blood glucose level is indicated. Liestyle changes with dietary modi�cation and regular physical activity are the cornerstone o weight management. Randomized studies have shown that calorie restriction and not macronutrient com position (proportions o carbohydrate, protein, and at) determine success in weight loss and weight maintenance. (See Gardner et al in the “Suggested Reading” list.) ��. Answer b.
Bariatric surgery has become well accepted in the management o medically complicated obesity mainly because o the bene�ts to the patient with type � diabetes mellitus. Remission rates or diabetes vary according to the operation perormed: Remission rates are highest with biliopancreatic diversion and duodenal switch (��%) and Roux-en-Y gastric bypass (��%) and lowest with laparoscopic adjustable gastric banding (��%). A patient considering bariatric surgery should be well inormed about the currently accepted criteria, risk actors that affect perioperative morbidity and mortality, and long-term management. (See Greenway et al in the “Suggested Reading” list.) ��. Answer a.
Acute thiamine de�ciency can occur in a patient with severe persistent vomiting. It has been reported in patients who have had bariatric surgery that involved a restrictive mechanism or weight loss, such as vertical banded gastroplasty, laparoscopic adjustable gastric banding, and Roux-en-Y gastric bypass. Early recognition and treatment are key to avoid neurologic complications and Wernicke encephalopathy. (See Serra et al in the “Suggested Reading” list.) ��. Answer c.
Health bene�ts o �sh oil supplementation vary according to the dosage o active docosahexaenoic acid (DHA) and eicosapentaenoic acid (EPA) consumed. At lower dosages (�,��� mg daily), omega-� atty acids are associated with a lower risk o cardiovascular mortality. Te same bene�t is observed when individuals consume � servings weekly o a atty �sh such as salmon. For individuals already consuming this amount o omega-� atty acids in their diet, there is no additional bene�t to additional omega-� atty acid supplementation. Higher doses o DHA and EPA (�–� g daily) are needed to lower triglyceride levels. (See Jacobson in the “Suggested Reading” list.) ��. Answer a.
Gynecomastia is the most common disorder o the male breast and accounts or more than ��% o all male breast masses. It may result rom a trivial cause, or it may be an early sign o a sinister disorder. Te basic mechanism o gynecomastia is relative estradiol excess, which can result rom decreased androgen production or effect, or an increase in estrogen production or effect. In adults, the � most common causes o gynecomastia are drugs and alcohol-related
�. ENDOCRINOLOGY QUESIONS AND ANSWERS
•
��
liver disease. Less common causes include recovery rom malnutrition or other serious chronic illness. Rarely encountered are hCG-producing tumors, adrenal tumors, and testicular tumors. In this patient, drugs and alcohol, gonadal ailure, hyperthyroidism, and prolactin-producing pituitary tumor have been excluded as causes. Tis patient has an hCG-producing tumor and increased estrogen production. Although the most common maniestation o testicular tumors is a testicular enlargement or mass, small testicular tumors may ofen be occult. O primary testicular tumors, ��% arise rom seminierous tubules (germ cell tumors) and about �% arise rom interstitial tissue. Germ cell tumors are the most common solid tumors in men between the ages o �� and �� years; these tumors occur primarily in white men and are the main tumors that secrete hCG. Stimulation o Leydig cells by hCG results in an increased secretion o both estrogen and androgen. esticular ultrasonography is the most appropriate step. A normal DHEA-S value excludes a diagnosis o an estrogen-producing adrenal tumor, a normal prolactin value excludes the diagnosis o a prolactin-producing pituitary tumor, and imaging studies o these glands are not warranted. Mammograms are not required or evaluation o symmetrical concentric gynecomastia; they are indicated in the evaluation o unilateral breast masses or when breast cancer is considered. Nothing in the presentation o this patient suggests an occult liver disease that would necessitate a liver biopsy. (See Braunstein in the “Suggested Reading” List.) ��. Answer e.
Headaches, pituitary insufficiency, and MRI �ndings consistent with a pituitary tumor in relation to pregnancy should always lead to consideration o lymphocytic hypo physitis. Absence o a high ACH level excludes primary adrenal insufficiency. Pituitar y tumors, whether prolactin producing or nonunctioning, are predominantly benign and slow growing. A pituitary tumor would have been present or a while and would probably have made the patient hypogonadotropic and inertile, so that she would not have presented initially in the postpartum state. Patients with Sheehan syndrome have a history o postpartum hemorrhage-induced hypotension or shock that requires blood transusion; the signs and symptoms o hypopituitarism are usually recognized early afer delivery, and the patient loses the ability to breast-eed. Lymphocytic hypo physitis is thereore the most likely diagnosis. (See Rivera in the “Suggested Reading” list.) ��. Answer d.
Major complications o pituitary surgery include postoperative hemorrhage, cerebrospinal �uid leak, meningitis, and visual disturbances; these occur in less than �% o patients and occur most requently in patients who have large tumors and not microadenomas. Diabetes insipidus, the most common disorder o arginine vasopressin (AVP) secretion in the postoperative state, occurs in less than ��% o patients and usually occurs early afer pituitary surgery ��
•
and lasts a ew days. Permanent diabetes insipidus is rare. ransient excess AVP leading to syndrome o inappropriate secretion o antidiuretic hormone (SIADH) and hyponatremia occurs in ��% o patients within � to �� days postoperatively. Pregnancy in the �rst week afer transsphenoidal surgery is unrealistic and a pregnancy test is not warranted. I symptomatic postoperative hypothyroidism occurs, it usually occurs several weeks postoperatively. Te patient has been taking her hydrocortisone regularly; thereore, she would not have cortisol de�ciency, so measuring serum cortisol would be unnecessary. Although the patient may have persistent hyperprolactinemia afer surgery or micro prolactinoma, she would not be symptomatic in the �rst week postoperatively. Te most important consideration in this patient is SIADH; thereore, measuring the serum sodium level is the most important laboratory test. (See Lien and Shapiro in the “Suggested Reading” list.) ��. Answer e.
Multiple endocrine neoplasia (MEN) type � must be considered because o the amilial association described. Te conditions that may also be present in this patient, thereore, are pheochromocytoma (in ��%-��% o patients with MEN type �) and hyperparathyroidism (in ��%-��%). Both are important rom an operative perspective: Hemodynamic �uctuations can occur with pheochromocytoma, and hyperparathyroidism can be addressed with the thyroid surgery. Tereore, screening or both with highly sensitive biomarkers is important. Te testing or calcium, gastrin, and insulin is a starting point or abnormalities in MEN type �. Imaging the abdomen or diagnosing pheochromocytoma has a very poor speci�city (because o adrenal incidentalomas) without any signi�cant gain in sensitivity compared with measuring levels o plasma metanephrines. Genetic testing takes time (�–� weeks), and a positive test does not indicate the presence o � or both o these associated endocrinopathies. Normal blood pressure is present in about ��% o patients with pheochromocytoma (the percentage is higher among patients with amilial syndromes). (See Kloos et al and Raue et al in the “Suggested Reading” list.) ��. Answer b.
Hypocalcemia is more likely to occur in patients who have vitamin D de�ciency; thereore, the risk o hypocalcemia can be minimized by vitamin D and calcium supplementation. Te serum calcidiol (��-hydroxyvitamin D) concentration should be assessed beore the patient receives IV bisphosphonate inusions. Patients with vitamin D de�ciency (��-hydroxyvitamin D <�� ng/mL) should be treated beore the inusion. Results rom checking the PH level and perorming the parathyroid scan would be inadequate or diagnosing vitamin D de�ciency. In addition, having a normal serum calcium level excludes the diagnosis o hypoparathyroidism. Acetaminophen will not minimize the risk o hypocalcemia since it only affects the systemic in�ammation that occurs with IV bisphos phonates. Te risk o hypocalcemia is independent o the creatinine
MAYO CLINIC INERNAL MEDICIN E BOARD REVIEW: QUESIONS AND ANSWER S
clearance even though IV bisphosphonates should not be used in patients with a creatinine clearance less than �� to �� mL/min because o the risk o renal osteodystrophy. (See Black et al and Rosen and Brown in the “Suggested Reading” list.) ��. Answer c.
Iodine-induced hyperthyroidism can be easily overlooked and is likely more common than is realized. It usually maniests as apathetic hyperthyroidism, which can be masked even urther by the use o β-blockers. Te time rame or exposure to iodinated contrast agents a month earlier is typical or the development o thyrotoxicosis in patients with preexisting multinodular goiter. reatment involves antithyroid drugs and supportive measures. Te other choices can be excluded on clinical grounds. Lymphocytic thyroiditis is relatively rare in the elderly and is almost always associated with anti-PO antibodies, which were absent in this patient. Tis disorder can occur in patients who have Graves disease and preexisting iodine insufficiency. However, iodine insuffi ciency is rare in North America, and the multinodular goiter also argues against Graves disease. An autonomous toxic nodule evolves slowly over many years rather than arising abruptly. A SH-producing adenoma can be excluded by the suppressed SH value. (See Bahn Chair et al and Martin et al in the “Suggested Reading” list.) ��. Answer d.
FHH is an autosomal dominant disorder characterized by asymptomatic hypercalcemia, normal parathyroid hormone levels, and low urinary calcium levels. Te cause is usually an inactivating mutation in the calcium-sensing receptor, which makes the parathyroid glands less sensitive to calcium and increases the tubular reabsorption o calcium. Screening or FHH among amily members is the most reliable method o making the diagnosis, and genetic testing should be used only in atypical cases i amily members are not available. Tis patient’s sister had asymptomatic hypercalcemia with a calcium clearance to creatinine clearance ratio o �.���. For approximately ��% o patients, the ratio is less than �.��; in contrast, or primary hyperparathyroidism patients, the ratio is usually greater than �.��. Parathyroid scanning and ultrasonography will not reliably distinguish these � entities and should be used only or localization purposes i the diagnosis o primary hyperparathyroidism has been established. Given that the urinary creatinine value rom the ��-hour collection appears complete, there is no nee d to repeat it. (See Heath, and elt-Hansen and Brown in the “Suggested Reading” list.) ��. Answer a.
When PH values are not available, the most useul evidence o hungry bone syndrome is low calcium and phos phorus levels. Both calcium and phosphorus are moving rapidly into the unmineralized osteoid o this patient. Hungry bone syndrome occurs in patients with severe
disease (re�ected by the very large parathyroid gland and the signi�cant calcium and PH elevations), and it occurs early afer surgery. Continuing calcium therapy beyond symptom control will likely improve her overall bone mass. With the high calcium value preoperatively, it is quite unlikely that she had signi�cant vitamin D de�ciency to explain these �ndings. Te minimal access o parathyroidectomy avoids bilateral neck exploration, thus eliminating the possibility o surgical manipulation o all the parathyroid glands and its consequences. Te relative insufficiency o the remaining parathyroids rom prolonged suppression is not consistent with a low phosphorus level since hypoparathyroidism is associated with low renal phosphate clearance. Oral calcium supplementation might be decreased initially, but it should improve in the second day, and the phosphorus level should be normal. (See Brasier and Nussbaum, and ohme and Bilezikian in the “Suggested Reading” list.) SUGGESED R EADING Ain KB, Mori Y, Reetoff S. Reduced clearance rate o thyroxine-binding globulin (BG) with increased sialylation: a mechanism or estrogen-induced elevation o serum BG concentration. J Clin Endocrinol Metab. 1987 Oct;65(4):689–96. American Diabetes Association. Standards o medical care in diabetes: 2011. Diabetes Care. 2011 Jan;34(Suppl 1):S11–61. Bahn RS, Burch HS, Cooper DS, Garber JR, Greenlee CM, Klein IL, et al. Te role o propylthiouracil in the management o Graves’ disease in adults: report o a meeting jointly sponsored by the American Tyroid Association and the Food and Drug Administration. Tyroid. 2009 Jul;19(7):673–4. Bahn Chair RS, Burch HB, Cooper DS, Garber JR, Greenlee MC, Klein I, et al; American Tyroid Association; American Association o Clinical Endocrinologists. Hyperthyroidism and other causes o thyrotoxicosis: management guidelines o the American Tyroid Association and American Association o Clinical Endocrinologists. Tyroid. 2011 Jun;21(6):593–646. Epub 2011 Apr 21. Erratum in: Tyroid. 2011 Oct;21(10):1169. Bartalena L, Marcocci C, Bogazzi F, Manetti L, anda ML, Dell’Unto E, et al. Relation between therapy or hyperthyroidism and the course o Graves ophthalmopathy. N Engl J Med. 1998 Jan 8;338(2):73–8. Bell DS. Metormin-induced vitamin B12 de�ciency presenting as a peripheral neuropathy. South Med J. 2010 Mar;103(3):265–7. Black DM, Delmas PD, Eastell R, Reid IR, Boonen S, Cauley JA, et al; HORIZON Pivotal Fracture rial. Once-yearly zoledronic acid or treatment o postmenopausal osteoporosis. N Engl J Med. 2007 May 3;356(18):1809–22. Brasier AR, Nussbaum SR. Hungry bone syndrome: clinical and biochemical predictors o its occurrence afer parathyroid surgery. Am J Med. 1988 Apr;84(4):654–60. Braunstein GD. Clinical practice: g ynecomastia. N Engl J Med. 2007 Sep 20;357(12):1229–37. Cryer PE, Axelrod L, Grossman AB, Heller SR, Montori VM, Seaquist ER, et al; Endocrine Society. Evaluation and management o adult hypoglycemic disorders: an Endocrine Society Clinical Practice Guideline. J Clin Endocrinol Metab. 2009 Mar;94(3):709–28. Daly RC, Su P, Schmidt PJ, Pagliaro M, Pickar D, Rubinow DR. Neuroendocrine and behavioral effects o high-dose anabolic steroid administration in male normal volunteers. Psychoneuroendocrinology. 2003 Apr;28(3):317–31. Gardner CD, Kiazand A, Alhassan S, Kim S, Stafford RS, Balise RR, et al. Comparison o the Atkins, Zone, Ornish, and LEARN diets or change in weight and related risk actors among overweight
�. ENDOCRINOLOGY QUESIONS AND ANSWERS
•
��
premenopausal women: the A O Z Weight Loss Study: a randomized trial. JAMA. 2007 Mar 7;297(9):969–77. Erratum in: JAMA. 2007 Jul 11;298(2):178. Gerstein HC, Yusu S, Bosch J, Pogue J, Sheridan P, Dinccag N, et al; DREAM (Diabetes REduction Assessment with ramipril and rosiglitazone Medication) rial Investigators. Effect o rosiglitazone on the requency o diabetes in patients with impaired glucose tolerance or impaired asting glucose: a randomised controlled trial. Lancet. 2006 Sep 23;368(9541):1096–105. Erratum in: Lancet. 2006 Nov 18;368(9549):1770. Greenway SE, Greenway FL 3rd, Klein S. Effects o obesity surgery on non-insulin-dependent diabetes mellitus. Arch Surg. 2002 Oct;137(10):1109–17. Heath H 3rd. Familial benign (hypocalciuric) hypercalcemia: a troublesome mimic o mild primary hyperparathyroidism. Endocrinol Metab Clin North Am. 1989 Sep;18(3):723–40. Jacobson A. Beyond lipids: the role o omega-3 atty acids rom �sh oil in the prevention o coronary heart disease. Curr Atheroscler Rep. 2007 Aug;9(2):145–53. Kloos R, Eng C, Evans DB, Francis GL, Gagel RF, Gharib H, et al; American Tyroid Association Guidelines ask Force. Medullary thyroid cancer: management guidelines o the American Tyroid Association. Tyroid. 2009 Jun;19(6):565–612. Erratum in: Tyroid. 2009 Nov;19(11):1295. Knowler WC, Barrett-Connor E, Fowler SE, Hamman RF, Lachin JM, Walker EA, et al; Diabetes Prevention Program Research Group. Reduction in the incidence o type 2 diabetes with liestyle intervention or metormin. N Engl J Med. 2002 Feb 7;346(6):393–403. Lien YH, Shapiro JI. Hyponatremia: clinical diagnosis and management. Am J Med. 2007 Aug;120(8):653–8. Loke YK, Kwok CS, Singh S. Comparative cardiovascular effects o thiazolidinediones: systematic review and meta-analysis o observational studies. BMJ. 2011 Mar 17;342:d1309. Martin FI, ress BW, Colman PG, Deam DR. Iodine-induced hyperthyroidism due to nonionic contrast radiography in the elderly. Am J Med. 1993 Jul;95(1):78–82. Nathan DM, Buse JB, Davidson MB, Ferrannini E, Holman RR, Sherwin R, et al; American Diabetes Association; European
��
•
Association or Study o Diabetes. Medical management o hyperglycemia in type 2 diabetes: a consensus algorithm or the initiation and adjustment o therapy: a consensus statement o the American Diabetes Association and the European Association or the Study o Diabetes. Diabetes Care. 2009 Jan;32(1):193–203. Epub 2008 Oct 22. Raue F, Frank-Raue K, Grauer A. Multiple endocrine neoplasia type 2: clinical eatures and screening. Endocrinol Metab Clin North Am. 1994 Mar;23(1):137–56. Rivera JA. Lymphocytic hypophysitis: disease spectrum and approach to diagnosis and therapy. Pituitary. 2006;9(1):35–45. Rosen CJ, Brown S. Severe hypocalcemia afer intravenous bisphosphonate therapy in occult vitamin D de�ciency. N Engl J Med. 2003 Apr 10;348(15):1503–4. Sawka AM, Gerstein HC, Marriott MJ, Mac�ueen GM, Joffe R. Does a combination regimen o thyroxine (4) and 3,5,3 ′-triiodothyronine improve depressive symptoms better than 4 alone in patients with hypothyroidism? Results o a double-blind, randomized, controlled trial. J Clin Endocrinol Metab. 2003 Oct;88(10):4551–5. Serra A, Sechi G, Singh S, Kumar A. Wernicke encephalopathy afer obesity surgery: a systematic review. Neurology. 2007 Aug 7;69(6):615. Singer PA, Cooper DS, Levy EG, Ladenson PW, Braverman LE, Daniels G, et al; Standards o Care Committee, American Tyroid Association. reatment guidelines or patients with hyperthyroidism and hypothyroidism. JAMA. 1995 Mar 8;273(10):808–12. Stratton IM, Adler AI, Neil HA, Matthews DR, Manley SE, Cull CA, et al. Association o glycaemia with macrovascular and microvascular complications o type 2 diabetes (UKPDS 35): prospective observational study. BMJ. 2000 Aug 12;321(7258):405–12. elt-Hansen J, Brown EM. Te calcium-sensing receptor in normal physiology and pathophysiology: a review. Crit Rev Clin Lab Sci. 2005;42(1):35–70. ohme JF, Bilezikian JP. Diagnosis and treatment o hypocalcemic emergencies. Endocrinologist. 1996 Jan;6(1):10–18. oth PP. Fibrate therapy in the management o diabetic dyslipidemia: there is no ACCORD to be ound. Curr Atheroscler Rep. 2010 Sep;12(5):331–5.
MAYO CLINIC INERNAL MEDICIN E BOARD REVIEW: QUESIONS AND ANSWER S
�. ONCOLOGY QUESTI ONS AND ANSWERS
QUESIONS
not tender. Findings rom mammography, breast ultrasonography, and computed tomography o the chest are negative except or the presence o right axillary adenopathy. Biopsy o a lymph node shows a moderately differentiated adenocarcinoma o unknown primary origin. What should be the next step in evaluation o this patient?
Multiple Choice (choose the best answer)
�. An otherwise healthy ��-year-old man asks you about screening tests or colon cancer. He reports that his � older sisters had colon cancer at age �� and ��, his mother had colon cancer at age �� and endometrial cancer at age ��, and a maternal aunt had breast cancer at a young age. He notes that there was not a history o colon polyps in any amily member. What should you recommend or this patient?
a. Perorm a mediastinoscopy. b. Assume that non–small cell carcinoma is present, and treat with cisplatin-based chemotherapy. c. Clariy the histogenetic origin o the tumor by testing or tumor markers: carcinoembryonic antigen, cancer antigen ��–�, and neuron-speci�c enolase. d. Perorm breast magnetic resonance imaging (MRI). e. Recommend bilateral mastectomies.
a. Annual ecal occult blood testing b. Colonoscopy now and every � to � years thereafer c. Colonoscopy with random biopsies to look or in�ammatory bowel disease—i not present, ollow routine screening recommendations or average-risk Americans d. Prophylactic colectomy e. Reassurance only, since no polyps were ound in amily members
�. A ��-year-old woman presents or intermittent abdominal pain and bloating that has been getting worse or the past several months. She has been reading medical inormation on the Internet and is very concerned about ovarian cancer. She has no amily history o malignancy and has been otherwise healthy. She is not taking any medications. Physical examination �ndings are remarkable only or some tenderness to movement o the uterus. No pelvic masses are detected. She requests a serum cancer antigen (CA) ��� test; the result is �� U/mL (reerence range <�� U/mL). How should you advise her at this time?
�. A ��-year-old male smoker with a ��-pack-year history presents with anorexia, cough, and altered mental status. A chest radiograph shows a right-sided mass that, on bronchoscopy, is identi�ed as squamous cell carcinoma. On physical examination, the patient is thin, cachectic, dehydrated, and disoriented with no ocal neurologic de�cits. Te calcium level is elevated (�� mg/dL), creatinine is �.� mg/dL, and albumin is �.� g/dL. A bone scan shows only some degenerative changes. What is the most appropriate next step in the management o this patient? a. b. c. d. e.
a. Recommend combination chemotherapy with cisplatin and paclitaxel. b. ell her that this degree o CA ��� elevation occurs only in ovarian cancer. c. ell her that although the CA ��� elevation is concerning, multiple conditions can cause such an elevation, and urther investigation is warranted. d. Recommend exploratory laparotomy with total abdominal hysterectomy, bilateral salpingo-oophorectomy, pelvic lymphadenectomy, omentectomy, and aggressive surgical debulking o all disease. e. Recommend only observation now, and recheck the CA ��� level in � months.
Cisplatin-based chemotherapy Radiotherapy to the brain Intravenous �uids and bisphosphonates Dexamethasone ��� mg given as an intravenous push Emergent magnetic resonance imaging o the head
�. A ��-year-old woman with a history o hypertension, a ��-pack-year history o smoking, mild chronic obstructive pulmonary disease, and moderate obesity presents with a right axillary mass. She has a amily history o coronary artery disease and strokes. Current medications include a statin, a diuretic, and a β -blocker. On examination, she is moderately obese and in no distress. Her lungs have increased sound in the expiratory phase diffusely but no rank wheezing or other sounds. Findings rom examination o the heart, abdomen, and breasts are unremarkable. On lymph node examination, a palpable right axillary mass is �rm, mobile, and
�. A ��-year-old woman with recently diagnosed node-positive breast cancer presents to the emergency department with a temperature o ��.� °C. She reports having mild chills and ever but denies having nausea, vomiting, diarrhea, cough, or dysuria. Seven days ago she received her third cycle o doxorubicin and cyclo phosphamide chemotherapy; thus ar, she has tolerated ��
the cycles well. On physical examination, she is pleasant and appears atigued but in no distress, with the ollowing �ndings: blood pressure ���/�� mm Hg, pulse �� beats per minute and regular, respiration rate �� breaths per minute, and temperature ��.�°C. Te remainder o the examination is remarkable only or alopecia. A chest radiograph is clear o abnormalities. Urinalysis shows no leukocytes. Laboratory data include the ollowing: hemoglobin ��.� g/dL, leukocyte count �.� ×���/L, absolute neutrophil count �.��×���/L, and platelet count ��×���/L. She lives in town with her husband and � children (aged �� and ��). No one else is ill at home. At this time, what should you do? a. Admit her to the hospital to receive broad-spectrum antibiotics. b. Administer granulocyte colony-stimulating actor now. c. Send her home, and ask her to ollow up with her oncologist in the morning. d. Collect blood and urine samples or cultures, begin therapy with amoxicillin–clavulanate potassium and cipro�oxacin orally, discharge to home, and ask her to ollow up with her oncologist by telephone within �� hours. e. Obtain a throat swab or in�uenza virus.
�. A ��-year-old man with a history o stage II rectal cancer was treated with resection and combined chemotherapy and radiotherapy � years ago. He has recovered well rom the operations and treatments; he still has some rectal and bladder urgency but no incontinence. He recently retired rom his job as an office manager and is physically active. His hypertension is well controlled with a β-blocker, and his cholesterol levels are controlled with diet. Tere is no signi�cant amily history. His most recent ollow-up colonoscopy was done �� months ago and showed no evidence o recurrence or other disease. He is a lietime nonsmoker. On physical examination, he is thin and pleasant, and he appears �t. General examination �ndings are unremarkable. On rectal examination, his prostate eels normal and smooth without palpable masses. A stool sample is negative or heme. Liver unction test results are normal, the level o carcinoembryonic antigen (CEA) is within the reerence range, and the level o prostate-speci�c antigen is �.� ng/mL. He is concerned about late side effects o his prior therapy, speci�cally about the development o new cancers. What should you tell him?
��
•
a. He is at increased risk or secondary cancers o the bladder, prostate, and rectum, but the risk is only about � in �� at �� years. b. Tere is no need to worry since he has no increased risk or secondary malignancies. c. He is at high risk or lung cancer in the uture, so he should have routine screening chest radiographs. d. He is right to worry since secondary cancers are very common, and screening should be done at regular intervals. e. Tere is no longer a reason to screen or prostate cancer since he has had radiotherapy to this area.
�.
An ��-year-old man comes to your office or routine ollow-up care. He has a prior history o chronic obstructive pulmonary disease with a orced expiratory volume in � second o ��% o the predicted value, coronary artery disease with mild congestive heart ailure, hypertension, and type � diabetes mellitus. He denies having any urinary symptoms. He takes the ollowing medication: enalapril � mg twice daily, hydrochlorothiazide �� mg twice daily, lovastatin �� mg once daily, albuterol inhaler as needed, �uticasone propionate ��� mcg and salmeterol �� mcg inhalation powder daily, aspirin ��� mg daily, glipizide �� mg twice daily, and a multivitamin daily. On examination, he has poor breath sounds in all areas, distant heart tones, a normal abdomen, and edema (�+) o the lower extremities bilaterally. On rectal examination, he has an enlarged prostate with a �rm nodule. Te prostate-speci�c antigen level is �.� ng/mL, and transrectal needle biopsy shows adenocarcinoma (Gleason grade �). A bone scan shows only some changes consistent with degenerative disease. What can you tell him at this time? a. Radical prostatectomy is likely to improve his overall sur vival and decrease his chance o death rom prostate cancer. b. Given his lack o symptoms rom prostate cancer, combined with his age and comorbid conditions, a watchul waiting approach is reasonable. c. External beam radiotherapy is not effective against prostate cancer. d. Chemotherapy can be used to decrease his risk o recurrence o prostate cancer. e. Orchiectomy is the standard o care.
MAYO CLINIC INERNAL MEDICIN E BOARD REVIEW: QUESIONS AND ANSWER S
ANSWERS
�. Answer b.
MRI or routine screening should be limited to high-risk women since it has not been shown to be bene�cial or average-risk patients. Te detection o hormone receptors in the pathologic specimen has diagnostic and therapeutic implications. Serum tumor markers are rarely useul diagnostic tools (with ew exceptions). (See Chen et al and Saslow et al in the “Suggested Reading” list.)
With his amily history, this patient is at very high risk or colon cancer. It is unlikely that he would have a hereditary polyposis syndrome since no amily member had polyps. His amily history is highly concerning or hereditary nonpolyposis colorectal cancer (Lynch syndrome) because multiple �rst-degree relatives were affected at an early age and because there is a amily history o breast and endometrial cancer. Tis syndrome is associated with a deect in mismatch repair enzymes and leads to microsatellite instability. Screening with ecal occult blood testing is not adequately sensitive or patients at high risk—or even normal risk. In�ammatory bowel disease does signi�cantly increase the risk o colon cancer, but nothing in the patient’s history suggests that it is present. Prophylactic colectomy would be a consideration only i testing is positive or the deective gene. (See Rex et al and Umar et al in the “Suggested Reading” list.)
�. Answer c.
Further investigation is warranted to determine the cause o her discomort and the reason or her elevated CA ��� level. Tis nonspeci�c serum marker can be elevated in many benign conditions, such as endometriosis, pregnancy, menstruation, and peritonitis. Te positive predictive value o CA ��� or screening is only about �% to �%. Although very high levels (several hundred to several thousand units per milliliter) typically occur only in ovarian cancer, patients with endometriosis can have levels around ��� U/mL with stage IV disease. Chemotherapy is never indicated until there is tissue con�rmation o disease. Extensive resection would be indicated i ovarian carcinoma were diagnosed, and this would typically be ollowed by systemic chemotherapy. Since the patient is having signi�cant symptoms, observation only is not warranted. (See Partridge et al in the “Suggested Reading” list.)
�. Answer c.
Te patient needs intravenous �uids and restoration o intravascular volume along with bisphosphonates to correct the hypercalcemia. In the absence o any ocal neurologic de�cits, it is unlikely that his disorientation is due to metastatic disease; altered mental status is very common with hypercalcemia. I correction o the hypercalcemia reverses the altered mental status, central nervous system imaging is not required. Cranial radiotherapy should be given only afer metastatic disease is identi�ed in a patient with non–small cell carcinoma. Dexamethasone is used to decrease peritumoral edema rom intracranial metastases and is not indicated or this patient. Dexamethasone can help to signi�cantly correct hypercalcemia due to multiple myeloma or lymphoma, but it is unlikely to correct hypercalcemia due to squamous cell carcinoma. (See Behl et al and Haldanarson et al in the “Suggested Reading” list.)
�. Answer d.
Febrile neutropenia is common with many orms o chemotherapy. Te vast majority o patients have negative cultures. Patients who are medically stable, are able to maintain oral intake, are reliable or close ollow up, and live near a medical acility can be saely treated with an outpatient regimen. Afer neutropenia develops, there is no role or administration o growth actors. Observation alone is insuffi cient since the low absolute neutrophil count puts her at signi�cant risk or sepsis. (See Behl et al and Haldanarson et al in the “Suggested Reading” list.)
�. Answer d.
�. Answer a.
Women presenting with axillary lymph node metastases o adenocarcinoma o unknown primary origin should undergo thorough evaluation or breast cancer. Breast MRI has greater sensitivity than mammography or ultrasonography, and among women who have occult adenocarcinoma in the axillary lymph nodes, MRI can detect a primary lesion in up to ��% o patients. Although breast MRI is helpul or evaluation o women with adenocarcinoma metastatic to axillary lymph nodes, use o breast
Patients treated with pelvic radiotherapy or rectal or prostate cancer are at increased risk or secondary malignancies in the area, but this risk is low (estimated to be � in ��� at � years and � in �� at �� years). Patients should still undergo screening or cancers that they are at risk or, as long as their general health and other medical conditions warrant screening. Tis patient does not have an increased risk or lung cancer per se, but lung metastases are a common site ��
or recurrence o rectal cancer. Te pattern o recurrence is different rom that o colon cancer, which much more commonly metastasizes to the liver beore traveling to the lung. Te venous drainage o the rectum is into the inerior vena cava, while most o the colon’s venous drainage is to the portal system. Routine ollow-up or otherwise healthy patients who have colorectal cancer includes the ollowing: �) Evaluate with a history and physical examination every � months or � years and then every � months or a total o � years. �) Perorm a colonoscopy in � year. I results are abnormal, perorm another colonoscopy in � year; i results are normal, perorm a colonoscopy as clinically indicated. �) Determine CEA levels every � months or � years and then every � months or years � to �. �) Consider computed tomography o the chest, abdomen, and pelvis annually or � years i the patient is at high risk. (See National Comprehensive Cancer Network [NCCN] in the “Suggested Reading” list.) �. Answer b.
For patients older than ��, especially those with signi�cant comorbid conditions, it is unclear whether radical prostatectomy improves the patient’s overall survival and it certainly has adverse effects on quality o lie. Patients in good condition who are younger than �� do seem to have a survival advantage i treated with radical prostatectomy instead o watchul waiting; however, this elderly patient with multiple other illnesses is unlikely to bene�t rom aggressive treatment. External beam radiotherapy or brachytherapy would be a reasonable treatment option, but each carries a risk o impotence, rectal injury, and incontinence. Chemotherapy has no role except or patients with metastatic disease. Orchiectomy or hormonal therapies
��
•
are typically reserved or patients with metastatic or symptomatic disease. (See National Comprehensive Cancer Network [NCCN] in the “Suggested Reading” list.) SUGGESED RE ADING Behl D, Hendrickson AW, Moynihan J. Oncologic emergencies. Crit Care Clin. 2010 Jan;26(1):181–205. Chen C, Orel SG, Harris E, Schnall MD, Czerniecki BJ, Solin LJ. Outcome afer treatment o patients with mammographically occult, magnetic resonance imaging-detected breast cancer presenting with axillary lymphadenopathy. Clin Breast Cancer. 2004 Apr;5(1):72–7. Haldanarson R, Hogan WJ, Moynihan J. Oncologic emergencies: diagnosis and treatment. Mayo Clin Proc. 2006 Jun;81(6):835–48. Erratum in: Mayo Clin Proc. 2006 Nov;81(11):1509. National Comprehensive Cancer Network (NCCN) guidelines [Internet]. Fort Washington (PA): National Comprehensive Cancer Network; c2012 [cited 2011 Mar 20]. Available rom: www.nccn.org. Partridge E, Kreimer AR, Greenlee R, Williams C, Xu JL, Church R, et al; PLCO Project eam. Results rom our rounds o ovarian cancer screening in a randomized trial. Obstet Gynecol. 2009 Apr;113(4):775–82. Rex DK, Johnson DA, Anderson JC, Schoeneld PS, Burke CA, Inadomi JM; American College o Gastroenterology. American College o Gastroenterology guidelines or colorectal cancer screening 2009 [corrected]. Am J Gastroenterol. 2009 Mar;104(3):739–50. Epub 2009 Feb 24. Erratum in: Am J Gastroenterol. 2009 Jun;104(6):1613. Saslow D, Boetes C, Burke W, Harms S, Leach MO, Lehman CD, et al; American Cancer Society Breast Cancer Advisory Group. American Cancer Society guidelines or breast screening with MRI as an adjunct to mammography. CA Cancer J Clin. 2007 Mar-Apr;57(2):75–89. Erratum in: CA Cancer J Clin. 2007 May-Jun;57(3):185. Umar A, Boland CR, erdiman JP, Syngal S, de la Chapelle A, Ruschoff J, et al. Revised Bethesda Guidelines or hereditary nonpolyposis colorectal cancer (Lynch syndrome) and microsatellite instability. J Natl Cancer Inst. 2004 Feb 18;96(4):261–8.
MAYO CLINIC INERNAL MEDICIN E BOARD REVIEW: QUESIONS AND ANSWER S
�. HEMATOLOGY QUEST IONS AND ANSWERS
QUESIONS
�. A ��-year-old woman is admitted to the surgical service with severe arterial insufficiency o the right second toe. She has no prior medical history and takes no medications. Physical examination �ndings are normal except or mild splenomegaly and signs o early gangrene in the right second toe. All pulses are ull and equal throughout. Diagnostic testing results are shown in able �.Q�.
Multiple Choice (choose the best answer) ANEMIAS AND MYELOID MALIGNANCIES
�. A ��-year-old man is evaluated or exertional dyspnea. He recalls that � years ago he was told that he had anemia. In reviewing his records, you note that at that time his hemoglobin level was �.� g/dL and his hematocrit was ��% with an increased mean corpuscular volume (MCV); the remainder o his complete blood cell count was normal. On physical examination, he had conjunctival pallor, normal heart and lung �ndings, no lymphadenopathy, no hepatomegaly or splenomegaly, and no petechiae or ecchymoses. Diagnostic testing results are shown in able �.Q�.
Table �.Q� COMPONEN
Table �.Q� COMPONEN
FINDING
FINDING
Hemoglobin, g/dL
��.�
Hematocrit, %
��
Leukocyte count, ���/L Segmented neutrophils, % Band cells, % Lymphocytes, % Monocytes, % Basophils, % Eosinophils, %
��.� �� � �� � � � �,���
Hemoglobin, g/dL
�.�
Platelet count, ���/L
Hematocrit, %
��
Mean corpuscular volume, L ��
Mean corpuscular volume, L
��� (reerence range, ��–��)
Erythrocyte sedimentation rate, mm/h
��
Leukocyte count, ���/L Neutrophils, % Lymphocytes, % Monocytes, % Basophils, % Eosinophils, %
�.� �� �� � � �
Leukocyte alkaline phosphatase score
��� (reerence range, ��–���)
Serum erritin
Within reerence range
Serum iron
Within reerence range
Platelet count, ���/L
��
Within reerence range
Reticulocyte count, % o erythrocytes
�.� (reerence range, �.�–�.�)
Serum total iron-binding capacity Peripheral blood �lm
Absolute reticulocyte count, ���/L
��.� (reerence range, ��.�–��.�)
Increased large platelets with some clustering; leukocytes and erythrocytes are unremarkable
Peripheral blood �lm
Dimorphic erythrocyte population with pronounced macrocytes
Bone marrow aspiration and biopsy
Increased cellularity with increased and atypical megakaryocytes in clusters; reticulin staining is normal
Lactate dehydrogenase, U/L
��� (reerence range, ���–���)
Chromosomal analysis
Normal emale karyotype (��XY)
Which o the ollowing is the most likely explanation or these �ndings?
Which o the ollowing is the most likely diagnosis?
a. b. c. d. e.
a. Essential thrombocythemia b. Vasculitis c. Philadelphia chromosome–negative chronic myeloid leukemia (CML) d. Primary myelo�brosis (PMF)
Acute myeloid leukemia (AML) Vitamin B�� de�ciency Hemolytic anemia Myelodysplastic syndrome (MDS) Primary myelo�brosis ��
�. A ��-year-old ��-year-old man is admitted admitted to the the hospital hospital or an elective cholecystectomy. You are asked to see him because he had anemia on preoperative testing. He tells you that he has always been told by his physicians that he has mild anemia; his medical history is otherwise unremarkable. His vital signs are normal. His conjunctivae are mildly icteric, and the spleen is palpable in the lef upper quadrant. Findings on the remainder o the physical examination are normal. Diagnostic testin testingg results are shown in able �.Q�. Table �.Q� C O M P ON E N
F IN D IN G
Hemoglobin, g/dL
��.�
Hematocrit, %
��
Leukocyte count, �� ���/L Differential count
�.� Within reerence reerence ranges
Platelet count, �� ���/L
���
Mean corpuscular volume, L
��
Reticulocyte count, %
�
Absolute reticulocyte reticulocyte count, �� ���/L
��� (r ��� (re eer ereenc ncee ra rang nge, e, ��.�–��.�)
Peripheral blood �lm
Polychromasia with numerous microspherocytes
Which o the ollowing tests would most likely help con�rm the diagnosis? a. b. c. d.
Hemoglobin electrophor electrophoresis esis Osmotic ragility test Direct and indirect antiglobulin antiglobulin (Coombs) tests Bone marrow marrow aspiration aspiration and and biopsy
�. A ��-year-old ��-year-old black man with with sickle cell disease disease prespresents to the emergency department with abdominal pain, chest pain, and shortness o breath. His dyspnea evolved over �� hours afer a visit with his niece and nephew. His history is signi�cant or approximately � emergency department visits or hospital admissions per year or painul crises. Tree years ago, a go, he spent � weeks in the hospital afer an episode o acute chest syndrome. He has been taking hydroxyurea but only intermittently because o �nancial concerns. His pulse is ��� beats per minute and regular, his blood pressure is ���/�� mm Hg, his respiratory rate is �� breaths per minute, and his temperature is ��.� °C. Pulse oximetry shows ��% oxygen saturation with room air and ��% with � L o oxygen by nasal cannula. His lungs have scattered scatte red inspiratory crackles in the right midlung �eld. His spleen is not palpable. Te remainder o the physical examination �ndings are normal. Diagnostic testing testing results are shown in able �.Q�.
��
•
Table �.Q� C O M P ON E N
F IN D IN G
Hemoglobin, g/dL
�.�
Hematocrit, %
��
Mean corpuscular volume, L
��
Leukocyte count, �� ���/L Segmented neutrophils, % Band cells, % Lymphocytes, % Monocytes, % Basophils, % Eosinophils, %
��.� �� � �� � � �
Platelet count, �� ���/L
���
Creatinine, mg/dL
�.�
Peripheral blood �lm
Anisopoikilocytosis with multiple sickle cells
A chest radiograph shows a right middle and upper lobe air space in�ltrate. Te patient is given supplemental oxygen, adequate pain control, and intravenous antibiotics. Which o the ollowing should you now order? a. a. b. c. d. e.
Hydroxyurea Erythrocyte exchange exchange transusion Plasma exchange Anticoagulation with with unractionated unractionated heparin heparin Aggressive intraven intravenous ous �uid hydration
�. A ��-year-old ��-year-old man present presentss with with weakness weakness o his right arm and leg. His symptoms began yesterday and are now resolved. He also reports a �-month history o recurrent headaches and atigue. He is a nonsmoker. His medical history is signi�cant or high blood pressure. His blood pressure is ���/�� mm Hg, his oxygen saturation saturat ion is ��% on room air, his ace ace is plethoric, and a right carotid bruit is heard. Other �ndings on physical examination are normal. Diagnostic testing results are shown in able �.Q�. Table �.Q� C O M P ON E N
F IN D IN G
Hemoglobin, g/dL
��.�
Hematocrit, %
��
Mea ean n co corp rpuusc scuula larr vol voluume me,, L
��
Leukocyte count, �� ���/L Neutrophils, Neutrophil s, % Lymphocytes, % Monocytes, % Basophils, %
��.� �� �� � �
Platelet count, �� ���/L
���
Ery rytthro ropo poie iettin in,, mIU mIU/m /mL L
<� (r (re eeere ren nce ra ran nge ge,, �–� �–��)
MAYO CLINIC INERNAL MEDICIN E BOARD REVIEW: REVIEW: QUESIONS AND ANSWER S
Carotid ultrasonography ultrasonography shows a ��% stenotic lesion in the right carotid. Te patient is hospitalized and begins antiplatelet therapy. Which o the ollowing should you order next? a. JAK� V���F mutation testing b. Fluorescence in situ hybridization (FISH) or BCRb. BCR-ABL ABL testing c. Arterial blood gas analysis analysis d. Bone marrow marrow aspiration aspiration and and biopsy
�. A ��-year-old woman with a history history o systemic lupus erythematosus (SLE) presents with atigue. She has been receiving anti–tumor necrosis actor therapy and has been managing the SLE well. However, she has recently experienced worsening atigue. Her vital signs are normal. Her ace and conjunctivae are jaundiced, and she has a ading butter�y rash on her ace. Te spleen is pal pable on deep inspiratio inspiration. n. Diagnostic Diagnostic testing testing results results are shown in able �.Q�, and the peripheral blood �lm is shown in Figure �.Q�. Table �.Q�
Which o the ollowing is the best interp interpretation retation o these data? a. b. c. d.
�.
Te hemolysis is predominantly predominantly intravascular intravascular.. Te bone marrow marrow is not not responding responding to the anemia. Direct Coombs Coombs testing testing results results should be positive. positive. Urine Uri ne hemoglobin testing testing results should be positive.
A ��-year��-year-old old woman with active rheumatoid arthritis presentss with atigue and joint pain. present pa in. She received the diagnosis o rheumatoid arthritis � years earlier and has been taking prednisone �� mg daily and methotrexate with olate olate weekly. weekly. She has had chronic chronic atigue atigue and aneanemia. Her vital signs are normal. Her conjunctivae are pale, and she has active synovitis affecting both knees, her wrists, and elbows, with rheumatoid nodules on the extensor surace o her right orearm. Te remainder o the physical examination �ndings are normal. Diagnostic testing results are shown in able �.Q�.
Table �.Q� C OM P ON E N
C OM P O N E N
F IN DIN G
F IN D I N G
Hemoglobin, g/dL
�.�
Hemoglobin, g/dL
�.�
Hematocrit, %
��
Hematocrit, %
��
Mean corpuscular volume, L
��
Mean corpuscular volume, L
��
Leukocyte count, �� ���/L Differential count
�.� Within reerence reerence ranges
Leukocyte count, �� ���/L Neutrophils, % Lymphocytes, % Monocytes, %
��.� �� � �
Platelet count, �� ���/L
���
Platelet count, �� ���/L
���
Reticulocyte count, %
�
Eryth Ery thro rocy cyte te se sedi dime men nta tati tion on ra rate te,, mm/ mm/hh
��
��� ��.� .� (r (re eer eren ence ce ra rang nge, e, ��.�–��.�)
Er ythropoietin, mIU/mL
�� (reerence range, �–��)
�
Absolute reticulocyte reticulocyte count, �� �� /L
Erythrocyte Eryth rocyte sedi sedimen mentat tation ion rate rate,, mm/ mm/hh �� Lac acta tate te deh ehyd ydrrog ogeenas ase, e, U/L
���� (reerence ra �� ran nge, ��� ��– –�� ����)
otal bilirubin, mg/dL
�.� (reerence range, �.�–�.�)
Indirect bilirubin, mg/dL
�.�
Which o the ollowing laboratory �ndings are consistent with this condition? a. Elevated hepcidin, elevated erritin, elevated total iron-bindingg capacity (IBC), iron-bindin ( IBC), elevated serum iron b. Elevated hepcidin, elevated erritin, erritin, decreased IBC, ele vated serum iron iron c. Decreased hepcidin, elevated erritin, erritin, decreased IBC, ele vated serum iron iron d. Elevated hepcidin, elevated erritin, erritin, decreased IBC, normal serum iron e. Decreased hepcidin, elevated erritin, erritin, elevated elevated IBC, normal serum iron COAGULATION
Figure �.Q�
�. A ��-year-old ��-year-old man underwent right total knee replacement � days ago. Swelling has developed in his right lower extremity, and Doppler ultrasonography con�rms the presence o a right super�cial emoral vein thrombosis. His curren currentt medications include oxycodone and subcutaneous unractionated heparin. Results o preoperative tests, including a complete blood cell count �. HEMAOLOGY QUESIONS AND ANSWERS
•
��
and liver and kidney unction, were normal. Other laboratory data include the ollowing: hemoglobin ��.� g/dL, leukocyte count �.�× �.� ×���/L, and platelet count ��× �� ×���/L. In addition to stopping the use o subcutaneous heparin, what is the next most appropriate step in management o this patient?
Food and Drug Administration Administration (FDA) approval o dabdab igatran, which requires no monitoring, and he would like a prescription or this new drug. Which o the ollowing statements is true about the use o dabigatran in atrial �brillation compared with the well-managed use o wararin?
a. b. c. d.
a. Switching to dabigatran would result in superior outcomes. outcomes. b. Switching to dabigatran would result result in inerior outcomes. outcomes. c. Switching to dabigatran would provide no signi�cant c. bene�t. d. Dabigatran is FDA approved or postoperative postoperative thrombothrombo prophylaxis or knee and hip replacement replacement surgery. surgery. e. Dabigatran is FDA approved approved as an anticoagulant anticoagulant or patients patients who have received received a mechanical heart valve.
Start low-mole low-molecular-wei cular-weight ght heparin therapy. Start intravenous intravenous therapeutic doses o heparin. Start direct thrombin inhibi inhibitor tor therapy. Start aspirin therapy. therapy.
�. A ��-year��-year-old old man presents with deep vein thrombosis o the right emoral vein. Tree months ago, he received a diagnosis o systemic lupus erythematosus (SLE). In addition to con�rming SLE, laboratory testing also documented the presence o a lupus anticoagulant (LAC). Tere is no amily history o venous thrombosis. Current medications include hydroxychloroquine. Laboratory testing shows normal results or a complete blood cell count and or tests o liver and kidney unction. Special coagulation testing con�rms the persistence o an LAC. What W hat is the most reasonable duration o wararin anticoagulation or this patient? a. � months b. � year c. � months d. Long-term d. e. � weeks
��. A ��-year-old woman is brought to the emergency department afer having � witnessed tonic-clonic seizure. She had appeared conused or the preceding ew hours. On examination, she is ebrile and appears slightly conused; otherwise, neurologic and physical examination �ndings are normal. Laboratory testing results are shown in able �.Q��, and the peripheral blood smear is shown in Figure �.Q��. Table �.Q�� C O MP O N E N
��. A ��-year-old ��-year-o ld white woman woman has been admitted to the hospital with pulmonary embolism. She has no chronic illnesses and is receiving no medication medicationss except or combination estrogen-progesterone birth control pills that she started using approximately � year earlier. Results were normal or a complete blood cell count, baseline prothrombin prothro mbin time, activated partial thromboplast thromboplastin in time (aP), and tests o kidney and liver unction. Te patient is currently receiving therapeutic doses o intravenous unractionated heparin, and her aP is therapeutic at �� seconds. A panel o thrombophilia tests has been perormed. Which o the ollowing statements about her thrombophilia test results is correct?
F IN D IN G
REF ERENCE R ANGE
Hemoglobin, g/dL
�
��–��
Platelet count, �� ���/L
��
���– ���
Leukocyte count, �� ���/L
�
�.�–��.�
�.�
�.�–�.�
Creatinine, mg/dL
a. DNA-based testing or or actor V Leiden and prothrombin prothrombin mutations are reliable. G�����A mutations b. Low antithrombin antithrombin con�rms con�rms a hereditary hereditary de�ciency state. state. c. A positive result result on lupus lupus anticoagulant anticoagulant (LAC) testing testing con�rms antiphospholipid antibody syndrome. s yndrome. d. Low protein S con�rms con�rms the presence o a hereditary de�ciency state.
��. A ��-year-old man with chronic atrial �brillation has been treated with wararin. He has no other chronic illnesses and is receiving no other medications long-term except or lipid-lowering agents. agents. Results o his complete blood cell count and tests o renal and kidney unction are normal. He checks his prothrombin time monthly and has kept the international normalized ratio (INR) within the therapeutic range (�–�) or the duration o his therapy with wararin. He has heard about recent US
��
•
Figure �.Q��
What is the most appropriat appropriatee next step in management? a. b. c. d.
Red blood cell transusion Platelet transusion Gamma globulin administratio administrationn Plasma exchange
MAYO CLINIC INERNAL MEDICIN E BOARD REVIEW: REVIEW: QUESIONS AND ANSWER S
��. A ��-year-old man with chronic atrial �brillation has been receiving dabigatran �� mg twice daily or the past � months. He has not had any thrombotic or hemorrhagic complications. He has a history o colon pol yps, or which he needs to undergo a colonoscopy with possible polypectomy. Apart rom an irregular pulse, his physical examination �ndings are normal. Results were normal or a complete blood cell count and tests o renal and liver unction. Te calculated creatinine clearance is �� mL/min. For how long should dabigatran use be discontinued beore the colonoscopy? a. b. c. d. e.
No need to discontinue �� hours �� hours � days � days LYMPHOID MALIGNAN CIES
��. At her annual physical examination, an asymptomatic ��-year-old woman has lymphocytosis (��×���/L) with a normal hemoglobin level and platelet count. On examination, she has �-cm lymphadenopathy in the cervical region and no palpable liver or spleen enlargement. A peripheral blood smear shows identically appearing mature lymphocytes with smudge cells. Flow cytometry o the peripheral blood lymphocytes shows a monoclonal B population with dim expression o λ light chain and CD�� that is positive or expression o CD�, CD��, and CD��. Which o the ollowing is the best next step in her management? a. b. c. d. e.
Combination chemoimmunotherapy Chlorambucil therapy Allogeneic peripheral blood stem cell transplant Combination monoclonal antibody therapy Active monitoring or disease progression and complications
��. en years ago, a previously healthy ��-year-old woman presented to her physician with a �-month history o pruritis, drenching night sweats, unintentional weight loss, and nonproductive cough. On examination, she had �-cm cervical lymphadenopathy. A computed tomographic scan showed a ��-cm-diameter anterior mediastinal mass. An excisional biopsy o a cervical lymph node showed nodular sclerosing Hodgkin lym phoma. Afer she was treated with ABVD (doxorubicin [ Adriamycin], bleomycin, vinblastine, and d acarbazine) combination chemotherapy ollowed by involved �eld radiotherapy, the disease was in complete remission. Now you see her or the �rst time or an annual physical examination. Te disease remains in complete remission. Compared to her peers, this patient is at increased risk o which o the ollowing conditions? a. b. c. d. e.
Breast cancer Coronary artery disease Hypothyroidism Skin cancer All o the above
��. An ��-year-old man is admitted to the hospital ater alling on an icy sidewalk and racturing his hip. He undergoes open reduction and internal ixation o the racture. At surgery, there does not appear to be any bone disease at the racture site. he patient was previously asymptomatic. Physical examination indings are otherwise unremarkable. Serum protein electrophoresis and immunoixation show an IgM κ monoclonal protein (�.� g/dL). he complete blood cell count and serum creatinine levels are normal. Skeletal survey shows no additional bone deects. Which o the ollowing statements is true or this patient ? a. He has multiple myeloma and requires treatment. b. He has a lower risk o a clinically signi�cant lymphocytic or plasma cell malignancy than patients with an IgG monoclonal protein. c. He requires a radioisotope bone scan to evaluate his bone integrity. d. He requires regular ollow-up and serial measurements o his monoclonal protein level. e. He has a ��% annual risk o multiple myeloma.
��. A ��-year-old Arican American man was seen last week by his primary care physician or mild dyspnea. He has also noted intermittent peripheral edema. During the evaluation, an electrocardiogram showed low-voltage QRS complexes in the limb leads. Te troponin level was elevated (�.�� ng/mL). Tis �nding suggested the need or a coronary angiogram, which showed no signi�cant coronary artery disease. An echocardiogram showed diffuse lef ventricular thickening with a granular texture to the myocardium and a septal thickness o �.� cm (normal <�.� cm). Te complete blood cell count results were normal. Serum and urine protein electrophoresis and immuno�xation were unremarkable. Serum ree light chain levels were not increased. What is the most likely diagnosis? a. b. c. d. e.
AA amyloidosis Light chain–related amyloidosis Hypertrophic obstructive cardiomyopathy Amyloidosis due to transthyretin deposition Amyloidosis due to β�-microglobulin deposition
��. A ��-year-old man presented to his primary care physician or evaluation o atigue. He was previously healthy with the exception o chronic musculoskeletal low back pain, or which he occasionally takes nonsteroidal anti-in�ammatory drugs. On examination, he is pale. Complete blood cell count results are as ollows: hemoglobin �.� g/dL, mean corpuscular volume �� L, leukocyte count �.� ���/L, and platelet count ������/L. Results o the ecal occult blood test are positive. During upper and lower endoscopy, a �.��.�-cm ulcerative lesion is noted in the lesser cur vature o the stomach. Te lesion is biopsied and identi�ed as a MAL lymphoma. Which o the ollowing is characteristic o MAL lymphoma?
�. HEMAOLOGY QUESIONS AND ANSWERS
•
��
a. Most cases are treated with anthracycline-based chemotherapy. b. It is caused by chronic stimulation with Chlamydophila psittaci. c. Radiotherapy is necessary in most cases. d. It requently undergoes transormation to a large-cell lymphoma. e. Te combination o amoxicillin, omeprazole, and clarithromycin is the most appropriate �rst-line treatment.
��. A ��-year-old woman presented to the emergency department with new-onset back pain, conusion, and constipation over the past week. Her past medical history is signi�cant only or hypertension. On examination, she is slightly pale with slow cognition and point tenderness over the lumbar spine. Plain �lms o the lumbar spine show osteolytic lesions in L�, L�, and L�. Laboratory values are as ollows: hemoglobin �.� g/dL, leukocyte count �.����/L with a normal differential count, platelet count ������/L, creatinine �.� mg/dL, total calcium ��.� mg/dL, albumin �.� g/dL, and total protein �.� g/dL. What is the most likely diagnosis? a. b. c. d. e.
��. A ��-year-old man presents to the emergency department with a �-week history o progressively worsening abdominal pain and night sweats. Physical examination �ndings were signi�cant or palpable bilateral �-cm axillary lymph nodes and diffuse abdominal tenderness with no rebound or guarding. Computed tomography o the abdomen and pelvis showed retroperitoneal and mesenteric lymphadenopathy. Excisional biopsy o an axillary node was positive or diffuse, large B-cell lymphoma. Positron emission tomography showed �uorodeoxyglucose-avidity in the axillary, mesenteric, and retroperitoneal lymph nodes. Results o the bone marrow examination were normal. Which o the ollowing is the best next step? a. Combination therapy with rituximab, cyclophosphamide, doxorubicin, vincristine, and prednisone (R-CHOP) b. Observation c. Combination therapy with cyclophosphamide, doxorubicin, vincristine, and prednisone (CHOP) d. Autologous stem cell transplant e. Involved �eld radiotherapy
Metastatic breast cancer Hydrochlorothiazide use Multiple myeloma Primary hyperparathyroidism Milk alkali syndrome
��
•
MAYO CLINIC INERNAL MEDICINE BOARD REVIEW: QUESIONS AND ANSWERS
ANSWERS
�. Answer d.
hemolytic anemia, which produces positive Coombs test results, can cause spherocytes as well; however, the history o lielong anemia makes this diagnosis unlikely. A hemoglobin electrophoresis would help in diagnosing thalassemia or a hemoglobinopathy; however, these conditions do not maniest with microspherocytes on the peripheral blood �lm. Tere is no indication or a bone marrow biopsy since the reticulocyte response is appropriate and no other cytopenias are apparent. (See Gallagher in the “Suggested Reading” list.)
MDS most commonly maniests as isolated macrocytic anemia. MDS can evolve to include pancytopenia over several years; the typical peripheral smear �ndings include a dimorphic erythrocyte population (microcytes and oval macrocytes) with an overall prominent macrocytosis and an MCV around ��� L. Te chronicity o MDS—in particular, anemia preceding the diagnosis o pancytopenia by several years—is in contrast to the typically acute maniestation o AML, which is thereore an unlikely possibility in this patient. Primary myelo�brosis, a myeloprolierative neoplasm, causes �brosis in the bone marrow, resulting in extramedullary hematopoiesis and signi�cant splenomegaly, and typically does not cause a macrocytic anemia. Vitamin B�� de�ciency can cause a megaloblastic anemia and maniest with slowly evolving macrocytic anemia and eventually pancytopenia, but the peripheral smear would not show a dimorphic erythrocyte population. (See efferi and Vardiman in the “Suggested Reading” list.)
�. Answer b.
Te patient has acute chest syndrome, a sickle cell anemia complication that is an indication or urgent red cell (not plasma) exchange transusion to decrease the hemoglobin S level to less than ��% to ��%. Gentle �uid resuscitation is appropriate (along with oxygen support and antibiotics, since about one-third o acute chest syndrome events are initiated by or associated with bacterial pneumonia). Aggressive �uid resuscitation, leading to overhydration, might cause pulmonary edema and worsen the oxygenation. Pulmonary embolism is possible, but ull anticoagulation is not warranted until embolism is documented. Use o hydroxyurea might have prevented this crisis, but it is o no value or the acute condition. (See Vij and Machado in the “Suggested Reading” list.)
�. Answer a.
Extreme thrombocytosis may be reactive and occur with severe iron de�ciency or in�ammatory states (with elevated erythrocyte sedimentation rates) or afer splenectomy; patients are typically asymptomatic. Clonal thrombocytosis is related to a myeloprolierative neoplasm, which usually causes splenomegaly. ypical bone marrow �ndings include a hypercellular bone marrow with increased atypical megakaryocytes in clusters. Essential thrombocythemia may cause extreme thrombocytosis (platelet count >�,���×�� �/L); however, it can also occur less commonly with polycythemia rubra vera (typically with erythrocytosis), the cellular phase o PMF, or rarely CML. Te normal karyotype makes CML much less likely since it typically maniests with the Philadelphia chromosome t(�;��). Increased reticulin �brosis would have been seen on the bone marrow biopsy i the patient had PMF. (See efferi in the “Suggested Reading” list.)
�. Answer a.
Polycythemia may be secondary, as with erythropoietinmediated causes such as chronic hypoxemia, living at high altitude, and high oxygen affinity hemoglobinopathies. Polycythemia vera is a myeloprolierative neoplasm that can maniest with arterial thrombosis secondary to hyper viscosity rom the increased concentration o erythrocytes. Te low erythropoietin rules out erythropoietin-mediated causes, leaving the presumptive diagnosis o polycythemia vera. With JAK� V���F mutation testing o peripheral blood, results are positive or approximately ��% o patients who have polycythemia vera. FISH or BCR-ABL testing would screen or chronic myeloid leukemia, which does not maniest with polycythemia. Although bone marrow aspiration and biopsy would be helpul, it is not immediately necessary and could be considered later. (See Patnaik and efferi in the “Suggested Reading” list.)
�. Answer b.
When a patient presents with premature gallstones, one should consider whether they may be due to pigment gallstones rom chronic hemolysis causing indirect hyperbilirubinemia. Te presence o microspherocytes is consistent with hereditary spherocytosis, and the diagnostic test is an osmotic ragility test, which identi�es a congenital membrane deect. ypically, acquired warm autoimmune
�. Answer c.
Hematologic complications o SLE include anemia o chronic disease, pure red cell aplasia, and warm autoimmune ��
hemolytic anemia (WAIHA). Te presentation and laboratory data suggest hemolysis, and the blood smear shows spherocytes. Tese �ndings are consistent with WAIHA, which causes extravascular hemolysis. Te reticulocytosis suggests that the bone marrow response is adequate. In intravascular hemolysis, the urine is positive or hemoglobin. (See Packman in the “Suggested Reading” list.) �. Answer d.
Rheumatoid arthritis is a chronic in�ammatory disorder that may lead to anemia o chronic disease. Anemia o chronic disease results rom the effect o elevated cytokines on hematopoiesis, including upregulation o hepcidin, leading to increased erritin rom iron malutilization and downregulation o erroportin, the main iron exporting system. ranserrin is also downregulated, leading to decreased IBC and normal to decreased serum iron levels. (See Weiss and Goodnough in the “Suggested Reading” list.) �. Answer c.
Te timing and degree o thrombocytopenia are consistent with immune-mediated heparin-induced thrombocytopenia type II. Unractioned heparin and low-molecular-weight heparin are contraindicated. Aspirin would not be the sole management agent or established thrombosis. Te most appropriate step is to start a direct thrombin inhibitor. �. Answer d.
Presentation with a vascular thrombosis and persistence o a LAC or �� weeks or more satis�es the criteria or an antiphospholipid syndrome. Tis patient has a high risk or recurrent venous thrombosis on discontinuing anticoagulation; thus, long-term wararin is recommended with periodic reassessment or saety. ��. Answer a.
DNA-based testing is reliable or patients receiving heparin or wararin and or patients who have acute thrombosis. However, acute thrombosis and heparin can cause lower antithrombin activity results, which should be veri�ed at another time, when heparin and acute thrombosis are not actors. A single positive test result or LAC does not con�rm antiphospholipid syndrome; ollow-up testing at ��-week intervals is required to demonstrate persistence o LAC. Acute thrombosis and estrogen use can lower protein S levels; thus, abnormally low results require ollow-up con�rmation. ��. Answer c.
Among patients randomly assigned to receive dabigatran, overall outcomes were noninerior when compared with the well-managed use o wararin, thus providing no signi�cant advantages. Te group o patients that derived the most bene�t rom dabigatran was the group with INRs outside the recommended therapeutic range. Dabigatran is FDA approved only to reduce the risk o stroke and
��
•
systemic embolism in patients with nonvalvular atrial �brillation. ��. Answer d.
Plasma exchange is the treatment o choice or thrombotic thrombocytopenic purpura (P). Although red blood cell transusion may be indicated, it does not address the underlying pathogenesis o P. Platelets are thought to be contraindicated in P because o the theoretical possibility o worsening the P. Gamma globulin is ineffective in increasing the platelet count in P. ��. Answer d.
Dabigatran is cleared through the kidneys. It has a prolonged hal-lie in patients who have a creatinine clearance less than �� mL/min compared with patients who have a creatinine clearance greater than �� mL/min. ��. Answer e.
Chronic lymphocytic leukemia (CLL) is a clonal lym phoprolierative disorder o mature lymphocytes. Te clinical diagnosis requires a B-lymphocyte count o more than ����/L. Peripheral blood smears typically show smudge cells, which are lymphocytes that have broken during processing o the slide. Te clinical course o CLL is chronic in most patients. For those with early-stage disease, standard practice is to withhold treatment until the disease is active or progressive. However, patients need to be monitored or disease progression, autoimmune complications, inections, and second cancers. ��. Answer e.
Hodgkin lymphoma therapy is curative in about ��% o cases. However, there are late complications o therapy, particularly in those treated beore modern chemotherapy and radiotherapy. At �� years, the risk o death rom other causes surpasses that o risk o death rom Hodgkin lym phoma. Patients are at higher risk o secondary malignancies, cardiovascular disease, thyroid disorders, and inertility than the general population. Many o these conditions can be attributed to chemotherapy and radiotherapy. ��. Answer d.
Tis patient has monoclonal gammopathy o undetermined signi�cance (MGUS), the most common dysproteinemia. In MGUS, the M protein level is typically less than � g/dL, the bone marrow has less than ��% plasma cells, and the hemoglobin, creatinine, calcium, and bone radiographs are normal. Te risk o progression to a lymphocytic or plasma cell malignancy is about �% per year. Patients with an IgM or IgA monoclonal protein are at higher risk o progression than those with an IgG protein. Patients with MGUS need to be observed. ��. Answer d.
Te patient has senile cardiac amyloidosis. Tis syndrome is usually isolated to the heart with ew clinically signi�cant
M A YO C L I N I C I N E R N A L M E D I C I N E B O A R D R E V I EW : Q U E S I ON S A N D A N S W E R S
deposits elsewhere, and the echocardiographic �ndings are ofen out o proportion to the degree o symptoms. ransthyretin is the protein causing the amyloid deposits; most patients have wild-type transthyretin. ��. Answer e.
With combination antibiotic therapy, ��% o gastric MAL lymphomas are cured. In cases reractory to antibiotics, tumors may carry the t(��;��) translocation, and involved �eld radiotherapy is effective. Combination chemotherapy is reserved or advanced disease. Te majority o cases are associated with Helicobacter pylori inection. ��. Answer c.
Tis patient has multiple myeloma with evidence o end-organ damage rom the plasma cell prolierative disorder (hypercalcemia, renal ailure, anemia, and osteolytic bone lesions). Te other answer choices are possible causes o hypercalcemia, but only multiple myeloma accounts or all the presenting symptoms, including the elevated level o total protein. ��. Answer a.
Tis patient has advanced-stage, diffuse, large B-cell lym phoma, and R-CHOP chemotherapy is the standard o
care. Rituximab is an anti-CD�� monoclonal antibody that improves overall survival when added to CHOP chemotherapy or aggressive B-cell lymphomas. For patients whose disease relapses or is reractory, autologous stem cell transplant is the standard therapy. Radiotherapy can be used in combination with chemotherapy in early-stage (I-IIA) nonbulky disease but is not standard therapy or advanced disease. SUGGESED R EADING Gallagher PG. Red cell membrane disorders. Hematology Am Soc Hematol Educ Program. 2005:13–8. Packman CH. Hemolytic anemia due to warm autoantibodies. Blood Rev. 2008 Jan;22(1):17–31. Epub 2007 Sep 27. Patnaik MM, efferi A. Te complete evaluation o erythrocytosis: congenital and acquired. Leukemia. 2009 May;23(5):834–44. Epub 2009 Mar 19. efferi A. Annual clinical updates in hematological malignancies: a continuing medical education series: polycythemia vera and essential thrombocythemia: 2011 update on diagnosis, risk-strati�cation, and management. Am J Hematol. 2011 Mar;86(3):292–301. efferi A, Vardiman JW. Myelodysplastic syndromes. N Engl J Med. 2009 Nov 5;361(19):1872–85. Vij R, Machado RF. Pulmonary complications o hemoglobinopathies. Chest. 2010 Oct;138(4):973–83. Weiss G, Goodnough L. Anemia o chronic disease. N Engl J Med. 2005 Mar 10;352(10):1011–23.
�. HEMAOLOGY QUESIONS AND ANSWERS
•
��
This page intentionally left blank
��. NEPHROLOGY QUES TIONS AND ANSWERS
QUESIONS
Which o the hyponatremia?
Multiple Choice (choose the best answer)
a. b. c. d. e.
ELECTROLYTE DISORDERS
�. A ��-year-old man, a ormerly heavy smoker, presents with a blood-tinged cough and weakness. He uses an ipratropium bromide inhaler. On physical examination, his vital signs are normal, his jugular venous pressure is normal, and he has no edema. His laboratory values are as ollows: sodium ��� mEq/L, potassium �.� mEq/L, bicarbonate �� mEq/L, serum urea nitrogen � mg/dL, and serum creatinine �.� mg/dL. What should you obtain next? a. b. c. d. e.
ollowing
Table ��.Q� COMPONEN
Table ��.Q� FINDING
���
Potassium, mEq/L
�.�
Chloride, mEq/L
��
Bicarbonate, mEq/L
��
Serum creatinine, mg/dL
�.�
Serum osmolality, mOsm/kg
���
Urine osmolality, mOsm/kg
���
Tyrotropin
Within reerence range
Morning cortisol
Within reerence range
his
HCZ Decreased intravascular volume Beer drinking Chronic kidney disease Syndrome o inappropriate secretion o antidiuretic hormone
FINDING
Blood pressure, mm Hg
���/��
Pulse, beats per minute
�� (irregular)
�. A ��-year-old man with a history o hypertension and drinking alcoholic beverages (mostly beer) presents Respiratory rate, breaths per minute with episodic conusion, weakness, and imbalance. For Oxygen saturation with room air, % the past � weeks, he has been taking hydrocholorothiazide (HCZ) �� mg once daily. On physical examina- Weight, kg tion, his blood pressure is ���/�� mm Hg, his pulse is Mucosal suraces ��� beats per minute, his respiratory rate is �� breaths per minute, his jugular venous pressure is normal, the Mental status liver edge is palpable, and he has no edema. Laboratory Lung �elds test results are shown in able ��.Q�.
Sodium, mEq/L
to
�. An ��-year-old woman who is a nursing home resident with hypertension, multi-inarct dementia, dysphasia, and atrial �brillation was admitted because her mental status changed. Physical examination �ndings and laboratory test results are shown in able ��.Q�.
Serum osmolality value Urine osmolality value Tyrotropin and morning cortisol values Computed tomographic (C) scan o the chest Magnetic resonance imaging o the brain
COMPONEN
contributed
�� �� �� Dry Sleepy, oriented to name only Clear, no edema
Sodium, mEq/L
���
Potassium, mEq/L
�.�
Chloride, mEq/L
���
Bicarbonate, mEq/L
��
Serum creatinine, mg/dL
�.�
Urine osmolality, mOsm/kg
���
What is her estimated water de�cit? a. b. c. d. e.
��
�.� L �.� L �.� L ��.� L �.� L
�. For the patient in the preceding question, what is the maximum rate o correction or her hypernatremia in the next �� hours? a. b. c. d. e.
�� mEq/L daily �� mEq/L daily �� mEq/L daily �� mEq/L daily �� mEq/L daily
Table ��.Q� COMPONEN
�. A ��-year-old man presents with a history o generalized weakness or � week. Otherwise, he has hypertension, diabetes mellitus, and obstructive sleep apnea (he uses nocturnal continuous positive airway pressure). He has been taking urosemide �� mg twice daily or � years and metolazone �.� mg twice daily or � weeks. Physical examination �ndings and laboratory test results are shown in able ��.Q�. Table ��.Q� COMPONEN
Body mass index
FINDING
��.�
Blood pressure, mm Hg
���/��
Pulse, beats per minute
��
Edema
only medication she uses is arti�cial tears. Physical examination �ndings and laboratory test results are shown in able ��.Q�. A computed tomographic scan o the abdomen is shown in Figure ��.Q�.
Absent
Sodium, mEq/L
���
Potassium, mEq/L
�.�
Bicarbonate, mEq/L
��
Chloride, mEq/L
��
Serum creatinine, mg/dL
�.�
Calcium, mg/dL
�.�
Glucose, mg/dL
���
FINDING
Body mass index
��
Blood pressure, mm Hg
��/��
Pulse, beats per minute
��
Respiratory rate, breaths per minute
��
Edema
Absent
Sodium, mEq/L
���
Potassium, mEq/L
�.�
Chloride, mEq/L
���
Bicarbonate, mEq/L
��
Serum creatinine, mg/dL
�.�
Anion gap, mEq/L
�
Arterial blood gas pH P���, mm Hg
�.�� ��
Urinalysis pH Protein Glucose Red blood cells per high-power �eld
�.� race Negative �–��
Which o the ollowing did not contribute to this patient’s hypokalemia? a. Metolazone b. Furosemide c. ranscellular shif d. High aldosterone caused by a reduced intravascular volume e. Chronic kidney disease
�. o the patient in the preceding question, what should be given next? a. b. c. d. e.
Insulin intravenously Insulin subcutaneously Potassium intravenously Calcium intravenously Potassium orally ACID�BASE DISORDER S
�.
A ��-year-old woman who has had Sjögren syndrome or � years presents with diffuse muscle weakness. Te ��
•
Figure ��.Q�
MAYO CLINIC INERNAL MEDICIN E BOARD REVIEW: QUESIONS AND ANSWER S
What is the diagnosis? a. b. c. d. e.
Table ��.Q�
Proximal renal tubular acidosis (RA) Distal RA Idiopathic nephrolithiasis Hyporeninemic hypoaldosteronism Gout
COMPONEN
�. A ��-year-old woman presents with new-onset back pain, weakness, episodic light-headedness, and a recent spontaneous lef rib racture. Physical examination �ndings and laboratory test results are shown in able ��.Q�. Table ��.Q� COMPONEN
FINDING
FINDING
Mental status
Incoherent
Blood pressure, mm Hg
���/��
Heart rate, beats per minute
���
Respiratory rate, breaths per minute
��
Edema
Absent
Sodium, mEq/L
���
Potassium, mEq/L
�.�
Chloride, mEq/L
���
Bicarbonate, mEq/L
�
Glucose, mg/dL
���
Serum urea nitrogen, mg/dL
��
Serum creatinine, mg/dL
�.�
Arterial blood gas pH P���, mm Hg
�.�� ��
Serum osmolality, mOsm/kg
���
oxicity screen
Pending
Blood pressure, mm Hg
��/��
Pulse, beats per minute
���
Respiratory rate, breaths per minute
��
Heart examination
Unremarkable
Lung examination
Unremarkable
Edema
Extremities: pitting edema (trace)
Hemoglobin, g/dL
��.�
Sodium, mEq/L
���
What is the most appropriate next step?
Potassium, mEq/L
�.�
Chloride, mEq/L
���
Bicarbonate, mEq/L
��
Serum creatinine, mg/dL
�.�
a. b. c. d. e.
Phosphorus, mg/dL
�.�
Uric acid, mg/dL
�.�
Glucose, mg/dL
��
Arterial blood gas pH P���, mm Hg
�.�� ��
Urinalysis pH Protein Glucose Red blood cells per high-power �eld
�.� �+ �+ �–�
What is the diagnosis? a. Proximal renal tubular acidosis (RA) b. Distal RA c. Drug-induced diarrhea d. Hypoaldosteronism e. Multiple myeloma
�. A ��-year-old man with a history o polysubstance abuse was ound conused. Physical examination �ndings and laboratory test results are shown in able ��.Q�.
Calculate the osmolal gap. Examine a urine specimen microscopically. Initiate �-methylpyrazole (omepizole) therapy. Initiate hemodialysis. Do all o the above.
��. A ��-year-old woman with a history o substance abuse was ound unresponsive. Physical examination �ndings and laboratory test results are shown in able ��.Q��. Table ��.Q�� COMPONEN
FINDING
Blood pressure, mm Hg
���/��
Heart rate, beats per minute
���
Respiratory rate, breaths per minute
��
Edema
Absent
Sodium, mEq/L
���
Potassium, mEq/L
�.�
Chloride, mEq/L
���
Bicarbonate, mEq/L
�
Glucose, mg/dL
��
Serum urea nitrogen, mg/dL
��
��. NEPHROLOGY QUESIONS AND ANSWERS
(continued ) •
��
Table ��.Q�� �CONINUED� COMPONEN
FINDING
Serum creatinine, mg/dL
�.�
Arterial blood gas pH P���, mm Hg
�.�� ��
Serum osmolality, mOsm/kg
���
Anion gap, mEq/L
��
Table ��.Q�� COMPONEN
What is the most appropriate next step? a. b. c. d. e.
��. A ��-year-old man who receives nocturnal oxygen or severe chronic obstructive pulmonary disease was admitted or acute pneumonia. He has some nausea but denied vomiting or having diarrhea. Physical examination �ndings and laboratory test results are shown in able ��.Q��.
Calculate the osmolal gap. Examine a urine specimen microscopically. Initiate �-methylpyrazole (omepizole) therapy. Initiate hemodialysis. Do all o the above.
��. A ��-year-old man who has a history o polysubstance abuse presents with delirium. Reportedly, he ingested an unknown substance. Physical examination �ndings and laboratory test results are shown in able ��.Q��.
FINDING
Mental status
Somnolent
Blood pressure, mm Hg
���/��
Pulse, beats per minute
��
Respiratory rate, breaths per minute
��
Edema
Dependent (�+)
Bicarbonate, mEq/L
��
Arterial blood gas pH P���, mm Hg P��, mm Hg
�.�� �� ��
What is the patient’s acid-base status?
Table ��.Q�� COMPONEN
FINDING
a. b. c. d. e.
Acute respiratory acidosis Chronic respiratory acidosis Acute and chronic respiratory acidosis Acute respiratory acidosis and metabolic alkalosis Chronic respiratory acidosis and metabolic acidosis
Mental status
Disoriented
Blood pressure, mm Hg
���/��
Pulse, beats per minute
��� (normal heart rhythm)
Respiratory rate, breaths per minute
��
Lung �elds
Clear
Edema
Absent
Sodium, mEq/L
���
Potassium, mEq/L
�.�
Bicarbonate, mEq/L
��
Blood pressure, mm Hg
Chloride, mEq/L
���
Heart rate, beats per minute
��
Serum urea nitrogen, mg/dL
��
Respiratory rate, breaths per minute
��
Serum creatinine, mg/dL
�.�
Sodium, mEq/L
���
Arterial blood gas pH P���, mm Hg P��, mm Hg
Potassium, mEq/L
�.�
�.�� �� ��
Chloride, mEq/L
���
Bicarbonate, mEq/L
��
What is the patient’s acid-base status?
Serum creatinine, mg/dL
�.�
a. Metabolic acidosis with respiratory compensation b. Metabolic acidosis and respiratory acidosis c. Metabolic acidosis and respiratory alkalosis d. Acute respiratory alkalosis with appropriate metabolic compensation e. Chronic respiratory alkalosis and metabolic compensation
Glucose, mg/dL
��
Arterial blood gas pH P���, mm Hg P��, mm Hg
�.�� �� ��
��
•
��. An ��-year-old woman with no past medical history reports having nausea and occasional vomiting in the morning or about � months. She denies having diarrhea. Physical examination �ndings and laboratory test results are shown in able ��.Q��. Table ��.Q�� COMPONEN
FINDING
��/��
M A YO C L I N I C I N E R N A L M E D I C I N E B O A R D R E V I EW : Q U E S I ON S A N D A N S W E R S
What is the patient’s acid-base status? a. b. c. d. e.
Acute respiratory alkalosis Chronic respiratory alkalosis Acute and chronic respiratory alkalosis Metabolic acidosis and acute respiratory alkalosis Metabolic acidosis and chronic respiratory alkalosis ACUTE R ENAL FAILURE
��. A ��-year-old man who has a history o hypertension and metastatic prostate cancer presents with dizziness and weakness. He has no known history o kidney disease; his creatinine was �.� mg/dL � months ago. His medications include lisinopril-hydrochlorothiazide (�� mg/��.� mg daily), multivitamin daily, tramadol (�� mg �–� times daily as needed or pain), and leuprolide (�� mg intramuscularly every � months). His blood pressure is ���/�� mm Hg, his pulse is �� beats per minute, and his temperature is ��.�°C. On examination, he appears atigued, heart sounds are normal, lungs are clear, and there is pretibial edema (trace). Laboratory test results are shown in able ��.Q��. Renal ultrasonography shows no evidence o hydronephrosis.
��. A ��-year-old woman who had been previously healthy is evaluated or a rash on her lower extremities that has been present or � week. She has noticed tea-colored urine or several weeks. She takes no medications. Her temperature is ��.�°C, her pulse is �� beats per minute, and her blood pressure is ���/�� mm Hg. Palpable pur pura is present on both legs and eet. Te remainder o the examination �ndings are unremarkable. Results o laboratory studies are notable or creatinine �.� mg/ dL. Te erythrocyte sedimentation rate is �� mm/h. Antinuclear antibody, antibodies to double-stranded DNA, myeloperoxidase, and proteinase � assays are negative. Te C� complement level is low, and the results o cryoglobulin testing are positive. Urinalysis shows proteinuria (�+) and hematuria (�+). Urine microscopy shows �� to �� erythrocytes per high-power �eld (HPF) and � to �� leukocytes per HPF. Which o the ollowing viruses is most likely to be associated with this disorder? a. Epstein-Barr virus b. Cytomegalovirus c. Human immunode�ciency virus d. Parvovirus B�� e. Hepatitis C virus
Table ��.Q�� COMPONEN
FINDING
Hemoglobin, g/dL
��.�
GLOMERULAR DISEASE
Leukocyte count, ���/L
�.�
Platelet count, ���/L
���
Sodium, mEq/L
���
Potassium, mEq/L
�.�
Bicarbonate, mEq/L
��
Chloride, mEq/L
���
Serum creatinine, mg/dL
�.�
Serum urea nitrogen, mg/dL
��
Calcium, mg/dL
��.�
Albumin, g/dL
�.�
Phosphorus, mg/dL
�.�
��. A ��-year-old truck driver is reerred or evaluation o persistent asymptomatic microhematuria. He has not seen a physician since he was �rst told about blood in his urine during a Department o ransportation physical examination � years ago. At that time, he had a computed tomographic scan o the abdomen and pel vis, cystoscopy with retrograde pyelograms, and urine cytology. He was told last year that his blood glucose level was elevated. He has never smoked and takes no medications other than ibuproen ��� mg approximately twice monthly or headaches. Physical examination �ndings and laboratory test results are shown in able ��.Q��. Results o erythrocyte sedimentation rate, antinuclear antibody testing, testing or antibodies to myeloperoxidase and proteinase �, serum protein electrophoresis, and hepatitis B and C and human immunode�ciency serologies all are negative or normal. Renal biopsy is perormed.
Urinalysis Protein Microscopic examination Sodium, mEq/L Creatinine, mg/dL
�+ Occasional granular casts �� ��
Table ��.Q�� COMPONEN
What is the most likely cause o this patient’s acute renal ailure? a. Membranous nephropathy b. Acute tubular necrosis c. Dehydration d. Obstruction e. umor lysis syndrome
FINDING
Blood pressure, mm Hg
���/��
Pulse, beats per minute
��
Weight, kg
��
Height, cm
��� (continued )
��. NEPHROLOGY QUESIONS AND ANSWERS
•
��
Table ��.Q�� �CONINUED�
Table ��.Q�� �CONINUED�
COMPONEN
COMPONEN
FINDING
FINDING
Heart, lungs, and abdomen
Normal
Chloride, mEq/L
���
Jugular venous distention
Absent
Serum creatinine, mg/dL
�.�
Serum urea nitrogen, mg/dL
��
Creatine kinase, U/L
��,���
Pitting edema
Both lower extremities (trace)
Rashes
Absent
Serum creatinine, mg/dL
�.�
Fasting blood glucose, mg/dL
���
Spot urine microalbumin, mg/g Urinalysis Blood Protein ��-h total protein, g
Urinalysis Color Blood Protein Leukocytes Microscopic examination
�,��� �+ �+ �.�
What is the most likely cause o her renal ailure?
What is the most likely diagnosis? a. b. c. d. e.
a. b. c. d. e.
Minimal change nephropathy Membranous nephropathy Chronic interstitial nephritis Diabetic nephropathy IgA nephropathy
��. A ��-year-old woman presents to the emergency department with diffuse myalgias. Her past medical history is signi�cant or arthroscopic knee surgery � years ago. She has no history o recent trauma. She admits to using cocaine regularly. She takes an oral contraceptive tablet daily and ibuproen ��� mg � to � times daily as needed or pain, most recently this morning. On auscultation, the heart rhythm is regular with no murmur, rub, or gallop, and the lungs are clear. Findings on abdominal examination are normal. Tere is diffuse tenderness in both upper and lower extremities without any ecchymoses or rashes. Ankle edema (trace) is present bilaterally. Additional physical examination �ndings and laboratory test results are shown in able ��.Q��. Table ��.Q�� COMPONEN
FINDING
Blood pressure, mm Hg
���/��
Pulse, beats per minute
���
emperature, °C
��.�
Hemoglobin, g/dL
��.�
Leukocyte count, ���/L
�.�
Platelet count, ���/L
���
Sodium, mEq/L
���
Potassium, mEq/L
�.�
Bicarbonate, mEq/L
�� (continued )
��
•
Brown �+ �+ �–� Granular casts, epithelial casts, renal epithelial cells (��–��)
Rhabdomyolysis Acute interstitial nephritis Trombotic thrombocytopenic purpura (P) Membranoprolierative glomerulonephritis (MPGN) Anti–glomerular basement membrane nephritis
��. Oliguric acute renal ailure developed in a ��-year-old man afer lef emoral-popliteal bypass surgery. He had a computed tomographic angiogram o the lower extremity arteries � days ago and underwent lef emoral-popliteal bypass surgery or critical ischemia o the lef oot � days ago. During the procedure, his systolic blood pressure decreased to �� to �� mm Hg several times. Over the past �� hours, his urine output through the indwelling bladder catheter was ��� mL despite intravenous �uid administration; he has a positive �uid balance o � L postoperatively. His current medications include insulin, metoprolol, aspirin, sim vastatin, and entanyl patient-controlled analgesia. In addition to peripheral vascular disease, his past medical history is signi�cant or type � diabetes mellitus or � years, hypertension, hyperlipidemia, and coronary artery disease. Baseline creatinine was �.� mg/dL last month, and he had a urine microalbumin to creatinine ratio o �� mg/g (reerence range <�� mg/g) at that time. His blood pressure is ���/�� mm Hg, his pulse is �� beats per minute, and his temperature is ��.� °C. On physical examination, he has a regular cardiac rate and rhythm and a grade �/� systolic ejection murmur at the right upper sternal border without a rub or gallop, there are diffuse pulmonary crackles bilaterally, and the abdomen is sof and nontender. Both lower extremities have pitting edema (�+). Pedal pulses are diminished bilaterally. His creatinine was �.� mg/dL on postoperative day � and �.� mg/dL on postoperative day �. Urine Gram staining is negative, and the results o the urinalysis with microscopy are pending. Which o the ollowing sets o �ndings on urinalysis and urine microscopy would be most characteristic o this acute presentation?
MAYO CLINIC INERNAL MEDICIN E BOARD REVIEW: QUESIONS AND ANSWER S
a. Normal urinalysis; occasional hyaline casts b. Protein (�+); occasional ree at c. Protein (�+); blood (�+); �� to �� red blood cells; occasional red blood cell casts d. Protein (trace); blood (trace); positive nitrite and positive leukocyte esterase tests; � to � red blood cells; �� to ��� leukocytes; occasional leukocyte casts e. Protein (�+); less than � red blood cells; � to �� renal epithelial cells
is �� beats per minute, his respiratory rate is �� breaths per minute, his temperature is ��.�°C, and he weighs �� kg (body mass index ��.�). He appears atigued and is in no acute distress. His skin is pale with scattered reddish lesions on the lower legs. Te heart rate is regular with a grade �/� systolic ejection murmur. Scattered pulmonary crackles are present. Tere is no hepatosplenomegaly, and the abdomen is nontender with normal bowel sounds. Bilateral pretibial edema (�+) is present. Laboratory test results are shown in able ��.Q��. Chest radiography shows multiple small bilateral pulmonary nodules that appear slightly smaller than � months ago.
��. A ��-year-old man is ound to have a serum creatinine level o �.� mg/dL while undergoing an evaluation or bilateral lower extremity edema that had been present or approximately � weeks. He has a history o hypertension, hyperlipidemia, mitral regurgitation, and Barrett esophagus. Neither the patient nor any amily members Table ��.Q�� are known to have kidney disease. Other than the edema, COMPONEN FINDING he has had no symptoms and denies having hematuria, decreased urine output, increased urinary requency, Hemoglobin, g/dL �.� hesitancy, or urgency. His medications are metoprolol, � �.� simvastatin, omeprazole, and aspirin. His blood pressure Leukocyte count, ×�� /L is ���/�� mm Hg, his pulse is �� beats per minute, his Platelet count, ×���/L �� temperature is ��.�°C, his respiratory rate is �� breaths ��� per minute, and his weight is ��.� kg. He does not appear Sodium, mEq/L ill. His skin turgor is normal. His heart rate and rhythm Potassium, mEq/L �.� are regular. A �/� holosystolic murmur is present at ��� the apex, and there is no rub or gallop. His lungs are Chloride, mEq/L clear, and abdominal examination �ndings are normal. Bicarbonate, mEq/L �� Tere is pitting pretibial edema (�+ to �+) bilaterally. �.� Laboratory test results are as ollows: serum creatinine Serum creatinine, mg/dL �.� mg/dL, serum urea nitrogen �� mg/dL, hemoglo- Serum urea nitrogen, mg/dL �� � bin ��.� g/dL, leukocyte count �.� ×�� /L, and platelet �.� count ���×���/L. Urinalysis shows protein (�+), posi- Calcium, mg/dL tive results or leukocyte esterase, �� to �� leukocytes Phosphorus, mg/dL �.� per high-power �eld, and negative Gram staining. Renal �.� ultrasonography shows increased echogenicity bilater- Albumin, g/dL ally and no evidence o hydronephrosis. Venous Doppler Lactate dehydrogenase, U/L ��� ultrasonography o the lower extremities is negative or deep vein thrombosis. Which o the ollowing intervenWhich o the ollowing is the most likely cause o this tions is most likely to improve this patient’s acute renal patient’s acute renal ailure? ailure by addressing the underlying cause? a. b. c. d. e.
Initiate subcutaneous low-molecular-weight heparin therapy. Discontinue the use o omeprazole. Insert an indwelling urinary catheter. Decrease the metoprolol dose. Start cipro�oxacin therapy with ��� mg twice daily.
��. A ��-year-old man presents with a �-week history o weakness and atigue. wo years ago, he underwent right nephrectomy or renal cell carcinoma. Tree months ago, pulmonary metastases were diagnosed, and therapy with sunitinib was initiated. Other past medical history includes hypertension and stage � chronic kidney disease with baseline creatinine �.� mg/dL (estimated glomerular �ltration rate �� mL/min per �.�� m �). His medications are amlodipine �� mg daily, urosemide �� mg daily, and sunitinib �� mg daily or the past � weeks (ollowing � weeks without sunitinib afer the previous cycle). His blood pressure is ���/�� mm Hg, his pulse
a. b. c. d. e.
Trombotic microangiopathy Acute tubular necrosis Atheroemboli Microscopic polyangiitis Goodpasture syndrome
CHRONIC RENAL FAILURE
��. A ��-year-old woman who is doing well is seeing you or her annual checkup. Her medical history is signi�cant or type � diabetes mellitus or �� years, hypertension, and diabetic neuropathy. On physical examination, she is obese and has decreased sensation in her eet. Other physical examination �ndings and laboratory test results are shown in able ��.Q��. Her current medications are metormin �,��� mg twice daily, simvastatin �� mg daily, gabapentin ��� mg � times daily, and atenolol �� mg twice daily.
��. NEPHROLOGY QUESIONS AND ANSWERS
•
��
Table ��.Q��
What should you recommend?
Heart rate, beats per minute
a. b. c. d. e.
COMPONEN
Blood pressure, mm Hg
FINDING
�� ���/��
Height, cm
���
Weight, kg
��
Hemoglobin A�c, %
�.�
Serum creatinine, mg/dL
�.�
otal cholesterol, mg/dL
���
riglycerides, mg/dL
���
High-density lipoprotein cholesterol, mg/dL
��
Low-density lipoprotein cholesterol, mg/dL
��
What additional tests should you order or this patient? a. b. c. d. e.
Renal ultrasonography A ��-hour urine protein collection A ��-hour urine albumin collection Random urine albumin to creatinine ratio Creatinine clearance
��. A ��-year-old woman presents with atigue, decreased stamina, and loss o appetite. She has a ��-year history o type � diabetes mellitus; she has coronary artery disease and has had coronary artery bypass graf surgery; she received a diagnosis o breast cancer �� years ago and had a mastectomy and radiotherapy; she has hypertension, hyperlipidemia, and a history o gout. Her heart rate is �� beats per minute, her blood pressure is ���/�� mm Hg, her height is ��� cm, and her weight is �� kg. On physical examination, she is pale, she is in no acute distress, and she has lower extremity edema; otherwise, examination �ndings are normal. Laboratory test results are shown in able ��.Q��. Results o liver unction studies are normal. Te patient takes atenolol �� mg daily, lisinopril �� mg daily, NPH insulin �� units twice daily, simvastatin �� mg daily, and allopurinol ��� mg daily. Table ��.Q�� COMPONEN
FINDING
Hemoglobin, g/dL
�.�
Serum creatinine, mg/dL
�.�
Sodium, mEq/L
���
Bicarbonate, mEq/L
��
Chloride, mEq/L
���
Potassium, mEq/L
�.�
Uric acid, mg/dL
��
Hemoglobin A�c, %
�.�
Urine albumin to creatinine ratio, mg/g ���
•
Exchange atenolol with metoprolol. Increase the dosage o lisinopril to �� mg daily. Add losartan ��� mg daily. Increase the dosage o allopurinol to ��� mg daily. Add metormin �,��� mg twice daily.
��. A ��-year-old man has lower extremity swelling, decreased stamina, and atigue. He is obese and has a ��-year history o type � diabetes mellitus. His other medical problems include hyperlipidemia and recurrent gout attacks. He weighs ��� kg, he is ��� cm tall, his blood pressure is ���/�� mm Hg, and his heart rate is �� beats per minute. Laboratory test values are as ollows: serum creatinine �.� mg/dL, hemoglobin A�c �.�%, and urine albumin to creatinine ratio �,��� mg/g. He takes losartan �� mg daily, metormin �,��� mg twice daily, allopurinol ��� mg daily, simvastatin �� mg daily, and aspirin �� mg daily. What therapeutic intervention should you recommend that is effective in treating patients with diabetic kidney disease and can prevent its progression? a. Increase the losartan dosage to �� mg daily and monitor serum potassium and creatinine. b. Add hydrochlorothiazide ��.� mg daily. c. Stop the use o metormin and start insulin therapy. d. Initiate dietary modi�cation with protein and salt restriction. e. All o the above would be effective.
��. A ��-year-old man presents with acute chest pain. He has stage � chronic kidney disease (CKD) and a ��-year history o type � diabetes mellitus. He is a smoker and has hyperlipidemia and degenerative joint disease. His blood pressure is ���/�� mm Hg, and his heart rate is ��� beats per minute. His troponin level is elevated. His electrocardiogram shows S-segment elevation. Te patient is being transported to undergo coronary angiography. What should you recommend be done next? a. No treatment is indicated; proceed with coronary angiography. b. Start therapy with N -acetylcysteine �,��� mg twice daily with � L o oral hydration. c. Start therapy with N -acetylcysteine �,��� mg twice daily with � L o �.��% saline. d. Start therapy with N -acetylcysteine �,��� mg twice daily with � L o �.�% saline. e. Start therapy with N -acetylcysteine �,��� mg twice daily with � L o sodium bicarbonate.
��. A ��-year-old woman presents with anasarca, decreased stamina, and atigue. She has had type � diabetes mellitus or � years, hypertension, and hyperlipidemia. She quit smoking � years ago. She has a history o migraine headaches. She has pale skin and pitting edema (�+) o the thigh. On auscultation, breath sounds are mildly decreased in the bases bilaterally. Other physical examination �ndings and laboratory test results are shown
�,���
M A YO C L I N I C I N E R N A L M E D I C I N E B O A R D R E V I EW : Q U E S I ON S A N D A N S W E R S
in able ��.Q��. She is taking NPH insulin, simvastatin �� mg daily, aspirin �� mg daily, metoprolol ��� mg twice daily, and ibuproen ��� mg twice daily as needed. A renal biopsy was perormed. Table ��.Q�� COMPONEN
FINDING
Height, cm
���
Weight, kg
��
Blood pressure, mm Hg
��/��
Heart rate, beats per minute
���
Serum creatinine, mg/dL
�.�
Serum albumin, g/dL
�.�
otal cholesterol, mg/dL
���
Low-density lipoprotein cholesterol, mg/dL
���
High-density lipoprotein cholesterol, mg/dL
��
riglycerides, mg/dL
���
Hemoglobin, g/dL
�.�
Hemoglobin A�c, %
�.�
What renal biopsy �nding would be most likely? a. b. c. d. e.
Fusion o podocyte oot processes on electron microscopy Mesangial matrix expansion and cell prolieration Tickening o the glomerular basement membrane Mesangial nodular sclerosis All o the above
��. A ��-year-old man presents or an annual checkup with no medical concerns. He has had type � diabetes mellitus or � years, and he has a history o hypertension, degenerative joint disease with use o nonsteroidal anti-inlammatory drugs, and gout. His height is ��� cm, he weighs ��� kg, his blood pressure is ���/�� mm Hg, and his heart rate is �� beats
per minute. He is mildly obese; examination indings are otherwise unremarkable. Laboratory test results include the ollowing: serum creatinine �.� mg/dL, urine albumin to creatinine ratio �� mg/g, low-density lipoprotein cholesterol �� mg/dL, and hemoglobin A�c �.�%. He takes metormin �,��� mg twice daily, allopurinol ��� mg daily, losartan ��� mg daily, aspirin �� mg daily, simvastatin �� mg daily, a multivitamin daily, vitamin E daily, ish oil capsules �,��� mg twice daily, and sildenail �� mg as needed. Which o the ollowing is true about the patient’s current condition? a. His blood pressure and diabetes are controlled well, so his risk o death rom cardiovascular disease is not increased. b. His blood pressure and diabetes are controlled well, so he will not progress to end-stage renal disease (ESRD). c. He should begin renin-inhibitor therapy to urther reduce his risk o death rom cardiovascular disease and his risk o progression o chronic kidney disease (CKD). d. He should initiate or intensiy liestyle modi�cations, including diet, exercise, and weight loss. e. All o the above are true.
��. A ��-year-old man presents with atigue, bone pain, and arthralgias. His history is signi�cant or chronic kidney disease (CKD), lef nephrectomy or renal cell carcinoma, hypertension, and degenerative joint disease. His height is ��� cm, he weighs �� kg, his blood pressure is ���/�� mm Hg, and his heart rate is �� beats per minute. He has diffuse arthralgias and mild pitting edema o the lower extremities. Laboratory test results include the ollowing: serum creatinine �.� mg/dL, serum calcium �.� mg/dL, phosphorus �.� mg/dL, parathyroid hormone (PH) ��� pg/mL, and hemoglobin ��.� g/dL. His only medication is the extended-release ormulation o diltiazem ��� mg. What is your next recommendation? a. b. c. d. e.
Reer the patient or parathyroidectomy. Schedule a parathyroid sestamibi scan. Schedule an ultrasonographic study o the neck. Start vitamin D therapy. Determine the �,��-dihydroxyvitamin D level.
��. NEPHROLOGY QUESIONS AND ANSWERS
•
���
ANSWERS
�. Answer a.
Tis patient presented with a history suspicious or lung cancer, which is known to be associated with syndrome o inappropriate secretion o antidiuretic hormone. Serum osmolality is necessary to con�rm the presence o hypo-osmolar hyponatremia. Urine osmolality is necessary to con�rm the presence o arginine vasopressin (AVP); concentrated urine (absence o maximally diluted urine) would indicate the presence o AVP. Hypothyroidism and adrenal insuffi ciency could cause hyponatremia and should be ruled out. Te presence o AVP in a patient with normal intravascular volume, low serum osmolality, and lack o other known AVP stimulating actors (pain, nausea, and medications) would indicate ectopic AVP production. Given the patient’s history, high-resolution C o the chest is indicated to determine the presence o lung cancer, which could be the source o AVP.
aldosterone secretion, which creates an abnormal coupling o high distal sodium delivery and high aldosterone level, leading to urinary potassium loss. ranscellular shif would lead to a shif o potassium rom the intracellular compartment to extracellular compartments; this shif would not contribute to the patient’s hypokalemia. �. Answer c.
Te best choice is potassium given intravenously at an appropriate rate. Insulin given intravenously or subcutaneously promotes intracellular potassium shifing and would be expected to urther exacerbate hypokalemia. Te use o insulin should be avoided in patients who have hypokalemia. Administration o calcium intravenously would not solve the problem o hypokalemia. �. Answer b.
Tis is a classic scenario or distal RA, which is associated with calcium phosphate nephrolithiasis and nephrocalcinosis. Without treatment, the hypokalemia and non– anion gap metabolic acidosis can become severe. Patients with RA tend to have slightly low intravascular volume. Tereore, hyporeninemic hypoaldosteronism would not be a possibility. Proximal RA would not be associated with a severe reduction in serum bicarbonate or with nephrolithiasis.
�. Answer d.
HCZ blocks absorption o sodium and chloride in the distal convoluted tubule; the result is a mild degree o intravascular volume depletion, which stimulates secretion o arginine vasopressin (AVP). Beer is hypotonic and contains a large amount o ree water. When passing through the distal convoluted tubule, ree water is absorbed under the in�uence o AVP; the result is hyponatremia. Tereore, HCZ, decreased intravascular volume (which stimulates AVP), and beer drinking all contributed to the hyponatremia.
�. Answer a.
Tis patient likely has plasma cell dyscrasia (multiple myeloma or AL amyloidosis, or both). Te unusually low anion gap indicates the presence o positively charged paraproteins. Te presence o glucosuria with euglycemia indicates a proximal tubular dysunction. ypically, renin and aldosterone levels are high because the blood pressure is low.
�. Answer c.
Water de�cit is calculated as �.� × body weight (kg ) × [(sodium concentration/���) − �]. For this patient, the water de�cit would be �� × [(���/���) − �] = �.� L. �. Answer b.
�. Answer a.
Te correction o hypernatremia should be limited to no more than �.� mEq/L per hour or �� mEq/L daily. Te daily rate is a limit, not a target.
Te history indicates some type o intoxication. Te patient has a large anion gap, and the most appropriate next step would be to calculate the osmolal gap to rule out alcohol (methanol and ethylene glycol) intoxication. Urine microscopy would be helpul. Te presence o calcium oxalate crystals would indicate ethylene glycol intoxication. Te severe acidosis and large osmolal gap would require the initiation o hemodialysis. I ethylene glycol or methanol intoxication is suspected, �-methylpyrazole therapy should be initiated.
�. Answer c.
Te combination o a loop diuretic (urosemide) and a distal tubular diuretic (metolazone) is a powerul kaliuretic regimen. In patients who have reasonably good renal clearance, the combination inevitably results in hypokalemia. Volume depletion rom the diuretic effect increases ���
��. Answer a.
Te anion gap acidosis and massively elevated osmolal gap suggest small-molecule (ie, methanol) intoxication. Te severe acidosis and large osmolal gap require combination treatment with �-methylpyrazole (omepizole) and hemodialysis. ��. Answer c.
Tis patient has anion gap acidosis and respiratory alkalosis, which is typical or salicylate intoxication. ��. Answer c.
A change in P��� o �� mm Hg with a change in bicarbonate o � mEq/L would be consistent with a mixture o chronic and acute respiratory acidosis. ��. Answer b.
Tis patient’s arterial blood gas and serum electrolyte values are consistent with chronic respiratory alkalosis. In an ��-year-old woman, chronic respiratory alkalosis is consistent with pregnancy. Progesterone stimulates the respiratory center, causing chronic respiratory alkalosis. ��. Answer b.
Granular casts on urine microscopy and a ractional excretion o sodium that is greater than �% (�.�% in this case) are consistent with a diagnosis o acute tubular necrosis. Proteinuria would be higher in membranous nephropathy, which is associated with malignancies. umor lysis syndrome is associated with hyperphosphatemia and, typically, hypocalcemia. Absence o hydronephrosis on ultrasonography is not consistent with obstruction. ��. Answer e.
Cryoglobulinemia can develop in patients with asymptomatic hepatitis C inection, and this can cause a vasculitis involving small vessels (skin) and membranoprolierative glomerulonephritis.
would maniest with pyuria; patients with P have anemia and thrombocytopenia; and MPGN and anti–glomerular basement membrane nephritis would maniest with hematuria with or without red blood cell casts. ��. Answer e.
Te clinical maniestation o oliguric acute renal ailure afer hypotension during surgery (along with exposure to intravenous contrast material � days preoperatively) is typical o acute tubular necrosis. Urinalysis would show low-grade proteinuria, and the renal epithelial cells seen on microscopy would indicate renal tubular injury. ��. Answer b.
Sterile pyuria with acute renal ailure suggests acute interstitial nephritis. Discontinuing the use o omeprazole would be the most appropriate treatment since proton pump inhibitors are associated with this condition. Inserting a urinary catheter in the absence o hydronephrosis or a history o voiding dysunction would not be helpul. Low-molecular-weight heparin should be avoided in patients with this degree o renal unctional impairment and is not indicated in the absence o deep vein thrombosis. Decreasing the metoprolol dose in the absence o signi�cant bradycardia would not be bene�cial. (See Myers et al in the “Suggested Reading” list.) ��. Answer a.
Renal thrombotic microangiopathy is an important side effect o vascular endothelial growth actor inhibitors such as sunitinib and would maniest with hemolytic anemia, thrombocytopenia, and renal ailure. Microscopic polyangiitis can also maniest with anemia and acute renal ailure, but severe thrombocytopenia is not a eature. Hemolytic anemia and severe thrombocytopenia are not eatures o Goodpasture syndrome, atheroembolic renal disease, or acute tubular necrosis. (See Eremina et al in the “Suggested Reading” list.) ��. Answer d.
��. Answer e.
A common maniestation o IgA nephropathy is persistent asymptomatic microhematuria with various degrees o proteinuria. Membranous nephropathy and minimal change nephropathy typically maniest with nephrotic range proteinuria without signi�cant hematuria. Patients who have chronic interstitial nephritis and diabetic nephropathy do not present with persistent microhematuria. ��. Answer a.
Rhabdomyolysis precipitated by cocaine use can cause acute renal ailure rom acute tubular necrosis that results rom injury to renal tubular epithelial cells afer myoglobin is released by myocytes. Te urine sediment �ndings are characteristic o acute tubular necrosis. Urinalysis typically is positive or blood by dipstick since the assay detects myoglobin in addition to hemoglobin, but red blood cells are not seen on urine microscopy. Acute interstitial nephritis
In the United States, only ��% to ��% o patients with type � diabetes mellitus are evaluated or diabetic kidney disease (DKD) with testing or proteinuria. Patients with DKD and proteinuria have a higher risk o end-stage renal disease and have a high associated cardiovascular mortality. Initiation o timely screening and appropriate therapy can decrease the rate o progression o DKD. (See Te National Kidney Foundation Kidney Disease Outcomes �uality Initiative [NKF KDOQI] in the “Suggested Reading” list.) ��. Answer a.
Exchange atenolol with metoprolol because atenolol is cleared by the kidneys and may accumulate to adverse plasma levels in patients with stage � or � chronic kidney disease (CKD). Te patient has a Modi�cation o Diet in Renal Disease (MDRD) glomerular �ltration rate
��. NEPHROLOGY QUESIONS AND ANSWERS
•
���
(GFR) o �� mL/min per �.�� m �. Metoprolol is metabolized by the liver and thereore has ewer side effects. Patients with progressive CKD (stages � and �) have decreased elimination o medications that are excreted by the kidneys. Tese medications can accumulate and lead to adverse effects, such as bradycardia rom renally excreted β-blockers (eg, atenolol). Switching to medications metabolized in the liver is more advantageous in these patients. Furthermore, although inhibitors o the renin-angiotensin system decrease progression o CKD and diabetic kidney disease, their bene�t is limited in patients with progressive CKD. Te adverse effects o these agents rom decreasing the GFR and causing hyperkalemia might be harmul. (See Micromedix �.�, and Te National Kidney Foundation Kidney Disease Outcomes �uality Initiative [NKF KDOQI] in the “Suggested Reading” list.) ��. Answer e.
Te ollowing therapies are effective or treating chronic kidney disease or diabetic kidney disease: titration o angiotensin-converting enzyme inhibitor or angiotensin receptor blocker therapy, blood pressure control, glycemic control, and weight loss. (See Saiki et al in the “Suggested Reading” list.) ��. Answer d.
Patients with CKD have an increased risk o contrast-induced acute kidney injury (AKI) and high cardiovascular mortality. Recurrent episodes o AKI can lead to progression o CKD. Appropriate therapies or contrast-induced AKI prevention should be initiated. (See Calvin et al in the “Suggested Reading” list.) ��. Answer e.
Diabetic nephropathy is characterized by glomerulopathy involving all segments, including the mesangium, glomerular basement membrane, and podocytes. Some o the phenotypes o diabetic nephropathy are characterized by rapid progression. Even so, patients with diabetes can have non-diabetic kidney disease (IgA, membranous nephropathy, etc). Nearly ��% o renal biopsies in this patient population show diabetic nephropathy. (See P�ueger et al in the “Suggested Reading” list.) ��. Answer d.
Patients with CKD have a high cardiovascular mortality and risk or progression to ESRD. Patient management should ocus on cardiovascular risk reduction therapy,
���
•
including intensi�cation o liestyle modi�cations or diet, exercise, and weight loss. (See Keith et al in the “Suggested Reading” list.) ��. Answer d.
De�ciency o �,��-dihydroxyvitamin D is common among CKD patients, and initiation o vitamin D therapy to treat secondary hyperparathyroidism in CKD patients is indicated along with achieving the ollowing target PH levels: •
PH ��–�� pg/mL or patients with an estimated glomerular �ltration rate (GFR) o ��–�� mL/min per �.�� m� (stage � CKD)
•
PH ��–��� pg/mL or patients with an estimated GFR o ��–�� mL/min per �.�� m � (stage � CKD)
•
PH ���–��� pg/mL or patients who receive dialysis or who have an estimated GFR <�� mL/min per �.�� m� (stage � CKD)
(See Te National Kidney Foundation Kidney Disease Outcomes �uality Initiative [NKF KDOQI] in the “Suggested Reading” list.) SUGGESED RE ADING Calvin AD, Misra S, P�ueger A. Contrast-induced acute kidney injury and diabetic nephropathy. Nat Rev Nephrol. 2010 Nov;6(11):679–88. Epub 2010 Sep 28. Eremina V, Jefferson JA, Kowalewska J, Hochster H, Haas M, Weisstuch J, et al. VEGF inhibition and renal thrombotic microangiopathy. N Engl J Med. 2008 Mar 13;358(11):1129–36. Keith DS, Nichols GA, Gullion CM, Brown JB, Smith DH. Longitudinal ollow-up and outcomes among a population with chronic kidney disease in a large managed care organization. Arch Intern Med. 2004 Mar 22;164(6):659–63. Micromedex 2.0 [Internet]. ruven Health Analytics. c2012. Available rom: http://www.micromedex.com/. Myers RP, McLaughlin K, Hollomby DJ. Acute interstitial nephritis due to omeprazole. Am J Gastroenterol. 2001 Dec;96(12):3428–31. Te National Kidney Foundation Kidney Disease Outcomes �uality Initiative (NKF KDOQI) [Internet]. National Kidney Foundation. New York (NY). c2012. Available rom: http://www.kidney.org/proessionals/kdoqi/index.cm. P�ueger A, Abramowitz D, Calvin AD. Role o oxidative stress in contrast-induced acute kidney injury in diabetes mellitus. Med Sci Monit. 2009 Jun;15(6):RA125–36. Saiki A, Nagayama D, Ohhira M, Endoh K, Ohtsuka M, Koide N, et al. Effect o weight loss using ormula diet on renal unction in obese patients with diabetic nephropathy. Int J Obes (Lond). 2005 Sep;29(9):1115–20.
M A YO C L I N I C I N E R N A L M E D I C I N E B O A R D R E V I EW : Q U E S I ON S A N D A N S W E R S
��. ALLERGY QUESTIONS AND ANSWER S
QUESIONS
Within minutes, the area becomes red, painul, pruritic, and swollen. During the next several hours, the redness and swelling spread to the elbow. He denies having other symptoms; speci�cally, he denies having dyspnea, light-headedness, nausea, vomiting, or diarrhea. Tere are no skin maniestations elsewhere. What is the most appropriate management?
Multiple Choice (choose the best answer)
�. A ��-year-old woman presents with year-round symptoms o nasal congestion, rhinorrhea, and sneezing. Although the symptoms are present year-round, they worsen in the spring and all. Her symptoms are bilateral and have responded partially to treatment with over-the-counter antihistamines. In an effort to control her symptoms, which medication should not be used as part o her treatment plan? a. b. c. d. e.
Nonsedating antihistamine Leukotriene antagonist opical decongestant Intranasal corticosteroid Saline nasal spray
�. A ��-year-old woman is treated or Neisseria gonorrhoeae inection. She has had � episodes o Neisseria meningitidis inection. As a child, she had � episode o pneumonia. She has no history o skin inections or abscesses. What type o deect is the most likely underlying immunologic abnormality? a. b. c. d. e.
C� C�–� IgG Neutrophil chemotaxis Lymphocyte count
�. A ��-year-old man presents with a ��-year history o recurrent angioedema, particularly o the lips and eyes. Troat swelling occurred on � occasions. Te episodes seem to occur randomly, approximately monthly, and there is no association with ood, medications (including over-the-counter products), or environmental exposures. His ather reports having similar symptoms or years. Te patient is otherwise healthy. Current physical examination �ndings are normal, although he brings in photographs that show marked angioedema o his orbits and lips. What type o abnormality does this patient have? a. b. c. d. e.
a. Apply ice to the arm, provide symptomatic relie, and review strategies to avoid stinging insects. b. Perorm skin testing to bee only; i results are positive, administer immunotherapy. c. Perorm skin testing to apids (honeybee) and vespids (yellow jacket, wasp, and hornet); i any o the results are positive, administer immunotherapy. d. Perorm an in vitro test or IgE speci�c or apids (honeybee) and vespids (yellow jacket, wasp, and hornet); i results are positive, administer immunotherapy. e. Administer �.� mL epinephrine �:�,��� intramuscularly and observe.
C� C�q ryptase Lymphocyte count ��-hour urine N -methylhistamine
�. A ��-year-old man is stung on the distal right orearm by what he describes as a “bee.” No stinger is visible.
�. A ��-year-old woman presents in her �fh month o pregnancy with increasing asthma symptoms. Her asthma had been well controlled throughout pregnancy but worsened over the past � days with the onset o an upper respiratory tract inection. She has daily symptoms and nighttime awakenings due to dyspnea. She uses her albuterol inhaler every � hours and has adhered to her usual inhaler regimen o budesonide � puffs daily. She denies having ever or purulent mucus. On examination, she has scattered expiratory wheezes throughout all lung �elds, a respiratory rate o �� breaths per minute, oxygen saturation o ��%, and orced expiratory volume in � second (FEV�) o ��% o the predicted value, which improves to ��% afer albuterol. How should you change her therapy? a. b. c. d. e.
Add montelukast �� mg daily. Increase the dosage o budesonide to � puffs twice daily. Start the use o amoxicillin ��� mg � times daily. Discontinue the use o the budesonide inhaler. Add prednisone �� mg daily or � days.
�. A ��-year-old woman reports that she had pruritic hives and wheezing afer her second dose o intravenous penicillin approximately �� years ago. She has avoided penicillin since then and has not had any urther drug reactions. Currently, she is being treated or endocarditis, and high-dose penicillin is considered the drug o choice. What recommendation can be given or penicillin use?
���
a. Penicillin cannot be given; use an alternate medication. b. Skin test to the major determinant o penicillin; i results are negative, penicillin can be given. c. Skin test to the major and minor determinants o penicillin; i results are negative, penicillin can be given. d. Patch test to penicillin; i results are negative, penicillin can be given. e. Avoid penicillin and use a β-lactam antibiotic.
�.
A ��-year-old man presents with a �-year history o recurrent sinopulmonary inections documented by computed tomographic scan o the sinuses and chest radiographs. He does not recall having
���
•
recurrent inections, allergic rhinitis, or asthma beore the past � years. Sputum cultures have shown growth o Streptococcus pneumoniae and Haemophilus in�uenzae. Currently, he has a chronic productive cough. With which o the ollowing laboratory tests is this patient most likely to have abnormal results? a. b. c. d. e.
otal leukocyte count IgG level Neutrophil chemotaxis assay otal complement level IgE level
MAYO CLINIC INERNAL MEDICIN E BOARD REVIEW: QUESIONS AND ANSWER S
ANSWERS
�. Answer c.
Te patient has perennial and seasonal allergic rhinocon junctivitis. Various medications can help control the symptoms and are ofen used in combination. Tese include nonsedating antihistamines, intranasal corticosteroids, and leukotriene antagonists. Saline nasal spray has also been shown to improve symptoms. opical decongestants, such as oxymetazoline, should be avoided. Use o this medication or more than � consecutive days can result in rhinitis medicamentosa.
the absence o other symptoms, does not require the use o epinephrine. �. Answer e.
Tis pregnant patient is having a signi�cant �are o her asthma, and aggressive treatment is warranted by the decreased FEV �, increased use o albuterol, and wheezing. In this situation, systemic corticosteroids are required. Te main risk to the mother and child is hypoxia. Systemic corticosteroids, inhaled corticosteroids, long- and short-acting inhaled β-agonists, and leukotriene receptor blockers are acceptable or use in pregnant patients who have asthma. For ongoing management, adding montelukast or increasing the budesonide dosage may be helpul, but these options are not the most effective in treating an acute exacerbation.
�. Answer b.
Te patient has had recurrent Neisseria inections, which are most commonly associated with deects in the terminal complement components (C�–�). Deects in the other components o the immune system are associated with other types o inections. Hypogammaglobulinemia is associated primarily with upper and lower respiratory inections caused by encapsulated bacteria. Neutrophil deects are associated with skin and pulmonary abscesses.
�. Answer c.
Te clinical history is consistent with an IgE-mediated reaction to penicillin. Over time, the majority o patients lose their sensitivity to penicillin. Penicillin allergy can be assessed by skin testing to the major and minor determinants o penicillin. Tese are allergenic metabolites o penicillin. I the test results are negative, the patient can receive penicillin with the same risk o a signi�cant reaction as or persons in the general population who have never had a reaction. Skin testing to only the major determinant will miss approximately ��% o patients who are penicillin allergic. I the results o skin testing are positive, the patient is at high risk or an IgE-mediated reaction and penicillin should be given only under a desensitization protocol.
�. Answer a.
Te patient has hereditary angioedema resulting rom C� esterase inhibitor de�ciency. Te usual screening test or this is C� testing; C� levels are decreased in these patients. Te C�q level is decreased in the acquired orm o this disease but not in the hereditary orm. Te acquired orm occurs in older patients who do not have a amily history o the disease, and in the acquired orm there is ofen an asso ciated underlying malignancy, particularly a hematologic malignancy. ryptase and ��-hour N -methylhistamine are used to evaluate or mast cell disease, which does not maniest solely with intermittent angioedema. Te lymphocyte count is normal in hereditary and acquired angioedema and in the majority o patients with mast cell disease.
�. Answer b.
Recurrent sinopulmonary inections with common respiratory microorganisms are the primary maniestations o common variable immunode�ciency disease (CVID). Te main laboratory abnormality in CVID is a depressed level o IgG. Hypogammaglobulinemia predisposes to recurrent sinus and pulmonary inections. Other maniestations include autoimmune processes and inectious diarrhea. Te neutrophil chemotaxis assay is decreased in chronic granulomatous disease, which is characterized by recurrent skin and pulmonary abscesses. De�ciencies o the late-acting complement components (C�–�) typically maniest as recurrent inections with meningococci and gonococci.
�. Answer a.
Te patient has a large local reaction to a stinging insect. A large local reaction to a stinging insect is not considered a risk actor or a more serious reaction with a subsequent sting. Tereore, only symptomatic treatment is required or treatment. Skin testing to the apids and vespids would be required only i immunotherapy were being contem plated. Tis should be considered or all adults who have systemic reactions to the sting. A large local reaction, in
���
This page intentionally left blank
��. PSYCHIATRY QUESTIONS AND ANSWERS
QUESIONS
her hands are now raw rom all the scrubbing. With urther queries, she says that she eels compelled to count ceiling tiles in your examination room and she worries irrationally about her children’s health. She has sleep difficulties, and her anxiety is so high throughout the day that she has difficulty working. Which o the ollowing would not be an appropriate recommendation or this patient?
Multiple Choice (choose the best answer)
�. A ��-year-old man has a history o poorly controlled type � diabetes mellitus, coronary heart disease with Q prolongation (corrected Q interval ��� ms), and major depression. He presents to your office with a �-month history o worsening depressive symptoms, including severely depressed mood, tearulness, decreased concentration, hopelessness, middle insomnia, and passive suicidal ideation without a speci�c intent or plan. He has been taking sertraline ��� mg daily (the dosage was recently increased rom ��� mg daily) without much bene�t. He has been experiencing lie stresses including �nancial strain and his wie’s declining health. Which o the ollowing would not be a reasonable next step in managing his depression?
a. reat with bupropion titrated to ��� mg twice daily. b. reat with clonazepam �.� mg twice daily as a bridge until other treatment is effective. c. reat with �uoxetine �� mg daily. d. aper and eliminate her intake o caffeine. e. Recommend cognitive behavioral psychotherapy (CB).
a. Cross-taper his medication rom sertraline to amitriptyline. b. Cross-taper his medication rom sertraline to bupropion. c. Perorm a preanesthetic medical examination and reer the patient to a psychiatrist or consideration o electroconvulsive therapy (EC). d. Reer the patient to a psychologist or psychotherapy. e. Ask the patient about his access to �rearms and enlist amily members to remove them rom the home.
�. A ��-year-old woman has recently received a diagnosis o bipolar disorder and is now taking lithium carbonate ��� mg orally twice daily. She wants to know some o the long-term risks o taking this medication. Which o the ollowing is false about long-term management o bipolar disorder with lithium?
�. A ��-year-old woman presents to your office again afer �� years o intermittent severe lef lower quadrant abdominal pain. She denies having weight loss, ever, or chills. Te cause o her symptoms is not apparent rom previous workups, which included a complete blood cell count, electrolyte evaluations, urinalysis, computed tomographic scan o the abdomen and pelvis, colonoscopy, and gynecologic examination. She has previously been thoroughly evaluated or episodic dizziness, headaches, �ulike syndromes, back pain, and pain with intercourse. Te results o all these workups were negative. What is the most likely diagnosis? a. b. c. d. e.
a. Nonsteroidal anti-in�ammatory drugs (NSAIDs) should be avoided because o nephrotoxicity associated with taking the combination o an NSAID and lithium. b. Tere is an increased risk o birth deects in children o women taking lithium during pregnancy. c. Tyroid unction needs to be monitored because o the potential or thyrotoxicity. d. Acetaminophen should be avoided because o hepatotoxicity associated with taking the combination o acetamino phen and lithium. e. Lithium doses may need to be decreased temporarily i an acute illness leads to dehydration.
Conversion disorder Somatization disorder Hypochondriasis Body dysmorphic disorder Factitial disorder
�. A ��-year-old man with a history o schizophrenia has a history o mild congestive heart ailure and osteoarthritis. He harbors the paranoid belie that his wie is poisoning him, and he gets special messages rom the television. Tese symptoms have been partially controlled with a second-generation antipsychotic, olanzapine � mg orally daily. You recommend increasing the olanzapine dosage to �� mg daily. Which o the ollowing is false about the use o olanzapine in this patient?
�. A ��-year-old woman presents with anxiety about germs. She washes her hands �� to �� times a day, and
���
a. b. c. d. e.
He is at increased risk o neuroleptic malignant syndrome. He is at increased risk o parkinsonism. He is at increased risk o tardive dyskinesia. He is at increased risk o diabetes mellitus. He is at increased risk o anorexia.
�. A ��-year-old woman is brought to the emergency department by police afer she was discovered alone on a rural bike trail dancing nude. “I am the lizard queen!” she joyully proclaims. You learn that she has been acting quite erratically and sleeping poorly or the past week. Which o the ollowing is the least likely diagnosis? a. Major depression with psychotic eatures b. Bipolar disorder c. Substance dependence d. Schizophrenia e. Methamphetamine intoxication
���
•
�.
You contact the amily o the patient in the previous question and learn that she has a strong amily history o bipolar disorder. Te amily asks you questions about management o this patient. Which o the ollowing would not be appropriate? a. b. c. d. e.
Hospitalize on a psychiatric unit. Begin use o amitriptyline �� mg orally. Begin use o ziprasidone �� mg orally. Perorm a urine drug screen. Begin use o valproate sodium �,��� mg orally.
M A YO C L I N I C I N E R N A L M E D I C I N E B O A R D R E V I EW : Q U E S I O N S A N D A N S W E R S
ANSWERS
�. Answer a.
Somatization disorder entails a history o multiple somatic complaints over many years, including � pain symptoms, � gastrointestinal tract symptoms, � sexual symptom, and � neurologic symptom. Conversion disorder is marked by a neurologic symptom, such as a motor or sensory de�cit, and is unconscious in origin (unlike actitial disorder, in which the symptoms are consciously produced). Body dysmorphic disorder is characterized by a perception that a normally appearing body part is misshapen or otherwise has an abnormal appearance. Hypochondriasis is characterized by an irrational ear that one has an illness or serious disease.
Amitriptyline would be a poor antidepressant choice because the patient has Q prolongation (tricyclic antidepressants can prolong the Q interval) and diabetes (tricyclic antidepressants can cause increased appetite and weight gain). Bupropion would be a reasonable medication alternative. EC could also be considered because o the severity o his symptoms and the ailure o a medication trial. Psychotherapy may help treat the depression and help the patient deal with lie stresses. Most completed suicides are rom �rearms. �. Answer d.
�. Answer e.
Lithium is not hepatotoxic and may be used saely with acetaminophen. Lithium can cause nephrotoxicity, an effect that can be increased when taken concomitantly with NSAIDs. Lithium is a pregnancy category D medication (associated with cardiac and acial structural abnormalities) and should be avoided i possible during pregnancy, especially in the �rst trimester. Lithium can cause thyrotoxicity. Because it has a positive ion with a valence similar to that o sodium, lithium will be retained by the kidneys during periods o dehydration, rapidly reaching toxic levels in some patients. Renal unction, thyroid unction, and electrolytes should be checked periodically (at least annually) or all patients taking lithium.
Second-generation antipsychotics are associated with hyperglycemia and an increased risk o diabetes mellitus. Similar to traditional neuroleptics, second-generation antipsychotics may cause extrapyramidal symptoms, tardive dyskinesia, and neuroleptic malignant syndrome. Olanzapine is not usually associated with anorexia. �. Answer a.
Although any o the disorders mentioned can cause psychosis and odd behavior, major depression is unlikely. Patients who have major depression with psychotic eatures would be unlikely to appear animated and joyul; more ofen, they have ego-dystonic (very unpleasant) delusions, such as a belie that their amily hates them or that people are conspiring against them.
�. Answer a.
Tis patient has obsessive-compulsive disorder, which is responsive to selective serotonin reuptake inhibitors and CB. Because she is very anxious, taking a scheduled low dose o a benzodiazepine and eliminating consumption o caffeine may improve sleep and provide some relie while waiting or other treatments to become effective. Bupropion is not active at the serotonin receptor and will not provide any bene�t.
�. Answer b.
With this patient’s age, behavior, and amily history, bipolar disorder is high in the differential diagnosis. Antidepressants, especially tricyclics, would be contraindicated because they can induce or prolong mania. With the severity o her symptoms and her poor judgment, she poses a danger to hersel, so hospitalization is necessary. Ziprasidone or valproate sodium is a reasonable initial option to manage her symptom o mood elevation. Substance-induced psychosis or mania is high in the dierential diagnosis; thus, a drug screen would be important in the evaluation o this patient.
�. Answer b.
Somatization disorder is characterized by physical symptoms without an identi�able organic cause. Patients believe that they have a physical problem and are not consciously generating the symptoms or pretending to have them.
���
This page intentionally left blank
��. NEUROLOGY QUESTIONS AND ANSWER S
QUESIONS
except or mild symmetrical acial weakness. On motor testing, distal muscle weakness is greater than proximal muscle weakness. Vibratory sensation is decreased distally. Re�exes are absent. Te Babinski test is negative. Coordination is diffi cult to test because o weakness. Which o the ollowing would be expected on urther evaluation?
Multiple Choice (choose the best answer)
�. A ��-year-old woman is evaluated or dizziness. She has a history o well-controlled diabetes mellitus and treated hypertension. On neurologic examination, she has a moderate loss o all sensory modalities distally and to the knees and wrists symmetrically. Her blood pressure is ���/�� mm Hg in the supine position (with a heart rate o �� beats per minute). Upon standing, her blood pressure is ���/�� mm Hg (with a heart rate o �� beats per minute), and she complains o a “dizzy sensation.” Which o the ollowing interventions should be instituted �rst? a. b. c. d. e.
a. Magnetic resonance imaging (MRI) o the head showing multiple areas o increased � signal in the subcortical white matter b. Cerebrospinal �uid (CSF) examination showing an increased protein level and a normal cell count c. Ophthalmoscopic examination showing optic disc pallor bilaterally d. Elevated blood glucose concentration e. Electroencephalography showing lef temporal sharp waves
Administer �udrocortisone. Perorm magnetic resonance angiography. Discontinue use o antihypertensive drugs. Perorm a vestibular evaluation. Perorm magnetic resonance imaging o the cervical spine.
�. An ��-year-old man has an abrupt, painless onset o right upper limb weakness and “garbled” speech. On examination in the emergency department �.� hours afer the onset o symptoms, he has moderate weakness o the right deltoid, triceps, and intrinsic hand muscles and a mild right oot drop. His speech is halting, and he appears rustrated when trying to speak, but he ollows commands without diffi culty. His blood pressure is ���/�� mm Hg, with a heart rate o �� beats per minute. His electrocardiogram indicates atrial �brillation. What is the most appropriate next step in the evaluation and management o this patient?
�. A ��-year-old man has a �-month history o �uctuating difficulties with his speech and swallowing. He notes that with long conversations he has increasing diffi culty speaking; this is apparent during the history when his speech becomes nearly unintelligible. He describes several choking episodes and nasal regurgitation o liquids when swallowing. On examination, he has normal eye movements at baseline, but afer � minutes o sustained upgaze, asymmetrical ptosis (worse on the lef) and right hypertropia occur. Strength is initially normal, but with sequential strong contractions o the deltoid and iliopsoas muscles, mild weakness develops. Sensation, muscle stretch re�exes, coordination, and alternating motion rates are normal. Which o the ollowing would be the most appropriate next step in his evaluation? a. b. c. d. e.
a. Inusion o intravenous (IV) tissue plasminogen activator (tPA) b. Initiation o wararin therapy c. Computed tomography (C) o the head d. Inusion o IV labetalol e. Electroencephalography (EEG)
Autonomic testing, including tilt-table testing Urgent magnetic resonance imaging o the cervical spine Cerebrospinal �uid examination Muscle biopsy Serum acetylcholine receptor antibody testing
�. A ��-year-old woman has a �-week history o increasing clumsiness with both hands and imbalance that has led to several alls. Her amily has also noted that her speech “sounds drunk.” On neurologic examination, she has a wide-based, cautious, unstable gait. Alternating motion rates o the limbs are o normal requency and amplitude but are irregular and imprecise. Finger-to-nose testing is inaccurate, and the patient becomes tearul during the examination. Her strength, sensation, and muscle stretch re�exes are normal, and magnetic resonance imaging (MRI) o her brain is normal. Which o the ollowing is the most appropriate next step in her evaluation?
�. A ��-year-old woman presents with a �-day history o difficulty walking. She has also noted a tingling sensation in her �ngers and toes. On neurologic examination, she has a normal mental status and normal neurovascular examination �ndings. She cannot rise rom the couch without assistance and cannot ambulate without assistance. She appears somewhat short o breath. Cranial nerve examination �ndings are normal ���
a. Serum paraneoplastic antibodies, including Purkinje cell cytoplasmic autoantibody type � (PCA-�) (also known as anti-Yo) b. Consultation with a psychiatrist c. Cervical spine MRI with contrast medium d. Nerve conduction studies, including needle electromyo graphy e. Sural nerve biopsy
�. A ��-year-old man is evaluated or spells during which he appears alert but stares ahead, is unresponsive to the environment, and exhibits automatic lip-smacking movements. Results o his neurologic examination are normal. Magnetic resonance imaging o the head shows atrophy o the lef hippocampus. Results o awake electroencephalography (EEG) are normal. Te spells, which occur up to � times weekly despite treatment with maximally tolerated doses o carbamazepine and valproate over the past �� months, have prevented him rom driving and working. Which o the ollowing is the most appropriate approach?
c. Substitute phenytoin or carbamazepine. d. Perorm prolonged video-EEG monitoring or possible anterior temporal lobectomy. e. Discontinue use o valproate and consider vagus nerve stimulation.
�.
A ��-year-old obese woman has a history o depression, chronic obstructive pulmonary disease, constipation, and recurrent migraine headaches. Up to � years ago, the headaches averaged � attack every � to � months and responded to sumatriptan. Over the past � months, they have increased in requency to � to � attacks weekly. Results o neurologic and ophthalmoscopic examinations are normal. Which o the ollowing is the most appropriate prophylactic treatment or these headaches? a. Daily long-acting triptan b. Valproate c. opiramate d. Propranolol e. Amitriptyline
a. Substitute topiramate or valproate. b. Add gabapentin or levetiracetam.
���
•
MAYO CLINIC INERNAL MEDICIN E BOARD REVIEW: QUESIONS AND ANSWER S
ANSWERS
�. Answer c.
dysunction), and systemic symptoms, such as nausea and vomiting and diarrhea, may precede these symptoms.
Orthostatic hypotension is an important cause o “dizziness,” particularly in patients at risk o autonomic ailure, such as those with diabetic neuropathy or synucleinopathies (eg, parkinsonism). Te hallmark o orthostatic hypotension due to autonomic ailure is inability to increase the heart rate when the blood pressure decreases prooundly upon standing. Te �rst step in managing orthostatic hypotension is to correct potentially reversible causes, particularly the use o vasodilators, diuretics, and anticholinergics. Simple maneuvers, such as increasing salt and water intake, elevating the head o the bed, and perorming postural maneuvers, should be tried beore pharmacologic management.
�. Answer b.
Weakness may be caused by abnormalities in many areas o the central nervous system and peripheral nervous system. Central nervous system abnormalities causing weakness include supratentorial, inratentorial, and spinal cord lesions, each o which would cause an upper motor neuron pattern o weakness with increased re�exes and extensor plantar responses. Te weakness is usually associated with other �ndings that localize the lesion somewhere along the corticospinal tract. Disorders o the nerve roots (ie, radiculopathies) causing weakness are ofen associated with pain, they may be associated with decreased re�exes, and they are commonly associated with sensory complaints. Disorders o the muscle are associated with normal re�exes early in the disease but may be diminished later. Weakness is typically greater proximally than distally. Plantar responses are �exor, and there is no sensation loss. Neuromuscular diseases such as myasthenia gravis may cause atigable weakness o both the axial and the bulbar muscles. Re�exes are typically normal, and sensation is intact. Peripheral nerve disorders typically cause more diffuse weakness that is greater distally than proximally. Tey are ofen associated with some degree o sensory loss (there are more pure motor neuropathies that are associated with minimal sensory symptoms). Re�exes are ofen decreased early in the course and the plantar responses are �exor. Guillain-Barré syndrome (GBS) is an acute in�ammatory demyelinating disorder o the peripheral nerves and nerve roots. Patients present with subacute onset o weakness and sensory symptoms (eg, paresthesias) that slowly progess proximally rom the hands and eet. Te weakness usually begins in the legs but may occur simultaneously in the upper and lower extremities. Facial weakness occurs later in ��% o cases. Te re�exes are diminished early with “large �ber” joint position and vibratory sensation loss. Antecedent inection with viral or bacterial inections occur rom � to several weeks beore symptom onset. Further �ndings include abnormal CSF with an elevated protein level and a normal cell count (ie, albuminocytologic dissociation). In most GBS patients, these abnormalities can be ound within � week afer onset. MRI is not needed because the lesion is localized to the peripheral nerve. Electromyographic �ndings may be normal early but later show conduction block and nerve conduction
�. Answer e.
Myasthenia gravis is an autoimmune disorder o the neuromuscular junction. It causes atigable weakness o the axial musculature or bulbar muscles (or both). Diplopia, ptosis, speech slurring, and difficulty with chewing or swallowing are typical symptoms o the atigable weakness o the bulbar muscles. Additional diagnostic studies include electromyography, which shows a decrement in amplitude o compound muscle action potentials with repetitive stimulation. Computed tomography o the chest should be perormed because thymoma is occasionally associated with myasthenia gravis. Tymectomy has a role in the treatment o myasthenia gravis, particularly in a person with generalized myasthenia gravis and in younger patients who are not at signi�cant surgical risk. Acetylcholine receptor antibody testing is also useul or diagnosis. Te ensilon (edrophonium chloride) test involves the administration o edrophonium, a short-acting acetylcholinesterase inhibitor, which temporarily increases the concentration o acetylcholine in the synaptic clef, so that the signs o myasthenia gravis are temporarily improved. Atropine should be available because o the potential or bradycardia or other cholinergic symptoms. Autoimmune thyroid disease can also occur in myasthenia gravis, and appropriate blood tests should be perormed. Fatigable weakness argues against muscle disease, and muscle biopsy is not useul in myasthenia gravis. Another neuromuscular junction disease, botulism, occurs typically several hours afer ingesting ood containing Clostridium botulinum. Te patients present with diplopia, dysphagia, dysarthria, ptosis, and weakness o the jaw muscles. Autonomic symptoms can also occur (eg, constipation, dilation o the pupils, and urinary ���
velocity slowing, with �ndings more apparent on motor testing than on sensory testing; �brillation potentials indicating denervation also occur within � weeks o symptom onset. Patients with GBS are usually hospitalized and closely monitored or respiratory compromise. reatment is usually initiated with plasmapheresis or intravenous immunoglobulin and is effective particularly i it is started early. Close attention is also paid to deep vein thrombosis pro phylaxis, risk o aspiration pneumonia, constipation, back pain, and autonomic instability (variable heart rate and blood pressure). �. Answer c.
Te patient’s presentation suggests an acute ischemic cerebral inarction in the distribution o the anterior division o the lef middle cerebral artery, but C o the head is required to exclude the management-changing possibility o cerebral hemorrhage. Te commonly accepted window or IV thrombolysis is � hours, but imaging, laboratory tests, and a careul history cannot be sacri�ced to beat the deadline. Selected patients may be considered or IV tPA afer � hours, and intra-arterial intervention can be considered up to � or more hours afer the onset o symptoms. Anticoagulation does not typically have a role in the management o acute ischemic cerebral inarction, particularly beore imaging has excluded hemorrhage, although it will likely be considered or secondary prevention o stroke in this patient who has atrial �brillation. Te treatment o mild, asymptomatic hypertension in this patient could lead to neurologic deterioration. An EEG might be helpul i a postictal paresis is suspected (odd paralysis), but there is no history o seizure. �. Answer a.
Gait ataxia, limb dysmetria, and ataxic dysarthria all suggest a cerebellar disorder, and the subacute onset o the symptoms raises the likelihood o an autoimmune or
���
•
paraneoplastic mechanism. In this patient, the most appro priate next step among the choices listed is assessment or paraneoplastic antibodies, such as PCA-�, that may mediate a cerebellar syndrome. Tese antibodies and this syndrome are predictive o gynecologic malignancies such as ovarian carcinoma or breast carcinoma. Other diagnostic considerations or this patient might include structural cerebellar lesions such as a tumor (effectively excluded by the normal imaging), intoxication with anticonvulsant medications or alcohol, or inectious cerebellitis. �. Answer d.
Tis young man has temporal lobe epilepsy with complex partial seizures that is intractable since it is not responding to � medications. ypically, i a patient does not achieve seizure control with � medications, additional medical therapy will not help. In this case, the next consideration is or a surgical procedure to remove the epileptogenic portion o his brain. Continuous EEG monitoring is done in a hospitalized setting, ofen with withdrawal o the seizure medications, in order to map the seizure onset. I it localizes to the temporal lobe, there is about a ��% chance that the patient will be ree o seizures afer temporal lobectomy. Vagus nerve stimulation is done in patients who are not surgical candidates. �. Answer c.
Te requency o the migraine headaches is such that preventive medication is appropriate. Answer choices b through e would be reasonable or prevention o migraine, but triptans are abortive agents and would not be used daily. A patient’s comorbidities must be reviewed beore an appropriate preventive agent is chosen. Since this patient has lung disease, depression, and obesity, topiramate would be the best choice. Potential adverse effects o topiramate include paresthesias, cognitive clouding, taste perturbation (particularly with carbonated beverages), anorexia, and nephrolithiasis.
MAYO CLINIC INERNA L MEDICINE BOARD REVIEW: QUESIONS AND ANSWER S
��. DERM ATOLOGY QUESTIO NS AND A NSWERS
QUESIONS
�. An ��-year ��-year-old -old man has an itchy, new rash. At At the the nursing home where he lives, he has been treated with different antihistamines a ntihistamines and topical corticosteroids without response. A clinic visit is arranged. On examination, he has red papules and excoriations on the hands, groin, and axillae (Figure ��.Q�). He has nodular areas on his scrotum. What diagnosis should you consider ?
Multiple Choice (choose the best answer)
�. A patient presents with recurrent episodes o the skin �ndings shown in Figure ��.Q�. What is the most likely inectious association? a. Herpes simplex virus b. Pseudomonas c. Dermatophyte c. d. Borrelia burgdorferi e. Mycobact Mycobacterium erium marinum
a. b. c. d. e. e.
Chronic idiopathic urticaria Urticarial Urt icarial vasculitis Bullous pemphigoid Pemphigus vulgaris Scabies
Figure ��.Q�
�. A ��-year��-year-old old white woman presents with a acial rash that irst appeared last summer and returned during her recent trip to Arizona. Initially, she says that she eels well, but then she describes atigue and malaise. She has a amily history o rheumatoid arthritis. On examination, she is aebrile and she has no active synovitis. Both cheeks are erythematous and have small papules (Figure ��.Q�). here are no other concerning signs on examination. What is the diagnosis? a. Malar rash o lupus b. Seborrheic dermat dermatitis itis c. Allergic contact dermat dermatitis itis d. Rosacea d. e. Acne vulgaris
Figure ��.Q�. (Adapted rom Drage LA, Bundri Bundrick ck JB, Litin SC. S C. Clinical pearls in dermatology. dermatology. Mayo Mayo Clin Proc. Proc. ���� ���� Jul;��[�]:���–�. Used Used with permissionn o Mayo Founda permissio Foundation tion or Medical Medical Education Education and Research.)
���
Figure ��.Q�
�. A ��-year-old ��-year-old woman woman presents presents with a persistent, persistent, pruritic, red rash on her ace, chest, elbows, knees, and hands. Te rash began last summer and is unresponsive to topical corticosteroids. Her past medical history is signi�cant or asthma and migraines. For the past month, she has been having more problems with shortness o breath and headaches. Some o her skin �ndings are shown in Figure ��.Q�. She has mild wheezing. Neurologic examination �ndings are normal. Laboratory test results are positive or antinu antinuclear clear antibody and normal or the complete blood cell count, liver unction tests, and renal unctio unction n tests. What other laboratory testing would would be most important? a. b. c. d. e.
Figure ��.Q�
Chest radiography and mammography Patch testing Lupus serologies issue transglutamin transglutaminase ase testing testing Light testing
�. A ��-year-old man with chronic obstructive pulmonary disease has a signi�cant �are. You prescribe tapered dosages o corticoster corticosteroid oid and a course o azithro az ithromycin. mycin. A new rash develops (Figure ��.Q�). What is the most likely cause o the rash? a. b. c. d. e.
apered dosag dosages es o corticostero corticosteroid id Drug rash Pityriasis rosea inea corporis Mycoplasma pneumonia
Figure ��.Q�
���
•
MAYO CLINIC INERNAL MEDICIN E BOARD REVIEW: REVIEW: QUESIONS AND ANSWER S
�. A ��-year-ol ��-year-oldd woman presents with a nonhealing ulcer o the lower extremity. Tree weeks earlier, she noted a purple papule on her shin. It quickly enlarged, ulcerated, and became extremely painul. D ébridement was perormed at a local wound care center; aferward, the ulcer expanded to � times its original size. Te woman is now admitted or intravenous antibiotics and pain control. On her lef shin, she has a large, tender ulcer with a purple border (Figure ��.Q�). Her biopsy rom the wound care center was interpreted as abscess, but the culture results are negative so ar. What is the most likely diagnosis? a. b. c. d. e.
a. b. c. d. e. e.
issue transglutamin transglutaminase ase antibody antibody serology Creatine kinase measurement Perinuclear Perin uclear antineutr antineutrophil ophil cytoplasmic antibody antibody testing Human Hum an immunode�cie immunode�ciency ncy virus serology serology A ��-hour urine sample or measuring the level o �-hydroxyindoleacetic �-hydroxyindoleacet ic acid
Brown recluse spider bite Venous insuffi cien ciency cy ulcer Necrotizing Necrot izing asciitis Atypical mycobacteria mycobacteria inection Pyoderma gangrenosum
Figure ��.Q�
�.
A ��-year ��-year-old -old man presents with weight loss and mild anemia. He is also concerned about a new rash that is intensely pruritic and “burning.” He has received a diagnosis o irritable bowel syndrome rom his reerring physician and is being treated with �ber. He takes no other medication. He still has intermittent diarrhea and bloating but no other symptoms. On physical examination, you note the skin changes shown in Figure ��.Q�. What testing will help con�rm your diagnosis?
Figure ��.Q�
��. DERMAOLOGY QUESIONS AND ANSWERS
•
���
ANSWERS
�. Answer a.
�. Answer a.
Patients who have erythema multiorme present with target lesions that are ofen located on the palms and soles but may occur as generalized eruptions. Drugs such as sulonamides, barbiturates, and anticonvulsants are commonly associated with development o erythema multiorme, but recurrent lesions are most ofen linked to a herpes simplex inection. Although the skin signs o an active herpes simplex inection may be apparent, the outbreak may be subclinical. Discontinuing use o any culprit drugs and a trial o acyclovir or other appropriate antiviral would be appropriate in cases o recurrent erythema multiorme.
Dermatomyositis is an in�ammatory myositis with distinctive skin �ndings. Skin signs seen with dermatomyositis may include heliotrope rash, Gottron papules, photosensitivity, Gottron sign (erythema over the extensor suraces o the �nger joints, knees, and elbows), periorbital edema, scalp erythema and pruritus, violaceous erythema o sun-exposed or extensor suraces, periungual erythema and telangiectasia, cuticle hypertrophy, generalized pruritus, shawl sign, and holster sign. Patients may present with skin �ndings but without clinical weakness or muscle enzyme abnormalities (amyopathic dermatomyositis or dermatomyositis sine myositis). Dermatomyositis may be a skin sign o internal malignancy with a signi�cantly higher risk o lung, breast, or ovarian cancer. A workup or underlying malignancy should be pursued.
�. Answer e.
Scabies is caused by inestation with the mite Sarcoptes scabiei var hominis. Inection occurs as a result o direct skin-to-skin contact; omite transmission is uncommon. It causes epidemics in schools, hospitals, and nursing homes. Te rash results rom a hypersensitivity reaction to the mite protein. Te main complaint is pruritus, especially at night. Clinical eatures include in�ammatory, excoriated papules in the web spaces o the hands and eet, axillae, groin, and wrists and the areolae and submammary sites o women. Nodules or thickened areas in the scrotum are also helpul clues. Te pathognomonic �nding is a burrow, commonly located on �nger webs. Identi�cation o mites on a scabies preparation is diagnostic. In immunocompromised patients, a highly contagious orm o scabies may appear as a generalized scaling eruption (ie, crusted scabies, ormerly called Norwegian scabies). reatment o classic scabies includes the topical application o permethrin; oral ivermectin may be used to treat crusted scabies.
�. Answer a.
Te man has psoriasis vulgaris. Psoriasis may be precipitated by many actors, including inection (streptococcal phar yngitis, human immunode�ciency virus disease), stress, smoking, alcoholism, physical trauma (koebnerization), and medications. Common medications that cause �ares or unmask psoriasis include lithium, β-blockers, calcium channel blockers, angiotensin-converting enzyme inhibitors, some nonsteroidal anti-in�ammatory drugs, antimalarials, and tapered dosages o corticosteroid. Psoriasis maniests with sharply demarcated plaques with silver scale involving the elbows, knees, scalp, lumbar region o the back, and the gluteal clef. Nail signs (nail pits, oil spots, and onycholysis), scalp scale, gluteal clef pinking, and koebnerization (development o psoriatic skin disease at sites o trauma) are subtle signs that can help with the diagnosis. Psoriatic arthritis occurs in �% to �% o patients who have psoriasis o the skin and commonly maniests as asymmetric oligoarthritis or enthesitis.
�. Answer d.
Not all malar rashes result rom lupus. Rosacea is a common acial rash with red papules and small pustules on a base o erythema and telangiectasia. It commonly affects the central acial area. Sun, vasodilators, caffeine, and hot or spicy oods may trigger �ares. It may be associated with eye �ndings, but otherwise it has no systemic associations. Mild rosacea is treated with topical metronidazole, use o sunscreens, and avoidance o triggering actors. Anti-in�ammatory antibiotics, such as tetracycline, are used in more severe cases. Rhinophyma can be treated with laser and other surgical methods.
�. Answer e.
Pyoderma gangrenosum is primarily a clinical diagnosis based on the patient’s history and physical examination �ndings. Patients typically present with an expanding nonhealing ulcer on the lower extremities and may relate a history o trauma that initiated or worsened the lesion (ie, pathergy). Te ulcer is extremely painul and unresponsive to conservative wound care measures. On physical examination, the rapidly enlarging, necrotic ulcer has a purple undermined margin and a surrounding border o ���
erythema. Te histopathologic eatures are nonspeci�c, and on skin biopsy the neutrophilic in�ltrate may be inter preted as cellulitis or abscess. Pyoderma gangrenosum may occur in otherwise healthy people or may be a skin sign o internal disease. Important associations include in�ammatory bowel disease, IgA monoclonal gammo pathy, rheumatoid arthritis, chronic active hepatitis, and hematologic malignancies. reatment o pyoderma gangrenosum includes intralesional corticosteroids, oral corticosteroids, potassium iodide, dapsone, minocycline, azathioprine, and cyclosporine. Te response to corticosteroids is dramatic. �. Answer a.
Dermatitis herpetiormis maniests with intensely pruritic vesicles and papules occurring symmetrically over the knees, elbows, lower back, neck, and scalp. Te classically grouped (herpetiorm) vesicles are rarely intact because o the intensity o the pruritus and subsequent scratching by
the patient. Tus, on physical examination, excoriations, erosions, and crusts are more commonly noted than intact vesicles. Most patients with dermatitis herpetiormis have subclinical celiac disease (gluten-sensitive enteropathy). Adults may present with episodic or nocturnal diarrhea, �atulence, or weight loss. Abdominal discomort and bloating may lead to a mistaken diagnosis o irritable bowel syndrome. Biopsies show classic jejunal villous atrophy in ��% o the patients; the remainder have minor mucosal changes. Te standard or diagnosis o dermatitis herpetiormis is a skin biopsy that shows granular immunoglobulin A deposits in the dermal papillae on direct immuno�uorescence examination. Serologic tests or tissue transglutaminase antibody or endomysial antibody are highly sensitive and speci�c markers or an associated gluten-sensitive enteropathy. reatment o dermatitis herpetiormis includes a gluten-ree diet and dapsone. Dapsone provides rapid relie rom pruritus and the rash but has no effect on the enteropathy.
��. DERMAOLOGY QUESIONS AND ANSWERS
•
���
This page intentionally left blank
��. CROSS�CONTENT AREA QUESTIONS AND ANSWERS
QUESIONS Multiple Choice (choose the best answer) GERIATRICS
�. An ��-year-old man is brought in by his son. Te patient has no complaints, but the son is concerned that his ather’s memory is ailing. Te patient resides in an assisted living acility. He receives � meals daily and an aide helps him with housework weekly. His amily moved him there several months ago when it was clear that he could no longer live alone. Last week, a staff member ound him wandering in the street, dressed in only a bathrobe and pajamas (the outside temperature was �� °C). Te patient told the nurses that he was ollowing a man who had entered his room. He has a history o hypertension treated with lisinopril �� mg daily. His son states that he ound an empty pill vial at home, and it does not appear that the prescription has been re�lled recently. On examination, the patient’s blood pressure is ���/�� mm Hg. He is unkempt. He has a resting tremor and a slow, shuffling gait. His Mini-Mental State Examination (MMSE) score is �� points (out o ��). Six months ago, his MMSE score was �� points (out o ��). What is the most appropriate next step in his management? a. Perorm magnetic resonance imaging (MRI) o his brain. b. Start donepezil therapy. c. Advise the son to move him into a nursing home. d. Start low-dose haloperidol therapy or his psychotic symptoms. e. Start citalopram therapy.
�. A ��-year-old emale nursing home resident complains o urinary incontinence. She describes the sensation o having an urgent need to void. However, with her arthritic knees, she ofen cannot reach the toilet beore she loses urine. Tese urinary symptoms have been present or several years but have gradually worsened. She is now araid to go out in public. She has no dysuria or hematuria. She takes metoprolol, hydrochlorothiazide, and a stool sofener. Which o the ollowing is the most appropriate �rst step in the management o her incontinence? a. b. c. d. e.
Begin tolterodine therapy. ry a trial o topical estrogen. Reer to a urologist or urther evaluation. ry a trial o timed voiding. Discontinue the use o hydrochlorothiazide.
�. A ��-year-old woman undergoes a right L� oraminotomy and an L�-S� usion. Past history is signi�cant or mixed connective tissue disease. Preoperative medications are methylprednisolone, hydroxychloroquine, and wararin or symptoms o connective tissue disease (arterial occlusive disease was considered a component o the connective tissue disease). She is also taking nortriptyline and tramadol or back pain. Immediately postoperatively, she has mild hypoxemia (thought to be rom narcotics). On the evening afer surgery, she is agitated and awake all night. She is drowsy and difficult to arouse on postoperative day � and reuses to eat or participate in therapy. She does not sleep. On postoperative day �, she is not oriented to time or place and is complaining o chest pain. What is the most appropriate way to manage her symptoms? a. Vest restraints, bed rails, and benzodiazepines titrated to a dose that causes sedation b. Vest restraints, haloperidol or agitation, and ��-hour supervision c. Sleep enhancement protocol, amily support, and haloperidol or agitation d. Benzodiazepines or sleep and removal o urinary catheter e. Psychiatric consultation
�. An ��-year-old woman presents with a �-kg weight loss over the past � months. She had a myocardial inarction �� years ago but has been asymptomatic ever since. She has been treated with atenolol and aspirin, and her mild hypertension is well controlled with the atenolol. She ractured her hip when she ell on ice about � years ago, but she does not have a history o recurrent alls. She states that her mood is good. She does her own shopping, but �nds it hard to carry the groceries into the house because o generalized atigue. She lives alone and her only daughter lives � hours away. She does not have any gastrointestinal tract (GI) com plaints. Her only real complaint is atigue. Her body mass index is �� (it was �� at her previous examination � months ago). Vital signs are normal. Te remainder o the examination �ndings are normal. Her health screening is up-to-date and unremarkable. Laboratory test results are normal or erythrocyte sedimentation rate, complete blood cell count, and creatinine. What should be done next?
���
a. Perorm a complete GI workup, including computed tomography o the abdomen and upper and lower GI endoscopy. b. Discontinue the use o atenolol. c. Start antidepressant therapy. d. Help her arrange assistance with shopping and cooking. e. Recommend that she move in with her daughter.
�. A ��-year-old woman presents or her annual examination. She has no complaints. She has a history o hypertension, which is well controlled with hydrochlorothiazide. She is physically active. She received an in�uenza vaccination last year and plans to receive one again this year. She received her pneumococcal vaccination at age ��. She received her most recent tetanus toxoid �� years ago. She helps her daughter run a amily day care or �� children aged � months to � years. In addition to the in�uenza vaccine, what other vaccines should you recommend? a. etanus and diphtheria toxoids vaccine booster b. etanus and diphtheria toxoids and acellular pertussis (dap) vaccine c. Polyvalent pneumococcal vaccine d. Measles-mumps-rubella vaccine e. Hepatitis A vaccine
�. An ��-year-old woman is seen in the office or her annual examination. She has a history o osteoporosis and is taking alendronate. Her body mass index is ��. She is the primary caretaker or her ��-year-old husband, who has Parkinson disease. She is a nonsmoker. She had a normal colonoscopy at age ��. Which o the ollowing screening tests is appropriate? a. Spirometry b. Colonoscopy c. Carotid ultrasonography d. Hypertension screening e. Exercise stress test
�.
A ��-year-old man is brought to your office by his wie. He has a known history o Alzheimer disease and has had a rapid loss o memory. Terapy with donepezil was stopped because he had gastrointestinal tract side effects. He did not want to try memantine. He can no longer care or himsel. His wie, who has diabetes mellitus, osteoarthritis, and hypertension, provides all his care. She is eeling increasingly stressed. He has periods o agitation several times each day. On � or � occasions, he has struck his wie. He ofen resists getting dressed or eating. His wie ears that she can no longer continue to care or him at home. However, she is struggling with this decision because he has told her that he would rather die than live in a nursing home. What is the most appropriate way to manage his symptoms? a. He can no longer be cared or saely at home, so he should live in a nursing home. b. He should be enrolled in an adult day care that incorporates music therapy and aromatherapy. c. He should take risperidone. d. Low-dose lorazepam should be given as needed or agitation. e. He should take citalopram.
���
•
PREVENTIVE MEDICINE
�. A ��-year-old man presents to an urgent care clinic afer stepping on a piece o broken glass in his ront yard. Te glass is lodged in the sole o his lef oot. He says that he has received several tetanus vaccinations during his lie, but he does not remember all the dates. Immunization records show that he received his latest tetanus-diphtheria (d) vaccination � years ago. What urther therapy is needed or optimal tetanus prophylaxis? a. etanus immune globulin b. etanus toxoid and tetanus immune globulin now c. etanus toxoid d. etanus toxoid ollowed by tetanus immune globulin in � days e. No urther therapy
�. An asymptomatic, active ��-year-old man presents or a general medical examination. He does not have any medical problems, but he has a ��-pack-year smoking history. He has no amily history o colon cancer. He had his latest colonoscopy � years ago; results were normal without polyps. He received tetanus-diphtheria (d) vaccine and pneumococcal vaccine polyvalent � years ago and zoster vaccine live � years ago. Which preventive service should you recommend? a. Abdominal aortic aneurysm (AAA) screening ultrasonography b. Pneumococcal vaccine polyvalent c. d vaccine d. d vaccine and AAA screening ultrasonography e. etanus and diphtheria toxoids and acellular pertussis (dap) vaccine
��. A ��-year-old woman presents or a general medical examination. She is healthy but is concerned that her ather had colon cancer at age ��. She wants to know at what age she should undergo her �rst colonoscopy. What age should you recommend? a. b. c. d. e.
�� �� �� �� ��
��. A ��-year-old woman presents or a routine general medical examination. She has had long-standing, well-controlled asthma. She is otherwise healthy. She does not smoke or drink alcoholic beverages, and she works as an office secretary. Which o the ollowing should you recommend? a. b. c. d. e.
In�uenza and pneumococcal vaccination Human papillomavirus (HPV) vaccination In�uenza vaccination only Pneumococcal vaccination only In�uenza and HPV vaccination
M A YO C L I N I C I N E R N A L M E D I C I N E B O A R D R E V I EW : Q U E S I ON S A N D A N S W E R S
��. A ��-year-old woman presents to your oice or a routine general medical examination. She is healthy and does not have a amily history o breast cancer. She had menarche at age �� and her irst child at age ��, and she breast-ed her � children. She has never had a mammogram and asks about the need or mammography. What should you recommend or this patient ? a. b. c. d. e.
Perorm screening now and then annually. Perorm screening now and then at age ��. Begin screening at age ��. Perorm screening now and then biennially. Weigh the individual risks and values with the bene�ts and harms o screening.
��. A ��-year-old man presents or a general medical examination. He is healthy and does not routinely take any medications. He has had � lietime partners and has been in a monogamous relationship or � years. He is a nonsmoker, he drinks � or � beers each week, and he does not use illicit drugs. His ather had a myocardial inarction at age ��. His blood pressure in the offi ce is ���/�� mm Hg. Which o the ollowing screening services would be most appropriate to offer to this patient? a. b. c. d. e.
Blood chemistry panel Fasting blood glucose level Lipid panel Chlamydia screening Voluntary human immunode�ciency virus (HIV) testing
��. A ��-year-old healthy bus driver is planning to join a local gym with his wie. He does not exercise on a regular basis. He does not take daily medications or use tobacco. His ather died o a myocardial inarction at age ��. His blood pressure in the oice is ���/�� mm Hg, his pulse is �� beats per minute, and his body mass index is ��. His total cholesterol level is ��� mg/dL, and his high-density lipoprotein cholesterol level is �� mg/dL. His Framingham risk score is �%. He asks or heart testing beore starting his new exercise program. What urther testing should you recommend? a. b. c. d. e.
Electrocardiogram (ECG) Computed tomographic (C) coronary calcium scan No urther testing Exercise treadmill testing High-sensitivity C-reactive protein (hs-CRP) testing WO M E N ’S H E A LT H
��. A ��-year-old woman who you know well comes or her annual preventive gynecologic examination. She eels well. She has a history o type � diabetes mellitus, which she manages with her liestyle, and rheumatoid arthritis. She tells you that her rheumatologist prescribed adalimumab shortly afer her visit with you last year and that her symptoms are well controlled; she is pleased with the results. She went through
menopause � years ago without the use o hormone therapy. She has been ree o vasomotor symptoms or � months but comments that intercourse with her partner o � years is uncomortable, so she began using a vaginal lubricant with some improvement in symptoms. Otherwise, she is doing very well. Her pre ventive health screening tests are all up-to-date with the exception o her Papanicolaou (Pap) test; results o her latest test, done � years ago, were normal. On examination, she appears healthy and comortable. Her pelvic examination shows vaginal atrophy with no other �ndings. You recommend vaginal estradiol. A Pap test is perormed. wo days later, the results show atypical squamous cells o undetermined signi�cance (ASC-US). You schedule an appointment or her to return to the offi ce to discuss the results and your recommendations. Which o the ollowing should you recommend? a. Human papillomavirus (HPV) DNA testing and, i the results are positive, loop electrosurgical excision b. Loop electrosurgical excision c. Pap tests in � and �� months and, i the results still show ASC-US, colposcopy d. Vaginal estradiol or � months and then another Pap test e. Colposcopy
��. A healthy ��-year-old woman wants your recommendations or breast cancer prevention because her mother had breast cancer at age ��. Te patient perorms regular breast sel-examinations and practices breast sel-awareness, and she has noticed no changes. She began having mammograms at age �� and has had no abnormalities; her latest mammogram �� months ago was unremarkable, showing only dense tissue. Her past medical history is unremarkable. Her latest menstrual period was � month ago, but the period beore that one was � months earlier; beore that, her menses had been irregular. Family history is otherwise notable or ovarian cancer in her maternal aunt at age ��, breast cancer in her maternal grandmother at age ��, prostate cancer in her maternal uncle at age ��, and pancreatic cancer in her maternal great grandather at age ��. Findings on her physical examination, including breast examination, are normal. Te mammogram done the same day is normal and unchanged compared with prior mammograms. Dense tissue is still observed. You recommend a medical genetics consultation to urther evaluate her amily history because you are concerned about the possibility o hereditary breast and ovarian cancer syndrome. Tree weeks later, the patient returns and reports that her mother accompanied her to the genetics consultation. Te geneticist estimated that the patient’s mother had a ��% likelihood o a mutation; thus, the patient has a ��% likelihood. Te patient’s mother was tested and the patient has learned that her mother carries a truncation mutation in BRCA�. Which o the ollowing should you recommend that the patient consider as her next step?
�� . C R O S S � C O N E N A R E A Q U E S I O N S A N D A N S W E R S
•
���
a. b. c. d. e.
Annual digital mammogram and clinical breast examination Bilateral mastectomy and oophorectomy amoxien �� mg daily or � years Raloxiene �� mg daily or � years Genetic testing
��. A healthy ��-year-old premedical student presents to your office or urther evaluation o a change in her menstrual periods. She reports that her periods began at age ��. Until � months ago, her menses were regular or � years and occurred every �� to �� days. She eels well but comments that she has been stressed about taking the Medical College Admission est and has been staying up late studying or several weeks. She attributes her headaches and blurry vision to stress and lack o sleep. She also tells you that her weight has increased since reshman year, probably because she snacks while studying and eats ast ood too ofen. She says that she started exercising and dieting last year, but she has not lost weight. Records she brought rom a student health clinic visit or an ankle sprain reshman year note body mass index o �� and normal vital signs. Her blood pressure is ���/�� mm Hg, her heart rate is �� beats per minute and regular, and her body mass index is ��. She appears to be a healthy, somewhat overweight young woman who is comortable. Findings rom the physical examination, including a pelvic examination, are unremarkable. Which o the ollowing tests would be most inormative? a. b. c. d. e.
Serum β-human chorionic gonadotropin Tyrotropin and ree thyroxine Serum estradiol Serum prolactin Serum ollicle-stimulating hormone
��. Which o the ollowing vaccinations should not be administered during pregnancy? a. Human papillomavirus (HPV) b. Meningococcus c. Pneumococcus d. Hepatitis A e. etanus-diphtheria (d)
��. A ��-year-old woman presents to your office concerned about milky discharge rom the lef breast. She eels well and has no other concerns. She reports normal menstrual periods and no prior breast problems. Her sister had breast cancer at age ��. Te patient has been very worried and checks regularly to see i the discharge is still present. Her sel-examination �ndings are other wise unchanged. She has � children and �nished nursing the second child �� months ago. On examination, she appears comortable, she has no adenopathy, and her breasts are normal bilaterally, with no skin abnormalities and no palpable abnormalities. Several drops o milky �uid are easily expressible rom several ducts o the lef nipple. Te �uid tests negative or blood. Serum thyrotropin (SH) and prolactin (PRL) levels are normal. Results rom bilateral diagnostic mammography ���
•
and lef subareolar ultrasonography are unremarkable, except or mild duct ectasia, with no abnormalities identi�ed. Which o the ollowing should you recommend? a. Reassurance and recommendation to discontinue checking so requently b. Ductoscopy c. Breast magnetic resonance imaging (MRI) d. Subareolar duct excision e. Pituitary MRI
��. A healthy ��-year-old woman presents to your office or urther evaluation o vaginal symptoms. She reports that or the past � months she has had persistent vaginal irritation, with itching and burning and a somewhat thick discharge. She has normal menses. She has been in a monogamous relationship with the same partner or the past �� years. She uses oral contraceptives. She appears mildly uncomortable but otherwise well. Her examination is signi�cant or vulvar erythema without any lesions; she has a mildly thick discharge. Te speculum examination shows normal �ndings, but she is uncomortable during it and during the bimanual examination. A Papanicolaou test is perormed. Te pH o the vagina is �.�. Te whiff test is negative. Microscopy shows only a moderate number o large rods, and Gram staining shows gram-positive rods. Culture results are negative or Trichomonas vaginalis . What is the most likely diagnosis? a. b. c. d. e.
Vaginal candidiasis richomoniasis Contact dermatitis Bacterial vaginosis Lichen planus
��. A ��-year-old healthy woman asks or a recommendation or hot �ushes and vaginal symptoms. She com pleted natural menopause about �� months ago. She began having hot �ushes and sweats during perimeno pause, and the symptoms continued afer menopause. She wakes up multiple times at night because o hot �ushes and has to change her nightclothes at least once nightly. She is atigued and irritable. She is embarrassed at work and in social situations because she sweats through her clothes. She has ollowed all recommendations about clothing, diet, and her immediate environment, but her symptoms have continued. Soy and red clover have not helped, either. She has also had worsening vaginal symptoms with dryness and burning causing signi�cant discomort during intercourse despite use o vaginal lubricants. Tis has led to avoidance o intimacy with her partner. She reports exercising regularly, and she has never smoked. She has no signi�cant medical or amily history. On examination, she is a healthy-appearing woman in no distress. Her breasts are normal on examination. Pelvic examination shows signi�cant vaginal atrophy. Her mammogram rom � months ago was unremarkable. She has never had an abnormal result rom a Papanicolaou test, including
MAYO CLINIC INERNAL MEDICIN E BOARD REVIEW: QUESIONS AND ANSWER S
the latest one, done � years ago. Which o the ollowing is the most appropriate recommendation? a. Start use o vaginal estrogen tablet �� mcg twice weekly, and reassess within �� months. b. Add black cohosh �� mg daily, and reassess within �� months. c. Start use o venlaaxine ��.� mg daily, titrating up to �� mg daily as needed, and reassess within �� months. d. Encourage her to wait it out and use a different vaginal lubricant. e. Begin combination hormone therapy, and reassess within �� months.
��. A healthy ��-year-old woman presents to your office with a �-week history o a grape-sized breast lump that she elt while showering. She reports that the lump is nontender and she has not noticed any skin changes or nipple discharge. Te appearance o the breast is unchanged. Her latest menstrual period occurred last week and was normal. She has no prior medical problems, and her amily history is negative or malignancies. She has no notable risk actors or malignancies. On examination, she appears healthy. Breast examination is unremarkable with the exception o a �.�-cm �rm mass in the lateral portion o the right breast. A mammogram shows dense breast tissue that appears normal, symmetrical, and, when compared with a prior screening mammogram, stable. What should you recommend as the next step? a. Breast ultrasonography b. Reassurance c. Surgical consultation d. Follow-up mammogram midcycle in � to � months e. Breast magnetic resonance imaging
��. A ��-year-old nulliparous woman in good health presents or evaluation o bilateral breast pain. She reports aching, burning pain (pain level � on a scale rom � to ��) in both breasts; the discomort has been interering with sleep and intimacy. Her breasts eel heavy and uncomortable, especially with activity that moves them. She has not noted any other changes in either breast, but she does not do sel-examinations. Her latest menstrual period was � weeks ago, and her periods have been normal. She has not paid attention to whether there is any pattern to the pain, but her quality o lie has been affected since she avoids intimacy and exercise when her breasts hurt. Her mother had breast cancer at age ��. On examination, the patient is overweight and has large, symmetrical breasts that are normal in appearance. She has nodular, lumpy breasts bilaterally but no ocal lumps or discrete masses. Te nodularity is thought to be relatively symmetrical when the breasts are compared. What is the most appropriate next step? a. b. c. d.
Reassurance and breast support with a well-�tting bra Bilateral diagnostic mammography and ultrasonography Danazol ��� mg daily in divided doses Fine-needle aspiration biopsy rom the area o greatest pain
e. Bilateral diagnostic mammography, ultrasonography, and magnetic resonance imaging
��. Which o the ollowing statements is true? a. Rubella is the most common cause o congenital viral inection. b. A pregnant woman presenting with pulmonary edema and atrial �brillation has a high likelihood o having mitral stenosis. c. Te lietime risk o ovarian cancer is greater than ��% in a carrier o the BRCA� mutation. d. Dysunctional uterine bleeding is benign and requires no evaluation. e. Annual clinical breast examinations decrease breast cancer mortality. GENERAL INTERNAL MEDICINE
��. Your clinic has access to magnetic resonance imaging (MRI) technology. You are evaluating a patient who has a diabetic oot inection. Tere is no exposed bone. You are concerned that he has not responded well to antibiotic treatment, and you determine that his current pretest likelihood o having underlying osteomyelitis is ��%. Recently, you read an article that evaluated the role o MRI in diagnosing oot osteomyelitis. Te sensitivity o this test is ��%, and the speci�city is ��% (com pared with bone biopsy as the reerenced standard). I you use MRI or this patient, and it is positive or osteomyelitis, what is the likelihood that he indeed will have osteomyelitis on bone biopsy? a. ��% b. ��% c. ��% d. ��% e. ��%
��. Reer to the inormation in question ��. I the magnetic resonance imaging (MRI) �ndings were negative, what is the likelihood that this patient would not have osteomyelitis? a. ��% b. ��% c. ��% d. ��% e. ��%
��. You have recently read about a randomized controlled trial that compared the efficacy o wararin (international normalized ratio �.�–�.�) with that o aspirin (��� mg) in stroke prevention in a subset o elderly patients (�� years or older) who had atrial �brillation. Te patients were ollowed or � years. Te �nal results showed that the chance o major stroke, intracranial hemorrhage, or systemic embolism was �% among patients treated with wararin and ��% among patients treated with aspirin. From these results, what is the relative risk reduction (RRR) o a major event in the group treated with wararin compared with the group treated with aspirin?
�� . C R O S S � C O N E N A R E A Q U E S I O N S A N D A N S W E R S
•
���
a. �% b. ��% c. ��% d. ��% e. ���%
He is otherwise healthy. His creatinine value is �.� mg/ dL, and his platelet count is ��� ���/L. Which o the ollowing statements is correct?
��. Reer to the inormation in question ��. For the group treated with wararin (international normalized ratio �.�–�.�) compared with the group treated with aspirin, what is the absolute risk reduction (ARR) o a major event at � years? a. �% b. ��% c. ��% d. ��% e. ���%
��. Reer to the inormation in question ��. What is the number needed to treat (NN) or � years with wararin anticoagulation to prevent � major event? a. b. c. d. e.
� �� �� �� ��
a. Because the clot is con�ned to his cal veins, anticoagulation is not required, but ultrasonography should be repeated in � to � days to rule out proximal extension. b. A temporary vena cava �lter should be placed to decrease the risk o pulmonary embolism because anticoagulation would be too risky in this early postoperative situation. c. For convenience, oral dabigatran could be used to treat the clot because no laboratory monitoring would be needed. d. He should receive anticoagulants or � months; an appro priate low-molecular-weight heparin (LMWH) (eg, enoxaparin) dosage would be ��� mg subcutaneously once daily while administering wararin. e. He should receive anticoagulants or � months; an appro priate LMWH (eg, enoxaparin) dosage would be ��� mg subcutaneously once daily while administering wararin. GENERAL INTERNAL MEDICINE AND QUALITY IMPROVEMENT
��. A ��-year-old woman presents with acute pain in her let leg. Ultrasonography has conirmed a new thrombus in the let supericial emoral vein. She is otherwise healthy and states that she has not had recent surgery or trauma. Her only medication is con jugated estrogen �.��� mg daily or hot lushes. She does not have a personal or amily history o venous thromboembolism, and she has no major bleeding risks. Her creatinine value is �.� mg/dL. She has a supportive husband at home who can help her with activities o daily living. She weighs �� kg. What is the most appropriate next step in the management o this patient? a. Elevate the leg, apply an elastic bandage wrap to the leg, and administer anti-in�ammatory drugs until she has recovered. b. Discontinue use o conjugated estrogen, elevate the leg, apply an elastic bandage wrap to the leg, and administer anti-in�ammatory drugs until she has recovered. c. Hospitalize and treat with either low-molecular-weight heparin (LMWH) or unractionated heparin while administering wararin. d. Provide outpatient treatment with LMWH while administering wararin. e. Request computed tomography (C) o the chest to look or pulmonary emboli beore deciding on hospitalization or outpatient treatment.
��. A ��-year-old man weighing �� kg presents or evaluation o an acute increase in swelling and pain o his right lower extremity. One week earlier, he had a right total knee arthroplasty and is currently walking with a walker. Ultrasonography o his right lower extremity con�rms that a thrombus in the right anterior tibial vein extends into the popliteal vein. Te emoral vein is ree o clot. ���
•
��. You serve on the sentinel event review committee at your hospital. An event occurred in which a patient received an overdose o heparin. Your committee completes a root cause analysis and �nds that the error resulted rom a gap in physician knowledge about heparin dosing, the lack o an institutional consensus on heparin dosing, and a cumbersome order entry system. From the root cause analysis, which o the ollowing interventions is most likely to have a sustained effect? a. b. c. d. e.
An online education module on heparin dosing Distribution o a heparin dosing pocket card A heparin order set A new institutional policy on heparin dosing A physician education conerence on heparin dosing
��. An ��-year-old woman comes to your oice with a �-day history o a red let eye associated with dull, achy pain and tears. She denies having any trauma or visual problems. On examination, she has clear tears and a ocal area o redness in the medial sclera. What would you recommend or control o her symptoms? a. Acetaminophen and avoiding contact with others until the symptoms resolve b. Aspirin and reassurance c. Urgent reerral to ophthalmology d. Gentamicin eyedrops or � to �� days e. Corticosteroid eyedrops or � week
��. A disheveled ��-year-old college student comes into the university student health center. It is Monday and he has just returned rom a spring break trip to Florida. He complains o a bright red right eye associated with copious purulent discharge, which began abruptly last night. On examination, you note a brightly erythematous sclera associated with purulent discharge and a prominent right preauricular lymph node. In addition
MAYO CLINIC INERNAL MEDICIN E BOARD REVIEW: QUESIONS AND ANSWER S
to immediate reerral to the local ophthalmologist or oral and topical antibiotics, what other testing should be considered next? a. Allergy testing or pollen and and ragweed ragweed b. Screening or human immunode� immunode�ciency ciency virus/AIDS c. Urin Urinary ary polymerase chain reaction or Chlamydia trachomatis and Neisseria gonorrhoeae d. Rapid streptococcal antigen antigen test or group group A streptococci streptococci e. Complete blood blood cell count count with erythrocyte sedimentation sedimentation rate
��. A ��-year-old kindergarten teacher presents to your office with a �-week �-we ek history histor y o an upper up per respiratory respir atory illness. She initially noted a low-grade ever, achiness, sinus congestion congestion with clear discharge, and cough, which improved over �� days but then seemed to recur with a vengeance. Now she has a severe sore throat, enlarged glands, and ever. She does not have a cough or shortness o breath. On examination, she has a ever (��° (�� °C), tonsillar exudate, and cervical adenopathy, and she appears ill and uncomortable. Abdominal examination �ndings are unremarkable. unremarkable. Which o the ollowing is most appropriate? a. b. c. d. e.
Mononucleosis spot test Mononucleosis Troat culture and Gram stain Rapid streptococcal streptococcal antigen test Empirical penicillin penicillin V or �� �� days Chest radiograph
��. Which eatures could potentially distinguish between acute angle-closure glaucoma and acute anterior uveitis? a. Circumco Circumcorneal rneal injection b. Blurry vision c. Hypopyon c. d. Pain d. e. Unilateral presentation o symptoms
��.. A ��-year-old �� ��-year-o ld male patient presents with gradualgradual-ononset erectile dysunction (ED). He is in good health, and he takes no medications. On physical examination, there is no evidence o penile abnormalities or testicular atrophy. Visual �eld testing and cardiopulmonary examination �ndings are normal. Serum testosterone levels are normal. He reports that he has a healthy libido and is involved in a stable and otherwise satisactory marital relationship. What should you recommend? a. He should undergo coronary angiography to rule out a. vascular disease as a cause o the ED beore initiatin initiatingg any treatment. b. He should undergo undergo urther urther evaluation or hypogonadism as a likely cause o the ED. c. He should undergo undergo magnetic resonance resonance imaging imaging (MRI) o the brain to rule out pituitary disease as a cause o his ED. d. He should undergo undergo psychotherapeutic psychotherapeutic intervention intervention or perormance anxiety. e. He should take a phosphodiesterase-� (PDE-�) inhibitor � hour beore sexual intercou intercourse. rse.
��. Recently, several complaints have been �led by patients in your clinic about excessive wait times in the lobby. As a member o the quality team at your clinic, you are charged to study and �x this problem. Which o the ollowing improvement methodologies would be most successul at reducing wait times or patients in the clinic lobby? a. a. b. c. d. e.
LEAN Six Sigma Cause-and-effect diagramming Swim lane diagramming Failure mode and effects analysis
��. A ��-year-o ��-year-old ld woman with long-stan long-standing ding diabetes mellitus is admitted to your hospital service or pain control or a pelvic insuffi ins ufficiency racture. rac ture. She receives hemodihemo dialysis. An electrocardiogram shows that her corrected Q interval is ��� ms. What pain medication should be ordered or this patient? a. Fentanyl a. b. Methadone b. c. Propoxyphen Propoxyphenee napsylate d. Codeine d. e. Meperidine e. M E D I C A L E T H I C S A N D PA L L I A T I V E C A R E
��. For � years, you have been caring or a ��-year-old unmarried man. He has bipolar disease but has been aithul in taking his medication. He is intelligent and works as a computer programmer. He has not had an episode o signi�cant mania or depression or the past � years. He is a ormer smoker smoker,, but he quit quit smoking smoking when his bipolar disease was adequately treated. Recently, he presented with the complaint o a new cough without other symptoms o upper respiratory tract inection, and he reported that on a couple o occasions he coughed up blood in his sputum. Chest radiography, ollowed by computed tomography, showed that he had a solitary lung mass on the right side. Sputum analysis con�rmed the presence o adenocarcinoma. His evaluation suggested that the lesion was stage I, which would indicate a high probability o cure by resection (�-year survival about ��%). When you share the diagnosis and recommendations or operation with him, he declines the procedure. In act, he wants no treatment at all. He acknowledges that he will probably die o this cancer i he does not pursue treatment, but he says that he accepts this as his ate. He is oriented and does not appear to be manic or depressed. depressed. What should you do next? next ? a. Comply with with his reusal and schedule regular appointments appointments to pursue palliative care. b. Recommend psychiatric consultation to assess capacity b. because o his history o bipolar disease. c. Declare the patient patient incompetent incompetent because o his irrational choice and seek a court-appoin court-appointed ted guardian. d. Contact his parents and coworkers and enlist them to con vince the patient patient to pursue pursue treatment. treatment.
�� . C R O S S � C O N E N A R E A Q U E S I O N S A N D A N S W E R S
•
���
e. Schedule another appointment appointment in � week to discuss discuss the issues again afer he has had a chance to think about his situation.
��.. A ��-year �� ��-year-old -old man had a permanent pacemaker placed a year ago or symptomatic symptomatic complete heart block. He has now had a massive myocardial inarction and is responsive to only deep pain. His ejection raction is estimated estimated to be ��%, and recovery o additional myocardial unction is not expected. Te patien patientt has a living will stating that he would not want lie-sustaining treatments initiated or continued continued i his chances o recovery rom a serious illness were small. Aggressive treatments are being withheld according to the amily’s request in com pliance with the living will. Te amily now comes to you requesting that the pacemake pacemakerr be deprogrammed. What should you do? a. Comply with the request and deprogram the pacemaker. b. Give the amily the deprogrammer deprogrammer and and ask them to deprodeprogram the pacemaker. c. Request an ethics consultatio consultation. n. d. Reuse to comply with the the request because it would conconstitute actively taking the patient’s lie, and euthanasia and assisted suicide are illegal in your state. e. Reuse to comply comply with the the request because because the living living will is invalid in this circumstance.
��. A ��-year-old man is admitted to the hospital with an acute onset o severe bilateral pneumonia afer his third round o chemotherapy or diffuse large cell non-Hodgkin lymphoma. He is in respiratory ailure and consents to intubation. Afer intubation, he requires chemical paralysis to tolerate mechanical ventilation. Cultures and bronchoalveolar lavage do not identiy an inectious cause, and corticosteroids are added to broad-spectrum antibiotic therapy in case the patientt has cyclophosphamide-ind patien cyclophosphamide-induced uced pneumonit pneumonitis. is. Afer � week o treatment, a little progress is observed, although attempts to wean the patient rom chemical paralysis have ailed. At this point, the patien patient’s t’s wie and � children demand to meet with the treating physicians. Te wie and oldest son insist that use o the ventilator ventila tor be disconti discontinued nued and the patien patientt be allowed to die. Tey claim that he would not want this therapy and that it is torture or him. When queried, they indicate that there is no known advance directive and that neitherr o them has ever overtly discussed these sorts o neithe issues with the patient. At this point, the youngest son dissents and states that his ather is a �ghter and would want the the ventilator ventilator therapy to continue continue in in hopes that that he could improve. He states that his ather was always an aggressive businessman and overcame � previous cancers, lung cancer and prostate cancer. Te middle child, a daughter, is very conused and does not know what to think or what her amily would want. All again reiterate that none o them have ever discussed issues such as lie-sustaining treatment or goals o therapy with the patient. patien t. At this this point, what should the the physician do? do?
���
•
a. Negotiate Negotiate a time-limited period o continued continued aggressive intervention with the amily, and encourage them to reach a consensus. b. Stop the use o the ventilator as requested requeste d by the patient’s rightul surrogate, his wie. c. Reus Reusee to withdraw any treatment at any point because there is no advance directive. d. Request an ethics consultation. consultation. e. ell the amily members that they must obtain a court order beore use o the ventilator ventilator can be stopped.
��. A ��-year ��-year-old -old adolescent adoles cent boy is brought to the emeremergency department afer a motorcycle accident. Severe, continuous hemorrhage has decreased the patient’s hemoglobin value to �.� g/dL. His systolic blood pressure is �� mm Hg. He is unconscious. His parents arrive soon afer the ambulance and reuse or their son to receive any blood products, in keeping with their religious belies. Tey want to take him home, where they and their neighbors can pray or the boy. What should you do? a. Contact the hospital’s legal department to get a court order or transusions and operation. b. Avo Avoid id transusion and manage with crystalloid. c. Dismiss the patient to the amily against medical advice. d. Start transusions transusions immediately and prepare the the patient patient or the operation to stop the bleeding. e. Ask the hospital hospital chaplain to reason reason with the the parents. parents. P E R I O P E R AT I V E M E D I C I N E
��. A ��-year-old woman will be undergoing a lef total knee arthroplasty because o degenerative joint disease. You are asked by her orthopedist to evaluate her preoperatively. She has a history o hypertension, which is currently well controlled with medication. She also has chronic kidney disease, with a baseline creatinine value o �.� mg/dL, which is thought to be the cause o the hypertension. She does not have diabetes mellitus. She had a stroke � years ago, with no residual impairment. She does not have known coronary artery disease. Her unctional unctional capacity is limited by her knee pain. She cannot walk up a �ight o stairs without stopping. Her current medications are lisino pril, metoprolol, metopr olol, simvastatin, simv astatin, metormin, met ormin, and aspirin. Physical examination �ndings are remarkable or a blood pressure o ���/�� mm Hg and a heart rate o �� beats per minute. Her electrocardiogram shows a �rst-degree atrioventricular block and normal sinus rhythm. Te only laboratory test abnormality is the creatinine value. What should you recommend or urther preoperative testing? a. b. c. d. e.
No urther testing Dobutamine stress echocardiogram Dipyridamole thallium testing Exercise stress test ransthoracic echocard echocardiogra iogram m
MAYO CLINIC INERNAL MEDICINE BOARD REVIEW: QUESIONS AND ANSWERS
��. A ��-year-old woman is admitted to to the hospital with a hip racture, which she sustained while shoveling her driveway.. She has been in good health. She denies having driveway syncope, shortness o breath, chest pain, or palpitations beore the all. She has no history o diabetes mellitus, hypertension, coronary artery disease, or hyperlipidemia. She takes no medications. She does all her own house work, yard work, and snow shove shoveling ling with without out sympt symptoms. oms. On physical examination, her blood pressure is ���/�� mm Hg, and her heart rate is �� beats per minute and regular. Her lungs are clear. On cardiac examination, she has a coarse, ejection-type murmur at the second right intercostal border, which radiates to the carotids, and her carotid upstrokes are mildly diminished. Tere is no peripheral edema. What is the most appropriate next step in the management o this patient? a. Delay surgery until until an echocardiogram can be obtained. obtained. b. Recommend that the patient begin taking atenolol �� mg daily beore surgery. c. Cancel the surgery until a dobutamine dobutamine stress stress echocardiogram can be obtained. d. Advise the anesthesiologist anesthesiologist o the patient’ patient’ss condition. e. Send the patient to the telemetry unit or urther e. monitoring.
��. A ��-year-old man is is seen preoperatively preoperatively beore a nephrectomy or renal cell carcinoma. He has a history o chronic obstructive obstructive pulmonary disease (COPD) due to smoking, and he has had multiple unsuccessul attempts at smoking cessation. He currently smokes about �� cigarettes daily. He has a chronic cough but denies having dyspnea, and he walks � to � miles daily without symptoms. He has no other medical problems. He has had � exacerbations o COPD in the past � years, during which he was treated as an outpatient and did not require systemic corticosteroids. What is the best way to manage him preoperatively? a. Perorm spirometry and check arterial blood gases. b. Perorm ch chest est radiography, radiography, spirometry, spirometry, and a �-minute walk test. c. Perorm spirometry and check lung volumes. volumes. d. No urther urther testing testing is needed. needed. e. Delay his surgery or � weeks until until he has stopped smoking. smoking.
��.. A ��-year-old �� ��-year-o ld man is seen or preoperative medical evaluation beore elective cataract removal. He received � drug-eluting stents or symptomatic coronary artery disease � months ago. He is taking both aspirin (�� mg daily) and clopidogrel (�� mg daily). What is your advice to the patient and his eye surgeon about medication management in the perioperative period? a. Stop use o clopidogrel � days beore surgery, surgery, but continue continue use o aspirin. b. Stop use o clopidogrel and aspirin aspirin � days beore beore surgery. c. Stop use o aspirin aspirin � days beore surgery, surgery, but continue continue use o clopidogrel. d. Deer surgery or an additional additional � months. months. e. Contin Continue ue use o o both clopidogrel clopidogrel and aspirin. aspirin.
��. A ��-year-old woman has peripheral vascular disease and is scheduled or an aortoemoral bypass. Her past history includes hypertension, diabetes mellitus, and hyperlipidemia. She has no symptoms o coronary artery disease, but her activity is limited by claudication and arthritis in her knee. Current medications include metormin �,��� mg twice daily, metoprolol ��� mg twice daily, lovastatin �� mg daily, aspirin �� mg daily, and lisinopril �� mg daily. On examination, her heart rate is �� beats per minute, and her blood pressure is ���/�� mm Hg. Heart and lung examination �ndings are normal. She has no peripheral edema, but she does have markedly diminished pulses in her eet. Dipyridamole thallium testing shows a moderately sized, �xed inerior wall deect with peri-inarct ischemia and an ejection raction o ��%. Cardiac catheterization showed a ��% to ��% stenosis in the lef anterior descending coronary artery and in the right coronary artery. Which o the ollowing strategies should you recommend? a. Proceed with surger surgery, y, continuing her current medicati medications. ons. b. Reer her or coronary coronary artery bypass graf (CABG) surgery beore vascular surgery. c. Reer her or angioplasty and stent stent placement placement beore vascuvascular surgery. d. Cancel the surgery and try to manage her claudication d. medically. e. Prescribe perioperative nitrate therapy.
��. A ��-year-old man is being evaluated or a transurethral transurethral resection o the prostate or prostate cancer. He has a Starr-Edwards mitral valve and is taking long-term wararin or anticoagulation. His internatio international nal normalized ratio (INR) has been in the therapeutic range. Te urologist asks or recommendations about the perioperative management o his anticoagulation. Other medical history includes hypertension, type � diabetes mellitus, obesity, and peripheral arterial disease. Physical examination �ndings are unremarkable. His blood pressure is ���/�� mm Hg, and his heart rate is �� beats per minute. His medications include wararin, lisinopril, atenolol, metormin, hydrochlorothiazide, and atorvastatin. Preoperative laboratory studies show normal values or the complete blood cell count and electrolyte levels, creatinine �.� �.� mg/dL, and INR �.�. What is the most effective way to manage his anticoagulation? a. Stop use use o wararin wararin � days beore surgery. surgery. b. Stop use o wararin � days beore surgery, and initiate b. low-molecular-weight heparin (LMWH) therapy. c. Continue wararin therapy through the entire perioperative period. d. Stop use o wararin, wararin, and admit � days beore surgery surgery or inusion o unraction unractionated ated heparin. e. Stop use o wararin and begin aspirin therapy therapy ��� mg daily.
�� . C R O S S � C O N E N A R E A Q U E S I O N S A N D A N S W E R S
•
���� ��
��. A ��-year-old woman has degenerative joint disease in her lef knee and wants to have a total knee replacement. She has a history o type � diabetes mellitus, which is controlled with insulin. Her body mass index (BMI) is ��, and she has cirrhosis, which is thought to be rom nonalcoholic steatohepatitis. Her aspartate aminotranserase and alanine aminotranserase levels are twice the upper limit o the reerence ranges. Her Model or End-Stage Liver Disease (MELD) score is �. Her Child-urcotte-Pugh score is �. What should you recommend?
���
•
a. Proceed with surgery without any intervention. b. She should receive nonsurgical therapy since her surgical risk is prohibitive. c. She should be treated with a transjugular intrahepatic portosystemic shunt (IPS) beore surgery. d. She should receive a transplant evaluation beore surgery. e. Postpone surgery until her BMI is less than ��.
MAYO CLINIC INERNAL MEDICIN E BOARD REVIEW: QUESIONS AND ANSWER S
ANSWERS
�. Answer b.
GI workup is not indicated. She shows no signs o depression. Her blood pressure is well controlled, and there does not appear to be a good reason to discontinue the use o atenolol. Moving in with her daughter may be an option, but this would involve moving away rom a amiliar environment. Tere are many agencies that could assist this woman with her shopping and cooking needs.
Tis patient shows signs o dementia with Lewy bodies. Patients who have dementia with Lewy bodies have signs o parkinsonism, visual hallucinations, and rapid eye movement sleep disorder. Tey are extremely sensitive to neuroleptic agents, which may severely worsen the clinical course. Donepezil and therapy or Parkinson disease may be effective. MRI would not contribute anything to his management.
�. Answer b.
Tis patient has close contact with inants younger than � months. She should receive � dose o dap vaccine. She was born beore ����, so she is presumed to have immunity to measles, mumps, and rubella. She has already had � dose o pneumococcal vaccine and will not need another.
�. Answer d.
Tis patient has classic symptoms o overactive bladder with urinary urgency incontinence. Although tolterodine is likely to give the patient bene�t rom her symptoms, there are other, nonpharmacologic treatments that should be tried beore medications. opical estrogen may bene�t some patients with overactive bladder, but the evidence supporting bene�t is weak. Generally, the best approach starts with the least aggressive measures and then leads to more invasive, more expensive, and higher-risk procedures. Since this patient has overactive bladder associated with urinary urgency incontinence, bladder suspension surgery is unlikely to give her any bene�t; that procedure is or stress urinary incontinence. Although a reerral to an urologist is not inappropriate, there are certainly several treatment measures that the primary caregiver could attempt �rst. Tis patient may be a candidate or timed voiding, which is ofen successul in nursing homes. Te patient is toileted every � hours or at a requency determined by the patient’s voiding diary. Tis is a simple, noninvasive treatment option that effectively decreases the number o daily incontinence episodes.
�. Answer d.
Te US Preventive Services ask Force recommends hypertension screening or all adults. Screening or carotid artery stenosis and chronic obstructive pulmonary disease is not recommended or asymptomatic patients who have no risk actors. Tis patient has no signs or symptoms o coronary artery disease, and an exercise stress test is not indicated. Colonoscopy in patients older than �� is controversial and probably not indicated or patients who have had negative colonoscopy �ndings in the past. �. Answer b.
Tis patient may ultimately need to enter a nursing home, but that situation would be distressing or both the patient and his wie. Nonpharmacologic therapies such as music therapy and aromatherapy have been shown to be successul or patients who have moderate to severe dementia. It is unclear how long the effect will last, but those therapies should probably be tried beore nursing home placement. Medications have only limited success in treating behavioral symptoms o dementia, and nonpharmacologic therapy should be tried �rst.
�. Answer c.
Vest restraints ofen cause or worsen delirium and should not be used except in extreme circumstances. Benzodiazepines may also cause or worsen delirium. Haloperidol may be appropriate or short-term management o behavioral symptoms. Orienting strategies, amiliar objects and peo ple, and normalizing the sleep-wake cycle help to decrease the severity and shorten the duration o delirium.
�. Answer c.
For a clean minor wound, revaccination with d is required i the latest vaccination was more than �� years ago. However, or a dirty deep wound, revaccination with d is required i the latest vaccination was more than � years ago (able ��.A�). (See Centers or Disease Control and Prevention, ���� in the “Suggested Reading” list.)
�. Answer d.
Many elderly patients exhibit signs o railty or socioeconomic reasons. Tis patient has no GI symptoms, and a
���
Table ��.A� EANUS PROPHYLAXIS FOR CLEAN MINOR WOU ND S
EA NUS PRO PHY LA XIS FO R A LL OHE R W OUN DS
Tetanus-Diphtheriaa
Tetanus Immune Globulin
Tetanus-Diphtheriaa
Tetanus Immune Globulin
Unknown or <� doses
Yes
No
Yes
Yes
≥� doses
Nob
No
Noc
No
VACCIN AION HIS ORY
a
etanus-diphtheria-acellular pertussis (dap) may be substituted or tetanus-diphtheria i the person has not previously received dap.
b
Yes, i the latest dose was given more than �� years ago.
c
Yes, i the latest dose was g iven more than � years ago.
Adapted rom Centers or Disease Control and Prevention. Epidemiolog y and Prevention o Vaccine-Preventable Diseases. ��th ed. Washington DC : Public Health Foundation; c����. Chapter ��, etanus; p. ���–���.
�. Answer e.
Te Advisory Committee on Immunization Practices recently released an updated recommendation or dap vaccination: or adults �� years or older who have not pre viously received dap, a single dose o dap may be given instead o d vaccine. dap can be administered regardless o the interval since the latest tetanus vaccine. Te US Preventive Services ask Force recommends screening or AAA in men aged �� to �� who have smoked more than ��� cigarettes. Pneumococcal vaccination is not routinely repeated or otherwise healthy patients. Zoster vaccine live is administered once to adults who are �� or older; they do not require another vaccination. (See Centers or Disease Control and Prevention [CDC], ���� in the “Suggested Reading” list.) ��. Answer b.
Te American Cancer Society, the US Multisociety ask Force on Colorectal Cancer, and the American College o Radiology developed consensus guidelines or colon cancer screening. Accordingly, persons who have a �rst-degree relative who had colon cancer or adenomatous polyps beore age �� should start colonoscopy at age ��, or �� years beore the youngest case in the immediate amily, whichever is earlier. Tereore, this patient should start screening at age ��. (See Levin et al in the “Suggested Reading” list.) ��. Answer a.
Pneumococcal vaccination is recommended or adults aged �� to �� who have asthma or who are smokers. HPV vaccination is recommended or women until age ��. In�uenza vaccination is now recommended or everyone aged � months or older. (See Centers or Disease Control and Prevention; Advisory Committee on Immunization Practices and Fiore et al in the “Suggested Reading” list.) ��. Answer e.
Te US Preventive Services ask Force updated its recommendations on mammography in December ����: Te decision to start regular, biennial screening mammography beore the age o �� years should be an individual decision that accounts or patient context, including the patient’s values related to speci�c bene�ts and harms. (See ���
•
Screening or Breast Cancer, topic page in the “Suggested Reading” list.) ��. Answer e.
Te Centers or Disease Control and Prevention recommends that all persons between �� and �� years o age be tested or HIV regardless o recognized risk actors. Te US Preventive Services ask Force makes no recommendation or or against routinely screening or HIV in adults who are not at increased risk o HIV inection ( grade C recommendation). Given that this patient is otherwise healthy and has ew cardiac risk actors, he does not need asting blood glucose or lipid screening. Lipid screening is recommended at age �� or otherwise healthy men. Blood glucose screening is recommended or individuals with hypertension. A blood chemistry panel is not recommended as a routine screening test. (See Branson et al in the “Suggested Reading” list.) ��. Answer c.
Te US Preventive Services ask Force (USPSF) recommends against routine screening with resting ECG, C scanning or coronary calcium, and exercise treadmill testing. Te USPSF also concludes that there is insu�cient evidence to balance the risks and bene�ts o using nontraditional risk actors such as hs-CRP testing. C coronary calcium scanning and hs-CRP testing can be considered or asymptomatic individuals i their ��-year risk o a cardiovascular event is between ��% and ��%. Te patient in this question has a Framingham risk score o only �%. (See Screening or Coronary Heart Disease With Electrocardiography, topic page and Using Nontraditional Risk Factors in Coronary Heart Disease Risk Assessment, topic page in the “Suggested Reading” list.) ��. Answer c.
Te ���� guidelines or a postmenopausal woman with vaginal atrophy recommended short-term vaginal estrogen treatment and a ollow-up Pap test, which generally would yield normal �ndings. Tose guidelines, however, recommended that immunosuppressed women undergo colposcopy immediately because they had a higher risk o neoplasia. More recent guidelines (����) offer the same
M A YO C L I N I C I N E R N A L M E D I C I N E B O A R D R E V I EW : Q U E S I ON S A N D A N S W E R S
recommendations or postmenopausal women, immunosuppressed women, and women in the general population because additional data have shown that women with ASC-US have a low prevalence o invasive cancer. Loop electrosurgical excision is not appropriate or any population with ASC-US. HPV DNA testing and colposcopy are also reasonable choices or this patient (and the general public) but choice c is the least invasive and most cost-effective. Additional management guidelines or dierent populations and higher-grade cytologic �ndings appear in the article by Wright et al (see the “Suggested Reading” list). ��. Answer e.
Although bilateral mastectomy and oophorectomy may be a reasonable choice, this patient should be offered genetic testing because her mother is a known mutation carrier. I the patient declines, prophylactic surgery or more intense surveillance would be a reasonable option. Intense surveillance or a BRCA mutation carrier or or a �rst-degree relative o a carrier should include annual breast magnetic resonance imaging in addition to mammography, biannual clinical breast examination, annual gynecologic examination, biannual or annual testing or cancer antigen ���, and pelvic ultrasonography with appropriate counseling on the bene�ts, limitations, and potential harms o each test. Raloxiene would not be appropriate or � reasons: �) Tere are no saety data rom premenopausal or perimenopausal women, so it is indicated only or postmenopausal women, and this patient is perimenopausal. �) Tere are no data assessing whether raloxiene decreases breast cancer incidence among BRCA mutation carriers or women suspected o being BRCA mutation carriers. amoxien may be an option, but the data on tamoxien in BRCA� mutation carriers are not extensive and are inconclusive; however, the data that are available suggest a bene�t. (See Nelson et al in the “Suggested Reading” list.) ��. Answer d.
Although pregnancy is the most common cause o secondary amenorrhea and must always be considered and excluded as a cause o amenorrhea (secondary or primary), prolactinoma is the cause o secondary amenorrhea in � o � women. Tis patient does not complain speci�cally o diplopia, but she reports visual changes that she attributes to stress and lack o sleep and may not realize that she is experiencing diplopia. Te possibility o a prolactinoma should not be overlooked. Further, the peak incidence o prolactinomas is in the third to �fh decades o lie and is most common in women in that age range. esting or hypothyroidism should be considered i the clinical presentation suggests that hypothyroidism is possible. Although this patient has gained weight and is having diffi culty losing it, there are other reasons or her weight gain; weight gain due to hypothyroidism usually occurs with other symptoms and at an older age. (See Mindermann and Wilson in the “Suggested Reading” list.)
��. Answer a.
Te HPV vaccine is an inactivated viral vaccine that is likely sae (pregnancy category B), but it has not been studied well in this population, and current guidelines recommend against its use during pregnancy. Women inadvertently vaccinated during pregnancy should have their exposure to the vaccine reported to the Vaccine in Pregnancy Registry. Te other vaccines listed as answer choices are known to be sae during pregnancy. Hepatitis A, pneumococcal, and meningococcal vaccines should be administered to pregnant women who are considered to be at high risk rather than to all pregnant women. d should be administered to pregnant women whose latest booster was �� or more years earlier or who have a deep or dirty wound. (See Pickering et al in the “Suggested Reading” list.) ��. Answer a.
Te patient’s nonbloody nipple discharge is consistent with galactorrhea. With otherwise normal results or the examination, or the SH and PRL levels, and or the breast imaging and with no objective evidence o blood in the �uid, there is no need or urther breast imaging or any other testing. Tere is also no need or a surgical evaluation. Given the normal PRL level, the galactorrhea is not indicative o a prolactinoma and pituitary MRI is unnecessary. I there is uncertainty as to whether the secretion is milk, a at stain could be done, but this is rarely necessary. In addition to reassurance, the patient should be instructed to discontinue checking or discharge so requently to avoid stimulating the breast tissue to continue �uid production. (See Hussain et al in the “Suggested Reading” list.) ��. Answer c.
Although the sensitivity o the whiff test is relatively low, this patient’s vaginal pH is normal, effectively excluding bacterial vaginosis. Large rod-shaped bacilli in the vaginal secretions represent normal vaginal �ora (lactobacilli) and are not indicative o inection. No motile species are noted on microscopy, so Trichomonas is unlikely and the negative culture results con�rm the absence o Trichomonas. Te patient should eliminate potential irritants (laundry detergent, douching, vulvar sprays, ragranced cleansers, etc) and be reassessed i her symptoms do not resolve. (See Laine et al in the “Suggested Reading” list.) ��. Answer e.
Hot �ushes and night sweats (vasomotor symptoms) are the most requent menopause-related complaints. Although symptoms generally decrease over time, they persist or up to �� years in up to ��% o women, with ��% reporting symptoms or even longer. Vaginal atrophy and dryness due to menopause are also common symptoms. Tis patient’s quality o lie is signi�cantly affected by her vasomotor symptoms and vaginal symptoms. Since she has no signi�cant risk actors or the use o combination hormone therapy (breast cancer, smoking, or coronary artery disease), it is the best choice; combination hormone
�� . C R O S S � C O N E N A R E A Q U E S I O N S A N D A N S W E R S
•
���
therapy will address vaginal and vasomotor symptoms. Since she has an intact uterus, unopposed estrogen cannot be used because o the risk o uterine malignancy. I this patient’s symptoms had been primarily vaginal, vaginal estrogen alone would have been an appropriate choice. Some data have shown bene�t rom antidepressants (eg, venlaaxine) or vasomotor symptoms, but her symptoms are signi�cant and venlaaxine would not help the vaginal dryness. Tere is insuffi cient evidence or the effectiveness o herbal remedies. Given the severity o her symptoms and the negative effect on her quality o lie, waiting longer to see i her symptoms improve is not a reasonable choice. Te lowest effective dose o hormone therapy or the shortest duration in low-risk women is recommended, and its use should be reassessed at least annually. Initiation o hormone therapy should be avoided or women who smoke, have vascular disease, have a history o breast cancer, or are well past menopause. (See Utian et al in the “Suggested Reading” list.) ��. Answer a.
Most palpable breast masses are benign, but evaluation is necessary to exclude malignancy, even with a normal mammogram and the absence o any breast cancer risk actors. O the palpable malignancies, ��% to ��% are mammographically occult. In a woman older than ��, the likeliest cause is a �broadenoma, a cyst, �brocystic change, or hyperplasia. Evaluation should include mammography and ultrasonography with a strong consideration or biopsy o a solid mass (�ne-needle aspiration, core needle biopsy, or excision). Ultrasonography reliably distinguishes solid masses rom cystic lesions. A benign biopsy in a woman with a negative mammogram does not preclude careul ollow-up to assess or long-term stability. In women younger than ��, evaluation should begin with ultrasonography; i the results are abnormal, mammography should be perormed. In this situation, it is also acceptable to allow � or � menstrual cycles to pass beore reassessing clinically and proceeding with imaging i the palpable mass persists. Diffuse lumpiness or nodularity is usually rom �broc ystic change. Discrete cysts occur most ofen in women younger than ��. Both diffuse lumpiness and discrete cysts may be tender and can �uctuate with the menstrual cycle. Cystic masses should be aspirated; i the �uid is bloody or the cyst is recurrent afer prior aspiration, the �uid should be evaluated cytologically. Any palpable mass in a postmenopausal woman requires immediate evaluation to exclude malignancy without a period o observation. One o the most common causes o litigation is ailure to diagnose breast cancer. (See Health Care Guideline: Diagnosis o Breast Disease in the “Suggested Reading” list.) ��. Answer a.
Tis is a classic history o cyclic breast pain associated with the luteal phase o the menstrual cycle. ypically, women describe burning, aching pain and a heavy sensation that is most ofen located in the upper, outer quadrants and ofen radiates into the axillae and sometimes the upper ���
•
arm. When women keep a monthly chart, they note that the pain develops during the � weeks beore menstrual bleeding; afer menstruation starts, the pain decreases and typically resolves completely between cycles. For a woman who provides this history and has unremarkable clinical examination �ndings, no diagnostic testing is necessary. I a woman is eligible or a screening mammogram and i she is overdue or screening, an up-to-date screening mammogram should be obtained. Otherwise, mammography is not necessary or evaluation o this type o breast pain. ypically, reassurance and education are sufficient. A well-�tting support bra (eg, a sports bra) is very helpul; or some women, decreasing the intake o caffeine and salt can be helpul, although the evidence is inconsistent. In addition, there is some inconsistent evidence or the use o supplements. Despite multiple studies, the underlying causes are unde�ned. Breast pain can be characterized as cyclic, noncyclic, and extramammary. Noncyclic mastalgia is not associated with the menstrual cycles and is usually constant or intermittent. Tere are multiple causes o noncyclic mastalgia, and, although it is uncommon or breast cancer to maniest with only pain, ocal and persistent pain must be evaluated to exclude an underlying malignancy. Noncyclic breast pain usually occurs at a postmenopausal age, but it may occur earlier, and it is usually unilateral with an anatomical cause that can be identi�ed. In addition, multiple medications have been associated with noncyclic breast pain. Te cause o extramammary pain commonly involves the underlying chest wall, but other conditions may cause extramammary pain (eg, ischemia, pulmonary disease, thoracic nerve root irritation) and should be considered. Evaluation must include a careul history (including quality, location, severity, aggravating and alleviating actors, and the effect on quality o lie). Medications should be careully reviewed (including any supplements and herbs). A thorough clinical examination should include the chest wall. Focal, persistent pain should be evaluated with ultrasonography, and mammography should be included or women �� or older. Any solid mass should be evaluated with a biopsy. Afer exclusion o malignancy and other causes that require speci�c treatment, multiple strategies or management may be considered. Severe pain may warrant treatment, but ofen other measures are helpul, and medications can be avoided in most cases. Reassurance, a supportive bra, and discontinuation o an offending medication or supplement (i possible) are ofen sufficient. aking essential atty acids, vitamin E, and evening primrose oil or decreasing or eliminating the intake o salt and methylxanthines (eg, caffeine) may be helpul or some women, although the evidence is con�icting , and in multiple studies there was no bene�t compared with placebo. Multiple medications have been studied and are effective or the treatment o severe mastalgia, including danazol (US Food and Drug Administration approved), bromocriptine, nonsteroidal anti-in�ammatory drugs, and tamoxien. However, the side effects ofen cause patients
M A YO C L I N I C I N E R N A L M E D I C I N E B O A R D R E V I EW : Q U E S I ON S A N D A N S W E R S
to discontinue use, and medications are rarely necessary with other measures. (See Smith et al in the “Suggested Reading” list.) ��. Answer b.
Cytomegalovirus is the most common cause o congenital viral inection. Te lietime risk o ovarian cancer or a BRCA� mutation carrier is approximately ��%. Dysunctional uterine bleeding is a diagnosis o exclusion; thereore, abnormal uterine bleeding must be evaluated beore this benign diagnosis is made. Annual screening mammograms have been shown to decrease mortality related to breast cancer, but there are no such data or clinical breast examinations. ��. Answer c.
A �� table must be constructed to determine the positive predictive value or MRI among patients who have a pretest probability o ��% in an evaluation or oot osteomyelitis. As determined rom able ��.A��, the correct a answer is or ��/��� = ��%. a b
As discussed in the answer to question ��, the absolute risk reduction (ARR) is �%. Now you are asked to determine the NN in this randomized controlled trial. Te ormula 1 or NN is . Tereore, NN = = ��. Tat is, ARR 00 �� elderly people with atrial �brillation would need to be treated with anticoagulant therapy or � years to prevent � event. ��. Answer d.
Te super�cial emoral vein is in the deep venous system; thereore, this patient needs to be treated with heparin and wararin, overlapped or � days, until the international normalized ratio is greater than �.�. Outpatient treatment with LMWH has been approved by the US Food and Drug Administration or the treatment o deep vein thrombosis. Te patient has a good support system at home; thereore, it would be appropriate or her to be treated as an outpatient. Since she has no symptoms to suggest pulmonary embolism, C o the chest is not indicated. ��. Answer d.
Table ��.A�� DISEASE PRESEN
Positive Diagnostic est Result
��. Answer c.
DISEASE AB SEN
��
Negative
�� a b
a+b
c d
c+d
��
�� a+c b+d
���
���
�� a+b+c+d
���
���
Te patient has a clot in the popliteal system. Tis is considered a deep vein thrombosis (DV) and does need anticoagulation. Te patient is not at high risk o bleeding � week postoperatively; thereore, he does not need a vena caval �lter. Dabigatran is not approved by the US Food and Drug Administration or DV treatment. Tree months o treatment is adequate. Te dosage o enoxaparin in the outpatient setting is � mg/kg twice daily or �.� mg/kg once daily; or this ��-kg patient, ��� mg subcutaneously once daily would be appropriate. ��. Answer c.
��. Answer b.
Te negative predictive value or MRI is or ��/�� = ��%.
d c
��. Answer d.
You are asked to determine the RRR in a randomized controlled trial that showed that the anticoagulant therapy group had an experimental event rate (EER) o �%. Te aspirin group had a control event rate CER EE (CER) o ��%. Te ormula or RRR is . CER 10% − 5% In this trial, RRR = = ��%. 10% ��. Answer a.
You are asked to determine the ARR in a randomized controlled trial that showed that the anticoagulant therapy group had an experimental event rate (EER) o �%. Te aspirin group had a control event rate (CER) o ��%. Te ormula or ARR is CER − EER. In this trial, ARR = ��% − �% = �% over � years.
�uality improvement interventions have a hierarchy or sustainability. Education tools, such as online modules, conerences, pocket cards, and signs, tend to have a positive initial effect, but this effect is not lasting. Although an institutional consensus or policy may be a good idea, it is difficult to implement and develop consistent adherence. In this question, the order set is most likely to provide a sustained improvement. I it is integrated into a computerized order entry system, it will prompt physicians about correct dosing and thus decrease the risk o human error. ��. Answer b.
Tis patient has episcleritis, which is a sel-limited, possibly autoimmune-mediated in�ammation o the episcleral vessels. Although vision is unaffected and tearing may be minimal, rapid-onset redness associated with achy pain and tenderness is evident. In this scenario, one would also want to rule out the possibility o temporal arteritis, although it does not maniest with eye symptoms. I symptoms persist or recur, the patient should be reerred to an ophthalmologist.
�� . C R O S S � C O N E N A R E A Q U E S I O N S A N D A N S W E R S
•
���
��. Answer c.
Tis patient likely has gonorrhea and gonococcal conjunctivitis. Immediate ophthalmologic reerral is advised or treatment with topical antibiotics (bacitracin or cipro�oxacin) and systemic antibiotics (cefriaxone � g intramuscularly). Sexual symptoms and contacts should be reviewed and treated appropriately. ��. Answer d.
On the basis o the Centor criteria (ever, tonsillar exudates, tender anterior cervical lymphadenopathy, and lack o cough), this patient has a high probability o Group A β-hemolytic streptococcal (GABHS) pharyngitis. Guidelines rom the Inectious Diseases Society o America recommend rapid streptococcal antigen testing or adults with suspected GABHS inection (ie, � Centor criteria) and no testing or treatment or those at low risk (� or � Centor criterion). Patients at high risk (� or � Centor criteria) should receive empirical antibiotic therapy. ��. Answer c.
Both acute angle-closure glaucoma and acute anterior uveitis are emergent processes associated with rapid onset, circumcorneal injection, pain, and unilateral maniestation. Acute angle-closure glaucoma is associated with a tense affected eye on palpation in an older patient; acute anterior uveitis usually maniests in a young or middle-aged adult and is associated with the presence o in�ammatory cells and protein in the anterior chamber o the eye. Tis collection o pus (hypopyon) may occasionally be observed in severe cases o acute anterior uveitis. ��. Answer e.
You have already taken a good history and examined the areas most likely to physically contribute to ED. Te onset o symptoms was gradual in a patient who has excellent health and who likely has ED due to age-related changes. Tere is no need or MRI o the brain to rule out pituitary disease since his testosterone level and testicular examination are normal. PDE-� inhibitors are accepted as �rst-line therapy or patients in this category. I his symptoms persist, consultation with a sex psychotherapist would be reasonable to evaluate the possibility o anxiety related to sexual perormance or loss o sensate ocus (mental distraction). (See Beckman et al in the “Suggested Reading” list.) ��. Answer a.
��. Answer a.
Tis patient has renal ailure and a prolonged Q inter val. Fentanyl does not affect the Q and can be used with caution and at lower doses in renal ailure. Methadone prolongs the Q interval but can be used in dialysis. Propoxyphene, codeine, and meperidine should be avoided because they have metabolites that can accumulate in dialysis patients. ��. Answer e.
From all appearances, the patient is decisionally capable. Answer choice a is not completely inappropriate. Given that this is the �rst time the patient is conronting a horrible diagnosis, he may need to take some time to process the inormation and could change his mind. Answer choice b points to a very important issue—the patient’s capacity in light o his signi�cant psychiatric history. Ultimately, psychiatric consultation might be an important step, but it is probably not the best �rst response. Since the patient has just received horrible news, it might not be the best time to remind him o his psychiatric issues. Rather, it is best to give him time to re�ect and continue the conversation in the near uture. Answer choice d is not appropriate because such action would violate patient con�dentiality unless you received speci�c consent rom the patient to make such contacts. ��. Answer a.
Granting a request to discontinue a orm o lie-sustaining therapy is not the same as euthanasia or assisted suicide. In both euthanasia and assisted suicide, the cause o death is the lethal agent administered by the physician. With deactivation o a pacemaker, the cause o death is the underlying conduction deect or the pathologic disorder. Respect or patient autonomy—in this case, the right to be ree rom unwanted medical intervention—underpins the right to reuse or to request the withdrawal o treatments no longer serving the patient’s goals o care. Tere is no ethical or legal distinction between withholding a treatment and withdrawing it afer it has been initiated. Both are expressions o the patient’s desire to be lef alone, ree rom “unwanted touching.” ��. Answer a.
LEAN is a quality improvement methodology developed by oyota Motor Corporation to provide value by reducing waste. In medicine, wait times are the most common orm o waste. Six Sigma is an improvement methodology developed by Motorola to reduce deects. A cause-and-effect diagram (also known as a �sh bone diagram because o its shape) is a tool to complete and organize a root cause analysis. Swim lane diagramming is a quality improvement tool used to visually show who is responsible or various processes within a system. Failure mode and effects analysis ���
is a tool or classiying errors by severity and likelihood o recurrence or use in prioritizing quality initiatives.
•
It is not at all clear what the patient would want at this point. Te last directive rom the patient was that he wanted treatment. At the same time, the patient may indeed be in an irreversible situation in which eventual withdrawal o use o the ventilator would be appropriate. Te goals are to try to do what is best or the patient and to demonstrate a good-aith commitment to work with the amily. A reasonable time limit on the aggressive treatment could accom plish both and allow the amily members to resolve their differences.
M A YO C L I N I C I N E R N A L M E D I C I N E B O A R D R E V I EW : Q U E S I O N S A N D A N S W E R S
��. Answer d.
��. Answer a.
Te patient is a minor; thereore, under presumed consent, lie-sustaining interventions are to be pursued even in the ace o parental reusal. Although surrogates can reuse lie-sustaining therapy or adults or whatever reason, parents have not been allowed to reuse appropriate medical treatment, especially lie-sustaining treatment, o minors or religious convictions. In such cases, physicians are obligated to obtain a court order appointing a guardian and permitting the lie-sustaining treatment. In this case, there is not time to receive a court order beore intervening, so it is most appropriate to pursue the liesaving treatment while the legal issues are sorted out. ��. Answer a.
No urther testing is required. Although this patient is at increased risk or coronary artery disease because o her history o hypertension and chronic kidney disease, she does not have known coronary artery disease. Her hypertension is well controlled, and her heart rate is �� beats per minute. American College o Cardiology and American Heart Association guidelines suggest that patients who take β-blockers and have good heart rate control do not need urther noninvasive testing. ��. Answer d.
Tis patient has signs o mild to moderate aortic stenosis and no symptoms. She has an excellent unctional status and shows no symptoms o aortic stenosis. Perioperative β-blocker therapy is not indicated or patients with aortic stenosis. Tere is no reason to delay surgery to obtain an echocardiogram or to do a unctional stress test, since she easily achieves � metabolic equivalent tasks o work and is asymptomatic. Spinal anesthesia is relatively contraindicated in patients with aortic stenosis because o the risk o induced hypotension, so the anesthesiologist must be told o her condition. ��. Answer d.
Tis patient clearly has COPD, but his unctional status is good. Pulmonary unction tests are not indicated or preoperative pulmonary assessment unless the patient is having new symptoms. Te patient’s condition appears to be stable, and he does not need any urther testing. Although he would certainly bene�t rom smoking cessation, there is no reason to delay his surgery to accomplish this. ��. Answer e.
Cataract surgery can be done saely i a patient is taking anti platelet drugs, although there are ew data or clopidogrel. Stopping use o either aspirin or clopidogrel would put the patient at high risk o stent thrombosis in the time rame outlined. Deerring surgery would be an option, but it would require � months o additional time to allow the patient to receive both antiplatelet drugs or a ull �� months. At that time, clopidogrel therapy could be saely stopped or surgery, but aspirin therapy would be continued.
Tis patient has a reduced unctional status, and noninvasive stress testing preoperatively is appropriate, given that she will undergo a high-risk surgical procedure. Her cardiac catheterization shows noncritical disease. Her stress test results would be considered low-risk positive because she has only peri-inarct ischemia. Perioperative β -blocker therapy is appropriate or her, and her goal heart rate should be �� to �� beats per minute. Tere is no role or preoperative CABG surgery since she has noncritical disease. Perioperative nitrates are not indicated or a patient who is not having angina. ��. Answer b.
Tis patient’s Starr-Edwards valve presents a high risk o thromboembolism. In addition, his surgical procedure coners a high risk o bleeding. Simply stopping the use o wararin is inappropriate because o the thromboembolic risk. However, because o the bleeding risk, he should receive an agent that can be easily reversed— either unractionated heparin, which would require hos pitalization and would increase his risk o bleeding, or LMWH. Te most appropriate choice in this situation, because o cost and ease o administration, is LMWH. His creatinine level is in the reerence range, so there is no contraindication to the use o LMWH. Aspirin would not be sufficient to prevent thromboembolism in this situation. ��. Answer a.
Tis patient has relatively well-compensated liver disease and a low MELD score. A MELD score less than �� is associated with a low risk o perioperative death. Tere is no role or IPS in this situation. SUGGESED R EADING Beckman J, Abu-Lebdeh HS, Mynderse LA. Evaluation and medical management o erectile dysunction. Mayo Clin Proc. 2006 Mar;81(3):385–90. Branson BM, Hands�eld HH, Lampe MA, Janssen RS, aylor AW, Lyss SB, et al; Centers or Disease Control and Prevention. Revised recommendations or HIV testing o adults, adolescents, and pregnant women in health-care settings. MMWR Recomm Rep. 2006 Sep 22;55(RR-14):1–17. Centers or Disease Control and Prevention. Epidemiology and Prevention o Vaccine-Preventable Diseases. 12th ed. Washington DC: Public Health Foundation; c2012. Chapter 20, etanus; p. 291–300. Centers or Disease Control and Prevention (CDC). Updated recommendations or use o tetanus toxoid, reduced diphtheria toxoid and acellular pertussis (dap) vaccine rom the Advisory Committee on Immunization Practices, 2010. MMWR Morb Mortal Wkly Rep. 2011 Jan 14;60(1):13–5. Centers or Disease Control and Prevention; Advisory Committee on Immunization Practices. Updated recommendations or prevention o invasive pneumococcal disease among adults using the 23-valent pneumococcal polysaccharide vaccine (PPSV23). MMWR Morb Mortal Wkly Rep. 2010 Sep 3;59(34):1102–6. Fiore AE, Uyeki M, Broder K, Finelli L, Euler GL, Singleton JA, et al; Centers or Disease Control and Prevention. Prevention and
�� . C R O S S � C O N E N A R E A Q U E S I O N S A N D A N S W E R S
•
���
control o in�uenza with vaccines: recommendations o the Advisory Committee on Immunization Practices (ACIP), 2010. MMWR Recomm Rep. 2010 Aug 6;59(RR-8):1–62. Errata in: MMWR Recomm Rep. 2010 Aug 13;59(31):993. MMWR Recomm Rep. 2010 Sep 10;59(35):1147. Health Care Guideline: Diagnosis o Breast Disease [Internet]. Bloomington (MN): Institute or Clinical Systems Improvement; c2012. Available rom: http://www.icsi.org/breast_disease_diagnosis/diagnosis_o_breast_disease_2.html. Hussain AN, Policarpio C, Vincent M. Evaluating nipple discharge. Obstet Gynecol Surv. 2006 Apr;61(4):278–83. Laine C, Williams D, Wilson JF. Vaginitis and cervicitis. Ann Intern Med. 2009 Sep;151(5):IC3–1. Levin B, Lieberman DA, McFarland B, Smith RA, Brooks D, Andrews KS, et al; American Cancer Society Colorectal Cancer Advisory Group; US Multi-Society ask Force; American College o Radiolog y Colon Cancer Committee. Screening and surveillance or the early detection o colorectal cancer and adenomatous polyps, 2008: a joint guideline rom the American Cancer Society, the US Multi-Society ask Force on Colorectal Cancer, and the American College o Radiology. CA Cancer J Clin. 2008 May-Jun;58(3):130–60. Epub 2008 Mar 5. Mindermann , Wilson CB. Age-related and gender-related occurrence o pituitary adenomas. Clin Endocrinol (Ox ). 1994 Sep;41(3):359–64. Erratum in: Clin Endocrinol (Ox). 1994 Nov;41(5):700. Nelson HD, Huffman LH, Fu R, Harris EL; U.S. Preventive Services ask Force. Genetic risk assessment and BRCA mutation testing or breast and ovarian cancer susceptibility: systematic evidence review or the U.S. Preventive Services ask Force. Ann Intern Med. 2005 Sep 6;143(5):362–79. Erratum in: Ann Intern Med. 2005 Oct 4;143(7):547.
���
•
Pickering LK, Baker CJ, Freed GL, Gall SA, Grogg SE, Poland GA, et al; Inectious Diseases Society o America. Immunization programs or inants, children, adolescents, and adults: clinical practice guidelines by the Inectious Diseases Society o America. Clin Inect Dis. 2009 Sep 15;49(6):817–40. Erratum in: Clin Inect Dis. 2009 Nov 1;49(9):1465. Screening or Breast Cancer, opic Page [Internet]. Rockville (MD): U.S. Preventive Services ask Force; c2010 [cited 2011 Mar 23]. Available rom: http://www.uspreventiveservicestaskorce.org/uspst/uspsbrca. htm. Screening or Coronary Heart Disease With Electrocardiography, opic Page [Internet]. Rockville (MD): U.S. Preventive Services ask Force; c2012 [cited 2011 Mar 23]. Available rom: http://www.uspreventi veservicestaskorce.org/uspst/uspsacad.htm. Smith RL, Pruthi S, Fitzpatrick LA. Evaluation and management o breast pain. Mayo Clin Proc. 2004 Mar;79(3):353–72. Using Nontraditional Risk Factors in Coronary Heart Disease Risk Assessment, opic Page [Internet]. U.S. Preventive Services ask Force; c2009 [cited 2011 Mar 21]. Available rom: http://www.uspre ventiveservicestaskorce.org/uspst/uspscoronaryhd.htm. Utian WH, Archer DF, Bachmann GA, Gallagher C, Grodstein F, Heiman JR, et al; North American Menopause Society. Estrogen and progestogen use in postmenopausal women: July 2008 position statement o Te North American Menopause Society. Menopause. 2008 Jul-Aug;15(4 Pt 1):584–602. Wright C Jr, Massad LS, Dunton CJ, Spitzer M, Wilkinson EJ, Solomon D; 2006 American Society or Colposcopy and Cervical Pathology-sponsored Consensus Conerence. 2006 Consensus guidelines or the management o women with abnormal cervical cancer screening tests. Am J Obstet Gynecol. 2007 Oct;197(4):346–55.
M A YO C L I N I C I N E R N A L M E D I C I N E B O A R D R E V I EW : Q U E S I O N S A N D A N S W E R S
INDEX
Page numbers ollowed by f indicate a �gure; page numbers ollowed by t indicate a table
A abdomen active bowel sounds, 45 C scan, 94, 97 distention, 30 hyperactive bowel sounds, 70 tenderness (pain), 27, 29, 45, 70, 84, 109 abdominal aortic aneurysm, 14, 16, 23 abdominal hernia repair, 46 abdominal radiotherapy, 29 abdominal wall cellulitis, 50 ABIM Maintenance o Certi�cation (MOC) Examination, 1–8, 4 t ABVD combination chemotherapy, 87 acarbose, 74 acetaminophen, 17, 30, 34, 60, 65 achalasia, 32 Achilles tendonitis (enthesopathy), 61, 65 Achilles tendon swelling, 61 acid-based disorders answers, 102 questions, 94–97 acute angle-closure glaucoma, 129, 138 acute anterior uveitis, 129 acute chest syndrome, 89 acute eosinophilic pneumonia, 42 acute interstitial nephritis, 103 acute ischemic cerebral inarction, 116 acute ischemic stroke, 41 acute myelogenous leukemia (AML), 49, 55, 83, 89 acute pancreatitis, 31 acute renal ailure answers, 102–103 questions, 97 acute respiratory acidosis, 96 acute respiratory alkalosis, 96 acute respiratory distress syndrome, 42 acute thrombosis, 90 acute tubular necrosis, 103 acyclovir, 49 adalimumab, 65, 125 adenocarcinoma rectal, 80 stage I lesion, 129 adenocarcinoma o unknown primary origin, 79 adult pulmonary Langerhans cell histiocytosis, 42 Advisory Committee on Immunization Practices, 134 AL amyloidosis, 102 albuterol inhaler, 39, 80, 105, 107 alcoholic cirrhosis, 38 alendronate, 37 aliskiren, 17 alkaline phosphatase elevation, 29 allergic bronchopulmonary aspergillosis, 42 allergic rhinitis, 106 allergic rhinoconjunctivitis, 107
allergies answers, 107–108 questions, 105–106 allopurinol therapy, 61, 65, 100, 101 Alzheimer disease, 124 amenorrhea, 71 amenorrhea, secondary, 135 amitriptyline, 111 amlodipine, 99 amoxicillin, 55 amoxicillin-clavulanic acid, 53 amyopathic dermatomyositis, 120 anasarca, 100 anemia o chronic disease, 89–90 anemias answers, 89–90 autoimmune hemolytic anemia, 89 hemolytic anemia, 83, 103 iron de�ciency anemia, 28, 31, 32 macrocytic anemia, 89 megaloblastic anemia, 89 normochromic anemia, 59 questions, 83–85 sickle cell anemia, 47, 89 warm autoimmune hemolytic anemia, 90 angina, progressive, 15 angioedema, recurrent, 105 angioplasty, 72 anion gap acidosis, 103 ankle edema, bilateral, 98 ankle re�ex, absence, 68 ankle swelling, 61 ankylosing spondylitis, 61, 65 anorexia nervosa, 79 anterior myocardial inarction, 14–15 anthrax, inhalation, 53 antiarrhythmic therapy, 67 anti-CD20 monoclonal antibody, 91 anticoagulation therapy recommendation, 9 antihistamines, 105 nonsedating, 107 antimicrobial therapy, 45, 46 antipsychotics, second-generation, 109, 111 antirheumatic drugs answers, 64–65 questions, 60–61 apathetic hyperthyroidism, 77 apical systolic murmur, 11 apnea. See obstructive sleep apnea arginine vasopressin (AVP) secretion, 76, 102 arm/leg weakness, 84 arrhythmias and clinical syndromes answers, 18 questions, 9–11 arterial insuffi ciency, right second toe, 83 arthralgias, 32, 59, 62, 64 arthritis, 17 acute monoarthritis, 62 in�ammatory arthritis, 65
psoriatic arthritis, 62 asbestosis, 42 ascites, 30 Aspergillus, 55 aspirin, 14, 15, 27, 37, 68, 80, 98, 99, 100, 101, 130 assisted suicide, 138 asthma, 118 during pregnancy, 105, 107 ataxic dysarthria, 116 atenolol, 14, 15, 99, 103 atherosclerosis, 16 atorvastatin, 15, 68, 69, 70, 71 atrial �brillation, 12, 18, 29, 72, 93, 113 chronic, 86, 87 new-onset, 67 atrial septal deect (ASD), 11, 19 atrial septum shunts, 11, 19 atrioventricular block, �rst-degree, 130 atypical squamous cells o undetermined signi�cance (ASC-US), 125 autoimmune hemolytic anemia, 89 autoimmune hepatitis, 34 axillary adenopathy, 49 axillary lymph node metastases o adenoma o unknown origin, 81 axillary mass, right, 79 azithromycin, 52, 118 azoospermia, 44
B back pain chest/neck radiation, 15 chronic musculoskeletal, low back, 87 lumbar spine point tenderness, 88 new-onset, 95 bariatric surgery, 75 B-cell lymphoma, 88, 91 bee sting symptoms, 105 Bell palsy, 66 benign prostatic hypertrophy, 37, 71 benzodiazepines, 111, 133 β-agonists, 107 β-blockers, 21, 35, 77, 79, 139 β-lactam antibiotics, 52 bibasilar coarse rales, 36 bibasilar crackles, 35, 37 bilateral carotid bruits, 10 bilateral expressible galactorrhea, 71 bilateral hilar lymphadenopathy, 16, 41 bilateral pitting edema, 13 bilateral pulmonary emboli, 16 biliopancreatic diversion, 75 bioterrorism, 52–53 bipolar disorder, 109, 110, 129 bismuth subsalicylate, 31 bisphosphonates, IV, 72, 77 biventricular pacing, 21 bloating, postprandial, 29 blood glucose monitoring, 67
���
body mass index (BMI), 15, 17, 40, 60, 67–71, 68, 74 bone and parathyroid disease answers, 76–77 questions, 72–73 bradycardia, 18 BRCA1 mutation, 137 BRCA2 mutation, 125, 135 breast cancer, 50, 79, 81 node-positive, 79 breast enlargement, male, 71, 75–76 breast-eeding, 71 breast masses, 127, 136 breast pain, cyclic, 127, 136 breath sounds, diminished, 37 Bruce protocol stress test, 15 budesonide, 31, 105, 107 Buerger disease, 59, 60, 64 bupropion, 111
C CA 125 elevation, 81 calcitriol therapy, 73 calcium oxalate arthropathy, 65–66 calcium oxalate nephrolithiasis, 17 calcium supplement, 70, 73, 76 Candida albicans, 52 cardiac murmur, 18 f cardiac physical examination answers, 18–20, 19 f questions, 11–12 cardiogenic syncope, 11, 18 cardiology answers, 18–24 arrhythmia, clinical syndromes, 18 cardiac physical exam, 18–20, 19 f congestive heart ailure, 20–21 coronary artery disease, 21–22 hypertension, 24 myocardial inarction, 21–22 vascular medicine, 22–24 questions arrhythmia, clinical syndromes, 9–11 cardiac physical exam, 11–12 congestive heart ailure, 12–13 coronary artery disease, 14–15 hypertension, 17 myocardial inarction, 14–15 vascular medicine, 15–17 cardiomegaly, 30 carotid stenting, 23–24 carotid ultrasonography, 85 carvedilol, 11, 12 caspoungin, 55 cataracts, 61, 65 surgery, 131, 139 cefriaxone, 33, 46, 55 celiac disease, 32 Centers or Disease Control and Prevention, 134
central hypothyroidism, 74 central nervous system cryptococcosis, 54 central venous catheter-related bloodstream inections (CRBSIs), 50 cephalexin, 53 cephalosporin (third-generation), 33 cerebellar disorder, 116 cervical adenopathy, 49 cervical lymphadenopathy, 48, 87 chemical gastritis, 28 chemotherapy ABVD combination, 87 or breast cancer, 50 ebrile neutropenia rom, 81 R-CHOP chemotherapy, 91 or rectal cancer, stage II, 80 chest pain anterior, 72 atypical, 19 exertional, 11, 14, 15, 36 at rest, 11 retrosternal, 14 right-sided, 36 substernal, 35 chest radiography dense upper lef consolidation, 45 diffuse pulmonary in�ltrates, widened mediastinum, 46 questions, 39–40 choking episodes, 113 cholecystectomy, elective, 84 chronic hepatitis C, 30 chronic interstitial nephritis, 98 chronic kidney disease (CKD), 93, 94, 101, 130 stage 3, 99, 100, 103 stage 4, 103 treatment recommendations, 104 chronic lymphocytic leukemia (CLL), 90 chronic obstructive pulmonary disease (COPD), 39, 40, 43, 79, 80, 96, 114, 118, 131 chronic renal ailure, 47 answers, 103–104 questions, 99–101 chronic respiratory acidosis, 96 chronic respiratory alkalosis, 96, 103 Churg-Strauss syndrome, 42, 60, 64 Chvostek sign, 73 cipro�oxacin, 33, 50, 52, 55 circumcision, 55 cirrhosis, 30, 33, 38 claudication intermittent, 15, 16, 22 pseudoclaudication, 16, 22 clindamycin, 53 clonal thrombocytosis, 89 clopidogrel, 14, 15 Clostridium difficile inection, 49–50, 55–56 clumsiness, hands, 113 coagulase-negative staphylococci, 56 coagulation answers, 90 questions, 85–87 cocaine use, 98, 103 codeine, 129, 138 colitis, microscopic, 31 colon and pancreas answers, 31 questions, 27 colon cancer amily history, 134 recurrence pattern, 82 screening, 27, 79 colonoscopy, 27, 28, 82 colon polyps, 27, 87
common variable immunode�ciency disease (CVID), 107 community-acquired pneumonia, 7, 37, 52 computed tomography (C), 12 colorectal cancer, 82 diverticulosis without abscess, 27 amilial pheochromocytoma, 24 myasthenia gravis, 115 retroperitoneal/mesenteric lymphadenopathy, 88 right axillary adenopathy, 79 sinuses, 106 conusion, 93, 95 congestive heart ailure (CHF), 18, 67, 80, 109 answers, 20–21 questions, 12–13 constrictive pericarditis, 12, 20 continuous positive airway pressure (CPAP) therapy, 70, 94 contrast-induced acute kidney injury (AKI), 104 contrast nephropathy, 22 coronary angiogram, 12, 14, 15, 72, 87 coronary artery bypass graf surgery, 12, 100 coronary artery disease, 50, 68, 79, 80, 98, 100 answers, 21–22 questions, 14–15 coronary bypass surgery, 68 coronary heart disease, with Q prolongation, 109, 111 corticosteroids, 65, 107, 118 cough, 79 dry, 36 mild, 16 nonproductive, 37, 87 productive, 45, 106 recurrent, 129 severe, 16 CPAP (continuous positive airway pressure) therapy, 70, 94 CRB-65 score, 52 C-reactive protein, 47 CRES syndrome, 62, 66 critical care medicine answers, 41 questions, 35–36 Crohn disease, 29 cross-content areas answers general internal medicine, 137–138 geriatrics, 133 medical ethics, palliative care, 138–139 perioperative medicine, 139 preventive medicine, 133–134 women’s health, 134–137 questions general internal medicine, 127–129 geriatrics, 123–124 medical ethics, palliative care, 129–130 perioperative medicine, 130–132 preventive medicine, 124–125 women’s health, 125–127 cryoglobulinemia, 103 cryoglobulinemic vasculitis, 66 cryptogenic organizing pneumonia, 42 crystalloid, IV, 35 CURB-65 score, 52 Cushing syndrome, 75 cutaneous leukocytoclastic vasculitis, 62 cyclophosphamide chemotherapy, 79–80 cystic �brosis, 44 cystic undic gland polyps, 28 cystitis, 56 cytarabine, 49 cytomegalovirus, 48, 97, 137
D dabigatran, 86, 87, 90, 137 daptomycin, 56 daunorubicin, 49 death, patient acceptance, 129, 138 decongestants, topical, 107 deep vein thrombosis (DV), 16, 23, 137 delirium, 35, 41, 96, 133 dementia, multi-inarct, 93 dementia with Lewy bodies, 133 depression, 109, 114 dermatitis herpetiormis, 119 f, 121 dermatology answers, 120–121 questions, 117–119 dermatomyositis, 118 f, 120 dermatomyositis sine myositis, 120 dermopathy, 74 descending aortic dissection, 23 deviated nasal septum, 46 dexamethasone, 56–57, 81 diabetes insipidus, postoperative, 71, 76 diabetes mellitus, 13, 15, 16, 22, 24, 47 answers, 74–75 questions, 67–69 remission rates, 75 type 2, 9, 45, 50, 80, 98, 109, 125 diabetic kidney disease (DKD), 100, 103 diabetic nephropathy, 98 diabetic neuropathy, 99 diaphoresis, 69 diaphragmatic calci�cation, 37 diarrhea, 27, 28, 29, 32, 45 dicloenac-misoprostol, 61 dicloxacillin, 53 diffuse large cell non-Hodgkin lymphoma, 130 diffuse lung disease answers, 41–43 questions, 36–38 digital clubbing, 36 digoxin, 9, 11 dilated cardiomyopathy, 11, 20–21 dilated inerior vena cava, 38 diltiazem, extended-release, 101 distal renal tubular acidosis (RA), 95, 102 distal tubular diuretics, 102. See also metolazone diuretics, 79 diverticulitis, 27, 31 dizziness, 109, 113 DNA-based testing, 90 donepezil, 133 Doppler ultrasonography, 30, 99 doxazosin, 17 doxorubicin, 79 doxycycline, 55 dronedarone, 11, 18 drug-eluting stents, 131 drug-induced lung disease, 42 D-shaped lef ventricle, 38 dual energy x-ray absorptiometry (DEXA), 72 duodenal switch bypass, 75 Duplex ultrasonography, 16 dysphasia, 93 dysplasia, low-grade, 27 dyspnea, 12, 13, 37, 40, 45, 59, 62, 72, 87 acute onset, 16, 35, 36 exertional, 11, 83 progressive, 39, 84 dysproteinemia, 90
E early-stage (I-IIA) nonbulky disease, 91 edema intermittent peripheral, 87
��� • I N D E X
lower extremity bilateral, 60, 67 bilateral ankle, 98 bilateral pitting, 12 leg edema, 30 pitting, thigh, 100 pitting, pretibial, 99 pretibial, 37 periorbital edema, 120 peritumoral edema, 81 pulmonary edema, 13 trace, peripheral, 14 ejection murmur, harsh, 10 electrocardiogram (ECG), 9 f, 10 f, 11, 14 electrolyte disorders answers, 102 questions, 93–94 embolism answers, 43 questions, 38–39 enalapril, 12, 50, 80 endocarditis, 105 endocrine tumors, 32 endocrinology answers bone, parathyroid disease, 76–77 diabetes mellitus, 74–75 hyperlipidemia, 74–75 hypoglycemia, 74–75 obesity, nutrition, 75 pituitary, gonadal, adrenal disorders, 75–76 thyroid disease, 74 questions bone, parathyroid disease, 72–73 diabetes mellitus, 67–69 hypoglycemia, 67–69 obesity, nutrition, 70–71 pituitary, gonadal, adrenal disorders, 71–72 thyroid disease, 67 endometrial cancer, 79 endotracheal intubation, 37, 46 end-stage renal disease (ESRD), 101 Entamoeba histolytica, 55 enzyme-linked immunosorbent assay (ELISA), 48 epilepsy, temporal lobe, 116 episcleritis, 31, 137 Epstein-Barr virus, 48, 97 erectile dysunction, 71 gradual-onset, 129, 138 erythema chronicum migrans, 66 erythema multiorme, 117 f, 120 erythematous nodosum, 41, 46 erythematous sclera, 128–129 erythromycin, 53 Escherichia coli, 32, 55 esophagogastroduodenoscopy (EGD), 28, 30, 31 esophagus and stomach answers, 31–32 questions, 28 essential thrombocytopenia, 83 etanercept, 60, 65, 66 ethambutol, 47, 53 euthanasia, 138 exertional dyspnea (NYHA unctional class III), 11 exophthalmos, bilateral, 67
F amilial amyloidosis, 20 amilial hypocalciuric hypercalcemia (FHH), 73, 77 atigue, 30 ebrile neutropenia, 81
ecal leucocytes, 29 emoral-popliteal bypass surgery, 98 emoral-popliteal deep vein thrombosis (DV), 16 emoral, popliteal, posterior pulses, 15, 16 eno�brate, 70 entanyl, 138 entanyl patient-controlled analgesia, 98 evers, low-grade, 59 exoenadine, 72 �bromyalgia-like symptoms, 64 �brotic lung disease, 442 �nger telangiectases, 37, 43 �sh oil capsules, 48, 68, 69, 71, 75, 101 �uconazole, 55 �uoroquinolones, 55, 56 �uticasone propionate, 44, 80 olic acid supplement, 60, 61, 71, 85 oraminotomy, 123 osinopril, 70 4-methylpyrazole therapy, 102, 103 urosemide, 11, 12, 35, 94, 99, 102
G gabapentin, 99 gait ataxia, 113, 116 galactorrhea, 71, 135 gallstones, 31, 89 gastric banding, laparoscopic adjustable, 75 gastric cardia malignancy, 32 gastric MAL lymphoma, 91 gastroenterology and hepatology answers colon and pancreas, 31 esophagus and stomach, 31–32 liver, 33–34 small bowel and intestine, 32–33 questions colon and pancreas, 27 esophagus and stomach, 28 liver, 29–30 small bowel and intestine, 28–29 gastroesophageal re�ux disease (GERD), 31–32, 37 gastrointestinal tract (upper) bleeding, 30 general internal medicine answers, 137–138 questions, 127–129 gentamicin, 56 geriatrics answers, 133 questions, 123–124 giant cell arteritis (GCA), 59, 60, 64 Gilbert syndrome, 33 glaucoma, acute angle-closure, 129, 138 glimepiride, 68, 69 glipizide, 70, 80 glomerular disease answers, 103 questions, 97–99 glucosamine chondroitin sulate, 60, 65 gluten-sensitive enteropathy, 121 glyburide, 37 glycoprotein IIb/IIIa inhibitors, 21 goiter, multinodular, 77 gonococcal arthritis, 54 gonococcal co njunctivitis, 138 gonorrhea, 56, 138 Gottron papules, 120 gout, 61–62, 65, 101 pseudogout, 66 granulomatous disease, 41 Graves disease, 74, 77 Graves ophthalmopathy, 74 group A β-hemolytic streptococcal (GABHS) pharyngitis, 138 Guillain-Barré syndrome (GBS), 115–116
gynecologic examination, 125 gynecomastia, 71, 75–76
H H2 receptor blocker, 32
Haemophilus in�uenzae, 52, 56, 106
haloperidol, 133 hands hand clumsiness, 35, 113 rheumatoid nodules, 60 synovitis, proximal interphalangeal joints, 60 Hashimoto thyroiditis, 67 hCG-producing tumor, 75–76 headache migraine, 114, 116, 118 new-onset, 59 post-partum, 71 heart bibasilar crackles, 35, 37 cardiac murmur, 18 f �rst sound, normal, 11 ourth sound, 12, 13 holosystolic murmur, 99 second sound, reverse splitting, 10 systolic ejection murmur, 68, 99 systolic murmur, 35 third sound, intermittent, 12 heart block, third-degree, 50 heartburn, intermittent, 28 heart ailure due to biventricular dysunction, 12 with preserved ejection raction, 20 heart sounds Helicobacter pylori, 28, 31, 91 hematology answers anemias, myeloid malignancies, 89–90 coagulation, 90 lymphoid malignancies, 90–91 questions anemias, myeloid malignancies, 83–85 coagulation, 85–87 lymphoid malignancies, 87–88 hemoglobinopathies, hig h-oxygen affi nity, 89 hemolytic anemia, 83, 103 heparin, 90. See also low-molecular-weight heparin dosing consensus, 128 intravenous, 14 subcutaneous, 86 hepatic cytochrome P450 enzymes, 53 hepatitis, autoimmune, 34 hepatitis A inection, 135 hepatitis B inection, 30, 34 hepatitis C inection, 30, 48, 62, 66, 97 asymptomatic, 103 hepatocellular carcinoma (HCC), 34 hepatology. See gastroenterology and hepatology hepatopulmonary syndrome, 44 hereditary hemorrhagic telangiectasia, 44 hereditary nonpolyposis colorectal cancer (Lynch syndrome), 81 herpes simplex encephalitis, 57 Hickman catheter tunnel, 50 hip arthroplasty (total), 17 hip racture, 87, 131 histoplasmosis, 53 HIV inection, 47, 48–49, 54–55, 97 hoarseness, 28 Hodgkin lymphoma therapy, 90 honeycombing (lungs), 37 hopelessness, 109 hospital-acquired pneumonia, 46 hot �ushes, 126, 135 HPV vaccine, 135
INDEX
hunger, excessive, 69 hungry bone syndrome, 77 hydralazine, 21 hydration, IV �uid, 71 hydrochloroquine, 123 hydrochlorothiazide (HCZ), 15, 17, 63, 80, 93, 102 hydrochlorothiazide-triamterene, 68, 69, 70 hydrocortisone, 71–72 hydroxychloroquine, 61, 63, 65 hyperbilirubinemia, indirect, 33 hypergammaglobulinemia, 33 hyperlipidemia, 12, 14, 15, 16, 27, 29, 35, 61, 70, 100 answers, 74–75 �sh oil treatment, 48 questions, 67–69 hypernatremia, 94, 102 hyperparathyroidism, 73, 76 hyperplastic polyps, 31 hypertension, 12, 13, 14, 15, 16, 27, 35, 37, 40, 60, 61, 67, 70, 80, 100 answers, 24, 43 questions, 17, 38–39 with type 2 diabetes, 47 hyperthyroidism apathetic, 77 iodine-induced, 77 hypertrophic cardiomyopathy (HCM), 21, 41 hyperuricemia, 61 hypocalcemia, 72, 76–77 hypochondriasis, 111 hypogammaglobulinemia, 107 hypoglycemia answers, 74–75 questions, 67–69 hypogonadism, 67, 74 hypokalemia, 94 hyponatremia, 45, 93, 102 hypo-osmolar hyponatremia, 102 hypotension, 35, 37 orthostatic, 115 postpartum hemorrhage-induced, 76 hypothyroidism, 30, 67, 74, 102 hypoxemia, 37
I iatrogenic thyrotoxicosis, 74 ibuproen, 30, 101 idiopathic bronchiolitis obliterans with organizing pneumonia (BOOP), 42 idiopathic dilated cardiomyopathy, 21 idiopathic pulmonary �brosis (IPF), 42, 44 IgA nephropathy, 98, 103 IgE-mediated reaction to penicillin, 107 ileal resection, 29, 33 immune-mediated heparin-induced thrombocytopenia type II, 90 immunosuppressive therapy, 34 implantable cardioverter-de�brillator, 11, 12 inectious diseases answers bone, joint inections, 53–54 HIV inection, 54–55 joint inections, 53–54 pneumonia, zoonoses, travel, bioterrorism, 52–53 skin, sof tissue inections, 53–54 syndromes cardiovascular, bloodstream, CNS, 56–57 gastrointestinal tract inection, 55–56 sexually transmitted diseases, 55–56 urinary tract inection, 55–56
•
���
questions bone, joint inections, 46–47 HIV inection, 48–49 joint inections, 46–47 mycobacterial inections, 46–47 pneumonia, zoonoses, travel, bioterrorism, 45–46 skin, sof tissue inections, 46–47 syndromes cardiovascular, bloodstream, CNS, 50–51 gastrointestinal tract inection, 49–50 sexually transmitted diseases, 49–50 urinary tract inection, 49–50 inective endocarditis, 50 inerior vena cava (IVC) dilation, 38 inerior vena cava (IVC) �lter replacement, 16, 23 in�ammatory arthritis, 65 in�ammatory bowel disease, 31 in�ammatory synovial �uid, 54 in�iximab, 60, 65 in�uenza vaccination, 124 inrapopliteal arterial occlusive disease, 24 inguinal adenopathy, 49 inguinal hernia, 29 inhalation anthrax, 53 insomnia, 109 insulin, 15, 98 intercourse pain, 109, 126 intermittent claudication, 16 invasive dental procedures, 56 iodine-induced hyperthyroidism, 77 ionized calcium elevation, 72 ipratropium, 39 ipratropium bromide inhaler, 93 iron de�ciency anemia, 28, 31, 32 irritable bowel syndrome (IBS), 29, 32–33 isolated central nervous system (CNS) vasculitis, 60, 64 isoniazid, 47, 53 isosorbide dinitrate, 21
J jaundice, 29 jugular venous distention, 13 jugular venous pressure (J VP), 12, 20, 38
K kidney stones, recurrent, 73 knees acute monoarthritis, 62 arthroscopic surgery, 98 bilateral intermittent effusions, 62 joint swelling, 47 lef knee pain, 59 lef, total arthroplasty, 130 mild discomort, 61 osteoarthritis, 60 sudden onset pain, swelling, 62 synovitis, 85 total knee replacement, 85
L lamivudine, 34, 54 large joint monoarticular arthritis, 54 LEAN quality improvement methodolog y, 138 lef bundle branch block (LBBB), 11, 18 lef ventricular assist device (LVAD), 12 lef ventricular ejection raction (LVEF), 11, 12, 18, 20 lef ventricular ailure, 41 lef ventricular hypertrophy, 13 leg/arm weakness, 84 leg edema, 30
Legionella pneumophila, 37, 52
leukocytosis, 45 leukotriene receptor blockers, 107 leuprolide, 97 levo�oxacin, 46 levothyroxine, 67, 74 light-headedness, 95 limb dysmetria, 116 lip telangiectases, 37, 43 lisinopril, 11, 14, 15, 17, 27, 37, 67, 68, 69, 123, 130 lisinopril-hydrochlorothiazide, 97 lithium carbonate, 109, 111 liver answers, 33–34 questions, 29–30 liver Doppler ultrasonography, 30 liver lobe abscess, 55 lobar pneumonia, 48 Lögren syndrome, 41–42 loop diuretics, 102. See also urosemide loose stools, 27 loperamide hydrochloride, 31 losartan, 17, 70, 100, 101 lovastatin, 80 low-molecular-weight heparin (LMWH), 128, 131, 137, 139 low-voltage QRS complexes, in the l imb leads, 87 lumbar spine tenderness, 59 lung cancer, 102 lung contusions, bilateral, 35 lungs dense upper lef consolidation, 45 diffuse bilateral crackles, 45, 46, 98 inspiratory crackles, 45 right midlung �eld, 84 lupus anticoagulant (LAC), 86, 90 Lyme disease, 66 lymph node biopsy, 79 lymphocytic hypophysitis, 76 lymphocytic thyroiditis, 77 lymphocytosis, 87 lymphoid malignancies answers, 90–91 questions, 87–88 Lynch syndrome (hereditary nonpolyposis colorectal cancer), 81
M macrocytic anemia, 89 magnetic resonance imaging (MRI) arterial distribution inarctions, 60 breast cancer evaluation, 81 diabetic oot inection, 127 head: sellar mass with suprasellar extension, 71 lef hippocampus atrophy, 114 pituitary microadenoma, 71, 76 MAL lymphoma, 87–88, 91 mammography recommendations, 134 medical ethics, palliative care answers, 138–139 questions, 129–130 medullary thyroid carcinoma, 72 megaloblastic anemia, 89 membranoprolierative glomerulonephritis, 103 membranous nephropathy, 98 memory problems, 123 meperidine, 138 meropenem, 49 mesalamine, 27 mesenteric ischemia, 33 mesenteric lymphadenopathy, 88 metabolic acidosis, 96 metabolic alkalosis, 96
metormin, 15, 29, 33, 67, 68, 69, 70, 74, 99, 100, 101, 130 methicillin-resistant Staphylococcus aureus (MRSA), 47, 51, 53, 56 methimazole, 74 methotrexate, 60, 61, 85 methylprednisolone, 62, 123 metolazone, 94, 102 metoprolol, 9, 14, 15, 71, 98, 99, 101, 103, 130 microhematuria, asymptomatic, 97 microscopic colitis, 31 midsystolic click, 11, 19 migrant headache, 114, 116, 118 minimal change nephropathy, 98 Mini-Mental State Examination (MMSE), 123 mitral regurgitant murmur, 19, 19 f mitral valve disease, 29 mitral valve prolapse, 11, 19 mitral valve replacement, 51 Model or End-Stage Liver Disease (MELD), 132 monoamine (histamine-tyramine) poisoning, 53 monoarthritis, acute, 62 monoclonal gammopathy o undetermined signi�cance (MGUS), 90 motor vehicle collision, 36 MRSA prosthetic valve endocarditis, 56 mucormycosis, 55 multi-inarct dementia, 93 multinodular goiter, 77 multiple endocrine neoplasia (MEN), type 2, 76 multiple myeloma, 91, 102 multiple sclerosis (MS), 60 multivitamin supplement, 67, 70, 71, 72, 80, 97, 101 muscle weakness, 115 diffuse, 94–95 slowly progressive, 63 myalgias, 59, 64 diffuse, 98 myasthenia gravis, 115 Mycobacterium tuberculosis, 47 myelodysplastic syndrome (MDS), 83, 89 myeloid malignancies answers, 89–90 questions, 83–85 myeloprolierative neoplasm, 89 myocardial contusion, post-trauma, 41 myocardial inarction, 123, 125, 130 answers, 21–22 questions, 14–15
N nabumetone, 60 N -acetylcysteine, 44
naproxen, 61, 70 nasal passages congestion, 105 hyperemic, 46 nasal regurgitation o liquids, 113 nasal spray, saline, 107 necrotizing ascitis, type 1, 53 Neisseria gonorrhoeae, 107 Neisseria gonorrhoeae inection, 54, 105 Neisseria meningitidis inection, 105 nephrology answers acid-based disorders, 102 acute renal ailure, 102–103 chronic renal ailure, 103–104 electrolyte disorders, 102 glomerular disease, 103 questions acid-based disorders, 94–97
acute renal ailure, 97 chronic renal ailure, 99–101 electrolyte disorders, 93–94 glomerular disease, 97–99 neuroleptic agents, 133 neuroleptic malignant syndrome, 109, 111 neurology answers, 115–116 questions, 113–114 new-onset headache, 59 niacin, 29 nicotine dependence, 67 niedipine, 61 night sweats, 135 nitrourantoin, 37, 55, 56 nitrourantoin lung toxicity, 42 nitroglycerin, 14, 35 nocturnal continuous positive airway pressure, 94 node-positive breast cancer, 79 nodular sclerosing Hodgkin lymphoma, 87 nonalcoholic steatohepatitis, 38 nonarticular rheumatism, vasculitis answers, 64 questions, 59–60 non-Hodgkin lymphoma, 30, 130 nonishemic dilated cardiomyopathy, 11 nor�oxacin (oral), 33 normochromic anemia, 59 North American Symptomatic Carotid Endarterectomy rial (NASCE), 23 nortriptyline, 123 NPH insulin, 101 nucleoside, oral/nucleoside analogue, 34 nucleoside reverse transcriptase inhibitors (NRIs), 54 nutrition answers, 75 questions, 70–71 nutritional supplementation calcium, 70, 73, 76 �sh oil capsules, 48, 68, 69, 71, 75, 101 olic acid, 60, 61, 71, 85 multivitamin, 67, 70, 71, 72, 80, 97, 101 vitamin B12, 70, 75 vitamin D, 70, 72, 76 vitamin E, 71, 101
O obesity, 29, 68, 70, 100, 101. See also body mass index obesity and nutrition answers, 75 questions, 70–71 obsessive-compulsive disorder, 111 obstructive sleep apnea (OSA), 44, 70, 94 occupational lung disease answers, 41–43 questions, 36–38 oculomasticatory myorhythmia, 28 olanzapine, 109 oliguric acute renal ailure, 98, 103 omega-3 atty acids, 75 omeprazole, 37, 99 oncology answers, 81–82 questions, 79–80 onycholysis, 37 open wound/oul-smelling pus, 47 oral contraceptives, 74, 98 orchiectomy, 82 orlistat, 74 orthopedic injury repairs, 35 orthopnea, 12 orthostatic hypotension, 115 osteoarthritis, rheumatoid arthritis
��� • I N D E X
answers, 64–65 questions, 60–61 outpatient parenteral antibiotic therapy (OPA), 46–47, 53 ovarian cancer, 79 oxygen therapy, 44 oxymetazoline, 107
P pancreatitis, acute, 31 pansystolic murmur, 13 paradoxical embolism, 44 paranoid belies, 109 parathyroid adenoma, 73 parathyroidectomy, 73 parathyroid hormone (PH), 73, 77 parenteral antihypertensive drug choice, 17 paresthesias o the hands, 68 Parkinson disease, 124, 133 paroxysmal nocturnal dyspnea, 12 parvovirus B 19, 97 Pasteurella multocida, 53 patent oramen ovale (PFO), 11, 19 PCP prophylaxis, 55 PDE-5 inhibitors, 138 pelvic radiotherapy, 81 penicillin, 105 IgE-mediated reaction to, 107 percutaneous coronary intervention (PCI), 14–15, 22 pericardium myopathy, 11 perioperative medicine answers, 139 questions, 130–132 periorbital edema, 120 peripheral artery disease (PAD), 22 peripheral neuropathy, 20, 68, 98 peripheral neuropathy Lupus-like syndrome, 53 peripheral vascular disease, 47, 131 peritumoral edema, 81 periumbilical discomort, 29 pes anserine bursitis, 64 pheochromocytoma, 17, 76 Philadelphia chromosome-negative chronic myeloid leukemia (CML), 83, 89 physiologic sleep, 36 pigment gallstones, 89 pituitary, gonadal, adrenal disorders answers, 75–76 questions, 71–72 pituitary tumors, 71, 76 plantar ulcer, 47 plasma cell dyscrasia, 102. See also AL amyloidosis; multiple myeloma plasma cell prolierative disorder, 91 plethoric ace, 84 pleural effusion lef-sided, 37 right-sided, 37 pleural thickening, 37 pneumococcal pneumonia, 54, 56 pneumococcal vaccination, 124, 134 pneumocystis pneumonia (PCP), 49 pneumonia acute, 96 acute eosinophilic, 42 answers, 52–53 community-acquired, 7, 37, 52 cryptogenic organizing, 42 hospital-acquired, 46 lobar pneumonia, 48 organizing, with idiopathic bronchiolitis obliterans, 42 pneumococcal, 54, 56 pneumocystis pneumonia, 49 questions, 45–46
recurrent, 37 severe, bilateral, 130 ventilator-associated, 45 polyarthritis, 41 polycystic ovary syndrome, 67 polycythemia, 89 polymyalgia rheumatica (PMR), 59 polymyositis, 63 polysomnography, overnight, 44 polysubstance abuse, 95, 96 popliteal system clot, 128, 137 portal hypertension, 33 post-exertion syncope, 12 postpartum hemorrhage-induced hypotension, 76 postprandial bloating, 29 potassium, IV, 102 prasugrel, 21–22 pravastatin, 37, 48 prazosin, 37 prednisone, 60, 63, 64, 85 premature coronary artery disease, 22 premature gallstones, 89 premature ventricular complexes (PVCs), 13 presumed consent, or minors, 139 pretibial edema, 37 preventive medicine answers, 133–134 questions, 124–125 primary biliary cirrhosis (PBC), 33 primary hypothyroidism, 67 primary myelo�brosis (PMF), 83 procainamide, 9 prolonged Q interval, 138 propoxyphene, 138 propylthiouracil (PU), 67, 74 prostate cancer, 81 metastatic, 97 prostatectomy, radical, 35, 82 prostate enlargement, 80 prosthetic hip joint, 47 proteinuria, 60, 103 proton pump inhibitor (PPI) therapy, 28, 31–32, 37 proximal renal tubular acidosis (RA), 102 pruritic hives, 105 pruritic rash, 14, 87 pseudoachalasia, 32 pseudogout, 66 Pseudomonas aeruginosa, 52 psoriasis vulgaris, 118 f, 120 psoriatic arthritis, 62 psychiatry answers, 111 questions, 109–110 psychosis, substance-induced, 111 pulmonary artery catheterization, 37 pulmonary artery hypertension, 44 pulmonary diseases answers critical care medicine, 41 diffuse lung disease, 41–43 occupational lung disease, 41–43 signs, symptoms, chest radiography, 43–44 vascular disease, embolism, hypertension, 43 questions critical care medicine, 35–36 diffuse lung disease, 36–38 occupational lung disease, 36–38 signs, symptoms, chest radiography, 39–40 vascular disease, embolism, hypertension, 38–39 pulmonary edema, 13 pulmonary embolism, 35–36, 86
answers, 43 questions, 38–39 pulmonary hypertension, 66 pulmonary tuberculosis, 47 puri�ed protein derivative (PPD) skin test, 47, 53, 55 purpura, legs and eet, 97 pyelonephritis, 56 pyoderma gangrenosum, 119 f, 120–121 pyrazinamide, 47, 53 pyridoxine, 47
Q quality improvement interventions, 137, 138 �uantiFERON-B Gold test, 47, 53 quinapril, 68
R radial pulse diminishment, 15 rash heliotrope, 120 lower extremities, 97 pruritic hives, 105 pruritic rash, 14, 87, 119 R-CHOP chemotherapy, 91 rectal cancer, stage II, 80, 81 renal biopsy, 97, 101 renal cell carcinoma, 99, 101, 131 renal thrombotic microangiopathy, 103 renal ultrasonography, 99 respiratory illness, upper, 129 retrobulbar neuritis, 53 retroperitoneal lymphadenopathy, 88 rhabdomyolysis, 103 rheumatoid arthritis, 85 answers, 64–65 questions, 60–61 rheumatology answers, 64–66 antirheumatic drugs, 64–65 miscellaneous conditions, 66 nonarticular rheumatism, vasculitis, 64 osteoarthritis, rheumatoid arthritis, 64–65 spondyloarthropathies, 65 questions, 59–63 antirheumatic drugs, 60–61 miscellaneous conditions, 62–63 nonarticular rheumatism, vasculitis, 59–60 osteoarthritis, rheumatoid arthritis, 60–61 spondyloarthropathies, 61–62 rhinoconjunctivitis, allergic, 107 rhinorrhea, 105 rhomboid bireringent crystals, 66 rib ractures, 35, 95 riampin, 47, 53, 56 riaximin, 33 right (dormant) hand clumsiness, 35 right ventricular heave, 38 rituximab, 91 rosacea, 117, 118 f, 120 rosiglitazone, 74, 75 Roux-en-Y gastric bypass, 70, 75
S saddle pulmonary embolism, 35 salmeterol, 44, 80 sarcoidosis, 41 Sarcoptes scabiei var hominis, 117 f, 120 schizophrenia, 109 seborrheic dermatitis, 49 secondary amenorrhea, 135 second heart sound splitting, 10 sedentary liestyle, 9
INDEX
selective serotonin reuptake inhibitors (SSRIs), 111 senile cardiac amyloidosis, 90–91 septoplasty, 46 seropositive rheumatoid arthritis, 61 serum brain natriuretic peptide (BNP) levels, 12, 20 Sheehan syndrome, 76 shock, severe, 37 shunts in the atrial septum, 11 sickle cell anemia, 47, 89 sickle cell crisis, 54 sickle cell disease, 84 sigmoid polyp, 27 sildena�l, 101 simvastatin, 12, 14, 15, 27, 33, 61, 63, 67, 68, 98, 99, 100, 101, 130 sinopulmonary inection, 106, 107 sinus bradycardia, 11 sinusitis, recurrent, 40, 46 sinus tachycardia, 35 Six Sigma quality improvement method, 138 Sjögren syndrome, 94–95 skin dryness, lower extremities, 68 skin hyperpigmentation, 28 sleep/sleepiness disrupted, ragmented, 41 excessive, 36 insomnia, 109 obstructive sleep apnea, 44, 70, 94 small bowel and intestine answers, 32–33 questions, 28–29 small-molecule intoxication, 103 smoking/smoking cessation, 15, 79 snoring, 40 socioeconomic issues, 133 somatization d isorder, 111 sotalol, 11, 18 speech difficulties, 113 spironolactone, 11, 12, 21 splenomegaly, borderline, 30 spondyloarthropathies answers, 65 questions, 61–62 sputum, progressively purulent, 45 squamous cell carcinoma, 79 staphylococcal toxic shock syndrome, 53 Staphylococcus aureus, 52, 55, 56 Starr-Edwards mitral valve, 131, 139 statins. See individual statins stavudine-tenoovir-lamivudine-darunavir with boosted ritonavir with raltegravir, 48 steatorrhea, 32 stenotic lesion, right carotid, 85 sterile pyuria, 103 steroid myopathy, 66 Streptococcus pneumoniae, 37, 106 stroke, 11, 14, 41, 79 S-segment depression, 14, 17, 21, 22 subepithelial collagen band (colon), 27 substance abuse, 95, 96, 98 substance-induced psychosis, 111 suicidal ideation, 109 sulonylurea medications, 74 sumatriptan, 114 sunitinib, 99 superior mesenteric artery embolus, 33 superior vena cava syndrome, 12, 20 swallowing, �uctuating difficulties, 113 symptomatic carotid disease, 16 syncope, 9–10, 13, 35 post-exertion, 12 syndrome o antidiuretic hormone (SIADH), 76, 102 synovitis, 60, 85
•
���
syphilis, IgM and IgG, 50 syphilis, secondary, 56 systemic lupus erythematosus (SLE), 63, 85, 86, 89–90 systolic click, 11, 12, 19 systolic click murmur, 20 systolic ejection murmur, 13
tachycardia, 70 tadala�l, 71 akayasu arteritis, 59, 64 tamsulosin, 71 tardive dyskinesia, 111 tea-colored urine, 97 temporal arteritis, 137 temporal lobe epilepsy, 116 tenoovir, 54 testosterone gel, 67 tetanus-diphtheria (d) vaccination, 46, 124, 133, 134 thiamine de�ciency, acute, 75 3-vessel disease, 22 thrombocytopenia, 90 thrombocytosis, extreme, 89 thrombotic thrombocytopenic purpura (P), 90 thrombus lef super�cial emoral vein, 128 right anterior tibial vein, 128 thrush, 49, 55 thyroid disease answers, 74 questions, 67 thyroid lobe nodule, 72 thyrotoxicosis, 74, 77 tiotropium, 39, 44 tissue plasminogen activator (tPA), 35 MP-SMX prophylaxis, 55, 56 tobacco use, 16 odd paralysis, 116 toe (right second), arterial insuffi ciency, 83 tonic-clonic seizure, 86 topiramate, 116 tramadol, 97, 123 transusion-related acute lung injury (RALI), 36, 41 transient ischemic attack (IA), 16 transient ischemic attack (IA)-like attacks, 60 transsphenoidal microadenomectomy, 71 transthoracic echocardiogram, 9 travel, 52–53 treadmill ECG, 11 trimethoprim-sulamethoxazole, 55, 65 trochanteric bursa tenderness, 59 Tropheryma whipplei, 32 tubal ligation, 70 tuberculin skin test (S), 47 tuberculosis, 47 tubular adenomas, 27 tubulovillous adenoma, 27 tularemia, 53 umor lysis syndrome, 103 tumor necrosis actor (NF) therapy, 61, 65 2-block claudication, 16 2-pillow orthopnea, 11 type B aortic dissection, 15–16 type 2 diabetes mellitus, 9, 45, 50, 80, 98, 99, 109, 125
U ulcerative colitis, 27 upper respiratory tract inection, 105 urinary tract inection, 55 urinary urgency incontinence, 133 urine, tea-colored, 97